Sesap 17th Vol 1

  • Uploaded by: amr elfaresy
  • 0
  • 0
  • February 2021
  • PDF

This document was uploaded by user and they confirmed that they have the permission to share it. If you are author or own the copyright of this book, please report to us by using this DMCA report form. Report DMCA


Overview

Download & View Sesap 17th Vol 1 as PDF for free.

More details

  • Words: 108,456
  • Pages: 183
Loading documents preview...
Volume I content Abdomen Part I

001

Part II

014

Part III

025

Part IV

------------------------------------------------------------------------

040

Alimentary Tract Part I

052

Part II

069

Part III

084

Part IV

101

Breast Part I

117

Part II

129

Endocrine

141

Head & Neck

155

Legal/Ethics

165

Oncology

175

Abdomin Part I

ITEMS 1-25 For each question, select the best possible response 1. A 60-year-old man presents for evaluation of a small right inguinal hernia. The hernia is easily reducible and is not associated with untoward symptoms. It has not limited the patient's activities. The patient would like to pursue nonoperative management for now. What is the most likely indication for surgical management of the hernia in the future? A. Cosmesis B. Impotence C. Pain D. Strangulation E. Bowel incarceration

2. The most likely diagnosis is A. cirrhosis. B. lymphoma. C. sarcoidosis. D. tuberculosis. E. chronic pancreatitis. 3. Initial management of his ascites should include A. repeat paracentesis. B. surgical exploration. C. peritoneal-venous shunt D. low-fat diet with medium-chain triglycerides. E. somatostatin analogues and parenteral nutrition.

ANAWARS: B, D ANSWER: C Chylous ascites is uncommon, accounting for only 1 in 20,000 hospitalizations in tertiary referral hospitals. Patients with nontraumatic chylous ascites generally present with increasing abdominal girth, weight gain, and occasionally dyspnea. Occasionally weight loss, abdominal pain, early satiety, diarrhea, and malnutrition are observed. In the Western world, malignancy, most commonly lymphoma, and cirrhosis are the most common etiologies; infectious causes, such as tuberculosis and filariasis, predominate in developing countries.

Groin hernias in men are among the most common problems encountered by the general surgeon. A man's lifetime risk of developing a groin hernia is more than 25%. Asymptomatic male patients presenting with an inguinal hernia can safely be offered nonoperative management (watchful waiting). However, the majority will ultimately pursue surgical repair within 10 years, most often due to pain. Delay in repair is not associated with an increase in operative complications. The incidence of acute presentation for strangulation or bowel incarceration during the period of watchful waiting is very low.

Additional causes include blunt and penetrating trauma, surgical trauma (primarily procedures that involve manipulation in the retroperitoneum, principally in the periaortic area where lymphatic disruption is possible), orthotopic liver transplantation, abdominal and pelvic radiation, sarcoidosis, idiopathic retroperitoneal fibrosis, acute/chronic pancreatitis, Whipple disease, and other intestinal malignancies that have a propensity to invade the root of the mesentery, including carcinoid. Hypo- and hyperplastic congenital lymphatic abnormalities, malignancy, and nonaccidental trauma are causes of spontaneous chylous ascites in children.

Items 2-3 A 69-year-old man presents with progressive abdominal distention and dyspnea over the past 9 weeks. Additional symptoms include anorexia, weight gain (2.7 kg), generalized malaise, and intermittent nighttime fevers and sweats. Abdominal examination demonstrates marked distention without a palpable mass; the ultrasound demonstrates marked intraperitoneal fluid. The CT scan is shown in figure 2-3.1. Paracentesis yields a milky fluid with an extremely high triglyceride content. Fluid analysis for cell count, Gram stain, and cultures are negative. His albumin content is 2.9 g/dL (3.5-5.2 g/dL). His dyspnea has now resolved after initial paracentesis.

Children undergoing surgical procedures and those who sustain blunt or penetrating trauma can develop chylous ascites as can their adult counterparts. The most likely etiology of chylous ascites in the case presented is lymphoma. Chylous ascites with the associated night sweats and abnormalities at the root of the mesentery on the CT scan make this diagnosis the most likely. Characteristically, "milky" fluid is observed by paracentesis. The fluid contains extremely high levels of triglycerides, usually greater than 200 mg/dL, and albumin. Cell count, Gram stain, and culture should also be used to exclude an infectious etiology. Occasionally, other laboratory determinations are used, such as glucose, lactate dehydrogenase, and adenosine deaminase (a test for tuberculosis) but these should be used selectively. CT imaging is helpful in the identification of abnormalities of lymph nodes and other abdominal or thoracic tumors. Lymphangiography can identify lymphatic leakage, lymphatic fistulas,and abnormalities of the thoracic duct.

Figure 2-3.1. 1

When lipiodol is used with bipedal lymphangiography, the study may also be therapeutic, resulting in closure of lymphatic fistulae in cases refractory to other conservative treatment strategies.

What treatment modality will decrease her hospital length of stay? A. Nasogastric tube decompression B. Correction of electrolytes C. Intravenous fluids D. Gastrografin challenge E. Nothing by mouth

Management should be directed at the underlying etiology when possible. Initial treatment should include dietary manipulation by substituting medium chain triglycerides for dietary fat. The second line of nonsurgical therapy involves fasting and parenteral nutritional support with or without the use of a somatostatin analog. Reports indicate these measures are effective in nearly all cases. Repeated paracentesis is typically avoided due to the risk of infection and malnutrition, unless required to relieve respiratory distress. Surgical exploration and ligation of damaged lymphatic channels is reserved for cases that fail conservative approaches. Peritoneal venous shunts are not recommended, except in very selected cases, due to the morbidity associated with their use in chylous ascites.

ANSWER:

D

The treatment of adhesive partial small bowel obstruction in patients who do not undergo immediate operative intervention includes nasogastric tube (NGT) decompression, correction of electrolytes, intravenous fluids, and nothing by mouth. Compared with standard nonoperative treatment alone, the addition of the Gastrografin challenge results in decreased hospital length of stay and delineates the need for operative intervention. Gastrografin may be diagnostic and therapeutic due to its osmotic effects. A100 mL dose of Gastrografin is given with 50 mL of saline via NGT in a patient who is not vomiting after evacuation of gastric contents. An abdominal radiograph is obtained 8 hours later. If contrast has reached the cecum or the patient has bowel movements, the test is considered successful, the NGT is removed, and clear liquids are started. Surgical intervention is strongly considered if contrast has not reached the colon at 8 hours after administration of Gastrografin.

4. What endoscopic finding predicts the highest rebleeding rate in a patient who has a bleeding peptic ulcer? A. Clot B. 2-cm ulcer C. Visible vessel D. Pigmented spot E. Injection therapy required to stop bleeding

6. A 55-year-old man with acute cholecystitis is taken to the operating room for laparoscopic cholecystectomy. The surgeon performs a dissection to attain the critical view of safety. Which of the following images demonstrates the critical view of safety? A. Figure 6.1 B. Figure 6.2 C. Figure 6.3 D. Figure 6.4 E. Figure 6.5

ANSWAR: C Initial management of bleeding duodenal ulcers is endoscopic management and medical treatment with a proton pump inhibitor (PPI). Studies to date have identified the following predictors of rebleeding after endoscopic therapy: ulcer size larger than 2 cm, hypotension, initial hemoglobin less than 10 mg/dL, fresh blood in the stomach, and active bleeding. The greatest risk for recurrent bleeding, however, is a visible vessel. The most consistent location of bleeding ulcers is in posterior duodenal bulb, and patients often have comorbidities and are older in age (>60 years). If patients rebleed after initial endoscopic treatment, repeat endoscopy with intervention should be attempted. Angioembolization is often performed if repeat endoscopy fails. Approximately 15% of patients fail angioembolization and require surgical intervention. If surgery is performed, care must be taken to ligate the transverse pancreatic artery in addition to the gastroduodenal artery when oversewing the ulcer after anterior duodenotomy.

5. A 45-year-old woman presents with a 6-hour history of crampy abdominal pain and the inability to pass flatus or have a bowel movement. Her medical history is significant only for a total abdominal hysterectomy. She is afebrile. Her abdomen is distended and tympanitic, but she does not have peritoneal signs. She has a normal white blood cell count.

Figure 6.1.

2

of surgical diseases. Its adoption, however, was associated with an increase in the number of common bile duct injuries compared with open cholecystectomy, and this difference has persisted to the present day. The lack of depth perception and decreased tactile feedback inherent in the laparoscopic approach contributed to this increase in injuries, in addition to misperceptions related to anatomy. To address this issue, Steven Strasberg and Nathaniel Soper proposed a modification of a common method used for ductal identification during open Cholecystectomy, which they called the Critical View of Safety technique This technique of identification has 3 key components: (1) the complete clearing of the hepatocystic triangle of all fat and fibrous material; (2) the separation of the lower pa rt of the gallbladder from the cystic plate; and (3) the identification of 2 and only 2 structures (cystic duct and cystic artery) remaining attached to the gallbladder. The Critical View of Safety should be confirmed intraoperatively by demonstrating these 3 elements before clipping and dividing the cystic artery and duct. This confirmation should be treated like a "time out" in which the elements are pointed out by the surgeon to the operative team.

Figure 6.2.

If the Critical View of Safety cannot be attained during a laparoscopic cholecystectomy, the surgeon should proceed with other "bail out" options, such as open conversion or subtotal fenestrating cholecystectomy. In the image that shows the correct answer, all 3 elements of the critical view of safety are present (figure 6.5). In the remaining images, dissection is incomplete, and the images do not demonstrate all 3 elements of the critical view of safety (figures 6.6-6.9).

Figure 6.3.

7. A 36-year-old woman presents with increasing abdominal fullness, dull right- sided pain, and early satiety after completing a 3-month hiking tour of Central and South America. Ultrasonography reveals the presence of a 10-cm cystic lesion on the left side of the liver with irregular walls and internal septations forming loculi. CT scan demonstrates the 10-cm cystic lesion in sections HI and IVa with evidence of irregular papillary growths, thickened cyst walls, internal septations, and no calcifications. MRI reveals heterogenous signal intensity, with a hyperintense signal on T2-weighted images and no daughter cysts. What is the most appropriate management of this lesion? A. Oral antimicrobial B. Aspiration C. Surgical unroofing D. Surgical enucleation E. Anatomic resection

Figure 6.4.

Figure 6.5.

ANSWER:

D

This patient has a biliary cystadenoma. Biliary cystadenomas can constitute up to 5% of all hepatic cysts and 10% of cysts larger than 4 cm in size. Symptoms, like other cystic disease, are typically related to size and include anorxia, early satiety, and abdominal discomfort. Imaging demonstrates heterogenetous septations within the cyst, irregular papillary

ANSWER: E The introduction of laparoscopic cholecystectomy revolutionized general surgery, leading to the widespread adoption of minimally invasive techniques to treat a variety

3

growths, and thickened cyst walls. The cysts also typically show hyperintensity on MRI T2 weighting. Biliary cystadenomas are premalignant lesions that can progress to biliary cystadenocarcinoma. As such, they require surgical removal, either via resection with clear margins or enucleation at the level of the pseudocapsule. Anatomic resection is not required.

Items 8-10 Each lettered response may be selected once, more than once, or not at all. A. B. C. D. E.

Hepatic cystic disease is present in up to 5% of the population and includes congenital as well as acquired pathologies. Symptoms can include nausea, early satiety, and abdominal pain, or patients can be completely asymptomatic Congenital pathologies include simple liver cysts as well as variants of polycystic liver disease (PCLD). Simple cysts appear thin walled without any internal septations on CT scan or MRI. Treatment is via aspiration/sclerosis or surgical unroofing for symptomatic cysts.

Superior mesenteric artery syndrome Annular pancreas Pyloric stenosis Duodenal web Duodenal atresia

8. Narrowing of the aortomesenteric angle 9. Palpable "olive" in the right upper quadrant 10. Windsock deformity

ANSWERS:

PCLD has 3 variants based on the size and number of cysts present Type 1 has limited number of large cysts, type 2 has diffuse involvement of moderate-sized cysts with intervening normal parenchyma, and type 3 has diffuse involvement of moderate- and small-sized cysts without much normal parenchyma. CT scan and MRI demonstrate multiple cysts, usually more than 20, distributed in one of the 3 patterns. Treatment varies according to PCLD type: Type 1 disease can be treated with aspiration/sclerosis or unroofing, type 2 disease responds to surgical unroofing or resection, and type 3 disease typically requires transplantation after failure of medical management.

A, C, D

Although obstruction of the upper gastrointestinal tract is most commonly caused by a malignancy or complication of peptic ulcer disease, it can also result from congenital anomalies or acquired conditions. Both pediatric and general surgeons need to be familiar with the diagnosis and management of these less frequently occurring conditions that can cause obstruction of the gastric outlet or duodenum. Superior mesenteric artery (SMA) syndrome is an acquired condition typically seen after significant rapid weight loss or surgery for conditions such as scoliosis. The main anatomic feature is the narrowing of the angle between the aorta and the SMA from its normal 38 to 65° to less than 25°. This narrowing results in compression of the third part of the duodenum as it crosses between the aorta and the SMA. Affected patients typically present with nausea, vomiting, weight loss, and epigastric pain. The diagnosis can be made with fine slice CT imaging of the abdominal aorta and its branches, demonstrating an aorto-superior mesenteric artery angle of less than 25°. Initial treatment focuses on correction of electrolyte abnormalities and careful refeeding. Once the patient's nutritional status is restored, performance of a duodenojejunostomy can provide significant symptomatic relief and help restore a durable nutritional state.

Acquired cystic liver lesions include infectious and neoplastic processes. Hydatid cysts arise in the setting of parasitic infection from the cestode Echinococcus granulosis. Humans who eat contaminated food can act as accidental intermediary hosts in the lifecycle of the parasite. Hatched eggs travel through the vasculature to the liver where cysts develop with a germinal layer with a fluid-filled central cavity. Main cysts then produce daughter cysts with protoscoleces. Symptoms of hydatid disease include fever, malaise, and right upper quadrant pain. CT scan and MRI reveal thickwalled, calcified cysts with daughter cells in the periphery. Diagnosis is made via serologic testing. Treatment options include open surgical pericystectomy in complicated disease to cyst puncture, aspiration of contents, injection of sclerosing agents, and re-aspiration of contents (the PAIR technique). Antihelminthic drugs, such as albendazole or mebendazole, should be given in conjunction with either technique. Even though this patient has recently traveled to central and South America, the imaging findings are not consistent with echinococcal disease.

Annular pancreas is a congenital condition that results from the tethering of the ventral pancreatic bud to the duodenum, leading to a ring of pancreatic tissue encircling the duodenum. Annular pancreas is diagnosed with nearly equal frequency in children and adults. Children are often diagnosed on prenatal ultrasonography and typically present within the first 2 days of life with duodenal or gastric outlet obstruction. Symptoms in affected children include vomiting, bloating, and food intolerance Duodenal bypass is very effective at relieving the duodenal obstruction related to annular pancreas in children.

In addition to hydatid cysts, pyogenic liver and amebic abscesses are other infectious etiologies. Pyogenic abscesses are bacterial in origin, can have air-fluid levels on imaging, and should be treated with antibiotics and aspiration as needed. Entamoeba histolytica is the causative organism in amebic abscesses. It can be identified using serologic testing and is effectively treated with metronidazole.

The initial presentation of annular pancreas in adults is symptomatic upper abdominal pain. Adults with annular pancreas have a lower incidence of duodenal obstruction and a much higher incidence of complex pancreaticobiliary pathology. The diagnosis in adults is most commonly made by

4

endoscopic retrograde cholangiopancreatography and CT scan of the abdomen. Surgical treatment of annular pancreas in adults requires a broader array of potential options, including cholecystectomy, duodenal bypass, or pancreatic procedures.

11. A 55-year-old woman is admitted to the hospital with fevers and right upper quadrant pain. She recently moved to the area after living her entire life in India. Imaging is obtained (figure 11.1). Which of the following should the next step include? A. Percutaneous aspiration with Gram stain B. Indirect hemagglutination testing C. Needle biopsy D. Endoscopic retrograde cholangiopancreatography E. PET scan

Pyloric stenosis most commonly presents between 2 and 12 weeks of age. Affected infants often present with frequent, progressive, nonbilious vomiting, typically shortly after feeding. On inspection of the abdomen, peristaltic waves may be visible, progressing from the left upper quadrant across to the right. On palpation of the epigastrium, a palpable "olive" can often be detected, corresponding to the hypertrophied pylorus. The frequent vomiting leads to dehydration and electrolyte abnormalities, typically a hypokalemic, hypochloremic metabolic alkalosis. After correction of the volume and electrolyte abnormalities, surgical intervention is indicated. Extramucosal longitudinal pyloromyotomy reliably relieves the pyloric stenosis and gastric outlet obstruction. Both short-term and long-term outcomes are excellent after surgery for pyloric stenosis. A duodenal web occurs as a result of the incomplete recanalization of the duodenum during the eighth through tenth weeks of gestation. This results in the presence of a persistent membranous structure within the lumen of the duodenum that has the appearance of a windsock. The center of the web, or windsock, is typically an opening measuring 3 to 5 mm in diameter. Affected patients may present in childhood with abdominal distention and vomiting. Many patients, however, remain asymptomatic until adulthood when they present with abdominal pain in the epigastrium that is exacerbated by eating and relieved by vomiting. In both age groups, an upper gastrointestinal barium examination is diagnostic, demonstrating barium retention at the site of the web and mimicking the presence of a duodenal diverticulum. The preferred surgical option for the treatment of a duodenal web is duodenotomy with excision of the web. The complete failure of recanalization of the duodenum after the seventh week of gestation, either as a result of an ischemic insult or genetic factors, results in duodenal atresia.

Figure 11.1

ANSWER: B In the Western world, the most prevalent hepatic abscesses are bacterial (pyogenic), and mortality approaches 15%, due mostly to patient debilitation and persistence of the underlying disease. By contrast, in South-East Asia and Africa, amebic infections are the most frequent source. The diagnosis of pyogenic hepatic abscesses depends on imaging (ultrasound or CT scan), with confirmation by needle aspiration for bacteriology studies. Sonography and CT scans lead to diagnosis in more than 90% of cases and can often pinpoint the etiology. The sensitivity of triphasic enhanced multislice CT scan is superior to that of sonography. There is no role for PET scan in the diagnosis of hepatic abscesses.

The diagnosis is often made on prenatal ultrasound demonstrating polyhydramnios. Postnatal presentation is characterized by persistent bilious emesis, gastric distension, and poor feeding. The classic finding on a plain radiograph of the abdomen is a "double bubble" sign due to distension of the smaller proximal duodenum and larger stomach and the absence of gas in the bowel distal to the duodenum.

The World Health Organization estimates that Entamoeba histolytica is responsible for 50 million infections and 100,000 deaths per year worldwide. Although amebic hepatic abscesses are the most frequent extraintestinal complication of E. histolytica infection, it affects less than 10% of patients with the disease. Amebic hepatic abscesses caused by E. histolytica must always be considered when the patient has lived or traveled in endemic areas, because intestinal infection often passes unnoticed. Classic amebic hepatic abscesses present with hepatomegaly and fever. The diagnosis of E. histolytica depends on identification of specific antibodies in the serum via serologic tests: indirect hemagglutination, enzyme-linked immunosorbent assay, indirect immunofluorescence, and the latex agglutination technique. Combining 2 different techniques increases sensitivity and specificity to nearly

Due to the high potential for aspiration, contrast studies of the upper gastrointestinal tract should not be performed. A nasogastric or orogastric tube should be placed to decompress the stomach and proximal duodenum. Once the infant is clinically stabilized, a duodenoduodenostomy or duodenojejunostomy should be performed. With proper surgical correction, the long-term prognosis for duodenal atresia is quite good, provided that no other significant congenital anomalies are present.

5

100%. Serology allows confirmation of diagnosis, and needle aspiration should not be performed.

of guidelines had the goal of improving specificity for malignancy detection, while raising the threshold for surgery and shifting the paradigm toward a more nonoperative approach.

Metronidazole is the standard treatment Percutaneous drainage should be performed if the abscess is large (>10 cm diameter), subcapsular, in prerupture status, or superinfected or if there is resistance to medical treatment. There is no role for endoscopic retrograde cholangiopancreatography in the diagnosis or treatment of amoebic hepatic abscesses, because this can lead to cholangitis and clinical decompensation.

A recent study compared the Fukuoka to the AGA guidelines and found that they produced similar surgical outcomes— both have a fair positive predictive value for the diagnosis of advanced neoplasia, and both miss cases of invasive cancer and high-grade dysplasia. This is not surprising, because the guidelines are relatively similar. Both recommend surgery for worrisome cytology or a dilated pancreatic duct in the presence of a solid component in the cyst. The Fukuoka guidelines add jaundice to these "high-risk" criteria. The patient presented has none of these features, and therefore neither of the operations listed is indicated.

12. A 53-year-old woman presents with vague abdominal pain. A CT scan shows a 3.5-cm cyst in the head of the pancreas that contains some solid component with thickening in the wall (figure 12.1). There is no pancreatic duct dilation. The rest of the scan is unremarkable. The patient's abdomen is nontender without masses, and she is anicteric. What is the next step for this patient? A. B. C. D. E.

By contrast, both sets of guidelines list "worrisome" features for which further workup is indicated. These include dilated pancreatic duct without solid cyst component or a solid cyst component without duct dilation A cyst larger than 3 cm is also listed as worrisome in both guidelines, whereas the Fukuoka guidelines add a history of pancreatitis and a thickened enhancing cy wall Per the AGA recommendations, patients having 2 worrisome features, such as the patient . presented, should have further workup with endoscopic ultrasound with fine needle aspiration for cyst cytology. Core needle biopsy of the cyst wall is not necessary because the diagnostic goal is to sample the cyst contents. Per AGA guidelines, patients without worrisome features should undergo repeat imaging in 1 year, with MRI preferred over CT scan. Without a diagnosis, cyst enucleation or pancreatoduodenectomy is not appropriate

Core needle biopsy of the cyst wall CT scan in 1 year Fine needle aspiration cytology of the cyst Pancreaticoduodenectomy Cyst enucleation

13. A 48-year-old Asian woman presents with postprandial right upper quadrant pain and jaundice. Liver function tests are normal. An ultrasound study demonstrates multiple gallstones and a fusiform dilation of the extrahepatic biliary tree with normal intrahepatic ducts. Which diagnostic test would be the next most appropriate to clarify the biliary anatomy? A. CT scan B. Hepatobiliary iminodiacetic acid scan C. Magnetic resonance cholangiopancreatography D. Endoscopic retrograde cholangiopancreatography E. Duodenal aspirate for malignant cells

Figure 12.1.

ANSWER: C Pancreatic cysts are extremely common, with an estimated incidence as high as 41% of the general population. The Sendai guidelines were advanced in 2006 as a way to better determine which cysts might harbor neoplasia or dysplasia and should be respected. These guidelines proved to have a high detection rate for mucinous neoplasms but a low specificity for advanced neoplasia, resulting in too many resections. In 2012, the Sendai guidelines were revised into the Fukuoka guidelines. In 2015, the American Gastroenterological Association (AGA) published its guidelines for the management of asymptomatic pancreatic cysts. Admittedly the AGA guidelines were based on mostly low-level evidence but were thought to be needed given the serious outcome of a minority of pancreatic cysts and the need for clinical guidance of this complex problem. Both sets

ANSWER:

C

Cysts of the biliary tree, called choledochal cysts, are rare, occurring in fewer than 1/100,000 patients. They are more common in people of Asian descent and are 3 to 8 times more common in women than in men. The Todani classification system is based on cyst location and includes 5 types: type I (80-90% of all choledochal cysts), type II, type III, type IV (15-20% of all choledochal cysts), and type V or Caroli disease. Challenges to the Todani classification system are the fact that gallbladder diverticulum, choledochoceles, and Caroli disease are not related to other forms of choledochal cysts. Malignancy is most commonly associated with type I

6

and IV cysts, whereas type II, III, and V choledochal cysts have minimal neoplastic risk. Choledochal cysts present differently in adults and children. Children present with an abdominal mass and jaundice. Adults present with biliary or pancreatic symptoms and symptomatic cholelithiasis in 40 to 70% of patients. Malignancy occurs in 10 to 30% of adults with a choledochal cyst. Early surgical correction of choledochal cysts is recommended given the risk of malignancy.

Complex Hernias The fallowing factors define hernia as complex • BMI > 35 • Smoking • Diabetes mellitus • Steroid use • Congestive heart failure • Chronic obstructive pulmonary disease • Intraoperative enterotomy or bowel resection • Urgent procedure for incarceration or obstruction • Hernias that result from damage control laparotomies with sequential closing • Hernias with contaminated wounds from ostomies, fistulas, or prior surgical site infections • Recurrent hernias • Prolonged operative time

Most choledochal cysts are diagnosed by imaging because the common presenting symptoms are often nonspecific. With the current liberal use of ultrasound and CT imaging, endoscopic retrograde cholangiopancreatography (ERCP) is often not useful in the diagnosis of a suspected choledochal cyst, because filling of the dilated biliary tree is difficult to achieve to define the abnormal anatomy. Additionally, ERCP is more invasive than magnetic resonance cholangiopancreatography (MRCP). MRCP helps create a complete cholangiogram and has become the noninvasive procedure of choice for the diagnosis of choledochal cysts. As the quality of MRCP has improved, many surgeons now consider MRCP the only imaging technique needed for diagnosis and operative planning.

Figure 14.1. Complex hernias. factors for recurrence include tobacco use, hernias derived from trauma, malnutrition, and type 2 diabetes. The type of mesh used and the location of mesh placement are areas of controversy. Use of mesh as a bridge between fascial edges should be avoided due to its high recurrence rate. This will also not create a functional abdominal wall A retrorectus placement of mesh is favored, but onlay and underlay mesh placement can also be used. For larger hernias, retrorectus placement will require a fascial release such as a transverse abdominis rectus fascia or a component separation. This can be done using an open technique or laparoscopically.

Routine liver scans are not useful when trying to image the biliary tree. Duodenal aspirates are used to diagnose cholelithiasis when other diagnostic evaluations fail to identify a reason for a patient continued abdominal pain.

14. A 60-year-old man who underwent an urgent sigmoidectomy for diverticulitis 10 years ago now presents with an 8-cm ventral wall hernia. The sigmoidectomy was complicated by a wound infection. He has a BMI of 37 and hypertension controlled by one medication. He has never been a smoker. You are planning an open repair and are considering the use of mesh. Which of the following statements is true? A. A history of infection is a contraindication for mesh. B. A complex hernia requires mesh repair. C. Biologic mesh has a lower infection rate than permanent mesh. D. Cross-linked mesh is preferred. E. Non—cross-linked mesh will have the lowest recurrence rate.

ANSWER:

The use of biologic or permanent mesh depends on the contamination of the field. It is believed, but not proven, that a biologic mesh has risk of infection but a higher risk of recurrence. The permanent meshes have i.e. opposite concerns; but neither is ideal. The Ventral Hernia Working Group grad - helps determine the risk of infection and recurrence for hernia repairs. The grading scale includes hernia size. In general, surgeons place biologic mesh in fields where there is a high risk of contamination and place permanent mesh in low- to moderate-risk situations, in patients with a high risk due to patient factors (tobacco, obesity, type 2 diabetes), biologic mesh is often used. Cross-linking is an attribute used to describe biologic meshes. Cross-linking appears to increase durability, tensile strength, and resistance to bacterial contamination. There is no evidence, though, that cross-linking or non-cross-linking is superior to the other.

B

Ultimately, patient factors and hernia factors determine the likelihood of recurrence or complication. The complex hernia deserves mesh. The risk of infection be it from the hernia or from the patient, will determine if biologic or permanent mesh is to be used. Balancing the risk of infection versus recurrence will guide the surgeon toward permanent verus biologic mesh in this case.

The definition of a complex abdominal wall hernia is varied but depends both on hernia factors and patient factors (figure 14.1). The importance of the definition lies in the fact that complex hernias deserve mesh placement to help prevent recurrence. This patient has a complex hernia because of the previous wound infection, which increases the rate of primary hernia occurrence and recurrence after repair. History of infection does not preclude the use of mesh in this operation. This patient has several other risk factors for hernia repair failure, including a BMI greater than 35 and hypertension. Other risk

15. A 20-year-old diabetic woman with a BMI of 45 presents with episodic epigastric and right upper quadrant pain. The episodes of pain last 30 to 40 minutes.

7

She has visited the emergency department twice for evaluation. She denies nausea and vomiting. Her right upper quadrant ultrasound is normal, and her cholecystokininiminodiacetic acid ejection fraction is 35%. According to the Rome III criteria, which one of the following characteristics is necessary to establish the diagnosis of biliary dyskinesia? A. Young age B. Female sex C. Diabetes D. Obesity E. Episode duration

ANSWER:

B. C. D. E.

Hernia width Diabetes Obesity Chronic obstructive pulmonary disease

ANSWER:

B

When patients present for repair of ventral incisional hernias, they should be counseled with respect to expected outcomes. A specific topic for discussion is the risk of hernia recurrence. Patient comorbidities, the presence or absence of contamination, and hernia dimensions should all be considered. However, hernia width and contamination are the 2 most significant factors for recurrence. Hernia length, hernia location (midline versus lateral), and significant comorbidities (e.g., obesity, chronic obstructive pulmonary disease, diabetes, or smoking within 3 months of the operation) are not as important for predicting recurrence.

E

Rome III criteria define functional gallbladder and sphincter of Oddi disorders (table 15.1). Although the exact ejection fraction criterion may vary from institution to institution, 35 to 40% is common. To be defined as biliary dyskinesia, the patient must have episodes of duration lasting at least 30 minutes; episodes occurring at different intervals; and crescendo pain that is not relieved by bowel movements, postural change, or antacids. Strict adherence to the Rome III criteria may be valuable in determining indications for surgery with functional gallbladder disorders.

17. A 22-year-old, otherwise healthy woman presents with a 2-day history of vague abdominal discomfort and nausea. Within the past 12 hours, the discomfort has progressed to lower abdominal pain. Physical examination demonstrates localized suprapubic and right lower quadrant tenderness. Her white blood cell count is 17,000/mm3 (360011,200/mm3) with a left shift Complete blood count and serum chemistries are otherwise unremarkable. Urinalysis is normal Urine pregnancy test is negative. CT imaging is being considered. Which contrast should be given before scanning to optimize diagnostic accuracy, patient comfort, and efficiency? A. Oral B. Intravenous C. Oral and rectal D. Intravenous and oral E. Intravenous, oral, and rectal

Table 15.1. The Rome III criteria for functional gallbladder and sphincter of Oddi disorders. I. Functional gallbladder and sphincter of Oddi disorders*: 1. Episodes last 30 minutes or longer. 2. Recurrent symptoms occur at different intervals. 3. The pain builds up to a steady level. 4. The pain is severe enough to interrupt the patient’s daily activities or lead to an emergency room visit. 5. The pain is not relieved by bowel movements. 6. The pain is not relieved by postural change. 7. The pain is not relieved by antacids. 8. Exclusion of other structural diseases that would explain the symptoms.

ANSWER:

Supportive criteria: The pain may present with 1 or more of the following: 1. Associated with nausea and vomiting. 2. Radiates to the back and/or right infrascapular area. 3. Awakens patient from sleep in the middle of the night.

B

Acute appendicitis is a familiar disease for the general surgeon, and it is the most common indication for urgent intra-abdominal surgery. Over the course of the past 30 years, the inclusion of diagnostic imaging, particularly CT scans and ultrasound, into the assessment algorithm for acute appendicitis has reduced the frequency of negative appendectomies without increasing the frequency of appendiceal perforation. During these 30 years, CT imaging technology has evolved such that higher quality images allow for more precise interpretation. Recently, intravenous- only versus intravenous plus enteral (oral or rectal) contrast CT scans were compared for the assessment of acute appendicitis.

II. Functional gallbladder disorder (must include all of the following) 1. Criteria for functional gallbladder and sphincter of Oddi disorder. 2. Gallbladder is present. 3. Normal liver enzymes, conjugated bilirubin, and amylase/ lipase.

*Must include episodes of pain located in the epigastrium and right upper quadrant and all of the following. Table 15.1. The Rome III criteria for functional gallbladder and sphincter of Oddi disorder.

In patients being considered for appendectomy, enteral contrast does not improve accuracy of diagnosis compared with intravenous contrast alone. The elimination of enteral contrast may improve efficiency, patient comfort, and safety. Using only intravenous contrast may decrease time spent in the emergency department, which in turn translates into cost savings and improvement in the use of emergency medicine resources. Although the risks of enteral contrast are minimal,

16. A 60-year-old man with diabetes and chronic obstructive pulmonary disease is evaluated for a ventral incisional hernia repair. His BMI is 40. He has a midline hernia measuring 50 cm long and 25 cm wide. What factor poses the greatest risk for hernia recurrence after repair in this patient? A. Hernia length

8

they are not absent. Although unusual, intolerance or "allergy" to enteral contrast is possible, and there is always a risk of inadvertent administration of the wrong agent. Consumption of a large volume of oral contrast soon before induction of general anesthesia is not without risk of aspiration. Rectal catheterization for administration of rectal contrast can cause discomfort in patients who already have abdominal pain. Its use does not. improve the accuracy of the CT scan for appendicitis

A. B. C. D. E.

ANSWER B Meckel diverticulum is the most common congenital malformation of the gastrointestinal tract and is found in approximately 2% of the general population. Inversion of a Meckel diverticulum leaves the patient with the risk of malignancy, which occurs in up to 17%, as well as a lead point for an intussusception. Likewise, leaving it in situ leaves the patient with all of the attendant risks of hemorrhage, obstruction, intussusception, diverticulitis, and perforation. Simple diverticulectomy is an acceptable treatment if there is no inflammation or palpable abnormality. Given any inflammation, perforation, or palpable abnormality, segmental resection is the preferred treatment. Ileocecectomy is unnecessary.

18. A 54-year-old man presents with abdominal pain, elevated serum lipase, and a CT scan consistent with acute necrotizing pancreatitis. Which of the following statements regarding the management of necrotizing pancreatitis is true? A. Immediate surgery is an independent predictor of poor outcomes. B. Management via the open-abdomen approach is preferred. C. Anatomic resection and necrosectomy are associated with similar rates of postoperative exocrine and endocrine insufficiency. D. Percutaneous management cannot replace surgery. E. The step-up approach is associated with more delayrelated complications.

ANSWER:

suture inversion. segmental resection and anastomosis. to leave it in situ. diverticulectomy. ileocecectomy.

20. Which of the following statements is true regarding portsite recurrence in incidentally found gallbladder carcinoma? A. If an extraction bag is used, the chance of port-site recurrence is zero. B. Less than 1% of port-site metastases occur at a nonextraction port. C. If the gallbladder is removed intact without spillage, port-site recurrence is rare. D. Port-site metastases remain high relative to colon cancer. E. Prophylactic port-site excision is associated with decreased port-site metastasis.

A

The management of necrotizing pancreatitis has undergone a dramatic change over the last several decades. Management by open abdomen is now considered the last in a series of escalations within the "step-up approach." In many cases, percutaneous management has replaced surgery. Even when surgery is necessary, it can be limited to a retroperitoneal endoscopic approach. Anatomic resections, such as partial or total pancreatectomy, are associated with higher exocrine and endocrine insufficiency rates and mortality than necrosectomy. A minimally invasive step-up approach is associated with fewer major complications and deaths.

ANSWER:

D

Despite the use of extraction bags, port-site metastases still occur. In fact, approximately 50% of port-site metastases occur at nonextraction ports. Although it is intuitive that the gallbladder is best removed intact without spillage, there is no good evidence to support that this practice decreases portsite metastasis. Likewise, there is no good evidence that portsite excision decreases port-site metastases. Port-site metastasis in gallbladder cancer occurs in approximately 10% of cases. A randomized trial of open versus laparoscopic surgery for colon cancer revealed a less than 1% wound recurrence in both arms after 4.4 years.

19. The pictured lesion (figure 19.1) is discovered incidentally at laparotomy in a 35-year-old man after a motorcycle crash. There is a small palpable mass where the diverticulum interfaces with the bowel. The most acceptable management would be

21. Ten days after undergoing a Roux-en-Y gastric bypass, a 42-year-old man is unable to tolerate oral intake. He states that he has crampy abdominal pain and diarrhea within minutes of eating. The accompanying nausea and palpitations improve with lying down. He appears volume depleted, and his hemodynamics respond to intravenous fluids. An upper gastrointestinal study shows normal Roux-en-Y anatomy and no evidence of obstruction. Which of the following would you advise? A. Laparotomy B. Separation of liquids and solids at meals

Figure 19.1 9

C. Upper endoscopy D. 6 small meals a day E. Octreotide

ANSWER: B This patient is demonstrating classic symptoms of an early clumping syndrome. Having ruled out an early postoperative bowel obstruction, management of symptoms becomes the primary focus. Early dumping syndrome occurs within 1 hour after eating. Fluid shifts with release of gastrointestinal hormones are triggered by the rapid transit of food into the small intestine, resulting in diarrhea and vasomotor symptoms. This is distinct from late dumping, which occurs 1 to 3 hours after a carbohydrate load and causes a hyperinsulinemic response and subsequent hypoglycemia. Although both forms of dumping syndrome are frequent sequelae of gastric bypass, they are not believed to be the primary cause of bariatric weight loss.

Figure 22.1.

ANSWER: C This patient presents with a CT scan that shows a single cystic lesion of the liver (figure 22.2).

Dietary education and modification is usually a beneficial first step. Patients should be advised to reduce the amount of food consumed and delay fluid intake until at least 30 minutes after each meal. Rapidly absorbed carbohydrates should be eliminated and replaced with high-fiber, protein-rich foods. Fruits and vegetables are encouraged, and alcohol is discouraged. These patients should be advised to eat slowly and to lie down for 30 minutes after meals. Although not currently approved for the management of dumping, the addition of a somatostatin analogue is an effective treatment option for patients who have failed dietary modification. Octreotide effectively slows gastric emptying and improves hypoglycemia, thus affecting both early and late dumping symptoms. However, diarrhea, nausea, steatorrhea, and gallstone formation may limit its prolonged use.

Figure 22.2. Cystic lesions can be classified as follows: • Simple (solitary) cyst • Polycystic disease • Parasitic (echinococcal) cyst • Primary neoplastic - Mucinous cystic neoplasm (MCN) - Cystadenocarcinoma • Metastatic neoplasm (ovary, pancreas, colon, kidney) • Duct based (bile duct duplication, Caroli disease) • False cysts (trauma, infection, intrahepatic biloma)

22. A 44-year-old woman was involved in a minor motor vehicle collision. A CT scan of her abdomen was obtained as part of her initial assessment (figure 22.1). She has no abdominal symptoms, no history of jaundice, physical exam within normal limits, and transaminases and bilirubin within normal limits. The next step in the treatment of this patient should be A. B. C. D. E.

observation. repeat imaging in 1 year. multidisciplinary evaluation. percutaneous aspiration. hepatic lobectomy.

The key features on this CT scan are an isolated cystic lesion of the liver. The differential diagnosis for this type of cyst is a simple hepatic cyst, MCN, or metastatic neoplasm. Metastatic neoplasm uncommonly presents with an isolated cystic lesion. On CT Scan, the primary tumor can usually be identified if it involves the pancreas, ovary, or kidney. If colon cancer is the primary, further evaluation with colonoscopy is indicated. The main differential diagnosis is between a simple hepatic cyst and MCN. This is a difficult but important diagnosis to make because the treatment is unique for each diagnosis.

10

History and physical exam are not helpful because both can be asymptomatic or present with vague upper abdominal fullness and pain. Laboratory analysis is also not helpful because most present with normal transaminases and bilirubin. There may be an elevation of alkaline phosphatase. Ultrasound is good at identifying the lesion and subsequent follow-up but will not allow for differentiation of the 2 etiologies. On CT scan, simple cysts will have a thin wall and no internal septations. MCN can also present this way, but typically has a thicker wall and septations Treatment of a simple cyst involves unroofing of the cyst. This can be accomplished laparoscopically with minimal risk of recurrence and complication. MCN requires resection of the cyst in its entirety due to the risk of malignant transformation of the cyst wall.

Figure 23.1.

In this patient, the best course of evaluation is via a multidisciplinary team (MDT) composed of surgeons, hepatologists, oncologists, radiologists, and pathologists. An advantage of an MDT is that the team does not isolate one aspect of the disease process, but instead examines findings in relationship with the complete clinical context. Observation, repeat imaging, or hepatic lobectomy may be the outcome of MDT evaluation but would not be decided by one provider. Percutaneous aspiration is not used for simple cysts due to complications associated with its use; it is reserved for patients who cannot tolerate a surgical procedure.

ANSWER E Wound infections can be superficial or deep and can be associated with foreign material in the wound (figure 23.2). In patients with superficial infections without foreign material, the wound should be opened and treated either with dressing changes or packing. Negative pressure wound therapy would not be appropriate in the face of acute infection. Antibiotics are not indicated unless the patient has cellulitis or systemic signs such as fever or an elevated white blood cell count. Patients with deep wound infections without foreign material present are candidates for percutaneous drainage. Antibiotics are initiated based on culture results.

23. A 36-year-old woman underwent retrorectus prosthetic mesh repair of an incisional hernia 9 months ago. She presents with a small area of drainage with associated symptoms of fever and fatigue. On exam, her temperature is 38.2°C, and purulence can be expressed from the wound. This fluid is sent for culture. Her white blood cell count is 19,000/mm3 (3600-11,200/mm3). A CT scan is shown in figure 23.1. She is given a course of antibiotics with no change in her drainage. In addition to antibiotics, what is the most appropriate management of this patient? A. Open the wound and pack B. Open the wound and apply negative pressure wound therapy C. Place a percutaneous drain D. Explore the wound with removal of infected mesh and wound closure E. Explore the wound, debride the infected tissue, and remove all infected mesh

Figure 23.2. Wound infection in the face of foreign material is treated differently. The surgeon must eliminate not only the infected tissue but also the infected foreign material. Foreign material includes permanent suture or prosthetic mesh. After debriding a wound and mesh, the wound is not typically closed. Thus, the treatment for this patient with a mesh infection and systemic signs is to explore the wound, debride the infected tissue, and remove all infected mesh. Wellincorporated mesh often does not require removal.

11

24. After a routine chest x-ray, a 74-year-old woman is found to have a type III paraesophageal hernia (PEH). She has no history of abdominal pain or reflux symptoms. Which of the following statements is true regarding the management of her PEH? A. A watchful waiting approach can be taken. B. Mediastinal sac excision is unnecessary. C. A mesh repair is not associated with long-term complications. D. Fundoplication is not necessary when the hiatus is closed. E. Thoracic and laparoscopic approaches have similar complication rates.

ANSWER:

C

The approach to nutrition in the patient with pancreatitis has changed over the last few decades. The enteral route has become the preferred route. The concept of "resting" the pancreas has itself been laid to rest. Enteral nutrition is not only tolerated but also decreases systemic inflammation compared with parenteral nutrition. Several studies have compared gastric to jejunal feedings, and all concluded that gastric feedings did not exacerbate the disease process. No evidence supports the theory that parenteral nutrition shortens the course of pancreatitis compared with enteral feeding. No evidence supports enteral feeding being held for patients receiving neuromuscular blocking agents.

ANSWER: A Hiatal hernias are characterized as types I through IV. For type I, or sliding hernias, traditionally surgery is indicated only when reflux symptoms occur. Types II and III, generally characterized as paraesophageal hernias (PEH), have a component of the stomach located above the diaphragm. Type IV includes herniation of additional organs located above the diaphragm. Traditionally, surgery was indicated for asymptomatic PEH because of the risk of acute incarceration. However, a study of the national inpatient sample showed the annual probability of the need for emergency surgery was similar to the mortality risk for elective surgery, at about 1.5%. Further, the risk of mortality with emergency surgery was lower than expected, at just more than 5%. Therefore, today, watchful waiting in asymptomatic PEHs is appropriate. Controversy continues regarding the management when surgery is indicated. When comparing the laparoscopic approach to thoracic approach, the thoracic approach has an increased length of stay, higher risk of pulmonary embolism, and greater need for mechanical ventilation. Complete excision of the mediastinal hernia sac and fundoplication are associated with a lower risk of recurrence. Mesh repair is associated with a lower risk of recurrence but is also associated with long-term complications, including esophageal stenosis, mesh erosion, and infection. Therefore, current strategies include watchful waiting and surgery when symptoms dictate the need. Surgical strategy includes excision of the sac, primary closure of the crural defect, and fundoplication without routine use of mesh.

25. A 35-year-old woman presents with acute pancreatitis. She is hemodynamically normal. Which of the following statements regarding enteral nutritional support is true? A. Enteral nutrition is contraindicated. B. Enteral nutrition must be given distal to the ampulla of Vater. C. Enteral nutrition decreases systemic inflammation compared with parenteral nutrition. D. Enteral nutrition should be held in patients receiving neuromuscular blocking agents. E. Parenteral nutrition decreases the duration of pancreatitis compared with enteral nutrition

12

Abdomin Part II

ITEMS 1-25 For each question, select the best possible response

2. A 67-year-old man has a history of alcohol abuse and multiple bouts of acute pancreatitis. Over the course of the last year, he has developed chronic abdominal pain, which worsens with eating. He stopped drinking, but his symptoms have persisted. He denies diarrhea, and his fasting blood glucose is normal. A pancreatic protocol CT scan was performed (figure 2.1). What is the next most appropriate diagnostic step?

1. A 28-year-old woman was admitted to the hospital with a new diagnosis of idiopathic thrombocytopenic purpura. She was started on dexamethasone and intravenous immunoglobulin. Her platelet count did not respond, and she has now developed blood in her urine and stool. The next step in her treatment is? A. B. C. D. E.

A. B. C. D. E.

continue current therapy. plasmapheresis. eltrombopag. rituximab. splenectomy.

Magnetic resonance cholangiopancreatography Endoscopic ultrasound Endoscopic retrograde cholangiopancreatography Serum cancer antigen 19-9 Upper gastrointestinal study

ANSWER: C Thrombocytopenia is a condition characterized by either decreased platelet production, increased platelet consumption, or a combination of both. Platelet production is driven by the hormone thrombopoietin, whereas consumption or destruction can be multifactorial. Idiopathic thrombocytopenic purpura (ITP), also known as primary immune thrombocytopenia, is an acquired immunemediated condition. It is typified by an isolated thrombocytopenia in the absence of an underlying etiology. Treatment is usually initiated when the platelet count falls below a certain threshold or in the presence of bleeding. In patients with ITP, platelet destruction was initially believed to be the only etiology. It is now recognized that many patients also have a decreased level of thrombopoietin. First-line therapy for ITP consists of intravenous steroids, immunoglobulin, or both. With this treatment, 80 to 90% of patients will see a positive result. Unfortunately, only 1 in 5 will achieve a durable result. For the other 80%, second-line therapy will need to be initiated. Long-term use of steroids is associated with complications and thus would not be indicated.

Figure 2.1.

ANSWER: B

Second-line therapies include rituximab (a monoclonal antibody that targets CD20 on B-cells), thrombopoietin receptor agonists (romiplostim, eltrombopag), and splenectomy. Splenectomy has the risk of surgery as well as postsplenectomy sepsis. Although the results are durable, some patients find these risks to be too high. Rituximab was not studied in patients who have not had a splenectomy.

In this scenario, the patient has developed chronic pancreatitis. Chronic pancreatitis is manifested by pain, exocrine dysfunction, and endocrine dysfunction. The diagnosis of chronic pancreatitis is typically made clinically, by the presence of abdominal pain, diarrhea, steatorrhea, or diabetes mellitus.

Eltrombopag is an orally available, nonpeptide thrombopoietin receptor agonist studied in patients who have relapsed after first-line therapies. It is currently recommended in these patients in an attempt to bring platelet counts back to normal. After eltrombopag, patients can decide whether they would like to undergo splenectomy.

When a patient presents with chronic pancreatitis, the first step in management is lifestyle modification. This means modifying the diet to lower fat and abstaining from alcohol and tobacco. Various types of analgesia are also prescribed, including splanchnic nerve blocks.

Plasmapheresis is typically used in patients with antiphospholipid syndrome at risk for thrombosis. Because this is not the clinical scenario presented, it would not be considered a treatment option.

In addition to pain, chronic pancreatitis can be a harbinger of pancreatic neoplasm (figure 2.2). Therefore, patients should undergo CT scanning of the abdomen. If any abnormality is noted, the next step is endoscopic ultrasound.

14

Endoscopic ultrasound allows for the delineation of the pancreatic duct and biopsy of any abnormal mass identified. If no mass is seen on endoscopic ultrasound, endoscopic retrograde cholangiopancreatography can be undertaken. If a proximal stricture is identified, an attempt can be made to dilate it or to place a stent to determine whether there is symptomatic relief.

A variety of pathologic conditions can produce focal lesions within the spleen. J Urgent splenic conditions can occur due to trauma, as well as several nontraumatic conditions, including rupture associated with tumors or infectious causes, infarction, thrombosis, and torsion. B-mode with or without Doppler sonographic evaluation of the spleen can be used as a screening and cost-effective option to evaluate splenic pathology. Spontaneous rupture of the spleen can be caused by acute enlargement associated with infections, malignancy, vascular or hematologic diseases, or infiltrative processes. Ultrasound is a useful screening tool for hemoperitoneum, although it has less sensitivity (72-78%) for detecting splenic rupture A partially or completely obstructive thrombus within the main splenic vein or intrasplenic segmental branches can be detected with sonography. Splenic torsion is a rare condition and can be caused by congenital or acquired conditions that allow for splenic hypermobility or "wandering spleen." Ultrasound will confirm malposition of the spleen, usually with splenic engorgement and enlargement. The spleen will have heterogeneous echotexture with diminished or absent Doppler flow. Splenic abscesses are characterized on B-mode ultrasound as cystic or complex hypoechoic lesions, sometimes multifocal, some of which will contain gas or internal debris. Because the branches of the splenic artery are noncommunicating end arteries, their occlusion due to any etiology leads to segmental splenic infarction. On B-mode ultrasound, a region of infarcted spleen may be more difficult to interpret, appearing as an ill-defined wedge-shaped or round hypoechoic focus. Sulfur hexafluoride lipid-type A microspheres, approved by the US Food and Drug Administration in 2014 to opacify the left ventricle of adults with suboptimal echocardiograms, were approved in 2016 to characterize focal liver lesions in children and adults. Use has now extended to the spleen. With ultrasound contrast, areas of infarction do not enhance, are well defined, and can be used to better appreciate the wedge-shaped nature of the lesion.

Figure 2.2. Surgery is an option with chronic pancreatitis. Drainage-only procedures, such as Puestow or Frey, have the advantage of minimizing the risk of exocrine and endocrine insufficiency. Each of these procedures provides acceptable relief early, but results diminish with time. Resection with drainage may provide longer term relief. Finally, total pancreatectomy with or without islet cell auto transplantation is described for chronic pancreatitis. The indications for this operation are still not clear. Magnetic resonance cholangiopancreatography is not indicated in this patient because there is already known pathology of the ductal system. Serum cancer antigen 19-9 should not be used as a screening tool due to its low positive predictive value. An upper gastrointestinal study would provide no useful information for this patient.

4. A 45-year-old woman presents with 24 hours of right upper quadrant abdominal pain. She has no medical or surgical history. On examination, she is afebrile with anormal heart rate and blood pressure. She has mild tenderness to deep palpation in the right upper quadrant. Ultrasound demonstrates cholelithiasis with a common bile duct of 4 mm. The gallbladder wall is 2 mm thick, and there is no surrounding fluid. She has a negative sonographic Murphy sign Her labs include a white blood cell count of 10,500/mm3 (3600—11,200/mm3), aspartate aminotransferase of 80 IU/L (8-40 IU/L), alanine aminotransferase of 92 IU/L (7-55 IU/L), alkaline phosphatase of 125 IU/L (<95 IU/L), and a total bilirubin of 1.4 mg/dL (02.-1.9 mg/dL). Her amylase and lipase are normal. What is the best management strategy for this patient?

3. A patient presents with left upper quadrant pain. What pathologic process involving the spleen is poorly delineated on B-mode ultrasound? A. B. C. D. E.

Infarction Spontaneous rupture Venous thrombosis Torsion Abscess

ANSWER: A

A. Endoscopic retrograde cholangiopancreatography with sphincterotomy

15

B. Laparoscopic cholecystectomy C. Laparoscopic cholecystectomy with laparoscopic ultrasound D. Laparoscopic cholecystectomy with intraoperative cholangiogram E. Laparoscopic cholecystectomy with common bile duct exploration

2004,2005-2014), the clinical series demonstrated an increase in severity of injuries and the rate of associated vascular injury (3.6% vs 22.7%). The decision making associated with managing bile duct injury during laparoscopic cholecystectomy begins with early recognition of the injury and subsequent sepsis control. Sepsis control is a significant protective factor for future complications and anastomotic failure Recognizing the injury at the time of cholecystectomy can be a surrogate for expeditious sepsis control. Draining the injury is an approach to obtaining sepsis control before definitive repair of the injury. The recommended approach for repair of bile duct injury should involve hepatobiliarytrained surgeons with experience in managing this complication. Successful surgical treatment of the most severe injuries occurs

ANSWER: B This woman has signs and symptoms consistent with acute cholecystitis. Her ultrasound demonstrates cholelithiasis. Associated gallbladder wall thickening, pricholecystic fluid, and a sonographic Murphy sign are not required for a diagnosis of cholecystitis in the appropriate clinical setting. Furthermore, her history and laboratory profile suggest a less than 2% chance of choledocholithiasis. Mild elevations in liver transaminase levels can occur in the setting of acute cholecystitis, However, normal gamma-glutamyl transferase, alkaline phosphatase, and bilirubin levels essentially exclude choledocholithiasis. Consequently, both preoperative endoscopic retrograde cholangiopancreatography with sphincterotomy and common Bile duct exploration are unnecessary.

in 90% of cases when the treatment is performed at centers with the appropriate expertise. There is a role for endoscopic retrograde cholangiopancreatography in the least severe injuries. Interventions such as T-tube placement or fibrin glue do not have a role in primary repair. Many large series have explored the impact of intraoperative cholangiogram. on bile duct injuries during laparoscopic cholecystectomy. Challenges exist with many of these studies due to their retrospective design or the infrequent use of intraoperative cholangiogram. In a Swiss prospective registry, one-third of patients underwent intraoperative cholangiogram, and its use did not improve intraoperative recognition of bile duct injury. The role of intraoperative cholangiogram is further complicated by its interpretation by the surgeon. A published survey shows that more than 50% of surgeons were mistaken in interpreting a normal cholangiogram or an anatomical variant of the biliary tree.

Either intraoperative cholangiography or laparoscopic ultrasound is advocated as a low-risk endeavor that adds little time or expense to the procedure, enhances the understanding of intraoperative anatomy, and increases the probability of finding pathology when the pretest probability of choledocholithiasis is higher. Although the "training benefit" arguments may have merit, the literature does not strongly support the routine use of these adjuncts when the probability of choledocholithiasis is low. They are more appropriately applied in cases with an intermediate probability of choledocholithiasis (>3%). Consequently, this patient should undergo laparoscopic cholecystectomy with only expectant management of her common bile duct perioperatively.

6. A 71-year-old woman undergoes uneventful cholecystectomy. Permanent pathology shows a 0.5-cm fundic gallbladder cancer extending to the lamina propria. The next step in management should be A. B. C. D.

observation. adjuvant gemcitabine therapy. postoperative radiation therapy. resection of gallbladder bed (segment IV-V) with portal lymphadenectomy. E. right hepatectomy.

5. A 45-year-old obese woman presents for laparoscopic cholecystectomy for acute cholecystitis. After 2.5 hours of difficult dissection, the surgeon is clipping and dividing the cystic duct when the surgeon notes a transected structure with frank bile drainage. What is the most appropriate way for this surgeon to proceed?

ANSWER: A

A. Placement of a T-tube B. Intraoperative endoscopic retrograde cholangiopancreatography C. Laparoscopic primary bile duct repair D. Placement of fibrin glue over the transection E. Placement of a subhepatic drain

Gallbladder carcinoma is the most common malignancy of the biliary tract. It is the third most common gastrointestinal tract malignancy. Occurrence is significantly more common among women (71% vs 29%) and patients older than 40 years. Cholelithiasis is a primary etiological factor in gallbladder cancer. T1a gallbladder cancer extends only into the lamina propria; therefore, a simple cholecystectomy is sufficient for the treatment of these tumors. Cure rate after

ANSWER: E Findings from large population-based studies indicate an incidence of iatrogenic bile duct injury ranging from 0.2 to 1.5%. Looking at the pattern of injury and associated vascular injury rates over 2 consecutive time periods (1992-

simple cholecystectomy with negative margins ranges from 85 to 100%. Radical resection, including reresection of the hepatic bed (segments IVB and V) and portal nodal

16

lymphadenopathy, is recommended for T1b tumors detected incidentally postcholecystectomy. Radical resection is also recommended for T2 and T3 incidental gallbladder cancers. Extended hepatic resections are a potential approach when the tumor invades hepatic inflow vascular structures, particularly the right portal vein. Overall survival in the Surveillance, Epidemiology, and End Results database is 2.7 years, and surgical resection is associated with significantly improved survival compared with patients receiving no treatment or radiation alone.

effect, resulting in increased bleeding. Due to the location on the abdominal wall, typically below the semicircular line of Douglas, there is no reasonable way to apply direct pressure nor is exploratory laparotomy necessary.

8. A 64-year-old man with a history of pelvic radiation for rectal cancer presents with recurrent episodes of nausea, abdominal pain, abdominal bloating, and multiple loose bowel movements. Workup reveals no evidence of an infectious process, recurrent cancer, malabsorption, or obstruction. The next step in his management should be

Compared with surgical resection alone, surgical resection and adjuvant radiation is associated with a slightly longer survival. Only limited level I data support the use of adjuvant chemotherapy regimens. Recent clinical trials with gemcitabine-based therapies have shown improved efficacy over 5-fluorouracil regimens in patients with unresectable disease. These studies involve patients with more advanced disease —T2 or T3 versus early stage disease Tla.

A. B. C. D. E.

exploratory laparotomy. broad-spectrum antibiotics. loperamide. parenteral nutrition. fecal transplant.

ANSWER: C 7. A 71-year-old man, who takes warfarin for chronic atrial fibrillation, presents after he develops a tender left lower quadrant abdominal mass (20 x 18 cm) that remains present despite contraction of his rectus muscle. He is hypotensive with an international normalized ratio of 6.7 (0.8-1.2). Despite correction of coagulopathy and fluid resuscitation, he remains hypotensive, and a CT scan demonstrates a blush off the left inferior epigastric artery. What is the next step in his management? A. B. C. D. E.

Chronic radiation enteritis is recognized as a frequent and clinically important sequela of treatment of malignant disease. Diarrhea and crampy abdominal pain are the most common symptoms. The etiology of the diarrhea is poorly understood but may be related to bacterial overgrowth, changes in absorption (particularly of bile acids), or more rapid transit time. Loperamide improves bile acid absorption, slows transit time, and improves diarrheal symptoms. There are no consistent dietary guidelines. Parenteral nutrition is used only in severe cases of malnutrition due to poor absorption. Surgery in the setting of radiation enteritis should be avoided due to the frequency of dense adhesions and the risk for unintentional enterotomies and possible short gut. Data on the role of bacterial overgrowth in chronic radiation enteritis are inconsistent; hence, there is no role for fecal transplantation. Broad-spectrum antibiotics have not been proven to resolve the diarrhea.

Exploratory laparotomy Percutaneous drainage Angioembolization Abdominal wall pressure dressing Evacuation of the mass, vessel ligation, and closed drainage

ANSWER: C Rectus sheath hematoma arises from bleeding into the sheath of the rectus muscle due to trauma to the epigastric vessels or occasionally the rectus muscle itself.

9. A 59-year-old man presents with low-volume peritoneal carcinomatosis 3 years after undergoing curative resection without chemotherapy for stage II colon cancer (T3, NO, MO). After systemic chemotherapy, which of the following options results in the best survival?

Most frequently, these hematomas are localized inferior to the arcuate line, where the posterior rectus sheath is absent and the epigastric vessels are fixed and hence more vulnerable to shear forces. Although previously uncommon, spontaneous rectus sheath bleeding has increased due to the greater use of anticoagulant and antiplatelet medications. As a result, coughing, vomiting, and trivial trauma may cause these hematomas.

A. B. C. D. E.

In most cases, rectus sheath hematomas are self-limited and can be managed with observation alone. However, they may lead to hemorrhagic shock and cause mortality, particularly if associated with anticoagulation and antiplatelet therapy. In cases of ongoing bleeding and shock, angioembolization is the best treatment option. This method identifies the bleeding vessel that may be difficult if not impossible to see if the hematoma is opened directly. An open operative approach or percutaneous drainage may lead to release of the tamponade

Complete peritoneal cytoreduction surgery Hyperthermic intraperitoneal chemotherapy (HIPEC) C Tumor lysis with water lavage Cytoreduction surgery with HIPEC Palliative care

ANSWER: D Peritoneal spread of colorectal cancer was previously associated with a poor prognosis, with a median survival of 5 to 7 months. However, cytoreduction surgery with removal of as much tumor volume as possible, combined with hyperthermic intraperitoneal chemotherapy (HIPEC), demonstrates improved survival in selected patients compared with either surgery or HIPEC alone. Hyperthermia

17

itself is cytotoxic to cancer cells; when combined with chemotherapy, the effect is augmented. Chemotherapy is administered directly into the peritoneal cavity, allowing for higher concentrations to the tumor cells. Systemic toxicity is reduced due to the peritoneal—plasma partition. Although water osmotically mediates lytic effects that may be theoretically useful, the use of water alone is ineffective. Determining a patient's goals of care when confronted with peritoneal carcinomatosis is appropriate but in and of itself will not improve survival.

location is in the gastrinoma triangle bounded by the junction of the cystic duct and common duct, the second and third portion of the duodenum, and the neck and body of the pancreas. Octreotide scanning and endoscopic ultrasound combine for a diagnostic accuracy and localization success rate greater than 90%. MEN1 syndrome must be ruled out. Surgical resection is the treatment of choice. Somatostatinomas secrete somatostatin, which inhibits pancreatic and biliary secretions. Patients present with gallstones, diabetes, and steatorrhea. Diagnosis confirmed with a serum somatostatin level greater than 10 ng/mL Most are metastatic at the time of diagnosis, although resection and cholecystectomy are appropriate in many patients.

Items 10-12 Each lettered response may be selected once, more than once, or not at all.

13. A 65-year-old woman presents with painless jaundice. On CT scan, she is found to have a mass in the head of the pancreas consistent with pancreatic cancer (figure 13.1). Metastatic workup reveals no metastatic disease, and her CA19-9 is 100 U/mL (0-37 U/mL) after resolution of the jaundice with a biliary stent. Arterial anatomy includes a replaced right hepatic artery with tumor surrounding the artery and the tumor abutting less than 180° of the superior mesenteric artery. This tumor is categorized as

A. Insulinoma B. Gastrinoma C. Glucagonoma D. VIPoma E. Somatostatinoma 10. Watery diarrhea, hypokalemia, achlorhydria 11. Diabetes, necrolytic migratory erythema, deep venous thrombosis

A. B. C. D. E.

12. Hypoglycemia

ANSWER: D, C, A

resectable. borderline resectable. locally advanced. unresectable. regionally advanced.

Insulinoma is one of the most common pancreatic endocrine neoplasms. The typical presentation is known as the Whipple triad: symptomatic hypoglycemia when fasting, serum glucose less than 50 mL/dL, and relief of symptoms after glucose administration. Laboratory studies reveal a low blood glucose and elevated serum insulin level C-peptide levels should also be elevated to rule out surreptitious administration of insulin. Insulinomas are localized with CT imaging and endoscopic ultrasound. Most insulinomas are benign and amendable to simple enucleation. Glucagonoma causes a syndrome consisting of diabetes associated with necrolytic migratory erythema. Patients are at an increased risk for deep venous thrombosis. Diagnosis can be confirmed with a serum glucagon level greater than 500 pg/mL. Glucagonomas are primarily located in the head and tail of the pancreas and tend to be large with metastases at the time of diagnosis. Surgical removal with debulking if necessary is the treatment of choice.

Figure 13.1.

ANSWER: B

The vasoactive intestinal peptide-secreting tumor (VIPoma) causes a constellation of symptoms of watery diarrhea, hypokalemia, and achlorhydria (WDHA syndrome). Symptoms are often episodic, although the massive nature of the diarrhea (5 L/day) can cause electrolyte abnormalities such as hypokalemia. Most are located in the tail of the pancreas and have spread at the time of diagnosis.

Advances in the staging of pancreatic cancer have included a better understanding of the anatomical relationship of the tumor to major vascular structures near the pancreas. A subset of patients with advanced tumors benefits from vascular resection. These tumors are categorized as borderline resectable. Although classification systems have minor differences, conceptually, tumors that encase aberrant anatomy, like in this case, are considered borderline resectable. Patients with borderline resectable tumors are often treated with neoadjuvant therapy or entered into clinical

Gastrinoma causes the Zollinger-Ellison syndrome, characterized by excess gastrin production leading to acid hypersecretion and peptic ulceration. Diagnosis is made by a gastrin level greater than 1000 pg/mL. The most common

18

trials (see reference 2, table 1). The distinction between borderline resectable and locally advanced pancreatic cancer is determined by the degree of tumor—artery interface (abutment is defined as <180° and encasement is >180°). Borderline resectable pancreatic cancer is defined ss tumor abutment of less than 180° of the superior mesenteric artery or celiac axis. because of the clinical observation that induction therapy may sterilize at least the periphery of the tumor, thereby facilitating a complete resection.

laparoscopic liver resection (19% vs 31%) without increasing operating room time (123 minutes vs 120 minutes).

15. A 50-year-old healthy woman presents with 4 days of right upper quadrant pain associated with nausea and vomiting. Her white blood cell count is 15,000/mm3 (360011,200/mm3), and her liver and pancreatic function tests are normal. The right upper quadrant ultrasound reveals gallbladder wall thickening, stones, and pericholecystic fluid with a positive Murphy sign. What is most appropriate management of this patient in addition to starting intravenous antibiotics?

14. During a laparoscopic right hemicolectomy for colon cancer, a patient is found to have a solitary metastasis in segment II of the liver. Which of the following statements is true about the management of solitary colorectal liver metastases?

A. B. C. D. E.

A. 5-year survival is less than 5%. B. Surgical resection should not be considered for metastatic disease. C. Adjuvant systemic chemotherapy after liver resection significantly improves survival. D. Laparoscopic liver resection has fewer postoperative complications than open resection. E. Perioperative FOLFOX chemotherapy improves overall survival compared with surgery alone.

Delayed laparoscopic cholecystectomy Endoscopic retrograde cholangiopancreatography Placement of a cholecystostomy tube Immediate laparoscopic cholecystectomy E Lithotripsy

ANSWER: D The definitive management of cholecystitis is cholecystectomy, typically performed laparoscopically. However, controversy remains regarding the timing of a laparoscopic cholecystectomy. The concern lies in the fact that a longer duration of symptoms usually predicates more inflammation, with a higher incidence of suppurative or necrotizing cholecystitis. In addition, delayed surgical resection was previously associated with a higher conversion rate A randomized clinical trial of 86 patients found no difference in surgical complications after an early laparoscopic cholecystectomy compared with a delayed cholecystectomy that included a period of antibiotics. The complication rate was 15% versus 17% in the delayed group, with one bile duct injury in the delayed group. Additionally, 22% of patients were readmitted during the period of treatment with antibiotics, and 3 patients (7%) failed initial antibiotic therapy. These results suggest the safety and appropriateness of an early laparoscopic cholecystectomy even in patients presenting more than 72 hours after onset of illness. Systematic reviews also uphold these findings.

ANSWER: D A significant advance in the management of liver metastases from colon cancer is liver resection. The 5-year survival rate for all patients undergoing liver resection for liver metastases is 48 to 51%, with a cure rate of approximately 20%. Thus, liver resection must be considered for all patients with liver metastases. Systemic chemotherapy is the standard treatment for patients with metastatic colorectal cancer. The effects of systemic chemotherapy in the perioperative setting were investigated by the EORTC 40983 trial, which randomized patients to receive FOLFOX chemotherapy before and after surgery. In this trial of 364 patients, there was no difference in long-term survival between the groups of patients that received perioperative chemotherapy and those that had surgical resection, although there was an improvement in the progression-free survival Thus, although perioperative chemotherapy is still used in the management of resectable colorectal liver metastases, it may not alter overall survival for these patients. In addition, chemotherapy-associated steatohepatitis and sinusoidal dilation (oxaliplatin) can increase perioperative morbidity after surgical resection. Thus, it is important for the surgeon to be involved in decision making for these patients early. By contrast with perioperative chemotherapy, adjuvant (after resection) chemotherapy has not conclusively demonstrated an improvement in survival.

Percutaneous cholecystostomy tubes are effective methods for managing acute cholecystitis, especially in patients who are too sick to be taken to the operating room. There is no reason to defer an operation in this otherwise healthy patient. Because the liver and pancreatic function tests are normal, an endoscopic retrograde cholangiopancreatography would be an unnecessary intervention. Lithotripsy uses high-frequency waves that lead to dissolution of stones. Used previously via transcutaneous approaches, lithotripsy was not effective in managing this disease. Intraductal lithotripsy is being investigated via intraductal endoscopy, but risks such as bile duct perforation, cholangitis, and bleeding occur.

Laparoscopic hepatic resection was described more than a decade ago, and improvement in technology allows for safe and feasible liver resections. A randomized trial that examined the role of laparoscopic versus open liver resection found that surgical complications were reduced after

19

16. Which of the following statements best characterizes the role of a CT scan in the diagnosis of acute cholecystitis?

organ rupture and is one of the primary ways that the spleen becomes infected through intraperitoneal spread from a ruptured hepatic hydatid cyst. Mononucleosis is the most common infectious cause of atraumatic splenic rupture.

A. It is the first-line study in a patient with right upper quadrant pain. B. Gallbladder wall thickness of 1 cm is diagnostic for malignancy. C. It identifies complications of acute cholecystitis. D. It reliably demonstrates the presence of cholelithiasis. E. E It is superior to hepatobiliary scanning for assessing gallbladder obstruction.

The splenic artery is the third most common site of intraabdominal aneurysms; these are generally found in the distal aspect of the artery. The risk of aneurysmal rupture is approximately 2 to 3% and is increased in patients with portal hypertension, after liver transplantation, or during pregnancy. Patients with splenic infarction are often asymptomatic and diagnosed incidentally. Complications include acute febrile illness, abscess formation, splenic pseudocyst, splenic rupture, and hemorrhage. Impending splenic rupture should be considered when follow-up scans show progressive liquefaction or expansion of known infarcted splenic tissue. Splenic abscesses causing atraumatic splenic rupture are more commonly seen in patients with infarcts due to thromboembolic disorders. Malignant hematologic disorders, including non-Hodgkin lymphoma and leukemia, are the most common cause of atraumatic splenic rupture.

ANSWER: C Cross-sectional CT imaging has neither replaced ultrasound as the first-line study in patients with right-upper quadrant pain nor has it become superior to hepatobiliary scanning in the determination of gallbladder obstruction (i.e., cholecystitis). It is often obtained in the workup of more generalized abdominal complaints. Thus; surgeons are often faced with interpreting CT scan findings in patients with possible cholecystitis. Because gallstones are often radiolucent, they may not be seen on CT scan. Secondary signs of cholecystitis, such as gallbladder wall thickening or pericholecystic fluid, are readily seen on CT scan, as are complications of cholecystitis, such as gallbladder perforation, emphysematous cholecystitis, or an adjacent liver abscess. The most common and reliable CT scam finding in patients with gallbladder cancer is a mass replacing or obscuring the gallbladder, not gallbladder wall thickening.

Items 18-20 Each lettered response may be selected once, more than once, or not at all. A. B. C. D. E.

17. A 30-year-old woman presents with left upper quadrant pain. She denies any injury. Her ultrasound shows a splenic rupture. What is the most common cause of atraumatic splenic rupture? A. B. C. D. E.

Repeat colonoscopy in 6 months Repeat colonoscopy in 1 year Repeat colonoscopy in 3 years Repeat colonoscopy in 5 to 10 years Transanal mucosal resection

18. A 50-year-old man without family history of colorectal cancer after his complete, high-quality, index, asymptomatic screening colonoscopy: 5-mm hyperplastic polyp at 20 cm, 9mm tubular adenoma from sigmoid colon at 40 cm, and a 4mm tubular adenoma without dysplasia in the rectum

Splenic hydatid disease Drug induced Malignant hematologic disorders Splenic aneurysm Splenic infarction

19. A 50-year-old man without family history of colorectal cancer after his complete, high-quality, index, asymptomatic screening colonoscopy: 9-mm hyperplastic polyp at 20 cm, 20-mm tubular adenoma removed piecemeal from the sigmoid colon at 40 cm, and a 7-mm tubular adenoma in the rectum

ANSWER: C Spontaneous atraumatic splenic rupture is an infrequent but potentially life threatening condition that occurs mainly in histologically abnormal spleens. A systematic review described 6 major etiologic groups: neoplastic (30.3%), infectious (27.3%), inflammatory or noninfectious (20%), drug- and treatment-related (9.2%), mechanical disorders (6.8%), and normal spleen (6.4%). Most patients were treated with splenectomy (84.1%) but a minority were managed with successful splenic salvage (1.2%) or nonoperative treatment (14.7%). Splenomegaly, age over 40 years, and neoplastic disorders were associated with increased mortality on multivariable analysis.

20. A 50-year-old man without family history of colorectal cancer after his complete, high-quality, index, asymptomatic screening colonoscopy: 12-mm adenoma with high-grade dysplasia completely removed by cautery snare polypectomy from the distal rectum

ANSWER:

D, A, C

Guidelines for postcolonoscopy surveillance are applied to an asymptomatic, average-risk patient who has undergone a complete, high-quality exam. Patients are then stratified into low risk (any number of hyperplastic polyps, 1 or 2 subcentimeter adenomatous polyps) and high risk (3 or more adenomatous polyps, adenomatous polyp >1 cm,

Splenic involvement in hydatid disease ranges from 0.9 to 8% and is the third most common site, after liver and lung. Isolated involvement of the spleen is uncommon. Cyst rupture in hydatid disease seems to be more common than

20

adenomatous polyp with high-grade dysplasia, villous adenoma, and serrated polyps). High-risk individuals are recommended to have repeat colonoscopy in 3 to 5 years, depending on specific pathology and guidelines. Low-risk individuals are screened every 10 years. Patients who have had an adenoma larger than 1 cm removed in a piecemeal fashion should have a repeat colonoscopy within 6 months to ensure complete removal of the adenoma and to minimize risk of development of colorectal cancer.

Each lettered response may be selected once, more than once, or not at all. A. Hepatic resection B. Hepatic transplantation evaluation C. C Transarterial chemoembolization D. Percutaneous image-guided tumor ablation E. Best supportive or palliative therapy 22. A healthy 56-year-old man with chronic asymptomatic hepatitis C infection, serum bilirubin 1.5 mg/dL (0.2-1.9 mg/dL), and a solitary 7-cm hepatocellular carcinoma of segment HI of the liver found on surveillance ultrasound.

21. Which of the following statements regarding patients with ulcerative colitis and colorectal cancer is true?

23. A 75-year-old man with mild chronic kidney insufficiency and stable angina pectoris, normal liver function, and a biopsy-proven hepatocellular carcinoma of segment V of the liver. Dynamic contrasted MRI of the liver shows the main lesion is 2.7 cm, and there are 2 small satellite lesions near the main tumor.

A. Colorectal cancer is the cause of death in up to 5%. B. Forty percent of patients will have colorectal cancer after 30 years. C. Rates of colorectal cancer are lower than in patients with Crohn disease. D. Outcomes are better compared with all other patients with colorectal cancer. E. Endoscopic screening for colorectal cancer should begin within 8 years ' of diagnosis.

24. A 55-year-old woman with alcoholism, recently abstinent for 3 months with a multifocal hepatoma of segments IV, VI, and VII of the liver. Serum alpha fetoprotein level is 650 mg/dL (0-40 mg/dL), and serum bilirubin is 3.5 mg/dL (0.21.9 mg/dL). The main tumor is 5.5 cm in size, and 2 satellite lesions are 1.5 cm and 2.5 cm in size.

ANSWER: E Inflammatory bowel disease is strongly associated with the development of colorectal cancer. Patients with ulcerative colitis have an overall colorectal cancer rate of 5%. At 20 years, 8% of patients will be diagnosed with colorectal cancer. At 30 years after diagnosis, nearly 20% of ulcerative colitis patients will develop colorectal cancer. Colorectal cancer is the cause of death in 10 to 15% of patients with ulcerative colitis. Factors that appear to influence the rates of colorectal cancer development include extent (pancolitis > left sided > proctitis), severity of inflammation of colitis, and duration of disease. Family history of colorectal cancer and coexistent primary sclerosing cholangitis also increase risks for colorectal cancer nearly 50%. Crohn disease also predisposes patients to the development of colorectal cancer; only 6% of patients with Crohn disease develop colorectal cancer at years. The use of biologic agents, earlier surgical intervention, and more intense screening appear to reduce colorectal cancer rates.

25. A 45-year-old reformed alcoholic with good performance status and Child class B cirrhosis with a solitary 3.8-cm hepatocellular carcinoma of segment VIII of the liver. Contrasted CT scan shows splenomegaly

ANSWER: A, D, C, B The algorithmic approaches to the management of hepatocellular carcinoma (HCC) are outlined by 2 widely adopted classification systems. The Barcelona Clinic Liver Cancer system (BCLC) is the most commonly used system in the West and has applicability in US and European populations where alcoholic cirrhosis and hepatitis C predominates as a cause of liver disease (figure 22-25.1). The similar, but more aggressive, Hong Kong Liver Cancer classification (HKLC) is more applicable to Eastern populations where Hepatitis B predominates (figure 22-25.2).

Current American College of Gastroenterology guidelines recommend index screening within 8 years of diagnosis and screening every 1 to 3 years thereafter, with more than 33 random biopsies. Chromoendoscopy is better for determining extent of inflammation and increases the sensitivity over traditional white-light endoscopy. The presence of inflammatory bowel disease itself does not negatively affect colorectal cancer prognosis or survival, because they have similar 5-year survival rates of 50% compared with patients with colorectal cancer alone.

Items 22-25 21

Figure 22-25.1. Barcelona Clinic Liver Cancer (BCLC) staging and treatment strategy. The BCLC system establishes a prognosis in accordance with the five stages that are linked to first-line treatment recommendation. The expected outcome is expressed as median survival of each tumor stage according to the available scientific evidence. Note that liver function should be evaluated beyond the conventional Child-Pugh classification or the Model of End-stage Liver Disease (MELD) score. None of them serves to properly gauge the liver function status, and this evaluation should take into account biochemistry parameters as well as the compensated or decompensated status of the patient. Preserved liver function includes a group of patients with different degrees of liver function reserve that has to be carefully evaluated. For most treatment options, compensated liver disease (Child-Pugh stage A without ascites) is required to obtain optimal outcomes. The sole option that could be applied irrespective of liver function is liver transplantation. ECOG PS = Eastern Cooperative Oncology Group Performance Status. *Patients with end-stage cirrhosis due to heavily impaired liver function (Child-Pugh stage C or earlier stages with predictors of poor prognosis or high a MELD score) should be considered for liver transplantation. In these patients, hepatocellular carcinoma might become a contraindication if it exceeds enlistment criteria. Currently, sorafenib followed by regorafenib has been shown to be effective. Lenvatinib has been shown to be non-inferior to sorafenib, but no second-line option after lenvatinib has been explored. Both systems were validated in respective population studies. The use of surgery in early stage disease with preserved liver function is similar in both systems, while the use of other liver-directed therapies in more advanced disease or in individuals with poor performance status predominates in the BCLC system. The HKLC system is more aggressive in prescribing surgical resection or transplantation in the predominantly hepatitis B populations, where outcomes appear to be better compared with other causes of liver failure.

The 75-year-old man with mild chronic kidney insufficiency has stage A disease by imaging and preserved liver function but has advanced age and co-morbidities that would likely preclude orthotopic liver transplantation. Liver-directed therapies with curative intent such as ablation with microwave, radiofrequency, or ethanol would be indicated. The 55-year-old woman with alcoholism is considered a poor candidate for surgical resection or orthotopic liver transplantation despite a good performance status. She has a markedly elevated alpha fetoprotein, recent active alcohol abuse, and an intermediate stage B tumor. Transarterial chemoembolization as a palliative treatment would be indicated.

Applying the BCLC criteria to these patient descriptions, the healthy 56-year-old man with chronic asymptomatic hepatitis C infection has good performance status, preserved liver function and a solitary tumor and would be considered an early stage hepatoma or stage A HCC. Treatment with curative intent should be considered. Liver resection in this case is considered unless portal hypertension is present, then evaluation for orthotopic liver transplantation would be prescribed.

The 45-year-old reformed alcoholic with intermediate stage B HCC would be considered a good candidate for orthotopic liver transplantation, with a solitary tumor, elevated portal pressures (splenomegaly) and good performance status.

22

Figure 22—25.2. Hong Kong Liver Cancer classification prognostic classification scheme. EVM, extrahepatic vascular invasion/metastasis; LT, liver transplantation; TACE, transarterial chemoembolization; ECOG, Eastern Cooperative Oncology Group. Early tumor: <5 cm, <3 tumor nodules and no intra hepatic venous invasion; Intermediate tumor: (1) <5 cm, either >3 tumor nodules or with intrahepatic venous invasion, or (2) >5 cm, <3 tumor nodules and no intrahepatic venous invasion; and Locally advanced tumor: (1) ,<5 cm, >3 tumor nodules and with intrahepatic venous invasion, or (2) >5 cm, >3 tumor nodules or/and with intrahepatic venous invasion, or (3) diffuse tumor.

23

Abdomin Part III

there is no progression of disease, the patient can undergo pancreaticoduodenectomy with venous reconstruction if there is occlusion of the SMV/PV confluence. Standard pancreaticoduodenectomy or a pancreaticoduodenectomy with vascular reconstruction as the first step in his care is not appropriate due to his borderline resectable tumor. The patient has no evidence of biliary or duodenal obstruction, so biliary drainage or palliative bypass is not indicated.

ITEMS 1-25 For each question, select the best possible response. 1. A 65-year-old man with painless jaundice undergoes a triple-phase pancreas protocol CT scan. An ill-defined hypodense mass is seen in the pancreas phase in the pancreatic head. The mass extends to the superior mesenteric artery with less than 180° abutment and has contact along the superior mesenteric vein confluence with an irregularity of the vein. He is otherwise healthy. What is the next step in his management?

2. The risk of overwhelming postsplenectomy sepsis after splenectomy in childhood is highest when the spleen is removed for

A. Neoadjuvant chemoradiation therapy and restaging CT scan B. Pancreaticoduodenectomy C. Palliative biliary and enteric bypass D. Percutaneous transhepatic biliary drainage E. E Pancreaticoduodenectomy with en bloc resection of superior mesenteric artery and superior mesenteric vein with reconstruction

A. B. C. D. E.

trauma. benign hematologic disorder. abscess. malignancy. E iatrogenic injury.

ANSWER: ANSWER:

A

D

The primary risk of splenectomy is overwhelming postsplenectomy sepsis (OPSI); the risk is lifelong and greater due to encapsulated organisms. This risk of infection is increased in individuals who are immunosuppressed or immunocompromised or in those who are not properly immunized.

Surgical resection of pancreatic ductal adenocarcinoma offers the only chance of a curative outcome. However, only 10 to 20% of patients will present with a surgically resectable mass. Locally advanced tumors are defined as those with encasement (>180° contact) of the celiac axis, common hepatic artery, or superior mesenteric artery (SMA) or occlusion of the superior mesenteric vein (SMV) and portal vein (PV) confluence.

A comprehensive review of the benefits and risks of splenectomy for all ages described the specific disease processes as indications for splenectomy, including trauma, benign hematologic conditions (i.e., red cell disorders, thalassemia syndromes, homozygous sickle cell disease, platelet disorders), lymphoproliferative disease, myelofibrosis, rupture due to infections, and others. Regardless of indication, the adjusted short- and long-term risk of death after splenectomy is higher than in the general population. The best prognosis is associated with removal after trauma. The underlying condition and age at removal may influence the risk of OPSI, although the relative risk has decreased as prophylactic measures to prevent OPSI have improved. However, other complications of asplenia are surfacing, including the hypercoagulable state with associated venous thrombosis and pulmonary hypertension.

With vascular reconstruction, some tumors initially thought to be unresectable are now called borderline resectable. Borderline resectable tumors have no involvement of the celiac axis or hepatic artery bifurcation but do contact (1) the common hepatic artery, (2) less than 180° of the SMA, (3) greater than 180° of SMV or PV, or (4) Jess than 180° of the SMV/PV confluence with a vein irregularity or thrombosis with the potential for venous reconstruction. Patients with borderline resectable disease are at higher risk of having a positive surgical margin if they undergo initial resection. Neoadjuvant chemotherapy regimens have several advantages to standard adjuvant chemotherapy. Sequencing the treatment algorithm to deliver chemotherapy and radiation first ensures that a high proportion of patients receive chemoradiation therapy. Less than 50% of patients will undergo adjuvant therapy after pancreaticoduodenectomy. Additionally, delivering chemotherapy first can identify patients with aggressive tumor biology who will progress to metastatic disease and not benefit from an aggressive resection.

A true population-based incidence of postsplenectomy sepsis in children was performed at the Mayo Clinic using the Rochester Epidemiology Project, a population database that contains the medical records of 445,000 residents of Olmsted County, Minnesota. From 1966 to 2011,96 children underwent splenectomy (mean age = 12 years) for a wide range of indications, with trauma predominating, but including benign hematologic diseases, malignancy, infarction, abscess, before kidney transplant, and iatrogenic injury. Vaccination rate was 73% (38% complete, 35% partial). In the approximately 14-year follow-up period, 22 patients developed 62 postsplenectomy sepsis events, with

This patient with borderline resectable disease should undergo neoadjuvant chemoradiation therapy followed by restaging CT scan and measurement of CA19-9 levels. If

25

each patient's first event occurring approximately 3.5 years after splenectomy. Five deaths occurred. The cumulative probability of postsplenectomy sepsis 5 years after splenectomy was 18.3%, with an overall incidence of 1.7 per 100 person-years. Patients with underlying malignancy had a more than a 4-fold increased risk of postsplenectomy sepsis compared with trauma patients.

recurrence compared with underlay and onlay repairs. Sublay mesh also has a lower complication rate compared with the other techniques. Biologic mesh is independently associated with a higher rate of recurrence and would not be used unless there was a contraindication to synthetic mesh (i.e., a contaminated wound).

3. During the initial open repair of an 8-cm ventral hernia, the lowest risk of recurrence is with

4. A 43-year-old woman has 3 episodes of crampy right upper quadrant abdominal pain. An ultrasound reveals a pedunculated 7-mm polyp in the gallbladder neck and no other abnormalities. Her labs are normal. The next step in management is

A. B. C. D. E.

A. B. C. D. E.

primary repair with nonabsorbable suture. sublay mesh repair with synthetic mesh. sublay mesh repair with biologic mesh. underlay mesh repair with synthetic mesh. onlay mesh repair with synthetic mesh.

ANSWER:

B

cholestyramine. triphasic CT scan of the liver. serial ultrasound every 6 months. laparoscopic cholecystectomy. cholecystectomy with en bloc resection of adjacent liver.

ANSWER:

Ventral hernias continue to be a vexing problem for the general surgeon. The recurrence rate escalates with each attempt at re-repair, so the initial attempt at repair is the most important. For hernias larger than 4 cm, the use of mesh significantly reduces the risk of hernia recurrence. Thus, primary repair alone for an 8-cm hernia would not be indicated. Once a decision is made to use mesh, the remaining questions are mesh type (synthetic or biologic) and mesh location (onlay, inlay, sublay, underlay, figure 3.1).

D

Gallbladder polyps occur in approximately 5% of all ultrasound examinations. The concern with gallbladder polyps is the risk of malignancy, which is reported to be anywhere from 0 to 27%, leading to uncertainty in management. Polyp size is an important risk factor for malignancy. In general, cholecystectomy is recommended for polyps larger than 10 mm, because this size seems to correlate with an increased risk of malignancy. Other risk factors for malignant polyps include age older than 50, sessile morphology, and single polyp. Thus, laparoscopic cholecystectomy is recommended for any polyp larger than 10 mm and smaller polyps with risk factors (e.g., increase in size on serial ultrasound examinations, sessile morphology, or symptoms). Laparoscopic cholecystectomy is recommended in this patient even though the polyp is smaller than 10 mm, because she has symptoms possibly related to the polyp. Given her age, the polyp size, and the characteristic of her polyp (i.e., pedunculated), her risk of malignancy is very low. Thus, she does not require a triphasic CT scan or en bloc liver resection. Delaying her operation for serial ultrasound exams would be reasonable if she were not symptomatic. Cholestyramine is a bile acid sequestrant and would not treat this patient's symptoms (i.e., biliary colic due to the polyp). 5. A 64-year-old woman with chronic kidney disease had surveillance chest CT scan to follow a stable 1-cm pulmonary nodule. Lower cuts demonstrated a 3-cm right adrenal mass with nonenhanced attenuation of more than 10 Hounsfield units and washout of less than 60%. Further biochemical workup is consistent with the nodule being nonfunctional. With these findings, you should recommend

Figure 3.1.

A. repeat CT scan in 3 months. B. right adrenalectomy. C. percutaneous biopsy.

In 2 recent meta-analyses, sublay mesh placement had the highest probability of being the best treatment to prevent

26

D. no further management E. MRI.

ANSWER:

suspicious features, because adrenal resection would be appropriate regardless of the results of the biopsy. Repeat imaging can be undertaken when following an adrenal mass with benign features to assess for interval growth. However, in the setting of concerning imaging characteristics, repeat imaging may only serve to delay resection.

B

Adrenal incidentalomas are adrenal masses identified in an individual who has no clinical symptoms or signs suggesting adrenal disease. The prevalence of adrenal incidentaloma has increased in the past decades owing to the increasing use of abdominal imaging and the higher resolution of CT scanning. The actual prevalence, based on autopsy data, is approximately 6% of the general population. Prevalence tends to be higher in the older population and is bilateral in 10 to 15% of those identified.

6. A 33-year-old woman using oral contraceptives complains of right upper quadrant pain and is found to have a 4-cm hepatocellular adenoma. She is concerned because her uncle died of liver cancer. Your recommendation at this time is A. B. C. D. E.

An adrenal incidentaloma provides the physician an opportunity for early detection of a functional adrenal tumor or adrenal malignancy. Biochemical workup should be undertaken early to exclude a tumor with excessive secretion of cortisol, aldosterone, catecholamines, or androgens. The main concern of nonsecreting tumors of the adrenal gland is the potential for malignancy, primary or metastatic. The risk for adrenal cortical carcinoma is related to the size of the adrenal tumor, with the risk increasing significantly for tumors larger than 6 cm. No tumor size can exclude malignancy, yet the risk for adrenal cortical carcinoma in a tumor smaller than 4 cm with favorable CT imaging characteristics is exceedingly small.

liver resection. percutaneous biopsy. discontinue oral contraceptives. radiofrequency ablation angioembolization

ANSWER:

C

Hepatocellular adenoma is an uncommon benign liver tumor more commonly found in women than in men. Studies in pregnant women and those on oral contraceptives implicated a hormonal role in the growth, and possible regression, of hepatocellular adenoma. Although benign, hepatocellular adenoma carries a risk of rupture and life-threatening hemorrhage and is the second most common cause for spontaneous (nontraumatic) liver bleed. Risk of rupture is related to the size of the tumor. Risk of bleeding is increased in tumors larger than 5 cm, although tumors smaller than 5 cm can also bleed. Malignant transformation is rare but also more common in larger tumors (>5 cm). In most cases, hemorrhage can be treated effectively with angioembolization.

Size of the tumor alone is a criterion commonly used to determine whether a nonsecreting adrenal mass should be surgically resected. A retrospective review of nearly 200 consecutive adrenal gland resections showed that a threshold of 4 cm effectively identified the malignant tumors and reduced the unnecessary resections of benign, nonfunctional adrenal disease. Metastatic lesions to the adrenal gland can be of any size and are often smaller than primary adrenal cortical carcinoma. However, in most cases, there is a prior history of malignancy.

Diagnosis of hepatocellular adenoma can usually be made with MRI with gadolinium contrast injection or contrastenhanced multiphase CT scan. Thus, percutaneous biopsy is not needed and often discouraged due to the potential risk of bleeding.

Of the CT imaging characteristics, benign adenomatous features are associated with tumors with a fattier content High concentrations of intracellular lipid result in a lower attenuation in nonenhanced CT scans. A threshold of 10 Hounsfield units (HU) can distinguish adrenal adenomas and nonadenomatous lesions with high sensitivity and specificity. If the attenuation of the adrenal tumor is greater than 10 HU in a nonenhanced CT scan, then further investigation should be undertaken. Contrast washout should be calculated in postcontrast images to determine the likelihood of a benign adenoma. A washout greater than 60% favors adenoma.

Patients with hepatocellular adenoma who are taking oral contraceptives should discontinue their use as the first line of treatment. Most patients will have regression of hepatocellular adenoma after discontinuation of oral contraceptive, and no further intervention is necessary if hepatocellular adenoma is smaller than 5 cm or regresses to smaller than 5 cm. Liver resection is reserved for large tumors that remain larger than 5 cm despite other treatment or that have other features suspicious for hepatocellular carcinoma. Resection is also considered in male patients with hepatocellular adenoma of any size, owing to the higher risk of malignancy. Urgent surgery or angioembolization may be needed in cases of bleeding hepatocellular adenoma.

In this patient, a nonenhanced CT scan with attenuation greater than 10 HU and contrast washout of less than 60% are features that are concerning for a tumor that is potentially malignant, despite a tumor size smaller than 4 cm. Thus, adrenalectomy is recommended. An adrenal biopsy may be reasonable in differentiating benign from malignant adrenal masses in equivocal cases but is not indicated for a mass with

27

Radiofrequency ablation is commonly used to treat hepatocellular carcinoma. Radiofrequency ablation has a good safety profile and promising efficacy, although multiple radiofrequency ablation treatments may be required, depending on lesion size.

Because acute acalculous cholecystitis is not an obstructive disease of the biliary tree, endoscopic retrograde cholangiopancreatography with sphincterotomy is not needed for diagnosis or treatment. Similarly, magnetic resonance cholangiopancreatography can suggest the diagnosis and confirm the absence of gallstones but does not provide appropriate treatment.

7. A 70-year-old obese man is in the intensive care unit recovering from a myocardial infarction 10 days ago. He complains of new severe right upper quadrant pain associated with nausea. He is afebrile, his heart rate is 100 beats per minute, and his systolic blood pressure is 90 mm Hg. The exam is remarkable for right upper quadrant tenderness. His white blood cell count is 14,000/mm3 (3600-11,200/mm3). Liver function tests, including bilirubin, are normal. Right upper quadrant ultrasound demonstrates a distended gallbladder with a wall thickness of 8 mm, pericholecystic fluid, but no gallstones. Next treatment should include A. B. C. D. E.

8. A 70-year-old woman had a percutaneous endoscopic gastrostomy (PEG) placed 3 days ago at another hospital. CT imaging shows that the PEG transverses the transverse colon. She requires a PEG for enteral nutrition support, as she is unable to swallow. Optimal treatment for this complication is A. colonoscopy with PEG removal. B. removal of PEG by cutdown through the anterior abdominal wall. C. repeat upper endoscopy with PEG placement. D. jejunostomy placement. E. removal of PEG with colon repair and placement of a new gastrostomy.

laparoscopic cholecystectomy. open cholecystectomy. percutaneous cholecystostomy. magnetic resonance cholangiopancreatography. endoscopic retrograde cholangiopancreatography with sphincterotomy.

ANSWER:

ANSWER:

E

Inadvertent placement of a percutaneous endoscopic gastrostomy (PEG) into the colon can occur when the colon overlies the anterior gastric wall. When detected early, the malpositioned PEG should be removed, the colonic injury should be repaired, and a new gastrostomy should be placed. This procedure will allow both repair of the colonic perforation and accurate placement of a new gastrostomy for nutritional support.

C

Acute acalculous cholecystitis is a severe inflammation of the gallbladder in the absence of obstructing gallstones seen in patients with critical illness. It carries a high risk of morbidity and mortality. Diagnosis is often delayed given the focus on the critical condition of the patient, and a high index of suspicion is necessary. Patients variably present with right upper quadrant pain and tenderness, and diagnosis is suggested by typical ultrasonographic findings that include a distended gallbladder with thickened wall and pericholecystic fluid. Acute intervention is indicated and requires gallbladder removal (open or laparoscopic cholecystectomy) or drainage. Acute acalculous cholecystitis commonly develops in the presence of other organ system derangements, making percutaneous cholecystostomy an attractive treatment modality in place of cholecystectomy. Multiple reports demonstrate a high morbidity and mortality associated with open cholecystectomy in critically ill patients with acalculous cholecystitis, thus favoring a nonsurgical approach. Laparoscopic cholecystectomy, although less invasive than open cholecystectomy, requires general anesthesia, does not obviate the risks of surgery after a recent myocardial infarction (as in this patient), and is associated with high rates of conversion to open surgery in patients with acute acalculous cholecystitis.

PEG removal can be performed via colonoscopy. This technique is not optimal in this case in which the PEG was placed only 3 days ago, and there is no chronic tract to the skin with colocutaneous fistula. This patient requires repair of the colonic perforation. Attempts to remove the PEG by cutdown through the anterior abdominal wall will result in a larger colonic perforation, because the PEG has a larger internal bumper than is in the colonic lumen. Repeat upper endoscopy with PEG placement is inappropriate, because this patient already had a PEG complication with that technique, and visualization is required for gastrostomy placement. Jejunostomy is not indicated, because she has no contraindication to gastrostomy and gastric feeding. Methods to avoid misplacement of the PEG include reverse Trendelenburg position to move the colon inferiorly, adequate insufflation of stomach with air to displace the colon away from the anterior gastric wall, confirmation of both digital reflex (invaginate anterior gastric wall with external digital pressure and confirm endoscopically) and light reflex with transillumination.

Percutaneous cholecystostomy is the best treatment option in this patient. It provides an immediate treatment and can be performed under local anesthesia with ultrasound or fluoroscopic guidance.

28

9. A 48-year-old man was evaluated in clinic for recurrent large incisional hernia after ileostomy closure and primary incisional hernia repair 3 months ago. J He underwent initial laparotomy for perforated cecal diverticulitis with cecal resection, ileostomy, and mucous fistula 9 months ago. Physical exam findings and CT scan of the abdomen are shown (figures 9.1 and 9.2). You plan posterior; component separation with bilateral transversus abdominis release (TAR) for I hernia repair. What is the location for mesh implantation in TAR (posterior fl component separation)?

ANSWER:

C

Posterior component separation with transversus abdominis release (TAR) is a versatile, effective technique for ventral hernia repair. TAR avoids large skin flaps to access and divide the external oblique muscle. TAR starts by entering the posterior rectus sheath. A retrorectus plane is developed. This plane can be dissected from the central tendon of the diaphragm superiorly, to the space of Retzius inferiorly. The lateral dissection is extended to 1 cm medial to the linea semilunaris preserving the neurovascular bundles innervating the medial abdominal wall.

A. B. C. D.

Onlay over midline fascia and anterior rectus sheaths Onlay over midline fascia Sublay in retromuscular position Underlay extraperitoneal posterior to the posterior rectus sheath E. Intraperitoneal onlay mesh

The transversus abdominis muscle fibers are identified and divided to enter a retromuscular and preperitoneal plane. The dissection is then continued laterally to the psoas muscle. After bilateral TAR is completed, the posterior rectus sheath is closed in the midline to fully isolate the visceral contents from prosthetic mesh placement. Closure of the posterior rectus sheath also creates a space for the placement of a large piece of mesh in a sublay retromuscular position. After mesh implantation, the rectus muscle and anterior rectus sheath are closed above the mesh to restore the midline abdominal wall. The risk for mesh infection may be reduced with TAR due to native tissue presence on both sides of the prosthetic mesh. Intraperitoneal onlay mesh exposes the mesh to the visceral contents. In the anterior component separation technique, mesh is placed either as an onlay over the midline fascia closure or as an underlay in a retrorectus position, which carries potential risk of bowel herniation around the edges of the mesh.

10. For which of the following surgical procedures does acellular dermal matrix mesh demonstrate lower recurrence rates and fewer infectious complications than artificial mesh?

Figure 9.1.

A. Reinforcement of primary closure in ventral hernia repair B. B "Bridged repair" in ventral hernia defects greater than 15 cm C. Reinforcement of crural closure during paraesophageal hernia repair D. Parastomal hernia repair E. E Ventral hernia repair with concomitant enterocutaneous fistula repair

ANSWER:

E

The most common postoperative complications after herniorrhaphy are surgical site infections (SSIs) and hernia recurrence. These complications increase in frequency with increasing levels of wound contamination. Historically it was hypothesized that acellular dermal matrix may result in lower instances of infected mesh in contaminated wounds. Because acellular dermal matrix costs significantly more than synthetic mesh, some surgeons choose synthetic mesh even in a case with wound contamination. At the very least,

Figure 9.1.

29

financial considerations have led to a more selective use of acellular dermal matrix, limited to those herniorrhaphies most likely to benefit from biologic mesh.

(polydioxanone) is preferred over rapidly (2-3 months) absorbable suture (polyglactin). The rate of incisional hernia is lowest when small, closely spaced fascial bites are used. Studies of this technique documented no increase in fascial dehiscence rates. The technique that was documented as superior in 2 randomized trials uses 2-0 polydioxanone and small, closely spaced fascial bites. Fascial bites 5 mm deep and 5 mm apart resulted in an incisional hernia rate of 4.7 to 13% compared with 17.2 to 21% for the usual technique (large bites).

A study with 359 patients and 28-month follow-up demonstrated that the use of acellular dermal matrix results in lower rates of SSI when used for abdominal wall repair in the presence of heavy contamination, such as abdominal herniorrhaphies in conjunction with enterocutaneous fistula (ECF) repair. In these types of patients, acellular dermal matrix showed lower rates of infection, less hernia recurrences, and fewer complications compared to synthetic mesh. These data support the use of acellular dermal matrix rather than synthetic mesh for complex abdominal wall repair in the setting of heavy wound contamination or when ECF repair is performed concomitantly.

Based on these findings, polyglactin should be avoided, because it is a rapidly absorbable suture. Given that small bites have a reduced risk of incisional hernia, the depth and width of bites of 0.5 cm are most appropriate.

12. A 67-year-old man is recovering from pancreaticoduodenectomy for stage lb pancreatic adenocarcinoma (T2N0M0). The next step in his treatment should be

A separate study demonstrated that it was safe to repair both an abdominal wall hernia and ECF in a single operation. It also showed that the use of biologic mesh offered lower rates of fistula recurrence compared with synthetic mesh.

A. B. C. D. E.

In general, abdominal wall repair appears to have a lower rate of recurrence and SSI when primary facial closure is accomplished with mesh reinforcement rather than bridged mesh repair of the fascial defect. Even with the use of acellular dermal matrix, SSI and recurrence is high for bridged repairs.

ANSWER:

Multiple studies demonstrated high success rates in using synthetic mesh to augment crural closure, to repair parastomal hernias, and to reinforce primary closure in abdominal wall repair. Consequently, there appears to be no advantage in using the more expensive acellular dermal matrix in these settings.

ANSWER:

Size 0 1 2-0 0 0

Depth of bites (cm) 1 1 0.5 1 1

C

Pancreatic adenocarcinoma tends to have a poor prognosis. Complete resection allows for the highest chance for longterm survival. Numerous clinical trials have sought to improve outcome through neoadjuvant chemotherapy, radiotherapy, or a combination of the 2. Similar trials investigated these techniques in the adjuvant setting. Adjuvant chemoradiation is associated with higher toxicity, but not with an improvement in survival, and it is not recommended in node-negative patients. The only exceptions are patients with residual microscopic disease or nodepositive disease. These patients are known to benefit from adjuvant chemoradiation. Radiation therapy as a single adjuvant modality does not offer survival benefit in either a neoadjuvant or adjuvant role. Adjuvant chemotherapy is associated with a survival benefit and an acceptable toxicity. The recommended agents are gemcitabine or fluorouracil plus folinic acid. Gemcitabine is favored because of its lower toxicity. Therefore, routine surveillance without adjuvant chemotherapy is not recommended. Although the prospects of immunotherapy are promising, it remains under clinical investigation and is not recommended for routine use after resection outside of a clinical trial.

11. A 56-year-old man undergoes open low anterior resection. The lowest risk for abdominal wall hernia will occur by closing his midline laparotomy with which of the following strategies for a running closure? Material A. Polvglactin B. Polvglactin C. Polydioxanone D. Polydioxanone E. Polvglactin

chemoradiation. radiation. chemotherapy. routine surveillance follow-up. immunotherapy.

Width of bites (cm) 1 0.5 0.5 1 0.5

C

Incisional hernia is a significant issue after laparotomy. The etiology is multifactorial. The technique and choice of suture material can influence incisional hernia rate after laparotomy closure. Specifically, slowly (6-9 months) absorbable suture

13. A 55-year-old healthy man presents with jaundice (bilirubin =12 mg/dL; 0.2-1.9 mg/dL). His cancer antigen 199 level is 77 U/mL (0-37 U/mL). CT imaging reveals a 1.5-

30

cm pancreatic head mass, 1-cm dilated common bile duct, and a 0.9-cm pancreatic duct Fine needle aspiration of the mass produces cells suspicious for malignancy. The superior mesenteric artery appears to be uninvolved. The superior mesenteric vein-portal vein has regular contour and appears to be patent. The most appropriate next step in this patient's management should be A. B. C. D. E.

B. C. D. E.

ANSWER:

neoadjuvant FOLFIRINOX. neoadjuvant gemcitabine. neoadjuvant external beam radiation therapy. Whipple procedure. biliary decompression and neoadjuvant gemcitabine.

ANSWER:

magnetic resonance cholangiopancreatography. laparoscopic cholecystectomy. open extended cholecystectomy. open extended cholecystectomy, regional lymphadenectomy.

C

This patient has an incidental 11-mm gallbladder polyp. Most gallbladder polyps are benign and require no additional workup or treatment. However, a minority of gallbladder polyps represent adenomas that have the potential for malignant transformation to adenocarcinoma. Gallbladder cancer has a dismal prognosis unless it is detected at an early stage. Therefore, it is essential to diagnose and treat gallbladder cancer as early as possible.

D

This patient has a diagnosis of pancreatic adenocarcinoma. This diagnosis has a poor prognosis with a 5-year overall survival of only 5 to 7%. Unfortunately, 80% of patients present with incurable disease that is either locally advanced or metastatic. Patients with resectable or borderline resectable disease still have a 5-year overall survival of only 25%. The primary treatment for patients who present with resectable disease is surgery to resect the primary tumor and regional nodes. Surgical resection should be followed by adjuvant chemotherapy with either gemcitabine or fluorouracil. Adjuvant chemotherapy decreases recurrence and increases survival. No diefinitive data support the use of adjuvant radiation therapy in patients who have undergone a pancreaticoduodenectomy (Whipple procedure). There is no role for neoadjuvant radiation therapy.

Risk factors for gallbladder malignancy include age older than 50, primary sclerosing cholangitis, Indian ethnicity, and sessile polyp. Published single-institution studies that have followed patients with incidental gallbladder polyps support the very low risk of malignancy in patients with these lesions, especially when the polyp is less than 10 m in size, the patient has no risk factors, and the patient is asymptomatic. Published guidelines can be used to manage patients with incidental gallbladder polyps. Cholecystectomy is recommended if follow-up ultrasound shows that the polyp has increased in size by at least 2 mm or the polyp has reached 10 mm in size. If the polyp disappears, it is appropriate to cancel additional ultrasounds. Table 14.1 outlines recommended approaches to gallbladder polyps based on polyp size, presence or absence of symptoms, and presence or absence of risk factors. Without a diagnosis of invasive adenocarcinoma, no data support extended cholecystectomy or regional lymphadenectomy because the polyp is still confined within the gallbladder wall. When a patient is asymptomatic and has an ultrasound consistent with a gallbladder polyp, there is no indication for magnetic resonance cholangiopancreatography.

Patients with pancreatic adenocarcinoma in the head of the pancreas should undergo a Whipple procedure followed by chemotherapy. Patients who present with borderline resectable disease should be treated with neoadjuvant chemotherapy and biliary decompression, if needed. Several published guidelines define borderline resectable pancreatic adenocarcinoma (these recommendations are summarized in reference 3, table 1). The patient in this scenario has preoperatively resectable disease and therefore should undergo resection with curative intent as his initial treatment. Inspection of the bowel and liver should be done at the time of surgery before proceeding with resection.

Polyp size Symptomatic Risk factors Recommendations < 10 mm No No Follow-up ultrasound > 10 mm No No Cholecystectomy Any size Yes Yes or no Cholecystectomy < 6 mm No Yes Follow-up ultrasound > 6 mm No Yes Cholecystectomy Table 14.1. Recommended approaches to gallbladder polyps based on polyp size, presence or absence of symptoms, and presence or absence of risk factors.

14. A 67-year-old healthy man undergoes a CT scan of the abdomen after being involved in a motor vehicle collision. The scan is remarkable only for an incidentally noted 11-mm polyp in the fundus of the gallbladder. A right upper quadrant ultrasound confirms the CT findings with no evidence of cholecystitis. The lesion is homogeneous and isoechoic compared with the liver. The patient is hemodynamically normal, with no significant injuries related to the accident. The most appropriate next step in his management is

15. A 60-year-old woman presents with lower abdominal pain, nausea, and vomiting 2 years after a total cystectomy for bladder cancer. She is afebrile with normal vital signs. A

A. follow-up ultrasound at 6 months.

31

CT scan was obtained (figures 15.1,15.2, and 15.3). What is the most appropriate next step in management? A. B. C. D. E.

Re-siting of the ostomy Primary repair of the hernia Observation Herniorrhaphy with mesh Laparoscopic reduction with truss application

Figure 15.3

ANSWER:

D

Parastomal hernias remain a common complication in the long-term care of all ostomies. Wide variability in recurrence rates after repair exists, but repair at the original site with mesh reinforcement consistently yields the lowest failure rate. The keyhole mesh technique and the mesh flap technique are the most commonly performed. In the keyhole mesh technique, mesh is placed with a cutout allowing egress of the ostomy while providing wide coverage of the abdominal wall defect (figure 15.4). The mesh flap or Sugarbaker technique uses intra-abdominal placement of mesh over the hernia with a flap valve created at the edge of the mesh (figure 155). Both techniques can be performed laparoscopically, with the flap valve technique having somewhat lower recurrence rates.

Figure 15.1. Figure 15.4. Keyhole repair.

Figure 15.2.

Figure 15.5. Sugarbaker repair. 32

Parastomal hernias can be observed if asymptomatic; however, because this patient demonstrated intestinal obstruction, operative intervention is warranted. Moving the ostomy site is generally not recommended, because it requires a more complex operative procedure and creates the possibility of a second hernia site. Primary repair of parastomal hernias consistently has the highest recurrence rate. Laparoscopic reduction of the hernia with truss application does not provide hernia repair and is not definitive treatment.

A. B. C. D. E.

ANSWER:

Intrahepatic metastasis Biliary obstruction Retroperitoneal lymphadenopathy A fixed tumor shelf on the pelvis Diffuse peritoneal tumor seeding

ANSWER:

C

Approximately 80 to 90% of nonparasitic cystic masses of the spleen are primary cysts. Sometimes referred to as false cysts, they can be caused by trauma, infarct, or nonspecific bacterial infections. Each of these etiologies results in tissue injury with subsequent tissue degradation and formation of a fluidfilled cavity without a cellular lining. The remaining 10% of nonparasitic splenic cysts are either congenital, lined by epithelial-derived cells of uncertain source, or neoplastic, most commonly hemangiomas or lymphangiomas.

16. Which of the following is the most appropriate indication for hyperthermic intraperitoneal chemotherapy in patients with metastatic gastrointestinal adenocarcinoma? A. B. C. D. E.

Open splenectomy Open partial splenectomy Percutaneous aspiration Cyst sclerosis Percutaneous drain placement

The majority (70%) are symptomatic. Pain is thought to be due to the stretch of the splenic capsule from the mass effect of the cyst. Other symptoms include early satiety, nausea, vomiting, and weight loss. Physical signs may note splenomegaly.

E

After ruling out parasitic infection and evaluating for malignancy, percutaneous aspiration can reduce the mass effect and alleviate pain. Although recurrence is common, relief of pain after the aspiration confirms the cyst as the cause. Return of pain suggests the cyst has recurred. Surgical treatment at that point may be open or laparoscopic with partial or complete splenectomy. Some surgeons suggest open splenectomy in the setting of parasitic lesions to avoid the risk of intraperitoneal seeding. An alternative is percutaneous sclerosis in lieu of surgery to eliminate the risk of spreading daughter cysts.

Peritoneal surface malignancies can be either primary (e.g., peritoneal mesothelioma) or secondary (e.g., metastases of tumors from other sites, most commonly the gastrointestinal tract but also from the ovary or breast, as in lobular carcinoma). Reducing the overall tumor burden with a combination of surgery (cytoreductive phase) to remove gross disease combined with direct chemotherapy to the remaining tissues (hyperthermic intraperitoneal chemotherapy or HIPEC) has met with some success in selected patients with diffuse peritoneal nodules. Identifying those patients who would benefit is the subject of ongoing research. The best outcomes are in healthy patients with lowgrade tumors of appendiceal origin, in whom the peritoneal tumor burden can be maximally resected. The peritoneal carcinomatosis index is an intraoperative estimate of tumor burden throughout the abdomen and pelvis and correlates with outcome.

18. A 35-year-old man is seen after 12 hours of right lower quadrant discomfort, nausea, and vomiting. A CT scan is shown (figures 18.1 and 18.2). Biopsy of the lesion shown in the CT scan shows spindle cells. Which of the following statements regarding this patient is true?

Contraindications to HIPEC include tumors that are not surface nodules on the peritoneum and therefore will not be exposed to the intraperitoneal chemotherapy, such as intrahepatic metastasis or a pancreatic or bile duct tumor resulting in obstruction. Primary or metastatic disease located in the retroperitoneum, such as would be evident from a fixed tumor shelf, or in lymph nodes would also not be exposed to intraperitoneal chemotherapy and would not benefit from HIPEC.

A. Immunohistochemistry of the biopsy predicts prognosis. B. Nodal metastasis is more important to staging thanhistologic grade. C. Neoadjuvant chemoradiation is recommended. D. Local recurrence occurs in up to 60% of patients within 10 years. E. E Frozen sections at operation determine margins.

17. A 23-year-old woman presents with vague left upper quadrant abdominal pain. CT scan shows a 10-cm cyst in the spleen. She has no history of trauma. After ruling out parasitic infection and evaluating for malignancy, how do you determine whether the cyst is the etiology of her pain?

33

Figure 18.3.

Figure 18.1.

Malignancy or the relatively aggressive nature of these tumors depends on several pathologic characteristics, including size, histologic subtype (differentiation may require use of specific immunohistochemical markers), grade (to include differentiation, number of mitotic figures, and presence of necrosis), margin status, and evidence of

Figure 18.2.

ANSWER:

D

Figure 18.4

This patient has a sarcoma (figures 18.3 and 18.4). These soft tissue tumors arise from mesenchymal tissues such as fat, muscle, fibrous tissue, and blood vessels. Approximately 15% arise from the retroperitoneum. If suspected, preoperative percutaneous biopsy of a retroperitoneal tumor is discouraged, which allows the tumor capsule to remain intact. In addition, core biopsy is often not sufficiently

invasion of adjacent structures. The 8th American Joint Committee on Cancer tumor, node, and metastases staging for sarcomas now includes tailoring of size criteria to the primary tumor site. Lymph node involvement is relatively uncommon and contributes little to sarcoma staging. The efficacy of neoadjuvant chemotherapy for a high-risk sarcoma is widely debated. Several trials are ongoing, with a recent trial of epirubicin and ifosfamide showing promise for trunk and extremity tumors. Other neoadjuvant chemotherapy regimens, followed by resection with or without radiation, may improve the response rate for younger patients with high-grade, borderline resectable lesions to preserve function at the time of operation. Evidence for

predominant failure pattern is that of local recurrence, which occurs in 25 to 30% of patients at 5 years and 35 to 60% of patients after 10 years.

34

concomitant chemoradiation in the management of a retroperitoneal sarcoma, however, is lacking.

ascites leaking from the site. Which of the following statements is true regarding surgical repair of umbilical hernia in cirrhotic patients? A. Surgical repair should not be performed unless complications are present B. The recanalized umbilical vein should be ligated. C. The use of mesh is contraindicated. D. Placing a peritoneovenous shunt reduces wound complications. E. Surgical repair is done electively after a period of medical optimization.

19. A 40-year-old woman presents with fever and minimal right upper quadrant pain. Her medical history is notable only for diabetes mellitus. Laboratory findings include a white blood count of 11,000/mm3 (3600-11,200/mm3), direct bilirubin of 1.4 mg/dL (0-0.3 mg/dL), and total bilirubin of 2.4 mg/dL (0.2-1.9 mg/ dL). Abdominal ultrasound is unremarkable, but axial imaging notes air in the wall of the gallbladder. She undergoes a laparoscopic cholecystectomy the next morning and is noted to have gangrenous cholecystitis. What factor correlates with gangrenous cholecystitis? A. B. C. D. E.

ANSWER:

The physiologic effects of portal hypertension can be severe and profound. Collateral flow leads to the development of esophageal and gastric varices, significant hemorrhoid disease, and recanalization of the umbilical vein. Increases in intraabdominal pressure as a result of ascites can lead to umbilical and inguinal hernias. Umbilical hernias in patients with ascites will continue to enlarge. Approximately 1 in 5 patients with cirrhosis will have an umbilical hernia that, if left untreated, can result in significant morbidity, including bowel strangulation and skin breakdown with subsequent leakage of ascitic fluid. Patients are often hypoalbuminemic and coagulopathic, rendering them poor surgical candidates. Historically, umbilical hernias were not repaired unless necessitated by one of these complications. Surgical repair in the setting of these complications is highly morbid. Because advances in anesthetic techniques have reduced the operative risks, complication rates of less than 10% led to current recommendations for elective repair.

Female sex White blood count less than 15,000/mm3 C Total bilirubin greater than 2.0 mg/dL History of diabetes mellitus Age less them 45 years old

ANSWER:

E

D

Gangrenous cholecystitis affects approximately 1 in 5 patients who present with acute cholecystitis. Gangrenous cholecystitis is an unlikely progression of acute cholecystitis and is more common in elderly male patients with comorbid conditions such as diabetes mellitus. On presentation, fullthickness ischemic necrosis of the gallbladder is typically encountered. Laparoscopic attempts at removal are associated with a higher open conversion rate. These patients often have worse outcomes, including increased hospital and intensive care unit length of stay and mortality. Although treatment options are generally the same as for acute cholecystitis (cholecystectomy), early recognition is important to avoid complications related to delays in treatment.

Patients should be medically optimized before undergoing elective surgical repair. This includes free water restriction, diuretics, and large volume paracentesis (with infusion of albumin). Some series report good results with perioperative placement of a temporary peritoneal dialysis catheter or transjugular intrahepatic portosystemic shunt. With its associated risks and the availability of these other techniques, there is no current indication for placement of a perioperative peritoneovenous shunt.

Several risk stratification strategies exist, but signs and symptoms that should raise concern for gangrenous cholecystitis include the following: age older than 45, gallbladder thickness greater than 4.5 mm, white blood cell count greater than 15,000/mm3 (3600-11,200/mm3), history of diabetes mellitus, and male sex. Of note, up to half of patients with gangrenous cholecystitis will have a normal ultrasound. CT imaging is being used with increasing frequency when the diagnosis is in question. Several series reported an overall accuracy of CT imaging for gangrenous cholecystitis of nearly 90%. Signs on CT imaging that raise concern for the diagnosis are intraluminal or intramural gas, irregular gallbladder wall, gallbladder abscess, and absence of part or the entire gallbladder wall.

From a technical standpoint, the use of synthetic mesh is not contraindicated. Excellent results are documented with both onlay and retrorectus sublay repairs. Recurrence rates drop from around 15% with primary repairs to less than 3% with the use of mesh. A patent umbilical vein should not be ligated if encountered. A substantial portion of the patient's splanchnic flow may be diverted through such a vein, and ligation can cause decompensation.

20. A 54-year-old with a history of alcoholic cirrhosis and ascites presents with a reducible but painful umbilical hernia. He has no overlying skin changes and denies history of

21. An 18-year-old man is taken urgently for splenectomy and hepatorrhaphy after a motorcycle crash. Due to

35

coagulopathy and hemodynamic abnormalities, his abdomen is left open, and a temporary abdominal vacuum closure device is applied. He is taken to the intensive care unit and resuscitated. He is started on enteral nutrition on postoperative day 1. Enteral nutrition A. B. C. D. E.

B. Superficial surgical site infections are higher with open placement. C. Distal shunt failure rates are lower with laparoscopic placement D. Operative time is increased with laparoscopic placement. E. Open placement is preferred in patients who have received multiple shunts in the past.

will increase bowel distension. increases the risk for bowel ischemia. increases the odds of primary fascial closure. increases the risk of enteroatmospheric fistula. should be given as an elemental formula.

ANSWER:

ANSWER:

C

Ventriculoperitoneal shunt placement is the first-line neurosurgical treatment for long-term decompression and drainage of hydrocephalus. The likelihood of shunt revisions over the lifetime of the patient is high and can involve revision at either the ventricular (proximal) or peritoneal (distal) portion. Historically, the peritoneal portion was placed into the abdomen by a small laparotomy either in the right or left upper quadrant, and the fascia around the shunt insertion was secured to prevent leakage. Increasingly, more general surgeons are assisting in neurosurgery and gaining laparoscopic access to the peritoneum to place the shunt under direct vision with a much smaller insertion incision/puncture. Theoretical benefits of laparoscopic

C

Damage control surgery, involving staged completion of trauma and emergency surgical cases, improves mortality in certain settings. The main tenet involves rapidly achieving hemostasis and contamination control but forgoing definitive completion of the procedure until the patient's physiologic derangements are reversed. Often, this can be accomplished within 24 to 48 hours, and definitive fascial closure can be achieved at that time. Sometimes, however, several trips to the operating room are required, and definitive fascial closure is delayed. Damage control surgery does lead to some unique risks, including enterocutaneous fistula and large ventral hernias. Thus, the risks of staging an abdominal closure must be weighed against a prolonged index operation.

placement include the lower likelihood of shunt damage at the insertion site, direct placement into the pelvis or suprahepatic space, avoidance of any subcutaneous space overlying the fascial insertion site, and direct visualization of cerebrospinal fluid flowing through the shunt after placement.

Early (within 72 hours) enteral nutrition in the setting of an open abdomen is associated with increased primary fascial closure rates, decreased overall complications, and decreased mortality. These benefits appear to be mainly in the setting of patients without bowel injury. Interestingly, the average time to primary fascial closure appears to be longer in patients receiving enteral nutrition. For patients with a bowel injury, early enteral nutrition does not appear to have the same benefits, although there are no reported adverse effects related to early enteral nutrition in this population. Risks of enteroatmospheric fistula in the setting of an open abdomen are related to the presence of an anastomosis (especially large bowel), infection, or total fluid intake at 48 hours and not to the initiation of early enteral nutrition. There are no reports of enteral nutrition causing bowel ischemia in the setting of appropriately resuscitated patients. Also, there are no reported benefits of elemental or trophic feeds over full or standard feeds in the setting of an open abdomen.

22. A 24-year-old woman is determined by a neurosurgeon to require a ventriculoperitoneal shunt for hydrocephalus. Which of the following statements is true regarding the placement of the peritoneal portion (distal portion) of the shunt? A. Laparoscopic placement is beneficial only in morbidly obese patients.

36

Laparoscopic-assisted ventriculoperitoneal shunt placement is associated with shorter operative time and lower rates of distal shunt failure. Interestingly, the overall shunt failure rate is the same between the 2 techniques. These benefits are independent of patient BMI. There is no benefit to placing a shunt in a traditional, open fashion in the setting of multiple prior shunts. Accessing the peritoneum laparoscopically offers a more comprehensive view of the peritoneum to determine optimal shunt location. The rate of surgical site infection between open and laparoscopic shunt placements is the same in several studies.

ANSWER:

C

For most patients with acute cholecystitis, cholecystectomy is the preferred treatment. To better characterize acute cholecystitis and codify its management, the Tokyo guidelines were developed in 2007 and later revised (table 23.1). Findings in grade I cholecystitis are limited to inflammatory changes in the gallbladder but no associated organ dysfunction. Findings of grade II cholecystitis include leukocytosis, a palpable tender mass, and marked local inflammation on imaging but no associated organ dysfunction. Grade III includes grade I and II findings, as well as organ dysfunction: hypotension, mental changes, respiratory failure, oliguria, and coagulopathy.

“Severe” (Grade III) acute cholecystitis is accompanied by dysfunctions in any one of the following organs/systems: 1. Cardiovascular dysfunction

Hypotension requiring treatment with dopamine >5 pg/kg per minute, or any dose of dobutamine

2. Neurological dysfunction

Decreased level of consciousness

3. Respiratory dysfunction

PaO2/FiO2 ratio <300

4. Renal dysfunction

Oliguria, creatinine >2.0 mg/dL

5. Hepatic dysfunction

PT-1NR > 1.5

6. Hematological dysfunction

Platelet count <100,000/mm3

“Moderate” (Grade II) acute cholecystitis is accompanied by any one of the following conditions: 1. Elevated WBC count (> 18,000/mm3) 2. Palpable tender mass in the right upper abdominal quadrant 3. Duration of complaints >72 hours0 4. Marked local inflammation (gangrenous cholecystitis, pericholecystic abscess, hepatic abscess, biliary peritonitis, emphysematous cholecystitis) “Mild” (Grade 1) acute cholecystitis does not meet the criteria of “Severe (Grade III)” or “Moderate (Grade II)” acute cholecystitis. Grade I can also be defined as acute cholecystitis in a healthy patient with no organ dysfunction and only mild inflammatory changes i the gallbladder, making cholecystectomy a safe and low-risk operative procedure. ‘Laparoscopic surgery in acute cholecystitis should be performed within 96 hours after the onset WBC white blood cell Table 23.1. TG07 severity assessment criteria for acute cholecystitis. 23. Which of the following statements regarding percutaneous cholecystostomy used for acute cholecystitis is true?

The Tokyo guidelines recommend urgent cholecystostomy tube placement followed by antibiotics and eventual cholecystectomy for grade III cholecystitis.

A. The rate of readmission compared with laparoscopic cholecystectomy is lower. B. The rate of recurrent acute cholecystitis is less than 10%. C. Thirty-day mortality is higher than for laparoscopic cholecystectomy. D. Long-term survival is better than for laparoscopic cholecystectomy. E. Hospital stay is shorter than for laparoscopic cholecystectomy.

Interestingly, this management strategy does not seem to be followed by most physicians in the United States. A retrospective cohort study using Medicare enrollment and claims data looked at patients admitted with acute cholecystitis and organ failure (grade III), comparing those managed with cholecystostomy with those who did not have cholecystostomy (58% of whom fed cholecystectomy during the index admission). In that study, in-hospital mortality was similar, but the odds of 30- and 90-day mortality were significantly higher in patients who underwent

37

cholecystostomy. While well meaning, the Tokyo guidelines evaluate only the organ to be removed and not the patient. This could be one reason the US experience is not consistent with these suggestions.

25. A 55-year-old healthy woman presents to the emergency department with lower abdominal pain. A CT scan of the abdomen/pelvis shows a 2.5-cm mass in the pancreatic tail. Ultrasound-guided biopsy of the pancreatic mass shows a low-grade neuroendocrine tumor. What is the recommended treatment?

If cholecystectomy is not performed, acute cholecystitis will recur. As such, readmission rates associated with cholecystostomy tube placement are far greater than for cholecystectomy; readmission rate is greater than 20% for gallstone-related complications. Recurrence of cholecystitis was seen in up to 41% of patients with tube placement, with readmission rates of 23 to 41% compared with a readmission rate of 9% without cholecystostomy tube.

A. B. C. D. E.

ANSWER:

Length of stay is significantly longer if cholecystostomy tube is placed compared with no tube being placed and a laparoscopic cholecystectomy.

Onlay polypropylene mesh reinforcement Sublay polypropylene mesh reinforcement Primary surgical closure with continuous running suture Primary surgical closure with interrupted running suture Incisional negative pressure device

ANSWER:

D

Most neuroendocrine tumors of the pancreas (PNETs) are found incidentally and are nonfunctional. Surgical resection is the primary treatment for localized PNETs. For nonfunctioning tumors 2 cm or smaller and for insulinomas, enucleation is considered reasonable. Depending on the location, distal pancreatectomy (with possible splenic preservation) or pancreaticoduodenectomy may be necessary, even for small tumors. Management of lymph nodes is controversial for nonfunctional PNETs between 1 and 2 cm, because the risk of nodal metastases is low but measurable (reported 7-26%). Lymph node resection should be considered in these patients. Formal pancreatic resection, including regional lymph nodes, is recommended for tumors larger than 2 cm, functional PNETs (somatostatinoma, VIPoma, glucagonoma), or with involved lymph nodes. Total pancreatectomy is rarely necessary. Although observation is proposed as a reasonable option for small (<2 cm), nonfunctional, incidentally found tumors, it would not be recommended for this 2.5-cm tumor.

24. Which of the following fascial closure techniques minimizes the incidence of incisional hernia after an elective midline laparotomy? A. B. C. D. E.

Enucleation of the mass Total pancreatectomy Observation with repeat imaging in 6 months Distal pancreatectomy with splenectomy Octreotide

A

Incisional hernia is one of the most common long-term complications after abdominal surgery, with reports ranging from 5 to 20% in the general patient population and as high as 30% in high-risk patients, including individuals with a BMI greater than 30 or with abdominal aortic aneurysms. In these patients, prophylactic mesh reinforcement for an elective midline laparotomy may reduce incisional hernias. A systematic review of 12 randomized controlled trials reported that prophylactic mesh placed during midline laparotomy reduced the incidence of incisional hernia but led to increased rates of seromas and surgical site infections.

Octreotide is a synthetic analog of somatostatin. Most PNETs express somatostatin receptors. Octreotide is used in the treatment of locoregional unresectable NETs and patients with distant metastatic disease. This patient has resectable disease.

The PRIMA randomized controlled trial recently reported that onlay mesh reinforcement prevented incisional hernia better than sublay mesh reinforcement or primarily sutured incisions. In spite of a higher incidence of seromas in the onlay group, there was no associated increased rate of surgical site infections, mesh infections, reoperations, or readmissions. Given that onlay placement of a mesh is technically easier than sublay mesh reinforcement, laparotomy closure with onlay mesh reinforcement could become the standard treatment for high-risk groups regardless of surgical specialty. Negative pressure devices provide a safe and easy tool that preserves fascial integrity and elasticity and increases the likelihood of eventual primary fascial closure in critically ill patients who require temporary abdominal closure.

38

Abdomin Part IV

C. Quality-of-life scores are better at 2 years compared with repair. D. Incarceration requiring urgent repair is 10%. E. Operative repair is rarely required in men older than 65.

ITEMS 1-25 For each question, select the best possible response. 1. A 50-year-old woman presents with chronic constipation. On exam, fullness is appreciated in the left abdomen. A CT scan of her abdomen/pelvis shows a 25 x 15-cm lipomatous mass in the left retroperitoneum, with focal areas of increased density in the tumor mass, most consistent with liposarcoma. Surgical resection with

ANSWER:

Adult inguinal hernia repair is one of the most commonly performed surgical operations (28 per 10,000 North Americans per year). Approximately one-third of patients presenting with a hernia have minimal symptoms. The risk of incarceration versus the risk of chronic groin pain after elective repair must be balanced. Traditionally, even minimally symptomatic hernias were repaired because watchful waiting represented a risk for eventual incarceration and a later, riskier emergency procedure. In 2 trials of watchful waiting for male minimally symptomatic inguinal hernia, there was a crossover into the repair group of 72% at 75 years in the UK trial and 64% at 10 years in the US trial. These findings led to the current recommendation that although watchful waiting is a reasonable approach initially, most men will need eventual repair of inguinal hernias. The rate of acute incarceration requiring urgent repair during watchful waiting was low (2.4% in the US trial and 2.5% in the UK trial). There was no mortality or need for bowel resection in repairs for

A. B. C. D.

grossly negative margins improve overall survival adjuvant chemotherapy reduces local recurrence. adjuvant radiation therapy improves overall survival. next-generation tumor DNA sequencing helps guide adjuvant treatment. E. adjuvant imatinib reduces local recurrence.

ANSWER:

B

A

Surgical resection is the primary treatment for retroperitoneal liposarcoma. The tumors are often large and can involve adjacent viscera, including the bowel, kidney, and spleen. Given the large nature of these tumors, complete microscopic evaluation of the margins is not feasible in most tumors. Therefore, gross negative margins are more often reported in retroperitoneal sarcoma studies. In most patients, to achieve grossly negative margins, adjacent organs often need to be resected en bloc. Patients with grossly negative margins have a median survival of 103 months compared with 18 months for incomplete resections. Recurrence is common, even with extensive surgery, and these patients often must undergo several repeat resections.

acute incarceration in either trial. Older men crossed over to the surgery group at a significantly higher rate than younger men in both the UK and US trials. In a Dutch trial of watchful waiting versus repair in men older than 50 (mean age 65.1 years) with minimally symptomatic inguinal hernia, there was a crossover rate of 37.8% during the 36-month study period, with 5 cases (1.9%) of repair for acute incarceration and one case (0.4%) of acute strangulation requiring a bowel resection. There was one operative death in the crossover group due to cardiac arrest before surgery during the anesthetic induction. Overall, 36-month event-free survival was 80.9% after elective repair compared with 77.2% after watchful waiting by Kaplan-Meier analysis. Pain, discomfort, and quality-of-life scores were slightly better in the immediate repair group in the later part of the study. The Dutch study could not rule out a relevant difference in favor of elective repair versus watchful waiting, but the investigators believed that watchful waiting was still a reasonable alternative compared with surgery in men 50 and older.

Adjuvant chemotherapy is not recommended for resected retroperitoneal liposarcoma; anthracycline-based chemotherapy is typically reserved for metastatic patients. Adjuvant radiation is not well established for retroperitoneal liposarcomas. The practical challenges for retroperitoneal sarcoma compared with extremity sites include the extent of retroperitoneal tumors in the abdominal cavity that would necessitate a large radiation field and the proximity of the bowel, which limits the radiation dose. Adjuvant radiation reduces local recurrence in retrospective studies but does not improve overall survival. To date, there are no completed randomized controlled trials of adjuvant radiation in retroperitoneal liposarcoma. Routine next-generation tumor DNA sequencing has not shown utility in guiding adjuvant treatment for liposarcoma. Imatinib, a tyrosine kinase inhibitor, is used in gastrointestinal stromal tumors that harbor KIT and PDGFRA mutations but not liposarcoma.

Inguinal hernia lifetime risk is 27 to 42.5% in men and 3 to 5.8% in women. Groin hernia repair in women was not addressed in any randomized controlled trials or systemic reviews. Urgent repair of groin hernias in required in 17% of women with groin hernias versus 5% of men. Women are thought to be at higher risk of incarceration and strangulation due to the inability of examination or imaging to reliably differentiate femoral from inguinal hernias. Repair without watchful waiting is recommended in nonpregnant women; a laparoscopic approach may help avoid missing a femoral

2. Which of the following statements is true regarding watchful waiting of a minimally^ symptomatic inguinal hernia in adults? A. Repair will be needed in 50% at 10 years. B. Is not appropriate for a female patient.

40

hernia. In pregnant women with an inguinal lump and pain, color-duplex ultrasound can identify round ligament varicosity, which is more common than inguinal hernia.

27.3%, respectively; p = .875). Both human and animal studies showed safety and efficacy of permanent synthetic mesh during other clean-contaminated procedures, such as colectomy, ostomy creation, or gastric bypass. Randomized controlled trials of mesh placement during cleancontaminated procedures demonstrated lower hernia rates than primary repair without mesh-related complications. Although no randomized controlled trials were performed on small enterotomies with limited contamination of the operating field during adhesiolysis in LVHR, a moderate amount of anecdotal and retrospective evidence exists for a variety of management strategies.

3. Which of the following statements is true regarding laparoscopic ventral hernia repairs (LVHR)? A. LVHR has a lower recurrence rate compared with open ventral hernia repairs. B. A Veress needle has a higher complication rate compared with open Hasson technique during primary port placement. C. Concomitant implantation of a permanent synthetic mesh must be avoided after inadvertent enterotomy. D. A mesh overlap of 2 cm at hernia edges reduces recurrence. E. LVHR leads to fewer surgical site infections compared with open repair.

ANSWER:

The 2016 SAGES guidelines state strategies that may be safe and effective include primary repair, delayed placement of prosthetic mesh, and concomitant use of prosthetic mesh. Three randomized controlled trials and 4 meta-analyses of earlier trials showed fewer surgical site infections with LVHR compared with open repair. In a Cochrane analysis, LVHR reduced the risk of wound infection over open repair with a relative risk of 0.26.

E

Four meta-analyses, including a Cochrane review, reported no differences in recurrence rates between laparoscopic ventral hernia repairs (LVHR) and open ventral hernia repair, although follow up was less than 2 years in half of the trials. No difference was noted in major complication rates by technique of abdominal access (Veress needle, open Hasson technique, or direct trocar insertion without pneumoperitoneum) in multiple meta-analyses and randomized controlled trials for a variety of general surgical and gynecological laparoscopic procedures.

4. Which of the following statements regarding the use of anticoagulation therapy at the time of colonoscopy is true? A. The timing of discontinuing Vitamin K antagonists before an endoscopic procedure is dictated by the patient's creatinine clearance. B. Screening colonoscopy should be deferred when a patient is receiving warfarin 12 months after a deep venous thrombosis. C. Aspirin should be discontinued during the periendoscopic period. D. Elective polypectomy should be deferred in a patient on dual antiplatelet therapy for a coronary stent placed 3 months ago. E. Direct Factor Xa inhibitors can be safely continued during colonoscopy with planned polypectomy.

The 2016 SAGES Guidelines for Laparoscopic Ventral Hernia Repair recommends surgeons use the access technique that the patient anatomy allows and with which they are trained and most comfortable.

The appropriate amount of prosthetic mesh overlap beyond the hernia defect has not yet been determined. The Italian LVHR guidelines recommend a minimum of 3 cm of mesh overlap, but the 2016 SAGES guidelines suggest that an overlap of 5 cm or more may be desirable in larger defects. Current LVHR techniques typically use prosthetic mesh and are restricted to clean and clean-contaminated cases. This technique excludes active enterocutaneous fistula. Although hernia surgeons must take care during adhesiolysis to avoid inadvertent enterotomy and inspect the bowel after adhesiolysis, bowel injury can still occur. Several retrospective studies examining LVHR and enterotomies exist. A study of 33,832 patients from administrative data recommended not using mesh with any level of contamination due to a wound complication rate of 21.2% when mesh was used versus 17.5% (p <.001) without mesh use in clean-contaminated cases. This study had no difference in wound complication rates in contaminated cases (31.2 vs

ANSWER:

D

The management of patients receiving anticoagulation and being considered for an endoscopic procedure depends on the indication and method of anticoagulation as well as the invasiveness of the endoscopic procedure. In patients on short-term anticoagulation therapy with Vitamin K antagonists who need an elective colonoscopy, the procedure should be deferred until anticoagulation therapy is completed. Dosing adjustments for the use or temporary discontinuation of warfarin are not usually required based on patients' creatinine clearance. Often the anticoagulation was initiated for a remote event, such as a deep venous thrombosis 12 months earlier, and can be discontinued. Direct Factor Xa inhibitors, such as rivaroxaban, can be continued during lowrisk endoscopic procedures, but renal clearance must be considered because this will affect rivaroxaban levels.

41

Patients often receive single- and dual-agent antiplatelet therapy to prevent thrombotic events. This usually includes a combination of thienopyridines, such as clopidogrel, with aspirin. Aspirin or nonsteroidal anti-inflammatory drugs can be continued safely during colonoscopy with or without polypectomy, but thienopyridines should be discontinued 5 to 7 days before high-risk procedures such as a polypectomy. Aspirin monotherapy can be used until the thienopyridine is resumed. In the first 6 months after coronary stent placement, the risk of lethal coronary thrombosis is high if dual antiplatelet therapy is discontinued, and high-risk procedures should be deferred.

SPN tumors occur most commonly in women, with a mean age at diagnosis between 20 and 30 years. On imaging studies, SPN tumors are often well circumscribed, encapsulated, and heterogeneous with hemorrhagic and cystic degeneration on imaging studies. Cytology smears demonstrate abundant cells that are bland and uniform with moderate cytoplasm, but immunohistochemical staining demonstrates nuclear localization of beta-catenin diagnostic of SPN. MCN tumors are often unilocular mucinous tumors found almost exclusively in women, usually ages 40 to 60. These tumors are premalignant, and histologic examination demonstrates mucinous producing epithelium surrounded by a unique ovarian stroma.

Items 5-8

IPMN tumors are typically found in older men and women. The tumor mass detected on imaging is the result of mucinproducing epithelia lining the pancreatic ductal system. When occurring in the side branches, the tumors are spherical, whereas main duct tumors typically manifest as a dilated main pancreatic duct full of mucin. On endoscopic examination, this mucin will protrude through the ampulla giving a classic fish mouth appearance. Cytologic analysis identifies abundant mucin surrounding cells that may have premalignant dysplastic cell changes or carcinoma.

Each lettered response may be selected once, more than once, or not at all. A. Mucinous cystic neoplasm B. Main-duct intraductal papillary mucinous neoplasm C. Serous cystadenoma D. Pseudopapillary neoplasm E. Cystic neuroendocrine tumor 5. Mean age at diagnosis of 20 to 30 years 6. Fish mouth ampulla

Approximately 10% of pancreatic neuroendocrine tumors present as cystic masses. These tumors are predominantly unilocular cysts of approximately 3 cm in size located in the tail of the pancreas. Endoscopic ultrasound-guided aspiration biopsy of these cysts will identify straw-colored fluid and uniform cells that stain strongly for chromogranin A or synaptophysin, both markers of cells of neuroendocrine origin.

7. Fine needle aspiration positive for chromogranin A 8. Fine needle aspiration positive for periodic acid-Schiff staining

ANSWERS:

D, B, E, C

Cystic lesions of the pancreas are common, and autopsy series have identified them in up to 25% of individuals older than 65. An increasingly common clinical scenario is the identification of incidental cystic masses on imaging studies. Knowledge regarding the demographics and clinical presentation is necessary to guide testing and weigh the need for surgical intervention. Endoscopic examination with ultrasound and guided fine needle aspiration are commonly used and provide additional diagnostic information.

9. Which of the following statements regarding the management of gallbladder cancers is true? A. Staging laparoscopy is recommended before laparotomy for a potentially curative resection of gallbladder cancer. B. Lymphadenectomy for gallbladder cancer should include lymph nodes in the porta hepatis and interaortocaval groove. C. Cholecystectomy, en bloc hepatic resection, and lymphadenectomy with or without bile duct excision is recommended for T1a gallbladder cancer. D. A T1b lesion can be treated with liver resection alone. E. In a patient with a T1b tumor and positive cystic node, resection of the common bile duct is required.

The 4 major types of pancreatic cystic neoplasms are intraductal papillary mucinous neoplasms (IPMN), mucinous cystic neoplasms (MCN), serous cystic neoplasm (SCN), and solid pseudopapillary neoplasm (SPN). Pancreatic neuroendocrine tumors can also take a cystic form and should be considered. SCN usually present as multilocular cystic tumors that can appear as a mass on imaging in men and women older than 50. Pancreas-specific imaging and endoscopic ultrasound identify numerous loculations, and aspiration biopsy show clear fluid without mucin and benign appearing epithelial cells that contain glycogen that is positive on a periodic acid-Schiff stain.

ANSWER:

A

Surgery offers the best chance to cure patients with gallbladder cancer. Gallbladder cancers are often diagnosed only after inspection of the cholecystectomy specimen. For patients with T1a lesion, observation is acceptable. For T1b

42

or greater lesions, surgery is recommended and usually includes hepatic resection and lymphadenectomy with or without bile duct excision.

bowl necrosis is present at the index operation, ileocecal resection is appropriate. The timing of cholecystectomy and takedown of the cholecystenteric (usually cholecystoduodenal fistula) is controversial. Takedown of the fistula can be technically challenging and should not be done at the index operation.

Staging laparoscopy has a high yield for detection of occult metastatic disease and is recommended before laparotomy for a potentially curative resection of gallbladder cancer. The lymphadenectomy should include lymph nodes in the porta hepatis, gastrohepatic ligament, and retroduodenal regions. Interaortocaval nodes are considered a distant metastatic site, and positive nodes render the patient unresectable for cure.

Milking stones in the cecum should not be performed, because this can result in mucosal or serosal injury that may not be identified. A loop ileostomy alone does not address the primary pathology of small bowel obstruction secondary to an impacted gallstone. In addition, bowel necrosis will persist and may result in perforation and poor patient outcome.

Pathologic evaluation of the cystic duct margin is part of the inspection of the cholecystectomy specimen when gallbladder cancer is identified. Resection of the common bile duct is recommended if cancer cells are identified at the cystic margin. Presence of a positive cystic duct node is not an indication for common bile duct resection.

11. A 29-year-old man undergoes an elective, totally extraperitoneal laparoscopic hernia repair for a right-sided indirect inguinal hernia. A polypropylene mesh is fixed in place using mesh tackers. The surgery is uneventful, and postoperative recovery is complicated only by right groin pain that gradually worsens over the next 2 weeks. At 3 months, the pain is persistent despite oral analgesics. The pain is exacerbated by physical activity and twisting of the trunk away from the site of surgery and is associated with a burning sensation. What is the next step in the work up of this patient's groin pain?

10. A 79-year-old, morbidly obese woman presents to the emergency department with nausea, vomiting, and abdominal pain for the past 48 hours. She is tachycardic and hypertensive. Imaging demonstrates pneumobilia and multiple air-fluid levels suggestive of a small bowel obstruction. Surgical exploration during laparotomy demonstrates dense adhesions between the liver and duodenum, with the presence of a stone in the lumen of the terminal ileum. The overlying bowel at this transition point is dusky, with areas of patchy necrosis. What is the next step in the management of this disease process?

A. B. C. D. E.

A. Incision proximal to the site of obstruction with extraction of the stone and primary closure B. Incision proximal to the site of obstruction with extraction of the stone, primary closure of enterotomy, and cholecystectomy C. Milk stone into cecum D. Ileocecal resection with primary anastomosis E. Loop ileostomy

ANSWER:

MRI of the pelvis Regional groin nerve block Percutaneous nerve ablation Diagnostic laparoscopy Triple neurectomy with mesh removal

ANSWER:

A

Chronic groin pain after an inguinal herniorrhaphy can result from neuropathic or nonneuropathic etiologies. Nonneuropathic etiologies, such as hernia recurrence and meshomas, can cause chronic inguinal pain and should be ruled out early in the evaluation. An MRI of the pelvis will help identify nonneuropathic causes of his pain.

D

This scenario describes the development of a gallstone ileus secondary to egress of a gallstone through a cholecystenteric fistula. Obstruction generally occurs at the terminal ileum where the stone is unable to traverse the ileocecal valve. This obstructive pattern can lead to irreversible ischemia and necrosis.

Neuropathic pain is usually secondary to mesh entrapment of a nerve, neuroma formation after nerve injury, or inappropriate laparoscopic tack application. The physical examination may help elucidate which nerve to focus the assessment on. In certain situations, a regional nerve block may be both diagnostic and, at least temporarily, therapeutic. Percutaneous nerve ablation involves the application of a high-frequency, high-temperature electrical current to the affected tissue resulting in the destruction of nerve tissue and the prevention of pain transmission. The treatment is controversial because it can lead to the formation of neuromas and a neuritis reaction, which can lead to worsening of the symptoms. Before application of these procedures, ruling out nonneuropathic sources of pain is prudent.

Creating an enterotomy at a healthy portion of small bowel and milking the stone out proximally is reasonable if bowel compromise is not present. The enterotomy is created longitudinally and subsequently closed transversely once the stone is extracted. The surgeon should thoroughly explore the rest of the bowel to ensure no second stone is present. Most recurrences of gallstone ileus occur within 3 months and are often due to stones missed during the initial operation. When

43

Diagnostic laparoscopy may be considered to evaluate positioning of the mesh and tacks once nonneuropathic sources are ruled out and less invasive means of treatment have failed. Similarly, in the case of an open hernia repair, resection of the nerves of the groin (i.e., resection of the ilioinguinal, iliohypogastric, and genital branch of the genitofemoral nerves [triple neurectomy]) and the mesh may be necessary.

regression model based on a patient's bilirubin, creatinine, and international normalized ratio (INR). Hence, the MELD score was validated as the best single predictor of perioperative mortality in cirrhotic patients. A MELD score of 7 or less is associated with a mortality of 5.7%, 8 to 11 with a mortality of 10.3%, and 12 to 15 with a mortality of 25.4%. Mortality after 7 days is likely best predicted by the MELD score.

12. The most precise single predictor of operative mortality at 30 days in patients with liver cirrhosis is the

Items 13-14 A 42-year-old woman experiences right upper quadrant pain repeatedly after a fatty meal. A right upper quadrant ultrasound demonstrates an 8-mm lesion in her gallbladder. The lesion is fixed to the gallbladder wall and has no acoustic shadowing.

A. Acute Physiology and Chronic Health Evaluation IV (APACHE IV) score. B. American Society of Anesthesiologists (ASA) score. C. Child-Pugh class. D. Frailty index. E. Model of End-Stage Liver Disease (MELD) score.

ANSWER:

13. This lesion is most likely

E

The Acute Physiology and Chronic Health Evaluation IV (APACHE IV) score is calculated based on several clinical variables, which are inputted in a computer-generated logistical regression equation. It can help predict hospital mortality and length of stay in critically ill patients. The Frailty index helps providers assess an individual's physiologic resilience and general health condition. Frail patients have increased mortality compared with their more robust counterparts. No evidence suggests that the frailty index alone is superior in predicting mortality compared with the Model of End-Stage Liver Disease (MELD) score alone.

A. B. C. D. E. 14.

a cholesterol polyp. an inflammatory polyp. an adenoma. an adenomyoma. an adenocarcinoma. The most appropriate next step in management is

A. B. C. D. E.

repeat ultrasound in 6 months. CT scan. PET-CT scan. laparoscopic cholecystectomy. open cholecystectomy.

ANSWERS: A, D Gallbladder polyps can be seen on ultrasounds of the right upper quadrant and are noted in 4 to 7% of the population. When ultrasound is used to evaluate patients with abdominal pain, these polyps are increasingly encountered. Most polyps are quite small and usually benign. However, some polyps may be neoplastic in nature and may even harbor invasive cancer. In a series of 172 patients who had a resection for a gallbladder polyp, the most common diagnosis was a cholesterol polyp, noted in 63% of the cases. Of concern, however, is that nearly 8% of these lesions harbored a malignancy. Other series have demonstrated a similar distribution of polyps, with cholesterol polyps being the majority. Other less common polyps include inflammatory polyps, adenomas, and adenomyomas.

The American Society of Anesthesiologists (ASA) score is used to assess anesthetic risk in preoperative patients. It has no objective modification in relation to liver disease. Nevertheless, studies have shown that in cirrhotic patients, the ASA score accurately predicts mortality within 7 days of major surgery. None of these scores is designed to assess operative risk in patients specifically with liver failure. Higher accuracy is achieved using scores designed and validated specifically for perioperative risk assessment and risk stratification in cirrhotic patients. The Child-Pugh class uses subjective as well as objective criteria to assess perioperative mortality in cirrhotic patients. Specifically, a patient's serum bilirubin, albumin, prothrombin time, and degree of encephalopathy and ascites are assessed. Based on these values, a Child-Pugh class (A, B, or C) is assigned and perioperative mortality is estimated at10% for Child A, 30% for Child B, and 76 to 82% for Child C.

Because of the risk of malignancy, all gallbladder polyps require further evaluation. Most polyps containing cancer occur in patients older than age 50, are generally larger than 10 mm, and are usually symptomatic. Other risk factors for cancer include the presence of gallstones and a solitary polyp. In fact, cancer is almost exclusively found in patients with a solitary polyp. Multiple polyps are usually indicative of a benign process. Because of the risk of malignancy, all patients with polyps larger than 10 mm should undergo a

The MELD score addresses the drawbacks of the Child-Pugh score (has no subjective criteria, has weighted variables, and does not rely on arbitrary cut off values). It consists of a linear

44

cholecystectomy. A laparoscopic approach is reasonable for polyps in the 10-mm range, but if the suspicion of a gall bladder cancer is very high (i.e., with larger lesions), an open procedure should be considered. Smaller (i.e., <10 mm) polyps in patients who are asymptomatic can be followed with serial ultrasound, especially if there are no other risk factors present (i.e., age >50, solitary polyp, presence of gallstones). With the presence of any of these risk factors, cholecystectomy can be considered in low-risk surgical candidates but is not mandated. In low-risk asymptomatic patients, polyps smaller than 10 mm can be monitored by serial ultrasound to monitor for a change in size. These polyps are usually benign and rarely change in size. The exact interval of the ultrasound and when to stop monitoring is not fully defined, but it seems reasonable to obtain several scans to be confident there is no growth in the polyp. Patients with symptoms should undergo cholecystectomy regardless of polyp size and often have resolution of their symptoms. In this scenario, the polyp is smaller than 10 mm in a younger patient. Without symptoms, a repeat ultrasound in 6 months would be reasonable. In a patient with symptoms consistent with biliary colic, surgery would be recommended. Because this is a low-risk lesion, no additional imaging is needed, such as a CT scan or a PET-CT scan, which may be helpful if cancer is suspected. Similarly, an open approach is not required because the risk of an advanced malignancy is quite low. However, the surgeon should make an effort to prevent perforation during the dissection.

Figure 15.2

ANSWER:

E

Postoperative nausea and vomiting are not uncommon after major abdominal surgery. Most of these patients will have an adynamic ileus, which continues to be a major contributor to hospital length of stay. Some patients will have an early postoperative bowel obstruction, the majority of which will resolve with supportive care, including a nasogastric tube and intravenous fluids. Because both ileus and an early bowel obstruction are managed similarly, distinguishing between them is not clinically important. However, a subset of patients experience an early bowel obstruction secondary to incisional hernias. In these patients, the bowel within the hernia is at risk for strangulation; therefore, urgent surgery is required. Port site hernias are particularly troublesome given the narrow neck at the hernia site. Although ports 5 mm and smaller are rarely closed, it is advisable to close the larger port sites, especially those 10 mm and larger. Robotic ports are often 8 mm in size and not routinely closed. However, there are reports of early bowel obstructions through these sites.

15. A 52-year-old woman experiences nausea and vomiting 5 days after a robotic anterior resection for a rectosigmoid cancer. She is afebrile with normal vital signs. Her abdomen is distended but not tender. Her white blood cell count is 10,000/mm3 (3600-11,200/mm3). A CT scan is shown in figures 15.1 and 15.2. In addition to a nasogastric tube, the most appropriate next step in her therapy is A. observation. B. mu-receptor antagonist. C. motility agents. D. parenteral nutrition. E. reoperation.

In this patient, the CT scan is consistent with a port site hernia containing obstructed small bowel (figure 15.3). Because of the risk of strangulation, surgery is indicated. Alvimopan is a mu-receptor antagonist that selectively blocks the effects of opioids on the gastrointestinal tract and is used to decrease the length of ileus. It is often a component of an enhanced recovery program. Motility agents are used for patients with a prolonged ileus if there is concern for an underlying motility

Figure 15.1. 45

The modality of choice is liver ultrasound, which is inexpensive, safe, and reasonably sensitive. Ultrasound can be used every 6 months to monitor those patients at high risk. With this regimen, HCC can be detected early, improving overall survival. New nodules noted on an ultrasound require further investigation. Nodules smaller than 1 cm should have a repeat ultrasound in approximately 4 months to determine whether the nodule is growing. Lesions that are 1 cm or larger should have a 4-phase dynamic CT scan or an MRI. If the scans are not definitive, a biopsy is recommended. Once a diagnosis is made, treatment options depend on the size and number of lesions and the hepatic reserve of the patient. Resection is possible for small lesions in patients with wellcompensated disease. Liver transplantation is the standard of care for patients with more advanced liver disease. Other options include radio frequency ablation, transarterial chemoembolization, and sorafenib. Alpha-fetoprotein is a tumor marker for HCC but is not always elevated in patients with active disease, so it is not effective for screening. Both CT scan and MRI are used once there is a suspicious nodule but are not used for screening purposes. Liver function tests are not effective for HCC screening.

Figure 15.3. problem, although their routine use is no longer standard practice. Parenteral nutrition is indicated for patients who have a prolonged ileus but generally not this early in the postoperative period. Continued observation would be appropriate for an ileus or an early obstruction without evidence of a hernia but would not be appropriate for this patient.

17. A 68-year-old man presents 6 weeks after an exploratory laparotomy for unresectable pancreatic cancer. His abdominal pain, which was present preoperatively, was initially reasonably well controlled with increasing doses of both immediate release and long-acting oral morphine. However, he is now requiring increasing doses of oral morphine (>400 mg/24 hours), with attendant worsening side effects of constipation and confusion, as well as increasing abdominal pain. What is the best way to manage his pain?

16. A 52-year-old man has chronic hepatitis B without evidence of cirrhosis. What is the most appropriate way to screen for hepatocellular carcinoma? A. B. C. D. E.

Liver function tests Alfa-fetoprotein Ultrasound CT scan MRI

ANSWER:

A. Intrathecal placement of a programmable implantable drug delivery system B. Intravenous opioids via patient-controlled analgesia C. Transdermal fentanyl D. Methylnaltrexone E. Image-guided celiac plexus block

C

Hepatocellular carcinoma (HCC) is increasing in incidence worldwide and continues to be a major cause of cancerrelated, deaths. Liver tumors account for approximately 7% of all cancers worldwide, the majority of which are HCC. Most HCC is diagnosed in patients with chronic liver disease, especially those with cirrhosis. The most common risk factor is hepatitis B and C infection resulting in chronic liver injury. Alcohol and nonalcoholic steatohepatitis (NASH) are also known risk factors but are not as common as viral hepatitis. Patients with known risk factors for HCC should be screened. It is widely accepted that patients with well-compensated cirrhosis should have HCC screening. Similarly, patients with advanced cirrhosis who are awaiting liver transplantation should be screened. Somewhat more controversial are patients with chronic hepatitis B infection without evidence of cirrhosis. However, current guidelines recommend screening for patients with chronic hepatitis B infection even without evidence of cirrhosis.

ANSWER:

E

Pancreatic cancer causes severe pain in 50 to 70% of patients. The pain is multifactorial, with contributions from pancreatic duct obstruction/hypertension, neural invasion, and gland inflammation. Current management of pancreatic pain follows the World Health Organization 3-step ladder schema, beginning with nonopioid analgesics such as nonsteroidal anti-inflammatory drugs, and eventually progressing to opioid analgesics. When escalating doses of opioids are required, or when untoward side effects of opioid treatment begin to affect quality of life, alternative pain management regimens are indicated.

46

The pain in pancreatic cancer is thought to be transmitted through the celiac plexus. Targets for celiac axis destruction are the splanchnic nerves, which cross the diaphragm, enter the abdominal cavity, and form the celiac plexus, located around the celiac artery. Block can be accomplished through imaging (CT scan or endoscopic ultrasound) or intraoperatively. A celiac plexus block is safe and tremendously effective, with significant advantages over conventional opioid therapy for up to 6 months, including dramatic decreases in opioid use and attendant side effects. One study showed no major complications or deaths associated with the procedure.

nodes along the cystic duct, common bile duct, hepatic artery, and portal vein. Prophylactic port site excision offers no benefit for survival or recurrence. Surgical management is critical because, unlike chemoradiation, it can be curative. Simple cholecystectomy followed by adjuvant chemotherapy results in inferior survival. A positive cystic duct margin is an indication for common bile duct resection, but otherwise routine common bile duct resection does not improve outcomes. Because the incidence of lymph node involvement in a T2 cancer is relatively high and the surgical margin is close, observation would risk leaving disease behind.

Implantable programmable intrathecal drug delivery devices are also very effective in this setting, with 88.5% of patients experiencing a positive effect, including a minimum reduction in pain scores of 27% and a 50% reduction in drug toxicity. These pumps are extremely expensive, with total costs exceeding $50,000.

19. A 65-year-old Child Class A cirrhotic with known hepatocellular carcinoma presents with abdominal pain, a heart rate of 110 beats per minute, and a systolic blood pressure of 90 mm Hg. A CT scan shows tumor rupture and hemoperitoneum. Which of the following statements is true?

Patient-controlled analgesia with opioids would not eliminate the side effects of opioid use and is costly, averaging $4000 per month in the outpatient setting.

A. It is not amenable to curative resection. B. Urgent hepatic resection improves overall survival. C. Child Class A and B patients have equivalent overall survival. D. Tumor rupture with hemoperitoneum does not affect long-term survival E. Urgent resection of the lesion has 30-day mortality equivalent to transarterial embolization.

Transdermal fentanyl is an excellent choice as well but carries the risks of opioid-associated side effects and cost; patches range from $50 to $200 each. Methylnaltrexone is used for opioid-induced constipation and neostigmine-resistant Ogilvie syndrome. Although it would likely help this patient's constipation, it would not help alleviate his worsening pain.

ANSWER:

Rupture of a hepatocellular carcinoma leads to hemoperitoneum and can look very disturbing on imaging. It does not necessarily worsen long-term survival, however. Long-term survival is still based on the stage of the tumor at the time of rupture. These patients will often present hemodynamically abnormal. Their cirrhosis increases their morbidity and mortality. As such, urgent surgical resection of the tumor to control the bleeding has a high 30-day mortality, and patients with Class B cirrhosis have a worse prognosis than do those with Class A cirrhosis. Urgent resection does not improve overall survival from the tumor.

18. A 65-year-old woman undergoes a laparoscopic cholecystectomy for cholecystitis. Her final pathology shows an adenocarcinoma invasive into the subserosal soft tissue, less than 1 mm from the cauterized margin. A CT scan shows no residual or metastatic disease. The appropriate next step is A. port-site excision. B. chemoradiation. C. resection of liver segments IVb/V with portal lymphadenectomy. D. common bile duct resection with portal lymphadenectomy. E. observation.

ANSWER:

D

The best immediate therapy is transarterial embolization (TAE) to control the hemorrhage and allow resuscitation of the patient TAE has a lower mortality than urgent resection. This approach will control the bleeding, allowing the patient to recover before undergoing an elective resection if the tumor is amenable to it. The rupture does not necessarily mean advanced disease making it unamenable to resection.

C

Incidental gallbladder cancer found on histologic examination of the specimen after laparoscopic cholecystectomy is the most common presentation of this malignancy. Depth of invasion correlates with overall survival. Adenocarcinoma that invades the subserosa is a T2 tumor. These tumors require reoperation to resect the liver segments IVb/V and clear locoregional lymph nodes. Before resection, the peritoneal cavity should be evaluated for metastatic disease. Regional lymphadenectomy includes

20. A 51-year-old man presents with severe, boring, epigastric pain and nausea. His bilirubin is 1.2 mg/dL (0-0.3 mg/dL), his amylase is 500 U/L (23-85 U/L), and his lipase is 750 U/L (10-140 U/L). He does not drink alcohol. His right upper quadrant ultrasound is seen in figure 20.1. His symptoms resolve after 2 days of bowel rest. He tells you that

47

this is his second episode of this event and asks if there are any surgical options. What is the next best step in management? A. B. C. D. E.

their pancreatitis, 33% experienced another episode of pancreatitis. In the patient who is a good operative candidate and has had recurrent episodes of pancreatitis, the best option is for cholecystectomy before discharge from the hospital.

Magnetic resonance cholangiopancreatography Observation Cholecystectomy before discharge Cholecystectomy 6 weeks later Endoscopic retrograde cholangiopancreatography

21. The recommended treatment for a 5-cm gastrointestinal stromal tumor on the greater curvature of the stomach is A. B. C. D. E.

serial endoscopic surveillance. endoscopic submucosal resection. wedge resection. wedge resection with regional lymphadenectomy. total gastrectomy.

ANSWER:

Gastrointestinal tumors (GISTs) arise from the intestinal cell of Cajal in the subepithelium of intestinal organs, most commonly the stomach and small intestines. Surgical resection to grossly and histologically negative margins (RO resection) is the standard of care for GISTs. Given that GISTs typically do not have extensive intramural spread, complete organ resection (total gastrectomy) is not indicated. Wedge resection to negative margins is sufficient. The surgeon should avoid violating the tumor pseudo capsule and tumor spillage with this procedure.

Figure 20.1.

ANSWER:

C

In experienced hands, a laparoscopic approach is feasible for tumors in favorable locations, such as the greater curvature of the stomach. The incidence of nodal metastasis is very low, and, as such, regional lymphadenectomy is not indicated. Serial endoscopic surveillance is not an appropriate therapy for GISTs. Less invasive techniques, such as endoscopic submucosal resection around the pseudo capsule, are gaining acceptance. Currently, these techniques are feasible only for tumors smaller than 5 cm but are associated with a higher rate of microscopically positive margin (R1 resection).

C

Even though gallstones are not seen on his ultrasound, sludge should be treated like gallstone disease (figure 20.2). Microlithiasis is believed to be present and to be the cause of his symptoms. Esophagogastroduodenoscopy with ultrasound can be used to confirm microlithiasis, but it is probably unnecessary. Magnetic resonance cholangiopancreatography and endoscopic retrograde cholangiopancreatography would show other etiologies like biliary cystic disease or pancreas divisum, but these are unlikely to cause pancreatitis.

22. A 62-year-old man who abuses alcohol presents with tense ascites and a large umbilical hernia. The ascites has persisted despite maximal medical therapy and repeated paracentesis. The next step should be A. B. C. D. E.

continued observation. urgent umbilical hernia repair. transjugular intrahepatic portocaval shunt (TIPS). distal splenorenal shunt liver transplant.

ANSWER:

Figure 20.2

C

Cirrhotic patients with ascites have an estimated 40% lifetime risk of developing an umbilical hernia due to increased intraabdominal pressure, attenuation of abdominal wall musculature, and other factors. Nonoperative management and "watchful waiting" were once advocated as means of avoiding the high risk of postoperative liver failure. However,

The decision to operate versus observe is based on a 3-fold incidence of recurrent pancreatitis with observation versus patients undergoing cholecystectomy. The timing of the cholecystectomy is influenced by the degree of pancreatitis. In the case of mild to moderate pancreatitis, there is a benefit to early intervention. In patients who waited 6 weeks after

48

due to the attendant risks of incarceration, strangulation, rupture, and infection, herniorrhaphy is now considered the standard of care in affected patients. Optimal timing is crucial, and improved outcomes depend on aggressive control of ascites before elective intervention.

The overall malignancy risk is very low; therefore, surveillance imaging in patients should be limited to those who can benefit from pancreatic resection if the lesion develops a neoplasm. Patients with pancreatic cysts that would not warrant immediate resection should undergo surveillance with CT scan or MRI every 1 to 2 years as long as they are otherwise potential surgical candidates. PET-CT scan has no role in the diagnosis or investigation of pancreatic cysts. Measuring the carcinoembryonic antigen level in cyst fluid is useful in differentiating mucinous from nonmucinous cysts. However, the absolute carcinoembryonic antigen level is not predictive of current or future risk of malignancy. Although testing for molecular alterations in pancreatic cyst fluid is currently available, no studies identify how the test performs in predicting outcomes with regard to need for surgery, surveillance or interventions leading to improved survival.

In this scenario, the patient has no indication for urgent hernia repair. Medical correction of the ascites represents the first line of therapy. In patients with ascites refractory to these measures, the use of transjugular intrahepatic portocaval shunt (TIPS) improves intermediate and long-term survival compared with repeated paracentesis. TIPS followed by semielective repair appears to be associated with improved outcomes in patients with umbilical hernias and ascites. During follow-up, some patients may require liver transplantation, but transplantation is not indicated for ascites alone. Distal splenorenal shunt should not be used, because it tends to aggravate ascites.

24. Which of the following statements regarding splenic artery aneurysms is true?

23. A 70-year-old man presents after a fall. He undergoes CT scan (figure 23.1), which demonstrates a 2-cm cystic lesion without mural nodularity in the pancreas. He denies any prior symptoms. He has no history of weight loss or prior surgery. The next step should be A. B. C. D. E.

A. Infection is the most common cause. B. Pregnancy mandates elective repair. C. Splenic aneurysms rupture more commonly than other splanchnic aneurysms. D. 1.5-cm diameter splenic aneurysms have an increased risk of rupture. E. Mortality rate for splenic artery aneurysm rupture is 10%.

genetic testing of cyst fluid. endoscopic fine needle biopsy. quantify CEA level in cyst fluid. repeat CT evaluation in 1 year. PET-CTscan.

ANSWER:

B

Splenic artery aneurysms, while rare, have a higher than 25% mortality rate if ruptured. Risk of rupture depends on anatomic location, size larger than 20 mm, growth rate, and underlying disease, especially pregnancy and liver transplantation. Splenic artery aneurysms account for 60% of all visceral aneurysms. Although they are more than 4 times more common in women, risk of rupture is increased only during pregnancy. Pregnancy mandates elective repair. Etiologies of a splenic artery aneurysm include atherosclerosis (32%), medial degeneration (24%), abdominal trauma (22%), high blood flow during pregnancy, portal hypertension, connective tissue disorders such as Marfan and Ehlers Danlos syndromes, vasculitis, neurofibromatosis, and antiphospholipid syndrome.

Figure 23.1.

ANSWER: D 25. Which of the following techniques for an open repair of a 4-cm ventral incisional hernia is most likely to achieve a durable reconstruction?

Approximately 15% of patients undergoing abdominal MRI or CT scan for other indications are diagnosed with asymptomatic pancreatic cysts. The incidence of pancreatic cysts increases with age and may be as high as 25% in patients older than 70 years. Cyst size greater than 3 cm increases the risk of malignancy approximately 3-fold, and the presence of a solid component on cross-sectional imaging increases the probability of a malignancy approximately 8-fold.

A. B. C. D.

49

Onlay placement of mesh after primary defect closure Inlay placement of mesh without primary defect closure Sublay placement of mesh with primary defect closure Underlay placement of mesh with primary defect closure

E. Primary repair without mesh

ANSWER:

durable based on review of the literature. In addition, it has the added benefit of mesh separation from intraperitoneal content by the posterior sheath closure compared with underlay repair and can protect the mesh from contamination. However, the various mesh repairs have not been robustly compared in prospective randomized trials, and durability is also likely to be related to the quality of the dissection, placement of suture, and degree of tension on the repair.

C

Comorbidities and obesity in incisional ventral hernia patients undoubtedly contribute to poor long-term results after repair, but technical decisions contribute substantially to the outcome. A large body of evidence supports the use of mesh repair, particularly for a hernia of this size. Onlay repair places mesh superficial to the closed fascia, with the creation of devascularizing skin flaps to allow placement. Inlay repair is used when the fascia is not reapproximated and involves interposition of the mesh, sutured to fascial edges, within the hernia defect. This repair does not reconstruct the functional abdominal wall and does not allow for much tissue integration. Sublay mesh is superficial to the posterior rectus sheath but deep to the rectus musculature (also known as a retrorectus repair). This technically challenging repair requires dissection of this space, suture closure of the posterior sheath, mesh placement with laterally placed Ustitches through the anterior sheath, and closure of the anterior sheath. An underlay placement is intraperitoneal, with mesh located deep to the peritoneal lining and secured with laterally placed U-stitches through multiple fascial layers (figure 25.1).

Figure 25.1. Mesh location: A. inlay, B. onlay, C. underlay, D. sublay

The sublay mesh repair has the lowest rate of recurrence compared with onlay and inlay repairs and is likely to be most

50

Alimentary Tract Part I

Mesenteric angiography requires a bleeding rate of 0.5 mL/minute to detect extravasation into the gut, which is significantly higher than scintigraphy. It also does not define etiology or other nonbleeding stigmata. The procedure is risky in patients with contrast dye allergy or renal insufficiency. Combined with embolization, it can be therapeutic. Superselective embolization is preferred to reduce ischemic complications. Initial hemostatic rates approach 96%, with a 22% early rebleeding rate and a 17% major complication rate, including bowel ischemia, hematoma, femoral artery thrombosis, contrast dye reaction, acute kidney injury, and transient ischemic attack. Angiography is reserved for patients who continue to bleed massively despite resuscitation. More commonly, the bleeding rate will slow and CTA can be used to localize bleeding followed by angiography and embolization of bleeding locations. CT angiography is fast, allows for accurate arterial images, and can reveal contrast extravasation into any portion of the gastrointestinal tract In many institutions, this study has replaced scintigraphy and is proving to be better than angiography alone.

ITEMS 1-30 For each question, select the best possible response. 1. A 78-year-old previously healthy man presents with a 48hour history of painless hematochezia. After appropriate resuscitation, he continues to have intermitted hematochezia. A nasogastric tube aspirate contains bile. What is the next most appropriate step in his management? A. B. C. D. E.

Mesenteric angiography Tagged red blood cell scan Upper endoscopy Colonoscopy Exploratory laparotomy

ANSWER:

D

Lower gastrointestinal hemorrhage is an extremely common reason for hospitalization, particularly in the elderly. Hematochezia is defined as bleeding per rectum that is either bright red blood, clots, or burgundy stools. Most patients will stop bleeding spontaneously and can be worked up electively. For those who do not stop bleeding, efficient diagnosis and treatment of the hemorrhage are of paramount importance.

Surgery for lower gastrointestinal hemorrhage is usually not necessary, because most bleeding is either self-limited or successfully managed with medical or colonoscopic intervention. Indications for exploratory laparotomy include malignancy, diffuse hemorrhage unresponsive to medical intervention, and recurrent bleeding from a diverticulum.

In patients who continue to bleed over the first 24 hours of their hospitalization with a drop in hematocrit of at least 2 g/dL or a transfusion requirement of at least 2 units of packed red blood cells, urgent diagnosis and treatment are indicated. Risk factors for severe colonic bleeding include aspirin use, 2 comorbid illnesses, pulse greater than 100 beats per minute, and systolic blood pressure less than 115 mm Hg. Overall mortality is 2.4 to 3.9%, with independent predictors of inhospital mortality including age older than 70, intestinal ischemia, and 2 or more comorbid illnesses.

Nasogastric tube placement with aspiration of bile makes an upper gastrointestinal source less likely, but still possible. If colonoscopy is unrevealing, esophagogastroduodenoscopy would then be indicated.

2. Regarding screening for colorectal cancer in an averagerisk patient population,

Urgent colonoscopy is diagnostic and therapeutic in a patient with hematochezia.

A. stool testing for occult blood during a digital rectal exam is more accurate than home-based stool samples. B. fecal immunochemical tests are specific for blood originating in the colon. C. colonoscopy withdrawal time should be a minimum of 4 minutes to ensure adequate visualization of the mucosa. D. split-dose bowel preparation for colonoscopy results in poorer preparations and mucosal visualization. E. a multitarget stool DNA test has higher specificity and lower sensitivity than a fecal immunochemical test.

A bowel prep is useful but not always possible. The goal is to find the source of bleeding. Therapy can include epinephrine injection, thermal contact coagulation, argon plasma coagulation, hemostatic clips, or band ligation depending on findings. Bowel preparation increases visualization, increases diagnostic yield, and decreases the risk of perforation but should not delay the procedure. Early colonoscopy is associated with shorter length of stay, is most cost-effective, has a higher diagnostic yield than the other strategies in this situation, and is the procedure of choice.

ANSWER:

Nuclear scintigraphy detects acute bleeding at a rate of 0.04 mL/minute but does not identify other nonbleeding stigmata, does not provide an etiology for the bleeding, and is not therapeutic. It is only 66% accurate in locating a site of bleeding, and therefore is mostly useful as an adjunct in the workup of hematochezia.

B

Screening for colorectal cancer has evolved. The use of fecal occult blood testing is a methodology used in screening large populations for colorectal cancer. The standard procedure is for the patient to place a stool smear from 3 separate bowel movements on a guaiac card, which is then tested for a peroxidase reaction. False-positives are possible with the peroxidase reaction, such as ingestion of high-dose Vitamin

52

C. This home-based method has a much greater sensitivity than a digital rectal exam test performed in the hospital or physician's office.

also infected with H. pylori are at especially high risk of ulcer bleeding (6.13-fold). Cigarette smoking is a lesser factor also associated with gastroduodenal ulcers and likely potentiates the effects of NSAIDS and H. pylori.

To increase specificity of a stool-based test, the fecal immunochemical test (FIT) was developed. This test is specific for hemoglobin and is not influenced by other peroxidase reactions that result in false-positive results with standard guaiac testing. It also does not react with hemoglobin from upper gastrointestinal sources, making it specific for detecting blood in the colon.

Gastric adenocarcinoma, although uncommon, can present as a nonhealing ulcer. Bleeding is rare and usually presents as occult bleeding or anemia. Peptic ulcers, meanwhile, are responsible for nearly half of all acute upper gastrointestinal bleeding.

The multitarget stool DNA test (MSDT) examines a stool sample for several DNA abnormalities associated with colorectal neoplasms. This has been combined with FIT in commercial preparations. The MSDT with FIT yields a higher sensitivity but lower specificity for colorectal cancer than FIT alone, raising concerns for a higher false-positive rate. This test requires the collection of an entire bowel movement.

The relationship between ingested nitrates, nitrites, and nitrosamines and the risk of gastric cancer is complex. A large meta-analysis demonstrated that consumption of foods high in nitrates is associated with a decreased risk of gastric cancer, whereas high consumption of nitrites was associated with an elevated risk of gastric cancer. Neither, however, is specifically associated with bleeding ulcers.

Colonoscopy as a screening test has the advantage of being both diagnostic and possibly therapeutic. The quality of the bowel preparation has a direct effect on the utility of the colonoscopy. Various bowel preparations are used. Split-dose preparations with half of the prep administered the evening before and the second half administered on the morning of colonoscopy have consistently demonstrated that they are at least as good, if not superior, to day-before preps. Adequate insufflation and careful examination of the mucosa is paramount to achieve high sensitivity in detecting lesions. Because the best visualization is often achieved on withdrawal of the endoscope, minimal withdrawal time of 6 minutes is recommended as a quality metric for a complete colonoscopy.

4. A 48-year-old man presents with recurrent localized leftsided abdominal pain. His white blood cell count is 12,000/mm3 (3600-11,200/mm3), and CT scan of the abdomen reveals sigmoid diverticulitis with colon wall thickening and adjacent fat stranding. The patient is given oral antibiotics, and his symptoms resolve after 2 days. Subsequent colonoscopy is normal except for pandiverticulosis. He has had 2 similar episodes in the last 7 years. What is the most appropriate next step? A. B. C. D. E.

Observation Daily suppressive antibiotics Sigmoid colectomy with anastomosis Total colectomy with anastomosis Sigmoid colectomy with end colostomy

3. The greatest risk factor for bleeding peptic ulcer is A. B. C. D. E.

ANSWER:

gastric adenocarcinoma. nonsteroidal anti-inflammatory drugs. Helicobacter pylori infection. tobacco and alcohol use. consumption of foods high in nitrates.

ANSWER:

A

Patients with uncomplicated diverticulitis rarely require urgent surgical intervention. After symptom resolution, they often present to the office to discuss the pros and cons of elective sigmoid colectomy. In the past, the decision for colectomy was based on the number of attacks as well as the patient's age. As the understanding of diverticulitis evolved, elective surgery for diverticular disease declined significantly.

B

Historically, peptic ulcer disease was attributed to gastric acid hypersecretion or high emotional stress. The understanding of this disease has changed significantly with the recognition of Helicobacter pylori infection as a causative agent, as have the approaches to its treatment.

Diverticulitis is not a progressive disease, and the first attack is typically the worst attack. For the 13 to 23% of patients who do experience recurrence, the severity of disease usually mirrors that of the index attack. Therefore, elective colectomy should not be performed to prevent a future hypothetical emergency, because the risk of subsequent urgent colectomy with colostomy is less than 5% for patients with initially uncomplicated disease, which translates to 1 in 2000 patient years of follow-up.

Currently, H. pylori infection and the use of nonsteroidal antiinflammatory drugs (NSAIDS) are the greatest risk factors for gastric and duodenal ulcers. NSAID use alone is associated with the highest risk of ulcer bleeding (4.85-fold). H. pylori infection alone is associated with a lower risk (1.79fold). Individuals who are chronic users of NSAIDS who are

53

Age younger than 50 was previously thought to be an indication for elective surgery, because younger patients were presumed to follow a more aggressive disease pattern. However, this assumption is wrong, and young patients are currently treated the same as those older than 50. The number of attacks has also become less important, and the old rule of surgery after 2 to 3 attacks has been replaced by individualized decision-making for each patient. The decision for elective surgery is now based less on the number of attacks and more on the severity of attacks and the associated patient disability.

Current recommendations for antibiotic prophylaxis in cleancontaminated cases where there is preceding biliary colic within 30 days are for a single preoperative dose of cefazolin with no further doses to be administered after the wound is closed.

6. A 42-year-old man with Crohn disease is currently treated with corticosteroids, 5-aminosalicylic acid (5-ASA), and anti-tumor necrosis factor (TNF) therapy. He presents with fever and abdominal pain. Radiographic findings demonstrate an intra-abdominal abscess in the right lower quadrant. Percutaneous drainage is not possible, and open drainage is planned. Which of the following statements regarding perioperative considerations is true?

This patient had mild diverticulitis that resolved with outpatient therapy. He has demonstrated a relatively benign pattern of recurrence with 3 minor attacks in 7 years. Observation is the most appropriate next step. Daily suppressive antibiotics would be unnecessary and could predispose him to infectious colitides and other antibiotic side effects. Sigmoid colectomy would likely be offered in the future if he develops attacks that become more frequent or more bothersome. Total colectomy is not indicated for sigmoid diverticulitis, as even patients with pandiverticulosis will have a recurrence rate of less than 10% after sigmoid colectomy with coloproctostomy. Colectomy with end colostomy would be reserved for patients with pelvic sepsis who are not candidates for primary anastomosis.

A. Continuing corticosteroids will increase risk of abdominal complications. B. Anti-TNF treatment will delay wound healing. C. 5-ASA should be stopped at least 7 days before surgery. D. Nutritional status has no effect on postoperative outcomes. E. Starting thiopurine therapy before surgery will lower his risk of postoperative complications.

ANSWER:

In a prospective study of 209 patients with inflammatory bowel disease who were undergoing ileocecal resection, 35% of patients treated with corticosteroids in the 4 weeks before surgery experienced postoperative complications compared with only 19% of the untreated patients. This difference is mainly caused by a greater incidence of infection-related complications. These results are different from treatment with anti-tumor necrosis factor agents in the 3 months before surgery where there is no association with the incidence of overall postoperative complications.

5. A 72-year-old woman presents to the clinic after an episode of biliary colic due to choledocholithiasis. She was successfully treated with endoscopic retrograde cholangiopancreatography (ERCP), stone removal, and sphincterotomy. You are consulted to perform a laparoscopic cholecystectomy scheduled 1 week after the ERCP. She has no allergies. What is the recommended antibiotic for perioperative prophylaxis? A. B. C. D. E.

Cefazolin No antibiotics Vancomycin Gentamicin Ampicillin-sulbactam and metronidazole

ANSWER:

A

Findings in a prospective study showed an increased risk of abdominal septic complications in patients using thiopurines within 6 weeks of surgery who underwent a primary anastomosis (16% for thiopurine patients vs 6% in the control group). The risk of septic complications for patients prescribed thiopurine therapy increases to 24% if the patient has preoperative intra-abdominal sepsis or undergoes colocolonic anastomosis compared with only 4% when neither of these risks factors is present

A

Surgical site infections occur in the deep organ space, wound, or incision site after surgical interventions. Along with full body preoperative washing with soap or an antiseptic agent and skin preparation with an alcohol-based agent, preoperative antibiotic administration within 60 minutes before the surgical incision is an important factor in decreasing the risk of surgical site infections. The morbidity and cost of surgical site infections is substantial, thus using recommended guidelines to decrease their occurrence should be a priority. The case described is a clean-contaminated but high-risk biliary case given the previous symptomatic choledocholithiasis and instrumentation of the biliary tract with endoscopic retrograde cholangiopancreatography.

5-aminosalicylic acid (5-ASA) medications work directly on the bowel lining for patients with inflammatory bowel disease. These are distinct from aspirin (acetylsalicylic acid; ASA) and do not need to be stopped before surgery. Preoperative hypoalbuminemia is a strong predictor for postoperative morbidity and mortality. Hypoalbuminemia is associated with active inflammation, sepsis, and malnutrition.

54

7. A 52-year-old man presents with dysphagia and a longstanding history of gastroesophageal reflux disease. Barium swallow is shown in figure 7.1. Upper endoscopy shows a narrowing from 38 to 40 cm and esophagitis. A standard adult endoscope cannot be passed. After a dilation, the scope can be advanced and multiple biopsies are taken. Biopsies demonstrate reactive squamous mucosa with mild chronic and focal acute inflammation. At an office visit 2 weeks later, he is maintaining his weight but cannot eat solid food. The next step in management is A. B. C. D. E.

gastrostomy tube placement. repeated mechanical dilation. laparoscopic Nissen fundoplication. laparoscopic Heller myotomy. esophagectomy.

Figure 7.2. Dilation can be accomplished with mechanical bougies (mercury or tungsten filled, such as Maloney) or guidewireassisted polyvinyl bougies (Savary-Gilliard). Balloon dilators are passed through the scope, with or without a guidewire, and inflated with water or contrast. There is no clear advantage of one type over the other. Risk of perforation is minimized when the "rule of 3" is followed: do not increase the dilation diameter by more than 3 mm per session. This patient is maintaining his weight and has had only one dilation, so feeding tube placement would be premature. Furthermore, feeding tube placement may be suboptimal if esophageal resection may be required in the future, because it may compromise the stomach for use as an esophageal conduit. Laparoscopic Nissen is a consideration once the stricture is successfully dilated and GERD is the confirmed diagnosis. Heller myotomy is indicated for achalasia and would have no effect on a luminal stricture. Esophagectomy is the "last resort" for a refractory or recurrent stricture after all other options have been exhausted

Figure 7.1.

ANSWER:

B

The clinical scenario describes a difficult or potentially refractory reflux-associated esophageal stricture. Gastroesophageal reflux disease (GERD) is the cause of 60 to 80% of benign strictures. Malignancy must be considered and explored, but if absent, management of benign strictures is dilation therapy with either bougies or balloons.

8. A previous healthy 45-y ear-old man presents with 2 weeks of right lower quadrant pain. He is febrile, has a white blood count of 18,000/mm3 (3600-1 mm3), and is tender in his right lower quadrant. A CT scan shows a 6-cm fluid collection in his right lower quadrant consistent with an appendiceal abscess. After percutaneous drainage, which of the following statements is true?

Most strictures require only 1 to 3 dilations, but approximately one-quarter will require additional sessions.

A. B. C. D. E.

Strictures that are more likely to be challenging are longer (>2 cm), angulated, irregular, or severely narrowed (figure 7.2). A stricture is refractory if a diameter of 14 mm cannot be achieved over 5 sessions at 2-week intervals. A stricture is recurrent if a satisfactory diameter cannot be maintained for 4 weeks once the target diameter of 14 mm has been achieved. The patient must be maintained on acid suppression therapy to prevent stricture recurrence. Up to 40% may still recur.

Antibiotics are not necessary. The risk of appendiceal cancer is 20%. Interval appendectomy should be performed at 6 weeks. Colonoscopy should be performed. The failure rate of nonoperative treatment is 25%.

ANSWER: 55

D

Patients diagnosed with complicated appendicitis and an appendiceal abscess are best managed with percutaneous drainage and antibiotics. Although failure rates ranging from 13 to 20% are reported for all patients with complicated appendicitis, these series include patients with both appendiceal abscess and phlegmon. In published series analyzing only patients undergoing percutaneous drainage for perforated appendicitis with an associated abscess, the failure rate of nonoperative treatment is less than 5%.

However, one group retrospectively examined the objective outcomes between the 2 techniques. The findings were consistent with other small studies suggesting that the POEM procedure is equivalent to laparoscopic Heller myotomy in ameliorating the symptoms of achalasia. The risk of longterm procedural complications and of recurrent disease was equivalent. Although there is concern about the development of gastroesophageal reflux disease (GERD) with POEM given that a concomitant antireflux procedure is not performed, this group found no difference in the development of GERD between the 2 techniques. In a 5-year retrospective review of 1680 patients undergoing POEM, another group reported a major adverse event rate of 3.3%, which is at least equivalent to other studies examining the efficacy of laparoscopic Heller myotomy for the treatment of achalasia.

For patients managed nonoperatively at initial diagnosis, a discussion of interval appendectomy typically occurs at the postoperative visit Although interval appendectomy was once considered routine, the complication rate (2-23%) must be weighed against the risk of recurrent appendicitis (8%); risk of recurrence may be higher with an appendicolith. Routine interval appendectomy at 6 weeks is not currently recommended.

10. A 26-year-old is evaluated for painless rectal bleeding. The patient reports prolapse of tissue on defecation that must be reduced manually. Anoscopy demonstrates internal hemorrhoids confined to one quadrant of the anal canal. Despite dietary modification, oral stool softeners, and topical agents, he has continued bleeding and prolapse that requires manual reduction. Compared with conventional open or closed excisional hemorrhoidectomy, hemorrhoidectomy using an electrothermal tissue-sealing device is associated with a lower rate of

If appendectomy is not performed, neoplasm must be excluded, particularly in patients older than 40. Follow-up colonoscopy is recommended in these patients. In all patients with appendicitis, the incidence of appendiceal tumors is 2%. Over the age of 40, the odds of concurrent malignancy is approximately 15%. Subgroups with an increased incidence of appendiceal malignancy include (1) patients older than 40 (15% incidence), (2) patients with an inflammatory mass (1029% malignancy rate), and (3) patients with other malignancies (10% rate of appendiceal malignancy on interval appendectomy specimen analysis).

A. B. C. D. E.

9. Which of the following statements is true regarding peroral endoscopic myotomy (POEM) in the treatment of achalasia compared with laparoscopic Heller myotomy with fundoplication?

ANSWER:

A. The risk of developing postoperative symptomatic gastroesophageal reflux is significantly higher. B. The risk of recurrent achalasia is higher. C. The risk of esophageal leak is lower. D. Successful resolution of symptoms due to achalasia is equivalent. E. The myotomy includes the circular and longitudinal muscle layers of the affected esophageal segment.

ANSWER:

incontinence. postoperative bleeding. overall complications. urinary retention immediate postoperative pain.

E

This patient has grade III hemorrhoids, which are unresponsive to medical therapy. Thus, excisional hemorrhoidectomy is indicated. Although hemorrhoidectomy is generally regarded as the most efficacious therapy in this circumstance, postoperative pain with the traditional excisional hemorrhoidectomy can be significant. Hemorrhoidectomy using an electrothermal sealing device decreases the postoperative pain associated with the traditional technique. The limited thermal spread associated with the use of this technology is postulated to contribute to the potential for reduced postoperative pain.

D

Peroral endoscopic myotomy (POEM) has become commonplace in the treatment of achalasia. The technique is performed through a forward-viewing endoscope. After creating a longitudinal mucosal incision and creating a submucosal tunnel into the proximal stomach, the endoscopist divides the circular muscle layer of the esophagus, leaving the longitudinal layer intact

In a Cochrane analysis comparing traditional hemorrhoidectomy to excision using an electrothermal sealing device, rates of incontinence, postoperative bleeding, overall complications, and urinary retention were equivalent between the 2 treatment groups. The hospital length of stay was also no different between the 2 techniques. However, the reported pain scores at the first day after surgery were significantly lower in the electrothermal sealing device group, which persisted for the first week after surgery. This advantage dissipated by day 14 after surgical intervention.

To date, no prospective randomized trials comparing POEM with laparoscopic Heller myotomy have been published.

56

Patients undergoing hemorrhoidectomy using an electrothermal sealing device returned to work earlier than the traditional surgery cohort.

hemoglobin is 13.3 g/dL (12-15.5 g/dL). No adenopathy is present on imaging. Biopsy confirms squamous cell carcinoma. What feature of her presentation portends a poorer prognosis? A. B. C. D. E.

11. A 54-year-old woman develops an enterocutaneous fistula after a complex abdominal wall reconstruction. Workup demonstrates a mid-jejunal location, and output is 700 mL/day, despite taking nothing by mouth. The recommended first-line pharmacologic therapy to control her output is A. B. C. D. E.

ANSWER:

omeprazole somatostatin. codeine. clonidine. pancreatic enzymes.

ANSWER:

Female sex Tumor size G Distance from the dentate line Extent of circumferential involvement Tumor ulceration

B

Ever since Nigro's successful implementation, chemoradiotherapy with 5-fIuorouracil and mitomycin C has been the first-line treatment for anal carcinoma. Although effective, with complete remission rates approaching 90%, locoregional recurrences with metastatic spread do occur. Risk factors present at the time of diagnosis associated with recurrence include tumor size greater than 5 cm and more than two-thirds involvement of the anal canal circumference. These 2 factors are also associated with poorer overall prognosis. Additional identified predictors of decreased survival include male sex, presence of nodal disease, and hemoglobin less than 13 g/dL. Neither tumor ulceration nor distance of the tumor from the dentate line are associated with decreased survival.

A

Enterocutaneous fistulas (ECFs) can substantially increase morbidity after abdominal surgery, especially in the setting of high output (i.e., >500 mL/day). Such high output, along with fistula length less than 2 cm, lateral bowel wall location, the presence of multiple fistulas, and adjacent inflammation, are poor prognostic indicators for spontaneous closure of an ECF. Typically, the jejunum or ileum are the site of high output ECFs, given the large volume of fluids produced each day in the upper gastrointestinal tract (i.e., 1500 mL from the mouth/stomach, 1000 mL from the pancreas, 1000 mL from the biliary system, and 2000 mL from the small bowel).

13. A 72-year-old woman presents with lower abdominal discomfort. Her medical history is significant for aortic regurgitation, renal dialysis, diabetes, hypertension, and a stroke that occurred 5 years ago. A plain film of the abdomen demonstrates a porcelain gallbladder. Which of the following statements is true regarding a porcelain gallbladder?

Fluid resuscitation, electrolyte replacement, sepsis control, nutritional support, and control of output/protection of skin are key tenets in the treatment of ECFs. In the setting of highoutput ECFs, control of the fistula output is paramount Oral intake of hypotonic fluid should be restricted to 1000 mL per day, followed by pharmacologic therapy. Useful agents include proton pump inhibitors (PPIs) and histamine 2 receptor antagonists to help decrease gastric secretions, antimotility drugs to slow transit, and somatostatin analogues to decrease gastric and pancreatic secretions.

A. It is a marker for gallbladder cancer. B. The porcelain appearance is caused by cholesterol deposits. C. Observation is recommended. D. No association exists with cholelithiasis. E. Prophylactic cholecystectomy is required.

Current algorithms recommend starting with the antidiarrheal loperamide and a PPI as initial therapy. If this regimen is ineffective, codeine is the next therapy. If refractory hypomagnesemia develops in the setting of PPI therapy, a histamine 2 receptor antagonist is a second-line alternative to the PPI. In patients with fat malabsorption, pancreatic enzymes are beneficial. Somatostatin analogues are not firstline therapies. There is little evidence that clonidine represents an effective agent, and it is not recommended as first-line therapy.

ANSWER:

C

A porcelain gallbladder has calcium deposits in the wall (figure 13.1). Two types of deposits are present based on the amount of calcification: selective mucosal calcification and diffuse intramural calcification. A prevalence of 0.1 to 0.2% is reported and an association with gallbladder cancer is suggested, with reported rates being as high as 25%. Given this association, in the past, prophylactic cholecystectomy was recommended for a patient with a porcelain gallbladder.

12. A 54-year-old woman presents with an ulcerative 5.5-cm mass 05 cm proximal to the dentate line involving one-third of the circumference of the anal canal. On workup, her

57

A. B. C. D. E.

intraoperative endoscopy. technetium-99m-labeled red blood cell scan. visceral angiography. CT angiography. repeat colonoscopy.

ANSWER:

D

Multidetector CT (MDCT) scan is a method to identify lower gastrointestinal bleeding sites. CT angiography has become the preferred initial study to evaluate a patient for lower gastrointestinal bleeding and should precede any attempt at angiography. CT angiography can detect bleeding at rates as low as 0.3 mL/minute, which is better than angiography and only slightly worse than tagged red cell scans. Using CT angiography before angiography improves the likelihood of angiography identifying a site of bleeding. CT angiography appears to be better than nuclear scanning before angiography and can be performed more quickly. Currently, CT angiography is used in patients who are presumed to have ongoing blood loss but who are not hypotensive. These patients may require ongoing resuscitation but do not require blood products. CT angiography is used to localize the site of bleeding; then angiography is used for angioembolization. Despite a larger dye load for patients when CT angiography precedes angiography, there is no increase in renal impairment. Overall sensitivity to detect the site of bleeding is reported at 100%. The massively bleeding patient is still a candidate for early visceral angiography.

Figure 13.1. Porcelain gallbladder showing calcium deposits in the wall.

The reason for this possible increased rate of cancer was explained by the inflammatory changes that occur with the calcium deposits. The type of deposits may relate to cancer risk. More extensive intramural deposits cause mucosal sloughing, which reduces the rate of adenocarcinoma while the selective calcification yields to a continued inflammatory stimulus. Thus, a stronger recommendation for prophylactic cholecystectomy is made for the selective mucosal calcification pattern in an asymptomatic patient. These patients commonly have complaints of abdominal pain, which prompts an evaluation. A plain film will usually be adequate to identify right upper quadrant calcifications. Sonography is sufficient to confirm the diagnosis in most patients. Cholelithiasis is reported in 95% of patients with a porcelain gallbladder.

Selective mesenteric angiography can detect bleeding at a rate of greater than 0,5 mL/minute in a patient with active gastrointestinal bleeding. The major advantage of this approach is that it offers a diagnostic and therapeutic option. Additionally, the bleeding site is identified, which can guide future surgical intervention. Success rates vary from 30 to 90%, based on patient selection. Higher rates are associated with hypotension and ongoing blood transfusion requirements.

Given the rarity of this condition, the natural history of the disease is unknown, but the risk of gallbladder cancer is not as high as previously thought. A recent analysis found a rate of 6% (0-33%) in patients with porcelain gallbladder compared with 1% (0-4%) in matched patients. Prophylactic cholecystectomy may be appropriate for healthy patients but an observational approach is reasonable in a patient with multiple comorbidities. This patient's comorbidities and her lack of specific symptoms related to her right upper quadrant discomfort attributable to her porcelain gallbladder make observation appropriate.

Once a bleeding site is identified, treatment with vasopressin infusion or embolization is possible. Vasopressin is infused intra-arterially at a dose of 0.2 units/ minute; after 30 minutes, anotherangiogram is done to assess the bleeding. The maximum dose is 0.4 units/minute, because doses above this are associated with increased complications, including myocardial ischemia. Although clinical success rates are high, rebleeding occurs when the infusion is stopped. Because of this experience, embolization has become the preferred approach, if available.

14. A 63-year-old man is admitted for lower gastrointestinal bleeding. He is hemodynamically normal and undergoes colonoscopy. Colonoscopy demonstrates blood throughout the colon but does not localize the bleeding. He continues to pass bright red blood per rectum and requires 4 units of packed red blood cells over 6 hours. The next most appropriate step to localize the site of bleeding is

Catheter-based techniques offer superselective embolization with autologous dot, absorbable gelatin, microcoils, and other agents. The goal is to place the material as close to the bleeding site as possible. The material should be placed in the bleeding terminal mural branches, which should reduce the risk of colon wall necrosis. If this type of superselective embolization cannot be done, surgical intervention is

58

recommended. Success rates as measured by cessation of bleeding are 80 to 90% and rebleeding rates are 10 to 15%. Superselective embolization complication rates are 0 to 9% for minor and 0% for major, including colon wall necrosis.

15. A 63-year-old woman presents with a small bowel obstruction. Admission CT scan suggests mid small bowel adhesive obstruction. This is her third admission for obstruction in the last 4 years. She had a hysterectomy 15 years ago. What is a predictor of failure of nonoperative management of her bowel obstruction?

Mesenteric angiography will successfully identify the site of bleeding when active bleeding is occurring, but in many series, this is present in only 50% of patients. Angioembolization typically stops bleeding acutely, but recurrent bleeding is common, contributing to an overall success rate of 75%. Negative angiography is also associated with ongoing bleeding and frequently requires surgical intervention.

A. B. C. D.

Fecalization of the small bowel on CT scan Anterior parietal peritoneal adhesion on CT scan Free peritoneal fluid on CT scan Visualization of contrast in the colon on plain films at 7 hours E. Lack of a "beak sign" on presenting CT scan

Colonoscopy is commonly used to identify the bleeding site in patients with a lower gastrointestinal bleed. It is typically done after the patient is believed to have stopped bleeding. Bowel preparation is usually not necessary, but a bowel preparation can help visualization. Diverticula can be visualized with blood clots or even active bleeding. At times, the diagnosis is based on seeing diverticula and no other source of potential blood loss. A bleeding source is usually discovered in 80 to 90% of patients with lower gastrointestinal bleeding having a colonoscopy within 24 hours of the acute bleed. A benefit of colonoscopy is that therapeutic interventions can be performed when a bleeding site is localized. These include epinephrine injections, bipolar coagulation, or endoclipping. Success rates in stopping the bleeding are 70 to 100%. Rebleeding can occur and merits another endoscopic attempt, unless it is massive and ongoing. Complication rates are similar to those reported with any therapeutic colonoscopy. However, repeating a colonoscopy that was unrevealing the first time is not recommended.

ANSWER:

C

The management of small bowel obstruction continues to evolve. Today both partial and complete obstruction are recognized, and surgeons understand that, in some circumstances, the sun indeed may rise and fall before operative intervention is necessary. Abdominal CT scan is frequently used to help determine management strategy. Water soluble contrast is often used as a diagnostic and therapeutic tool. In the multi-institutional study from the American Association for the Surgery of Trauma, evidence of Gastrografin in the colon even beyond 8 hours was predictive of successful nonoperative management. Fecalization of the small bowel (small bowel feces sign) may be an indicator of the need for surgical intervention. However, a review from the Mayo Clinic found the opposite to be true. Fecalization was predictive of a 5-fold decrease in surgical intervention. This is also seen in a recent radiology review from France. These authors found that the presence of 2 or more "beak signs" (defined as "a fusiform tapering of the bowel at the site of the obstruction") predicted the need for surgical intervention. Current CT scan guidelines suggest an adhesion posteriorly located is more likely to need surgical intervention compared with anteriorly located adhesion, Finally, free intraperitoneal fluid is more likely to be associated with surgical intervention due to concern for strangulation and bowel ischemia.

Operative intervention with endoscopy is not recommend. This would require opening the small bowel or colon and passing the endoscope in an attempt to localize a bleeding site. Although possible, the lack of identification during the previous colonoscopy makes success with this procedure as unlikely as the repeat colonoscopy. The sensitivity of technetium-99m-labeled red blood cell scintigraphy in detecting the presence of lower gastrointestinal bleeding is 20 to 95%. This method will identify bleeding rates as low as 0.1 mL/minute. Its drawback is that localization of the bleeding point is not always possible. False-positives for actual bleeding location are reported in 10 to 60% of studies. A retrospective, singlecenter study compared CT angiography with tagged red blood cell scintigraphy in patients with acute lower gastrointestinal bleeding. The study found no significant differences between the 2 modalities with regard to patients' average hospital length of stay, blood transfusion requirements, incidence of acute kidney injury, or in-hospital mortality. What was different was the accuracy of localizing the bleeding site. CT angiography was able to identify the site in 53% of patients compared with 30% for red blood cell scintigraphy (p = .008)

16. A S7-ve.li old obese man presents with 3 days of left lower quadrant abdominal pain with abrupt worsening today, Laparoscopic lavage, irrigation of the abdomen with 4 L of saline, and pelvic drain placement are performed for Hinchey class III diverticulitis. Which of the following statements is true comparing laparoscopic lavage and drainage (ULD) with resection and end colostomy? A. Reoperation rate at the index admission is higher with UUD. B. More patients in the UUD group have an ostomy at 12 months. C. More patients in the LLD group require drainage of additional abscesses.

59

D. Initial length of stay is similar for both groups. E. 12-month mortality is higher for UUD.

ANSWER:

ANSWER:

D

In 1886, Reginald Fitz published a large series of autopsy data that supported the idea of progressive inflammation of the vermiform appendix to perforation. This led McBurney in 1889 to publish his treatise on the surgical management of appendicitis. Of importance is that the original Fitz publication noted that 30% of autopsy specimens suggested the spontaneous resolution of appendicitis. This finding, of course, occurred well before the development of antibiotics.

C

The use of laparoscopic lavage and drainage (LLD) in the management of acute diverticulitis remains controversial. Given that the primary short-term goal for patients with Hinchey class III diverticulitis and those with purulent peritonitis is to control the acute sepsis, LLD still plays a role. Three recent European randomized controlled trials addressed this issue: the DILALA Trial, LADIES Trial, and the SCANDIV Trial.

Uncomplicated appendicitis is defined as evidence of acute inflammation without perforation or appendicolith. It makes up approximately 80% of all patients presenting with acute appendicitis. Interest in the utility of antibiotic therapy alone for uncomplicated appendicitis is high. There have been several randomized controlled trials and much experience with this strategy in Europe and a current ongoing trial in the United States (CODA Trial).

Patients with purulent diverticulitis who undergo LLD are more likely to require drainage of additional abscesses compared with those who undergo local resection Although this is of concern given the intent to control sepsis, it does not appear to adversely affect outcome. The rate of reoperation in this group was not increased. Fewer in the LLD group had colostomy at 12 months. Considering that 30 to 40% of patients with acute diverticulitis who have ostomy formation never have their colostomy closed, this is a relevant consideration in favor of LLD. Initial length of stay is consistently shorter in the LLD group, and 12-month mortalities are comparable.

The 1-year rate of need for appendectomy in the population of uncomplicated appendicitis treated with antibiotics alone is approximately 25%. In the APPAC trial (APPendicitis ACuta), the failure rate was 27.3%; other randomized trials have found similar rates. The index hospitalization failure rate was only 5.8%, and the complication rate was lower for the antibiotic-only group compared with those who had appendectomy (7% vs 22%).

Although LLD is not indicated in each patient with perforated diverticulitis, advantages include shorter length of stay and a decreased rate of ostomy formation for some patients. If sepsis can be controlled with LLD, this treatment affords the opportunity for elective surgical consideration.

In an economic subanalysis of the APPAC trial, the overall societal costs, including lost time for work at 1 year, were less for antibiotic treatment alone compared with initial index appendectomy ($5000 vs $8100). In both groups, diagnostic costs and medicines played a relatively small role, whereas productivity losses where the most significant component of the costs incurred.

17. A 15-y ear-old girl presents with 2 days of right lower quadrant pain. CT scan reveals uncomplicated acute appendicitis. Surgery is recommended, but the family is concerned about the cost because they have no health insurance. They are curious about antibiotic-only management of her appendicitis. Which of the following statements is true regarding the nonoperative management of uncomplicated acute appendicitis?

Although still controversial, evidence is compelling enough to consider antibiotics alone for the treatment of uncomplicated appendicitis, especially when cost considerations are of great importance to the patient.

18. A 63-y ear-old woman is diagnosed with a mid-rectal cancer. Metastatic workup is negative. Clinically, she has T3, N1 disease. She is given induction chemoradiation therapy. On follow-up evaluation, she has no evidence of her rectal disease. Which of the following statements is true regarding a complete response to induction therapy for rectal cancer?

A. Most patients who undergo nonoperative treatment with antibiotics require appendectomy within 1 year. B. Those patients who require appendectomy for failed nonoperative management have a high rate of complications. C. During the initial treatment period, patients who are treated with antibiotics are more likely to have complications than those who are treated with initial appendectomy. D. The overall societal costs for antibiotic therapy are lower than the costs for surgical management. E. Costs associated with laboratory testing, imaging, and medications account for the majority of cost incurred with nonoperative management.

A. The pathologic complete response rate is greater than the clinical response rate. B. If the tumor recurs, it will do so after 24 months. C. Nodal status is a predictor of recurrence. D. T status is not a predictor of recurrence. E. Patients with a higher pretreatment carcinoembryonic antigen level are likely to have recurrent disease.

ANSWER: 60

A

Neoadjuvant chemoradiation therapy is delivered to shrink and downsize low and mid rectal cancers to improve the chance of resection, reduce residual disease, and increase the number of patients with anal continence. It also provides complete pathologic response in 10 to 30% of patients. Additionally, some 15 to 20% of these with clinical residual disease have complete pathologic response after surgery.

20. Eosinophilic esophagitis A. B. C. D. E.

Complete pathologic responders, based on restaging biopsy, can be followed for evidence of recurrence. Studies show that initial T-stage is the best predictor of local recurrence, and neither nodal status nor higher pretreatment carcinoembryonic antigen portends a higher risk of local recurrence. Finally, although most local tumor recurrences occur within the first 12 months, many occur later. Therefore, local evaluation continues to be important in follow-up.

ANSWER:

Classic endoscopic findings include edema, decreased vascular mucosal markings, longitudinal furrowing, concentric rings (sometimes called "trachealization"; figure 20.1), white plaques (figure 20.2), crepe paper mucosa, and strictures or narrow caliber esophagus. Submucosal masses are not characteristic of EOE.

Young age Female sex Anemia Jejunal location of tumor 5 lymph nodes removed

ANSWER:

B

Eosinophilic esophagitis (EOE) is the leading cause of dysphagia and food impaction in children and young adults, with a peak age of incidence of 30 to 50 years. It is a local immune-mediated esophageal disease, with eosinophil predominant inflammation. There is no causal or temporal relationship with inflammatory bowel disease, connective tissue disorders, esophageal atresia, or hypereosinophilic syndromes. The presence of gastroesophageal reflux disease (GERD) symptoms and dysphagia are common in EOE. How EOE and GERD symptoms interplay is not clear, but it may be relative acid hypersensitivity induced by EOE.

19. A 40-year-old woman presents with anemia. Her stool is guaiac positive. Esophagogastroduodenoscopy and colonoscopy are normal. Capsule endoscopy demonstrates an ulcerated mid jejunal mass. She undergoes segmental small bowel resection inclusive of the mass and a primary anastomosis. Pathology demonstrates a stage I adenocarcinoma; 5 lymph nodes, which are removed and all of which were negative for malignancy; and clear margins. Which factor in this patient is associated with poor prognosis? A. B. C. D. E.

is associated with inflammatory bowel disease. presents with dysphagia and reflux symptoms. is characterized by submucosal masses on endoscopy. symptomatically improves with antireflux surgery. has a peak incidence in the elderly.

E

Figure 20.1.

Small bowel cancer is rare and comprises less than 5% of gastrointestinal malignancies. Primary malignant small bowel tumors include adenocarcinomas, neuroendocrine tumors (NETs), gastrointestinal stromal tumors (GISTs), and lymphomas. Small bowel adenocarcinomas comprise 30 to 40% of these malignancies. Bowel obstruction and gastrointestinal hemorrhage are the primary presenting symptoms of small bowel adenocarcinoma. When gastrointestinal bleeding is present, CT scan with enteroclysis or video capsule endoscopy is used to make the diagnosis, because conventional upper and lower endoscopy will be normal.

Figure 20.2.

Prognosis is poor. The only potential for cure is complete surgical resection (RO) with locoregional lymph node resection. After complete resection, lymph node invasion is the main prognostic factor. For jejunoileal tumors, a multivariate analysis linked mortality to advanced age, advanced stage, ileal location, the recovery of fewer than 10 lymph nodes, and the number of positive nodes. Sex and anemia were not considered.

Many patients have improvement of symptoms with proton pump inhibitors—up to 50% with 8 weeks of therapy. Topical steroids are also effective, administered as swallowed puffs

61

from inhalers, suspension, viscous slurry, or effervescent tablets of fluticasone or budesonide. There is no evidence that antireflux surgery has any role in managing EOE.

The indications for surgical treatment of gastroesophageal reflux disease are as follows: •

Extraesophageal complications of reflux such as cough, aspiration, or chest pain • Persistent symptoms despite compliant medical management • Adverse effects to antireflux medication • Adequate control of symptoms but desire for surgery due to social or lifestyle issues (e.g., cost of medication or not desiring lifelong medication regimens) • Structural complications of gastroesophageal reflux such as Barrett esophagus or peptic stricture This patient has a clear diagnosis of gastroesophageal reflux on his workup. His indication for surgery is his failure of medical therapy as well as the presence of Barrett esophagitis. Continued antireflux therapy is incorrect due to the duration of time he has failed proton pump inhibitor treatment. The other options do not treat the actual disease process. Endoscopic mucosal resection or ablation would not be indicated in a patient with a surgical indication for gastroesophageal reflux and Barret esophagitis. His Barret esophagitis without dysplasia requires endoscopic surveillance but at 3- to 5-year intervals, not 6 months.

21. A 35-year-old man presents with epigastric pain of 4 days duration. He says the pain was initially worse and has slowly been improving. His temp is 38.0°C and his heart rate is 95 beats per minute. His white blood cell count is 12,000/mm3 (3600-11,200/mm3). He has a normal mental status. A CT scan of the abdomen reveals a small, contained, anterior duodenal perforation. Which of the following statements is true regarding nonoperative management? A. His age confers a higher risk of failure. B. Helicobacter pylori treatment is unnecessary. C. Nonoperative treatment is best reserved for the very elderly moribund patient D. CT scan findings support a nonoperative approach. E. Duodenal stenting is necessary.

ANSWER:

D

The patient presented has a 4-day history of abdominal pain and is not overtly toxic. In fact, he reports that his pain is improving. The key radiologic finding is that of a small, contained, duodenal perforation. This patient is physiologically relatively well and has radiologic features consistent with successful nonoperative management. Half of patients with perforations will spontaneously seal and avoid surgery. Patients age 70 and older are at higher risk for failure of nonoperative management. The moribund elderly patient may undergo nonoperative management as an alternative to expected death in the operating room or from surgery. The salvageable older patient belongs in the operating room. Empiric therapy in patients with peptic ulcer disease is often used to dear the stomach of Helicobacter pylori. Duodenal stenting is not a treatment for benign duodenal perforation.

23. A 62-year-old man presents to the emergency department with abdominal pain and distention. He has not passed flatus or stool for the last 12 hours. He has no surgical history. Vital signs include a temperature of 37.8°C, heart rate of 82 beats per minute, and blood pressure of 130/85 mm Hg. On exam, he is tender in the right abdomen with distention but no guarding or rebound. A CT scan is performed (figure 23.1). The most appropriate intervention for this patient is A. B. C. D. E.

22. A 55-year-old man presents with confirmed gastroesophageal reflux. He has been on appropriate acid suppression therapy (e.g., proton pump inhibitors) for 6 months and admits his symptoms have not improved. An esophagogastroduodenoscopy shows Barrett esophagitis without dysplasia. What is the most appropriate next step? A. Continued antireflux treatment B. Repeat endoscopy in 6 months with 4-quadrant biopsy every 1 to 2 cm C. Fundoplication D. Endoscopic mucosal resection E. Endoscopic ablation of the Barrett mucosa

ANSWER:

C

62

intravenous neostigmine. colonoscopic decompression. sigmoid resection. ileocecectomy. loop ileostomy.

Figure 23.1 Figure 23.2. ANSWER:

D

Because of the risk of ischemia of the cecum, resection is the treatment for cecal volvulus. Whether performed laparoscopically or open, ileocecal resection and anastomosis is currently the treatment of choice for most patients. Cecopexy is an option but is associated with high recurrence rates, and cecostomy tube placement has a very high complication rate. Indications for cecopexy are limited, and indications for cecostomy tube placement do not exist.

Abdominal pain, distention, and obstipation are the hallmarks of colonic Obstruction. This obstruction can Be functional or mechanical. Initial evaluation of these patients includes a history, physical exam, and laboratory analyses. These will typically not lead to the etiology, so the next step is imaging. In this patient, the CT scan shows the classic diagnostic findings of a cecal volvulus. Cecal volvulus is the second most common volvulus seen after sigmoid volvulus and accounts for 25 to 40% of colonic volvulus. Cecal volvulus is responsible for 1 to 1.5% of all intestinal obstructions.

Sigmoid volvulus would have a different appearance on CT scan. Treatment includes colonoscopic decompression if the patient is not severely ill, followed by elective sigmoid resection. Colonoscopic decompression is rarely effective for cecal volvulus. functional obstruction, or pseudo obstruction, results from anomalies in the parasympathetic and sympathetic balance in the colon. It also has a different appearance on CT scan and is treated with intravenous neostigmine or colonoscopic decompression. There is no role for loop ileostomy in these patients.

Diagnosis of cecal volvulus is typically made with imaging. Plain abdominal radiograph can show the classic coffee bean sign with the axis pointing at the cecum. Unfortunately, only 20% of cecal volvulus patients will have this finding on plain film. Barium enema is also diagnostic, with the classic "bird's beak" sign being seen in up to 88% of patients. However, this study can often not be completed easily and is limited after hours. CT scan has become the diagnostic test of choice. In the image shown, the tapering of the proximal and distal bowel is visible in addition to the "coffee bean" sign (figure 23.2).

24. A 45-year-old woman was admitted 2 weeks ago with a diagnosis of necrotizing pancreatitis and a peripancreatic abscess. A drain was placed, and antibiotics were started. Drain output has continued to be moderate and purulent. The patient suddenly becomes hypotensive; on examination, her drain output is now bloody. What intervention is the appropriate next step?

63

A. B. C. D. E.

Intravenous beta-blocker Upper endoscopy Video-assisted retroperitoneal exploration Angiography Exploratory laparotomy

ANSWER:

D

This patient presents with hemorrhage associated with necrotizing pancreatitis. Hemorrhage can be due to arterial or venous etiologies. Arterial etiologies include ruptured pseudoaneurysm, hemorrhagic pseudocysts without pseudoaneurysm, or direct bleeding from vessel injury from drain placement. The first step in the management of this patient is resuscitation, including blood products. The next step is angiography and embolization. This plan has become the standard of care over the last 2 decades, and its use has significantly improved outcomes. However, surgery needs to remain part of the equation, because up to 33% of patients will not have control of bleeding with angiography and will require exploratory laparotomy. Clearly, angiography with embolization is the preferred therapeutic option.

Figure 25.1.

ANSWER:

A

This patient presents with postpolypectomy syndrome. Postpolypectomy syndrome occurs when the electric current used during polypectomy extends beyond the mucosa into the muscularis propria and serosa. The serosal irritation leads to a localized inflammatory response and manifests clinically as a localized peritonitis. Although it can occur with any polypectomy, it is more common when a sessile polyp is removed. Patients usually present within 12 hours of the procedure, although symptoms can rarely occur days afterward. Clinically, patients will complain of pain, sometimes associated with fever. On exam, there will be localized pain, with or without guarding. White blood cell count may be elevated. Because there is concern for perforation, most patients will undergo imaging. Although an abdominal radiograph may show free air, this could be benign and associated with the colonoscopy itself. The definitive test for postpolypectomy syndrome is a CT scan, which will show localized inflammation in the section of the colon where the polyp was removed (figure 25.2).

Venous etiologies include portal vein thrombosis, splenic vein thrombosis, or superior mesenteric vein thrombosis. Each of these can produce venous hypertension in the splenic system, resulting in esophageal or gastric bleeding. This bleeding would not be seen in the drains but instead in the gastrointestinal tract. Treatment for this type of hemorrhage typically includes beta-blockers to decrease venous pressure and upper endoscopy to manage the bleeding. This is not the type of bleeding this patient is presenting with; thus, these choices are not correct. Video-assisted retroperitoneal exploration is used in the management of necrotizing pancreatitis and is not indicated for an acute bleed secondary to pancreatitis.

25. A 51-year-old man undergoes screening colonoscopy during which a 1.5-cm sessile polyp is identified in the right colon and removed with a hot snare. He returns to the emergency department 8 hours later with right-sided abdominal pain. Vital signs include a temperature of 38.1°C, heart rate of 90 beats per minute, and blood pressure of 135/85 mm Hg. On exam, he is very tender to palpation in the right mid abdomen, without guarding or rebound. His white blood cell count is 10,900/mm3 (3600-11,200/mm3). CT scan shows pericolonic inflammation of the right colon without free air (figure 25.1). The treatment plan for this patient should be A. nothing by mouth, intravenous antibiotics. B. repeat CT scan with rectal contrast. C. colonoscopy. D. diagnostic laparoscopy. E. exploratory laparotomy.

Figure 25.2 64

Further diagnostic studies are not needed at this time, so repeat CT scan with rectal contrast or repeat colonoscopy would not be indicated. This patient should be started on intravenous antibiotics, his oral intake should be held, and he should be followed clinically. As the patient improves, he should be started on a diet and transitioned over to oral antibiotics.

of prospective randomized trials addressing the optimal route of nutritional supplementation before surgery. A retrospective study analyzed the benefits of 3 months of preoperative enteral nutrition in patients with Crohn disease requiring surgery. After 3 months, the enteral nutrition group showed improvements in nutritional parameters, and the rate of intra-abdominal septic complications up to 3 months postoperatively was significantly lower than the control group who were maintained on a normal diet.

Patients can present with diffuse peritonitis. This is an indication for immediate surgical intervention. Additionally, if the exam worsens or the patient develops peritonitis, operative intervention is indicated, either laparoscopically or with a laparotomy. The bowel is inspected; if a perforation is identified, it can either be repaired or resected.

These data suggest that enteral nutrition is a viable alternative to parenteral nutrition before surgical resection. Adiposederived mesenchymal stem cells are effective in treating complex perianal fistulas, but they have not been studied in the treatment of anastomotic fistulas in patients with Crohn disease.

26. A patient with Crohn disease requires an elective resection for a stricture. His laboratory data demonstrate a low albumin. Which of the following statements about your operative plan is true?

27. A 72-year-old man presents with a 24-hour history of abdominal distention and no flatus. Five days earlier, he was diagnosed with a 5-cm near-obstructing sigmoid colon cancer. His gastroenterologist wants to place a self-expanding stent. Which of the following statements about stenting in this patient is true?

A. Parenteral nutrition should be initiated before surgical resection. B. Stapled side-to-side anastomosis has no difference in overall complication rates compared with handsewn endto-end anastomosis. C. A macroscopic, disease-free margin of 2 cm is associated with higher recurrence rates than a 12-cm margin D. Injecting the intestinal anastomosis with adipose-derived mesenchymal stem cells is associated with a lower incidence of anastomotic fistulas. E. Resection and primary anastomosis is associated with a higher risk of postoperative complications.

ANSWER:

A. Mortality of subsequent surgical resection is increased. B. Likelihood of successful laparoscopic resection is increased. C. Surgical site infection risk increases. D. Migration is reduced if the lesion is located at the rectosigmoid junction. E. No impact on quality of life is achieved compared with surgical decompression.

B

ANSWER:

The management of patients with Crohn disease undergoing surgical resection of a stricture has been studied extensively. In a prospective study of patients undergoing ileocecal resection, no difference was found in the cumulative risk of postoperative complications in the 30 days after surgery compared with patients undergoing a 1-stage procedure versus a 2-stage procedure.

B

Large-bowel obstruction secondary to cancer occurs in up to 30% of patients with colonic malignancies. Most obstructing colorectal cancers are left sided and potentially amenable to colonoscopic intervention. Several studies have explored the impact of stenting as a bridge to elective surgery versus emergency surgery for these patients. These studies showed no difference in 30-day mortality, but they did show an improvement in quality of life 1 week after stent placement compared with a surgery group without stent placement. In addition, the ability to successfully perform laparoscopic resection is significantly increased after stenting compared with emergency surgery (67.2% vs 31.4%, respectively). When comparing stent placement with emergency surgery, there is no difference in anastomotic leakage, but the incidence of surgical site infections is significantly lower in the stent group. Stent migration is a complication of stent placement for malignant colonic obstruction. Migration occurs in 3% of patients, with the rectosigmoid reported as the most common site for stent migration.

A randomized controlled trial of 139 patients with Crohn disease showed no difference in overall complication rates, anastomosis-associated complications, or reoperation rates between stapled side-to-side and hand-sewn end-to-end anastomosis. In addition, no difference was seen in surgical recurrence rates when the macroscopic disease-free margin was limited to 2 cm versus 12 cm. However, the presence of microscopic inflammation in a resection margin independently predicts short-term intra-abdominal septic complications in patients with Crohn disease undergoing ileocolic resection and primary anastomosis. Although the association between malnutrition and poor postoperative outcome is well documented, there is a paucity

65

28. A 36-year-old man presents with an anal fissure in the posterior midline refractory to medical management for 3 months. What is the next step in treatment? A. B. C. D. E.

A. B. C. D. E.

Anal dilation External sphincterotomy V-Y advancement flap Lateral internal sphincterotomy Fissurectomy

ANSWER:

Intravenous hydration Exploratory laparotomy Comfort care Colonoscopy Visceral angiogram

ANSWER:

A

Pneumatosis is defined as gas located within the bowel wall. Increased use of CT scan imaging over plain film radiography indicates this condition is more common than previously thought. The pathogenesis is poorly understood, and its clinical significance varies from benign to bowel wall necrosis. The combination of pneumatosis with portal venous gas or a lactic acidosis is closely associated with advanced intestinal infarct. Other factors suggesting the need for exploratory laparotomy include vasopressor support, hypotension, peritonitis, and acute kidney injury. In the absence of these findings, intravenous hydration and serial examination is all that is required. Neither colonoscopy nor visceral angiogram is required. Visceral angiogram would be indicated if the patient presented with symptoms of ischemic colitis, such as pain out of proportion to physical exam, fever, and bloody diarrhea.

D

Anal fissures typically present with a tearing pain during defecation and are associated with minimal red blood or blood streaks on the toilet tissue. Fissures appear as an ovalshaped, longitudinal tear in the anal canal, distal to the dentate line. Chronic fissures (duration of 6-8 weeks) typically have chronic inflammatory changes, a skin tag at the distal fissure margin, and a hypertrophied anal papilla proximally. Almost 90% of fissures arise in the posterior midline, but they can occur in the anterior midline, particularly in women. Lateral fissures should raise concerns for Crohn disease, syphilis, anal carcinoma, or tuberculosis. Initial treatment is nonoperative, consisting of sitz baths, psyllium fiber, and bulking agents. This approach is successful in 50% of acute fissures. Topical nitrates are only marginally more effective than placebo, and evidence on topical calcium channel blockers is not conclusive. Botulinum toxin has healing rates similar to placebo, and there is no consensus on dosage, site of administration, or number of injections.

Given her unremarkable abdominal exam, this patient has benign pneumatosis, most likely associated with her chronic obstructive pulmonary disease. Her labs indicate a possible volume depletion requiring hydration. Benign pneumatosis would not be an indication to proceed to comfort care.

Should these medical approaches fail, lateral internal sphincterotomy is the surgical treatment of choice. This procedure offers superior healing and lower incontinence rates than posterior fissurectomy. Although balloon dilation and advancement flaps have been tried, these procedures are not as effective as a lateral internal sphincterotomy. Anal dilation is also associated with higher rates of major incontinence. Division of the external anal sphincter results in incontinence. Fissurectomy has similar healing times and complication rates as internal lateral sphincterotomy but frequently requires additional medical or surgical treatment to complete healing.

30. A 56-year-old woman with frequent heartburn and antacid use presents with a 3-year history of intermittent dysphagia to solid food only. When this occurs, she feels as if the food becomes stuck in the retrosternal area. She has otherwise been healthy, has no significant medical history, and has no other complaints. Esophagram reveals a smooth, short stricture located in the distal esophagus. A biopsy does not demonstrate cancer. Esophageal manometry is normal. Which of the following statements is true regarding treatment of this stricture? A. Dilation is contraindicated B. Lumen size after dilation does not correspond with symptom relief. C. Proton pump inhibitors reduce the need for subsequent dilation. D. Injection of the stricture with corticosteroids is superior to dilation. E. Placement of an expandable metal stent is first-line treatment.

29. An 83-year-old woman with a medical history of chronic obstructive pulmonary disease presents with crampy abdominal pain and diarrhea. On exam, she has normal vital signs with a mildly distended, nontender abdomen. A CT scan of the abdomen demonstrates pneumatosis of the cecum and right colon. No portal venous gas or pneumoperitoneum is present. Laboratory data demonstrate a white blood cell count of 11,000/mm3 (3600-11,200/mm3), hematocrit of 51% (3751 %), bicarbonate of 22 mmol/L (20-29 mmol/L), and creatinine of 1.1 mg/dL (0.4-1.3 mg/dL). What is the next appropriate step in her treatment?

ANSWER:

C

Benign strictures of the esophagus have a significant negative effect on patients' quality of life. These strictures may lead to

66

weight loss, malnutrition, and aspiration. The initial treatment of choice is endoscopic peroral dilation with bougies or balloons; approximately 40% of patients require only a single dilation. The exact mechanism leading to increased luminal size is unknown but is probably related to stretching or splitting of the stricture. Generally, the size of the lumen at the conclusion of the dilation correlates with symptom relief. In patients with peptic strictures, the coexistence of esophagitis may be a contributing cause of dysphagia. Patients who demonstrated healing of their esophagitis with proton pump inhibitors after dilation had complete resolution of their dysphagia, whereas those with continued esophagitis required repeat dilations. Topical corticosteroids are secondline therapy in eosinophilic esophagitis, after acid suppression. There is no current role for injectable steroids in peptic stricture. Expandable metal stents are successful for palliation of malignant esophageal obstruction, but when used for benign strictures result in multiple complications, such as in-growth of granulation tissue with subsequent obstruction, stent migration, and perforation.

67

Alimentary Tract Part II

decrease in postoperative pain. The return to activity is the same as in traditional laparoscopic techniques. In general, operating time is longer, but the

ITEMS 1-30 For each question, select the best possible response. 1. A 21-year-old man presents with 1 day of right lower quadrant abdominal pain. He is hemodynamically normal, has a temperature of 38°C, and a white blood cell count of 12,500/mm3 (3600-11,200/mm3). On examination, he has localized peritonitis in his right lower quadrant A CT scan demonstrates acute appendicitis without a surrounding phlegmon or abscess. He asks your opinion on treatment with antibiotics only. Compared with operative management, antibiotic therapy for acute uncomplicated appendicitis has a A. B. C. D. E.

conversion to an open technique is not higher, suggesting that it takes longer to do the operation via a SILS approach but adequate visualization and maneuvering can be achieved. The one disadvantage of the SILS technique is an increased incisional hernia rate of approximately 8% versus 1% using traditional laparoscopic techniques. This is likely because SILS incisions use a 2.5- to 3-cm incision versus a 1.0- to 1.5cm incision seen from traditional laparoscopic ports. The use of robot-assisted laparoscopic techniques in SILS does not improve the incisional hernia rate, because a larger incision is still used.

similar length of stay higher overall cost. higher readmission rate within 1 year. higher rate of early implications. longer duration of pain for the patient.

ANSWER:

3. You perform an uneventful appendectomy on a 22-year old man with uncomplicated appendicitis. You see him 1 week later in clinic for his postoperative follow-up visit He has no complaints and reports that he is doing well. The pathology report notes a 0.8-cm neuroendocrine carcinoid tumor at the tip of the appendix with clear margins. The next appropriate step in management is

C

Acute appendicitis can be classified as either complicated or uncomplicated. Complicated appendicitis is defined as perforation; the presence of an abscess, phlegmon, or an appendicolith; or suspicion of a tumor. This distinction is important, because recent literature suggests nonoperative management of uncomplicated appendicitis with antibiotics may be equivalent or superior to operative management

A. B. C. D. E.

Patients with uncomplicated appendicitis who undergo nonoperative management with antibiotics have a higher readmission rate within 1 year, primarily for recurrent appendicitis. Recurrence of appendicitis after nonoperative management occurs in up to one-third of patients. Length of hospital stay tends to be slightly longer in the nonoperative group although overall cost ends up being lower. No difference in minor or major complications or duration of pain is noted between the groups.

ANSWER:

Neuroendocrine cancer accounts for 60% of all appendicular tumors. The appendix is the most common site for gastrointestinal carcinoid tumors. Most are smaller than 1 cm in diameter and located near the tip of the appendix. They are indolent and possess no specific tumor characteristics to aid in the preoperative diagnosis. Tumors smaller than 1 cm found on retrospective pathologic analysis and with negative margins are considered cured by appendectomy. Ileocecectomy is overtreatment for this patient. No further workup, staging, or postoperative surveillance is required. Right hemicolectomy is recommended for tumors larger than 2 cm. Treatment is controversial for tumors 1 to 2 cm in diameter. Some practitioners advocate simple appendectomy, whereas others recommend right hemicolectomy. Right hemicolectomy should be considered for tumors with positive

less postoperative pain. longer operating times. lower incisional hernia rates. faster return to activity. increased conversion rates.

ANSWER:

D

Appendix cancer is rare, with an occurrence of 1.2 cases per 100,000 people. Most cases present as acute appendicitis and are diagnosed incidentally during pathologic examination. No established risk factors exist for appendix cancer. Classification of tumors can be divided into neuroendocrine carcinoma, colonic-type adenocarcinoma, mucinous adenocarcinoma, and goblet cell adenocarcinoma. Treatment varies based on the classification and size of the tumor.

2. A 34-year-old man presents with symptomatic cholelithiasis. He is interested in the newest techniques for surgery. Compared with traditional laparoscopic cholecystectomy, single-incision laparoscopic surgery is associated with A. B. C. D. E.

colonoscopy. ileocecectomy. intraperitoneal chemotherapy. no further workup right hemicolectomy.

B

The use of single-incision laparoscopic surgery (SILS) has recently gained popularity. The only proven benefit, though, is improved cosmesis. Despite only one incision, there is no

69

margins, appendiceal base location, or invasion into the mesoappendix.

deposits reveals no cancer. What is the most appropriate definitive treatment for this patient?

Colonoscopy is recommended for all patients with colonictype adenocarcinoma to rule out synchronous colorectal lesions. Right hemicolectomy should be considered for all patients with colonic-type adenocarcinoma. Intraperitoneal chemotherapy is the treatment of choice for mucinous adenocarcinoma in which peritoneal dissemination has occurred.

A. B. C. D.

Observation Systemic chemotherapy Right hemicolectom Cytoreductive surgery with hyperthermic intraperitoneal chemotherapy E. Referral to hospice

ANSWER: 4. In comparing laparoscopic gastrostomy versus percutaneous endoscopic gastrostomy (PEG) tubes, which of the following statements is true?

This patient has low-grade carcinoma peritonei, previously referred to as disseminated peritoneal adenomucinosis. Presence of mucin in the peritoneum can lead to the pseudomyxoma peritonei syndrome, which results in massive distension of the abdomen with mucinous ascites and malnutrition. Although pseudomyxoma peritonei syndrome is common with appendiceal neoplasms, it also can occur due to pancreatic, ovarian, and gastric tumors.

A. Complication rates are similar in adults. B. Early pneumoperitoneum is seen only after laparoscopic gastrostomy tube placement. C. Laparoscopic gastrostomy tube placement is associated with more complications in children younger than 5 years of age. D. Previous fundoplication is a contraindication to PEG tube placement. E. Mortality is higher after PEG tube placement.

ANSWER:

D

This patient has a low-grade appendiceal mucinous neoplasm. This neoplasm has no malignant potential and thus does not spread via lymphatics. Therefore, the prognosis of such patients is determined more by the peritoneal disease than by nodal disease. Management of the primary lesion includes a margin-negative resection only.

A

The management of the peritoneal mucin is most effective by the appropriate use of parietal peritonectomy procedures, such as a right hemidiaphragmatic peritonectomy and an omentectomy with a bursectomy to cytoreduce all disease. Historical series suggest a significantly improved survival for patients who undergo cytoreductive surgery and hyperthermic intraperitoneal chemotherapy (HIPEC) compared with serial cytoreductive surgeries. Patients who undergo complete cytoreduction and HIPEC have an estimated median survival of 70% at 20 years. Given the low malignant potential for this condition, there is limited role for systemic chemotherapy.

The 2 most common techniques used to perform gastrostomy include the percutaneous endoscopic gastrostomy (PEG) and laparoscopic gastrostomy tubes. PEG tubes are generally used because they are minimally invasive, are low cost, and have a low complication rate. In addition, PEGs can be performed without general anesthesia, even at the bedside. The drawbacks of a PEG tube include hollow viscus perforation or injury, liver injury, and failed tract formation. Laparoscopic gastrostomy tubes require the use of general anesthesia and are more expensive. Despite these differences in techniques, both procedures have similar complication rates in adults with similar mortality rates.

6. A 60-year-old man undergoes an endoscopy for reflux disease. He is being treated with proton pump inhibitors. His endoscopy reveals a 1-cm subepithelial lesion on the lesser curvature of the stomach, 3 cm from the gastroesophageal junction with a smooth appearance on endoscopic ultrasound with no high-risk features. Needle-guided core biopsy reveals a CD 117+ tumor. What is the most appropriate management?

Patients who undergo PEG tubes can present with pneumoperitoneum, which results from either a failed tract or air insufflation that occurs during the needle access of the stomach. In children, PEG tubes appear to have a slightly higher complication rate than laparoscopic gastrostomy tubes, regardless of prior fundoplications. However, prior fundoplications are not a contraindication for PEG tubes. If a transillumination window can be seen, then this technique is equally safe and feasible.

A. B. C. D. E.

5. A 45-year-old man presents with acute appendicitis. During his laparoscopic appendectomy, he is found to have a mucinous neoplasm of the appendix with mucinous peritoneal deposits over the right hemidiaphragm. Surgical pathology reveals a low-grade appendiceal mucinous neoplasm with no invasion, and a biopsy of the mucinous

Endoscopic submucosal dissection Laparoscopic partial gastrectomy Subtotal gastrectomy with modified D2 node dissection Stop proton pump inhibitor therapy Observation

ANSWER: 70

E

This patient has an incidentally detected gastrointestinal stromal tumor (GIST). GISTs are the most common mesenchymal tumors of the gastrointestinal tract and are derived from the intestinal cells of Cajal. Immunohistochemical staining for tyrosine kinase, c-kit (also known as CD-117) shows receptor positivity in approximately 75 to 80% of GIST tumors, whereas 8% have mutations in the PDGFRα gene. Only 10 to 30% of GISTs are clinically malignant, although almost all have malignant potential. Microscopic GISTs are noted in almost 22% of autopsies and 35% of surgically resected stomach for other cancers. Thus, incidental GISTs are very common.

Several risk factors are associated with an increased incidence of metastatic lymph nodes. These include size greater than 2 cm and undifferentiated-type histology. With advances in laparoscopy and endoscopy, endoscopic mucosal dissection provides a less-invasive treatment approach to treating early gastric cancer. However, endoscopic mucosal dissection is limited to mucosal cancers smaller than 2 cm in diameter due to the risk of lymph node metastasis. A metaanalysis of 15 randomized controlled trials did not show strong evidence that chemoradiotherapy provides a 5-year survival rate over chemotherapy alone in patients with nongastroesophageal junction gastric cancer.

Management of localized GIST tumors is surgical resection with negative margins with no node dissections. Tumors of the stomach have a lower malignant potential than those of the small intestine or rectum, Tumors that are 1 cm in size are referred to as mini GISTS. These tumors have very low malignant potential and are managed by endoscopic ultrasound surveillance. High-risk features include the presence of echogenic foci, irregular borders, or ulceration. Endoscopic removal of GISTs is not well supported by evidence and risk perforations.

8. A 65-year-old woman presents with epigastric fullness, pain, nausea, vomiting, and recent weight loss. Upper endoscopy demonstrates a diffusely thickened stomach wall Biopsy reveals diffuse large B-cell lymphoma and is negative for Helicobacter pylori. CT scan shows a diffusely thickened stomach and perigastric lymph node enlargement. No other masses or lesions are noted on further staging with PET scan. What is the most appropriate next step in management?

Gastric carcinoids are tumors of the stomach that occur due to hypergastrinemia that occurs with achlorhydria due to proton pump inhibitors. These tumors can be managed by stopping proton pump inhibitors. These tumors are not CD117 positive but rather express neuroendocrine markers.

A. B. C. D. E.

7. A 52-year-old Chinese woman presents with 4.5-kg weight loss. An endoscopic biopsy of a lesion in the antrum demonstrates a 2.5-cm diameter signet ring cell carcinoma. In determining her management, which of the following statements is true?

ANSWER:

A

The incidence of gastric cancer has decreased over the last century, but the incidence of proximal gastric cancer has recently risen. Adenocarcinoma is the most common malignancy, accounting for 90% of all gastric cancers globally. Other etiologies include lymphoma, carcinoid, and sarcoma. Overall prognosis for patients with gastric cancer is poor, with 5-year survival rates ranging between 20% and 25%. Early-stage disease has a much better prognosis than regional or metastatic disease.

A. The patient should be treated with definitive chemoradiation. B. An endoscopic submucosal dissection is adequate. C. Laparoscopic gastrectomy increases risk of anastomotic leak compared with open resection. D. Laparoscopic gastrectomy limits the ability to perform lymph node dissection. E. Laparoscopic exploration for metastatic disease is necessary before attempts at curative resection.

ANSWER:

Chemoradiation Treatment for H. pylori Imatinib Total gastrectomy and perigastric lymph node sampling Total gastrectomy with splenectomy and perigastric lymph node removal

Gastric lymphoma is the second most common primary gastric malignancy and accounts for 5% of gastric cancers. However, the incidence has been rising in recent years. Up to 98% of primary gastric lymphomas are of B-cell origin, and nearly 60% are aggressive diffuse large B-cell lymphoma. Patients presenting with diffuse large B-cell lymphoma should undergo chemoradiation therapy. Surgical therapy is reserved for treatment of complications of the disease, such as perforation, bleeding, or obstruction.

E

Early gastric cancer is increasing in diagnosis, and management options continue to evolve. In a multicenter, randomized controlled trial, laparoscopic gastrectomy was compared with open gastrectomy. Intraoperative and postoperative complications, including anastomotic leak rates, were comparable between the 2 groups. The status of lymph node metastasis is another important factor for determining the treatment modality. Lymph node retrieval between open and laparoscopic gastrectomy was comparable, with an average of 36 lymph nodes retrieved, independent of technique.

B-cell lymphomas associated with mucosa-associated lymphoid tissue (MALT) are more likely (up to 90%) to be associated with chronic Helicobacter pylori infection. They demonstrate a high level of responsiveness to H. pylori treatment when in early stages. H. pylori is also commonly associated with diffuse large B-cell lymphoma, but treatment

71

of H. pylori alone is not recommended due to the aggressive nature of diffuse large B-cell lymphoma.

Acute upper gastrointestinal bleed from esophageal varices in a cirrhotic patient is managed with upper endoscopy and variceal injection or banding. Banding is preferred. Medical management of the portal hypertension is also used.

Imatinib is used to treat gastrointestinal stromal tumors (GIST) and chronic myelogenous leukemia. Imatinib has no role in treatment of gastric lymphoma.

10. A 54-year-old man presents to the emergency department with 4 days of lower abdominal pain, fatigue, and low-grade fevers. On exam, his abdomen is distended and tender in the right lower quadrant. His white blood cell count is 18,000/mm3 (3600-11,200/mm3). CT scan demonstrates a 5cm, rim-enhancing abscess in the right lower quadrant with extensive inflammatory changes in the surrounding tissues. The appendix is only partially visualized. There is a small appendicolith near the appendiceal base. Intravenous antibiotics are started. Which of the following represents the optimal management of this patient?

9. Angiography with embolization has become an accepted management strategy for select cases of acute gastrointestinal bleeding. Which of the following scenarios represents the most appropriate patient for use of this approach? A. Lower gastrointestinal bleed from a sigmoid diverticula B. Lower gastrointestinal bleed from radiation proctitis C. C Acute perioperative upper gastrointestinal bleed after Roux-en-Y gastric bypass D. Upper gastrointestinal bleed from a peptic ulcer after failed endoscopic therapy E. Upper gastrointestinal bleed from varices in a cirrhotic patient

ANSWER:

A. B. C. D. E.

D

Gastrointestinal bleeding is the most common gastrointestinal reason for hospital admission in the United States. Both upper and lower gastrointestinal bleeding are associated with significant morbidity and a 2 to 4% hospital mortality. Principles of initial management include cessation of antiplatelet and anticoagulation therapy where possible, early endoscopic localization, and endoscopic therapy. For patients with diverticular bleeding, endoscopic clips are the preferred first-line therapy. Radiation proctitis can be managed effectively with local interventions, such as endoscopic argon beam coagulation or formalin application. Early postoperative bleeding after Roux-en-Y gastric bypass generally resolves with cessation of venous thromboembolism chemoprophylaxis or other anticoagulation or antiplatelet agents. If it persists, upper endoscopy may localize a source of anastomotic bleeding that can be managed with clipping or other endoscopic means. Surgical intervention may be required if bleeding persists or if the diagnosis is uncertain.

Laparoscopic irrigation Open abscess drainage with appendectomy 14 days of intravenous antibiotic Right hemicolectomy Percutaneous abscess drainage

ANSWER:

E

In this patient with perforated appendicitis and an abscess, percutaneous drainage remains the preferred initial approach. Antibiotics alone are unlikely to achieve resolution of this walled-off, rim-enhancing collection. Antibiotics alone might be curative with abscesses smaller than 3 cm, but they would be unlikely to succeed given the size of this abscess, regardless of duration of treatment. Recent pediatric evidence supports immediate appendectomy in cases of complicated appendicitis. In addition, one small randomized trial in adults indicated laparoscopic abscess drainage and appendectomy is safe in experienced hands. However, the preponderance of adult data suggest that although operation (either laparoscopic or open) may permit drainage of the abscess, the perioperative risks (wound infection, ileus, anesthetic risk) are greater than with percutaneous drainage. Furthermore, in this patient, any attempt to remove the appendix would likely result in significant bleeding and perhaps even a fecal fistula or need for an ileocecectomy or right hemicolectomy (10%).

In patients with recurrent or refractory nonvariceal upper gastrointestinal bleeding, angiographic treatment should be considered. Microcoils allow for precise placement into actively bleeding vessels while preserving collateral perfusion. If. hemorrhage continues, these can be combined with other hemostatic agents such as absorbable gelatin. These focused approaches minimize the risk of bowel ischemia or infarction (1-4%) while achieving clinical hemostasis in up to 96% (ranges: 44-94% for upper gastrointestinal bleeding and 63-96% for lower gastrointestinal bleeding). Thus, angioembolization has become the preferred approach for definitive intervention in select patients when first-line therapy fails.

Laparoscopic irrigation alone is not an adequate intervention in this case because it is associated with a need for secondary procedures (e.g., percutaneous drainage). Although there may be some concern for malignancy given this patient's age, an immediate right hemicolectomy is not justified because the overall risk of malignancy is relatively low (1.2%). Instead, the risks and benefits of an interval appendectomy can be weighed in an elective setting after the abscess has resolved and a colonoscopy has been performed.

72

11. A 28-year-old man presents with worsening perianal pain. He is on chemotherapy after a bone marrow transplant for acute leukemia. He is hospitalized for neutropenic fever treated with broad-spectrum antibiotics and antifungal agents. He is lethargic and appears pale and diaphoretic. His abdomen is slightly distended but soft and nontender. His perineal examination is notable for a 10-cm area of erythema extending from his anus. He has extreme tenderness adjacent to the anal canal, but there is no fluctuance. His white blood cell count is 300/mm3 (3600-11,200/mm3) with an absolute neutrophil count of 150/mm3 (1500 to 8,000/mm3). A CT scan of his abdomen and pelvis demonstrates an enlarged spleen and some inflammatory changes in the perineum but no intra-abdominal or perirectal fluid collections. What is the best approach to managing this patient? A. B. C. D. E.

ANSWER:

More than 200,000 bariatric surgical procedures are performed annually in the United States, and long-term complications are seen. This incidence mandates that all surgeons be aware of the different surgical procedures being done, different postoperative anatomy, clinical presentations of late complications, and necessary workup and treatment options. Late complications of these procedures can include band slippage or erosion, gastric pouch dilation, bowel obstruction, marginal ulcer, incisional hernia, and gallstone disease. The incidence of bowel obstruction after Roux-en-Y gastric bypass is 4.4%, more than half of which are caused by internal hernia, followed by roux limb compression, and adhesions, and less commonly gastrojejunal anastomotic stenosis, incisional hernia, or intestinal intussusception.

Hyperbaric oxygen treatment Image-guided aspiration Steroids Continued broad-spectrum antibiotics Incision and drainage

ANSWER:

D

Symptomatic internal hernia occurs in 1.6 to 4.1% and is most commonly at the mesojejunal mesenteric window (56%), followed by mesenteric defect posterior to the roux jejunal limb known as the Petersen defect (27%) and the mesocolic window. Symptoms of central colicky abdominal pain occur in 82% of patients. Nausea, vomiting, and distention are also common. During the evaluation and resuscitation of these patients, it is important to identify what type of surgery the patient had and to ask about the character and volume of emesis, which can help identify the location of the obstruction. Most patients have a history of intermittent, colicky abdominal pain before presenting with an acute surgical emergency.

D

This neutropenic patient has a severe perineal cellulitis for which broad-spectrum antibiotics should be continued. External exam or by cross-sectional imaging provide no evidence of an abscess that would warrant either imageguided aspiration or incision and drainage. If a collection were apparent on exam or on imaging, incision and drainage would be the management of choice, even in the setting of neutropenia.

Workup requires rapid exclusion of peritonitis or ischemia, which would mandate immediate laparoscopy or laparotomy. Imaging with abdominal x-rays, contrast swallow examination, and CT scans can help identify the site of obstruction. In the absence of peritonitis in a hemodynamically normal patient, a conservative approach may be appropriate and may afford the opportunity to review the case with a bariatric surgeon. If conservative management fails, internal hernia and adhesive obstructive disease can be surgically managed either using laparoscopy or laparotomy. Anastomotic strictures can often be endoscopically dilated before being revised surgically if endoscopic dilation is unsuccessful.

Hyperbaric oxygen therapy remains a controversial therapy in managing acute infections. No evidence supports the use of steroids in the management of acute perineal infections in either immunocompetent or immunocompromised patients. If the patient does not improve with continued antibiotics or if he deteriorates further, a necrotizing soft tissue infection should be considered. In such cases, wide surgical debridement would be the preferred management.

12. Regarding the late or long-term complications from Roux-en-Y gastric bypass, which of the following statements is true? A. Adhesive bowel obstruction is the most common cause of bowel obstruction. B. Central colicky abdominal pain is an uncommon presenting complaint. C. Herniation through the Peterson window is the most common type of internal hernia. D. Internal hernia is responsible for more than 50% of postoperative bowel obstruction. E. Gastrojejunostomy anastomotic stricture most often requires surgical revision.

13. A 50-year-old, otherwise healthy man presents with acute onset of foreign body sensation in the esophagus after eating dinner. Which of the following statements is true regarding the workup and management of this condition? A. Esophageal food impaction is the most common indication for urgent endoscopy. B. Complication rate of flexible endoscopic retrieval of food impactions is 20%.

73

C. Less than 25% of patients will have underlying esophageal pathology. D. Flexible endoscopic retrieval is successful in less than 75% of cases. E. Barium swallow is not recommended.

ANSWER:

percutaneous drainage of abscesses larger than 4 cm. An evaluation of 136 patients with perforated diverticulitis with pneumoperitoneum or abscess found that only 3.7% required initial surgery, with another 5% requiring surgery for failure of nonoperative management Of the 27 patients with free air remote from the site of perforation, 25 (92.5%) were successfully treated without surgery. A study of 8 patients who were hemodynamically normal with distant pneumoperitoneum had a 100% success rate of nonoperative therapy.

E

Upper endoscopy for foreign body obstruction is the second most common urgent indication after gastrointestinal hemorrhage. Twenty-five percent of patients who require urgent upper endoscopy will have impacted food bolus. Ingestion of a foreign body occurs most frequently in pediatric patients and in adults with a psychiatric history. Most food bolus obstructions pass spontaneously, but roughly 15% will require urgent upper endoscopy. Approximately three-quarters of patients with food bolus impaction will have an underlying pathology such as stricture, eosinophilic esophagitis, malignancy, or motility disorders. Medical therapy for food bolus is not often successful and should riot delay the definitive diagnostic and therapeutic intervention: flexible endoscopy. Flexible endoscopy is successful 95% of the time with a less than 5% complication rate. Barium swallow is not recommended, because it decreases visualization at endoscopy and has a risk of aspiration.

This patient does not have a drainable fluid collection, so percutaneous drainage is not warranted. However, up to 40% of these patients will ultimately develop an abscess larger than 4 cm, so percutaneous drainage may become indicated, depending on the patient's clinical progress. Sigmoid colectomy would be warranted only in the setting of uncontrolled intra-abdominal sepsis or failure to improve with nonoperative management. Primary anastomosis is safe in select patients with complicated diverticulitis, with the decision for colostomy versus anastomosis based on intraoperative findings including the degree of inflammation and contamination. A colorectal anastomosis is often protected by a diverting loop ileostomy. Laparoscopic lavage and drainage was previously considered a popular alternative to resection, but it has lost favor. Many of the retrospective studies evaluating the technique had significant selection bias and included patients with early disease (Hinchey I/II) who would have presumably done just as well with nonoperative therapy. The LADIES trial was the first randomized controlled trial to compare laparoscopic lavage with sigmoidectomy. It focused entirely on Hinchey III patients. Unfortunately, the lavage arm of the trial was forced to close early due to concern for very high rates of morbidity, including a 39% risk of serious short-term adverse events and a 20% rate of same-stay surgical reintervention.

14. A previously healthy 44-year-old man presents with acute abdominal pain. Physical exam reveals tenderness localized to the left lower quadrant without guarding or rebound. White blood cell count is 17,000/mm3 (360011,200/mm3), and a CT seem of the abdomen and pelvis demonstrates perforated sigmoid diverticulitis with small foci of free intraperitoneal air around the sigmoid colon and above the liver. What is the most appropriate initial management? A. B. C. D. E.

Bowel rest and antibiotics Percutaneous drainage Sigmoid colectomy with primary anastomosis Sigmoid colectomy with end colostomy Laparoscopic washout and drainage

ANSWER:

15. A 79-year-old woman with chronic constipation presents to the emergency department with complaints of bloating and nausea. On examination, she has a soft and moderately distended abdomen, but no abdominal tenderness. Her white blood cell count is 12,000/mm3 (3600-11,200/mm3), and an abdominal CT scan shows a sigmoid volvulus. What is the most appropriate initial treatment?

A

This patient is healthy with a reliable abdominal exam. There is localized tenderness without evidence for diffuse peritonitis. Therefore, the most appropriate initial management is bowel rest and antibiotics even in the presence of free intraperitoneal air. Conversely, if the patient had signs of uncontrolled sepsis, was immunocompromised, or had an otherwise unreliable exam, initial surgical exploration would be warranted, with the subsequent procedure dependent on operative findings.

A. B. C. D. E.

Bowel prep for colonoscopy Hydrostatic enema Endoscopic detorsion Sigmoidopexy Sigmoid colectomy

ANSWER:

In the absence of diffuse peritonitis, most patients with complicated diverticulitis can be treated successfully with a nonoperative approach combining bowel rest, antibiotics, and

C

Sigmoid volvulus is a common and well-described disease in which the colon twists around a narrow and elongated mesentery leading to large bowel obstruction and vascular

74

compromise. This twist occurs in a counter-clockwise fashion in 70% of cases. Patients are usually elderly with a history of constipation, and present with abdominal pain and distention. Abdominal x-rays reveal a massively distended colon with a "bent inner tube" sign, and CT scan reveals mesenteric swirling with a closed loop sigmoidal obstruction.

primary wound closure, and lowest with leaving the wound open and allowing healing by secondary intention. Unfortunately, open wounds can take a long time to heal and are often associated with poor cosmesis. Pursestring wound closure has been extensively studied for wound management after ileostomy takedown, including several well-designed randomized controlled trials and 2 recent meta-analyses. Pursestring closures have lower SSI rates compared with primary closures (2-3% vs 15-30%), similar times to complete wound healing, and improved cosmetic satisfaction scores.

This patient presents with volvulus in the absence of peritonitis. The most appropriate initial treatment is endoscopic detorsion. This treatment changes the situation from urgent to elective, allowing patient optimization for surgery, endoscopic visualization of the proximal colon, and the ability to operate on a nondilated colon.

Negative pressure wound therapy can be expensive and involves ongoing specialized wound care with nursing visits 2 to 3 times per week. The cost of negative pressure wound therapy cannot be justified over pursestring wound closure for routine wounds. Similarly, delayed primary closure requires a second procedure and would not be preferred over pursestring wound closure.

Detorsion can be accomplished with either rigid or flexible proctosigmoidoscopy. The benefit of using the rigid scope is the ability to pass a rectal tube directly through the inside of the scope into the sigmoid colon to prevent retorsion, which occurs in up to 90% of patients if a tube is not used. The benefit of flexible endoscopy is improved visualization and the ability to detorse more proximal volvuli, because 24% of patients will have torsions too proximal to be reached with the shorter 25-cm rigid sigmoidoscope. In this situation, a suture can be passed through the end of the rectal tube, tied to itself to create a loop, then grasped with endoscopic forceps to safely drag the tube up along with the endoscope.

17. A 39-year-old woman has undergone incision and drainage of a perirectal abscess 3 times in the last year, all of which occurred in the same location. She presents again with anal pain, swelling, and leukocytosis. Exam under anesthesia reveals a recurrent perirectal abscess with an associated highlying transsphincteric fistula in the left anterior position. What is the most appropriate initial management?

Bowel prep is not appropriate in the presence of a large bowel obstruction and may lead to worsening obstruction and perforation. Hydrostatic enemas are less successful than endoscopy for detorsion, and they do not allow the surgeon to inspect the bowel wall for ischemia.

A. B. C. D. E.

Sigmoidopexy is an acceptable treatment for volvulus after detorsion but is associated with high rates of recurrence and therefore should be reserved for patients who cannot tolerate colectomy. Sigmoid colectomy would be the definitive treatment of choice, but it would only be used as initial treatment if there was concern for bowel ischemia.

ANSWER:

B

This patient has a classic presentation of a cryptoglandular fistula with multiple abscesses in the same location. Drainage alone will allow for sepsis resolution, but the patient will most likely present with more abscesses in the future unless the fistula is addressed. This is a high-lying transsphincteric fistula; fistulotomy is inappropriate, because it would be associated with an unacceptably high risk for fecal incontinence, especially in a female patient with an anterior fistula.

16. A 64-year-old woman with rectal cancer and previous proctectomy has completed her adjuvant chemotherapy. She undergoes an elective takedown of her loop ileostomy. What is the most appropriate way to address the patient's surgical wound after fascial closure? A. Leave skin open, pack the wound, and allow healing by secondary intention. B. Leave skin open for 1 week, then perform a delayed primary linear closure C. Apply negative pressure wound therapy. D. Close skin with a pursestring suture. E. Close skin primarily in a linear fashion.

ANSWER:

Drainage alone Drainage with placement of a noncutting seton Drainage with fistulotomy Drainage with an endorectal advancement flap Drainage with placement of a fistula plug

For this patient, the best initial approach is to drain the abscess and place a loose, noncutting seton to facilitate drainage, stent the fistula open, and prevent recurrent abscess formation. This can be followed 8 to 12 weeks later with a definitive repair. Many different materials are acceptable to serve as a seton, but silastic vessel loops are most commonly used.

D

An endorectal advancement flap is a suitable muscle-sparing approach for complex fistulas with a success rate of 66 to 87%. It is not appropriate in the setting of an acute abscess

Surgical site infections (SSIs) occur in up to 40% of patients after ileostomy takedown. The SSI rate is highest with

75

and should instead be performed in a delayed fashion after sepsis resolution. Fistula plugs also require sepsis resolution before placement but are less frequently used due to reported healing rates of approximately 24%.

A. B. C. D. E.

18. A 52-year-old woman undergoes her initial screening colonoscopy. She has no known medical problems or gastrointestinal complaints, and her family history is negative for colon polyps or colon cancer. The endoscopist finds a 1.1cm pedunculated polyp in the sigmoid colon, which is removed completely with a hot snare. Pathology reveals this polyp to be a tubular adenoma. When should this patient undergo a subsequent surveillance colonoscopy? A. B. C. D. E.

ANSWER:

D

This patient has developed fulminant Clostridium difficile colitis, which is defined as pronounced leukocytosis and hypotension requiring vasopressor support. Fulminant C. difficile colitis is a rare presentation of C. difficile colitis, occurring in only 2 to 3% of patients but associated with serious complications and death. Once a patient has progressed to fulminant C. difficile colitis, a subtotal colectomy with end ileostomy should be performed. Mortality and complication rates are lower for subtotal colectomy (11-56%) compared with segmental colectomy with or without anastomosis (14-100%).

6 months 1 year 3 years 5 years 10 years

ANSWER:

oral and rectal vancomycin. segmental Colectomy with primary anastomosis fecal microbiota transplant. subtotal abdominal colectomy with end ileostomy. laparoscopic loop ileostomy with intraoperative colonic lavage.

Laparoscopic loop ileostomy with intraoperative colonic lavage attempts to preserve the colon in patients with C. difficile colitis. Single institution series and a retrospective multi-institutional study demonstrated safety compared with subtotal colectomy. However, this minimally invasive technique is likely best used in patients with C. difficile colitis who have not progressed to organ dysfunction or failure

C

Different guidelines exist for colorectal cancer screening as well as subsequent surveillance after discovery of colorectal polyps. The US Multi-Society Task Force (MSTF) created consensus guidelines that are frequently updated and provide supporting evidence with the recommendations. MSTF separates patients with polyps found on screening endoscopy into 2 groups: (1) low-risk adenomas (1-2 tubular adenoma <10 mm in size), and (2) high-risk adenomas (villous histology, high-grade dysplasia, size >10 mm, 3 or more adenomas).

The standard medical management of C. difficile colitis consists of discontinuing any antibiotics associated with the development of the colitis and initiating oral or intravenous metronidazole or oral vancomycin. Vancomycin enemas can be used for adjunctive therapy along with oral vancomycin. The recommended duration of therapy is 10 to 14 days. However, once a patient has progressed to fulminant C. difficile colitis, antibiotic therapy alone is unlikely to be effective. Fecal microbiota transplant is an option for patients with recurrent C. difficile infections. It currently does not have a role in patients with fulminant C. difficile colitis.

For low-risk adenomas, the recommended surveillance interval is "5-10 years," which is somewhat ambiguous. However, for high-risk adenomas, there is more certainty and also more supporting evidence for MSTF recommendations. For adenomatous polyps of at least 10 mm, high-quality evidence supports recommending a surveillance interval of 3 years. Moderate-quality evidence supports a 3-year interval for other high-risk adenomas as well. An interval of 6 months is appropriate for a large polyp removed in a piecemeal fashion to verify complete removal. An interval of 1 year is indicated for surveillance after complete resection of a cancerous polyp with a margin greater than 2 mm.

20. A 45-year-old woman presents with peritonitis. She has normal vital signs except that her heart rate is 110 beats per minute. Her white blood cell count is 16,000/mm3. CT scan of her abdomen and pelvis confirms perforated sigmoid diverticulitis. In addition to intravenous antibiotics, what is the optimal surgical management of this patient?

19. On postoperative day 3 after a femoral popliteal bypass, a 64-year-old man develops abdominal distention and diarrhea. The next day, he is transferred to the intensive care unit with hypoxemia and a white blood cell count of 35,000/ mm3 (3600-11,200/mm3). He is diagnosed with Clostridium difficile colitis and started on intravenous metronidazole. That night, he becomes oliguric and is started on low-dose norepinephrine and vasopressin. A CT scan of the abdomen and pelvis shows pancolitis. The best next step in the management of his clinical condition is

A. B. C. D. E.

Sigmoid resection, colostomy (Hartmann procedure) Sigmoid resection, primary anastomosis and ileostomy Diverting proximal colostomy Diverting loop ileostomy with percutaneous drain Laparoscopic peritoneal lavage and pelvic drain

ANSWER: 76

A

Three small randomized controlled trials compared the Hartmann procedure to colon resection and primary anastomosis for perforated diverticulitis. The first trial randomized 34 patients to primary anastomosis and 56 patients to Hartmann procedure. There was no difference in mortality, morbidity, or stoma reversal. The second trial demonstrated no difference in mortality, but it was stopped early for an interim safety analysis of more serious complications with stoma reversal after Hartmann procedure (20% vs 0%). This trial reported a higher rate of stoma reversal after primary anastomosis with diverting ileostomy (90% vs 57%, p = .005). The most recent DIVERTI study enrolled 102 patients and found no difference in mortality or morbidity, and stoma reversal was higher in the primary anastomosis arm (96% vs 65%).

underwent stoma reversal within 12-months. Approximately 14% of laparoscopic lavage patients required a stoma; 48% obtained gut continuity within 12 months, whereas 36% underwent elective sigmoidectomy. The options of percutaneous catheter drainage, diverting proximal colostomy, and diverting loop ileostomy with percutaneous drain are not correct, because they would not provide optimal source control of the perforated sigmoid diverticulitis in this patient with peritonitis.

Items 21-23 21. A 44-year-old man who reports binge drinking presents with acute pancreatitis. He has been sick for a day. Systolic blood pressure is 85 mm Hg and improves with 2 L of crystalloid. He is afebrile, and his serum creatinine is 25 mg/dL (0.4-13 mg/dL). A CT scan is obtained (figure 21.1). The next step in management is

Multiple randomized trials (SCANDIV, DILALA, LADIES) compared laparoscopic peritoneal lavage with primary colon resection for acute perforated diverticulitis. Two trials [DILALA (Diverticulitis - Laparoscopic Lavage versus Resection (Hartmann Procedure) for Acute Diverticulitis with Peritonitis) and LADIES (Laparoscopic Peritoneal Lavage or Resection for Generalised Peritonitis for Perforated Diverticulitis)] separated patient cohorts based on purulent or fecal peritonitis and enrolled only Hinchey grade III diverticulitis patients for laparoscopic lavage after initial diagnostic laparoscopy confirmed the diagnosis and Hinchey grade. The SCANDIV (Scandinavian Diverticulitis) trial, in contrast with the DILALA and LADIES trials, randomized patients before diagnostic laparoscopy and included all Hinchey-grade diverticulitis cases. The choice of procedure (laparoscopic versus open resection or Hartmann procedure versus primary resection and anastomosis) in the SCANDIV trial was determined by surgeon preference, which may have resulted in increased risk of bias.

A. B. C. D. E.

A meta-analysis, which included the 3 randomized controlled trials above, confirmed a 3-fold higher reintervention rate within 30 days postoperatively in the lavage group [45/159 (28.3%) in lavage group versus 13/148 (8.8%) in the resection group]. This analysis concluded that leaving the diseased colon in place was associated with persistent intra-abdominal sepsis and increased need for reintervention in the laparoscopic lavage group.

pancreatic necrosectomy. parenteral antibiotics. percutaneous drainage. percutaneous aspiration for Gram stain and culture. observation.

Figure 21.1. 22. After a lengthy hospitalization, he is discharged and doing well—pain free, tolerant of a low-fat diet, and gaining weight A follow-up CT scan at 3 months interval is shown (figure 22.1). The next step in management should be

A 2017 meta-analysis included 589 patients recruited from the same 3 randomized controlled trials as well as 4 comparative studies (85% as Hinchey III). The laparoscopic lavage group had younger patients with higher BMIs and lower American Society of Anesthesiologists grades but comparable Hinchey classification and previous diverticulitis rates. No significant differences were noted for mortality, 30day reoperations, and unplanned readmissions. Laparoscopic lavage had higher rates of intra-abdominal abscesses (p .001), peritonitis (p = .002), and increased long-term emergency reoperations (p <.001). Benefits of laparoscopic lavage included shorter operative time, fewer cardiac complications, fewer wound infections, and shorter hospital stay. Overall, 90% had stomas after colon resection, and 74%

A. B. C. D. E.

77

observation. endoscopic transgastric cystgastrostomy. laparoscopic Roux-en-Y pancreatic cyst-jejunostomy. pancreatic enzyme supplementation. percutaneous drainage.

After recovery from the acute insult, some patients develop persistent fluid collections in the retroperitoneum or pseudocysts (figure 21-23.4). Management of these peripancreatic fluid collections is symptom driven. In the absence of pain with oral intake, early satiety, or ongoing pancreatic leak from duct disruption, observation is reasonable. If necessary, drainage can be accomplished surgically, endoscopically, or image guided. If the patient is in positive nitrogen balance and shows no signs of pancreatic enzyme insufficiency, supplementing the diet with enzymes is not necessary.

Figure 22.1 23. Six months after the initial consult, he is abstinent of alcohol, asymptomatic, and doing well. His interval CT scan is shown (figure 23.1). His pseudocyst has resolved, but he has developed splenic vein thrombosis and gastric varices indicating sinistral hypertension. His labs show mild thrombocytopenia. The next step in management is A. B. C. D. E.

Figure 21-23.4. pancreatic pseudocyst

laparoscopic splenectomy. oral anticoagulation. esophageal vascular transection. observation. distal splenorenal shunt.

A common sequelae of acute necrotizing pancreatitis is splenic vein thrombosis. This phenomenon is much more commonly noted with modern cross-sectional imaging (figure 21-23.5). It was previously thought that left-sided elevated visceral venous pressures (sinistral hypertension) would lead to gastroesophageal varices and bleeding and thus mandated prophylactic splenectomy. Observational studies show that clinically significant variceal bleeding requiring splenectomy is actually uncommon (4%); therefore, operative intervention is indicated only for clinically unmanageable hemorrhage from varices. Chronic sinistral hypertension does not routinely warrant systemic anticoagulation, especially in the face of varices, nor are any portosystemic shunting procedures necessary.

Figure 23.1.

ANSWER:

E, A, D

The management of acute necrotizing pancreatitis has evolved to a nonsurgical approach. Initial management of acute pancreatitis includes judicious resuscitation and evaluation for any end-organ dysfunction. In the absence of any signs of sepsis, antibiotics and debridement are not routinely indicated in the initial inflammatory phase of the illness, even though 20 to 30% of patients develop sterile pancreatic necrosis.

Figure 21-23.5. Gastric varices 78

thought to be appropriate for immediate surgery, multiagent chemotherapy with neoadjuvant intent or palliative care is reasonable. An assessment of patient fitness for surgery and a review of imaging by a surgeon with expertise in hepatopancreatobiliary surgery is critical for patient treatment planning. Hepaticojejunostomy with gastrojejunostomy (double bypass) is rarely indicated as a primary treatment option and should be considered in a palliative situation if other treatment modalities are not possible.

24. A healthy 72-year-old woman develops obstructive jaundice. A contrasted CT scan is notable for dilation of the biliary and pancreatic ducts but is otherwise unremarkable. An endoscopic retrograde cholangiopancreatography exam (figure 24.1) shows a tight stricture in the pancreatic portion of the common bile duct, and no gallstones are found. What is the best option for management of this patient? A. B. C. D. E.

Neoadjuvant multiagent chemotherapy Placement of a metallic uncovered stent Palliative care consult Hepaticojejunostomy with gastrojejunostomy Referral to a hepatobiliary surgeon at a multidisciplinary cancer center

25. A 70-year-old man presents to outpatient clinic with complaints of pneumaturia and recurrent urinary tract infection. CT scan demonstrates sigmoid diverticulitis (figures 25.1 and 25.2). Which of the following statements is true concerning the management of this patient? A. Colonoscopy is indicated to rule out malignancy. B. Sigmoid resection by laparoscopic approach is contraindicated. C. Cystoscopy is mandatory before surgical treatment. D. End colostomy or loop ileostomy is preferred treatment, H E. Barium enema is required to confirm location of colovesical fistula.

Figure 24.1.

ANSWER:

Figure 25.1

E

Nationwide in the United States, less than 30% of patients with potentially resectable peripancreatic malignancies ever come to surgery for treatment of their disease. There is even more disparity in the use of surgery as a primary treatment in minorities, underprivileged patients, and those older than 65. Proximity to a National Cancer Institute designated center is independently related to the use of surgery as a treatment for peripancreatic malignancy. Also, pancreatic resection is independently associated with prolonged disease-specific survival in large observational studies of localized malignancies. In this patient, treatment with curative intent includes placement of a temporary plastic biliary stent and evaluation for surgical intervention for cure. If after evaluation in a multidisciplinary format, a patient is not

Figure 25.2.

79

ANSWER:

B. C. D. E.

A

Diverticular disease of the colon is common. Colovesical fistula remains a rare complication of diverticulitis that typically involves the sigmoid colon. Other potential causes of colovesical fistula are trauma, radiation, inflammatory bowel disease, and malignancy. Because of the potential risk for a locally invasive colon cancer presenting with colovesical fistula, a colonoscopy should be performed in all patients presenting with a clinical suspicion for a colovesical fistula.

annual surveillance endoscopy. measurement of Vitamin B12. continuation of PPI therapy. treatment for Helicobacter pylori.

ANSWER:

D

Gastric polyps are found in nearly 4% of individuals undergoing esophagogastroduodenoscopy. Fundic gland polyps represent the overwhelming majority of these polyps in the US population, with fewer than 1% being gastric adenomas. They are sporadic and typically few in number and almost exclusively occur in the absence of Helicobacter pylori infection. Although patients taking proton pump inhibitors (PPIs) may have large numbers of these polyps that can regress with discontinuation of PPIs, the causative role of PPI therapy is not conclusive. Thus, typical, small polyps (<1 cm) confirmed to be fundic gland polyps do not require discontinuation of PPI therapy or annual surveillance endoscopy.

Most patients present with varying degrees of pneumaturia, fecaluria, and recurrent urinary tract infections. CT scan findings of proximity of bowel to bladder and air in the bladder are consistent with colovesical fistula (figure 25.3), Cystoscopy is often performed in patients with genitourinary problems and can demonstrate chronic inflammation, colon contents in the bladder, or a frank fistulous opening. However, a patient presenting with typical history, symptoms, and GT scan findings does not require a cystoscopy before definitive surgical

Pathologic confirmation of a single or few small polyps by endoscopic biopsy is sufficient, without need for further endoscopic surveillance. There is a small risk of dysplasia that increases in larger polyps (>1 cm); thus, PPI therapy is usually discontinued for larger polyps after endoscopic removal. Partial gastrectomy is not indicated for small fundic gland polyps without dysplasia, because malignant transformation is rare. Atrophic gastritis, but not fundic gland polyps, is associated with the presence of Vitamin B12 deficiency. Adenomatous polyps, conversely, are neoplastic lesions that occur sporadically or in association with familial adenomatous polyposis. These lesions are at an increased risk of malignant transformation and warrant more aggressive treatment.

Figure 25.3. Treatment. Similarly, a barium enema is not required preoperatively to confirm location of the colovesical fistula.

27. A 55-year-old man presented with upper gastrointestinal bleeding. He underwent diagnostic esophagogastroduodenoscopy, which confirmed bleeding from a type HI gastric ulcer. Epinephrine injection was used for endoscopic hemostatic therapy, and intravenous proton pump inhibitors were initiated. Recurrent upper gastrointestinal bleeding occurs 12 hours later, and he is hemodynamically normal. What is the best next step in his management?

Definitive treatment is resection of the involved colon with restoration of gastrointestinal continuity, with or without repair of the involved bladder. A laparoscopic approach is standard and is associated with reduced postoperative pain a and hospital length of stay. A diverting stoma may be necessary in select cases in which there is heavy pelvic contamination or vascular compromise, but it is not preferred.

A. B. C. D. E.

26. A 65-year-old woman is on proton pump inhibitors (PPIs) for chronic gastroesophageal reflux. Upper endoscopy was completely normal, with the exception of a solitary 0.5-cm gastric polyp that was completely excised. The pathology report was consistent with a fundic gland polyp without dysplasia. The next step in management is

Vagotomy and pyloroplasty Repeat endoscopic hemostatic therapy Selective angioembolization Vagotomy with antrectomy Duodenotomy with oversew of the ulcer

ANSWER:

A. partial gastrectomy.

80

B

The modified Johnson classification contains 5 types of peptic ulcers (table 27.1). In this case, a type HI ulcer is located in the prepyloric area and is associated with high add levels (figure 27.2).

Type Location

Incidence Acid level

I

In the antrum, near the lesser curve

55%

Normal

II III

Body of stomach and duodenum

25%

High

Prepyloric ulcer (within 2-3 cm of pylorus) 15%

High

IV V

Gastric ulcer in proximal stomach or cardia 5% Anywhere, induced by chronic NSAID use

Normal Normal

Surgery is not indicated as the next step in management, because randomized controlled trials show that repeated endoscopic treatment reduces the need for surgery without increasing mortality risk and are associated with fewer complications than surgery in patients with peptic ulcers and recurrent bleeding after initial endoscopic bleeding control. Angioembolization is reserved for nonvariceal upper gastrointestinal bleeding that cannot be managed endoscopically.

28. A 41-year-old woman underwent laparoscopic appendectomy for appendicitis diagnosed by CT scan. Surgical pathology of the resected specimen confirmed lowgrade appendiceal mucinous neoplasm (LAMN) confined to the muscularis propria (Tis). The next step in management is

Table 27.1. Endoscopic hemostatic therapy is indicated to control bleeding, but injection of dilute epinephrine alone is inadequate. A meta-analysis of 16 randomized controlled trials compared injection of diluted epinephrine alone with injection followed by a second modality. Combination treatment led to substantial reductions in rate of recurrent bleeding (from 18.4% to 10.6%), surgery (from 11.3% to 7.6%), and mortality (from 5.1% to 2.6%).

A. B. C. D. E.

Other meta-analyses showed that a second modality should be added to injection treatment Evidenced-based recommendation issued in 2012 from the National Clinical Guideline Centre are as follows: • •

right hemicolectomy. ileocecectomy. laparoscopy in 6 months. no further therapy. hyperthermic intraperitoneal chemotherapy.

ANSWER:

D

The Peritoneal Surface Oncology Group International (PSOGI) recognized persistent lack of uniform diagnostic terminology in appendiceal mucinous neoplasia. PSOGI convened a working group of 71 participants (surgical pathology, surgical oncology, medical oncology) from 13 countries to discuss appendiceal mucinous neoplasia and adopt a consensus diagnostic terminology.

Do not use adrenaline as monotherapy for the endoscopic treatment of nonvariceal upper gastrointestinal bleeding. For the endoscopic treatment of nonvariceal upper gastrointestinal bleeding, use one of the following: o a mechanical method (for example, clips) with or without adrenaline. o thermal coagulation with adrenaline. o fibrin or thrombin with adrenaline.

The definition of low-grade appendiceal mucinous neoplasm (LAMN) is a mucinous neoplasm with low-grade cytology. "High-grade appendiceal mucinous neoplasm" was proposed as the new term for lesions without infiltrative invasion but with high-grade cytologic atypia. The term "cystadenoma" is no longer recommended and "mucinous adenocarcinoma" is reserved for lesions with infiltrative invasion. Pseudomyxoma peritonei is characterized by mucinous ascites and peritoneal implants generally originating from a perforated mucinous tumor of the appendix. Patients with pTis (LAMN) do not develop tumor recurrence and are essentially cured by appendectomy. No further therapy is indicated. Right hemicolectomy, ileocecectomy, hyperthermic intraperitoneal chemotherapy, and repeat laparoscopy in 6 months are not indicated.

29. What is the best treatment option for a hemodynamically normal patient with a third recurrence of Clostridium difficile colitis who has persistent abdominal cramping and watery diarrhea despite treatment with oral vancomycin and fidaxomicin?

Figure 27.2. Classification of gastric ulcers 81

A. Adding loperamide and simethicone and continuing 4 more weeks of oral vancomycin B. Probiotics including Saccharomyces boulardii and Lactobacillus C. Diverting loop ileostomy with colonic lavage D. Total colectomy E. Fecal transplantation

ANSWER:

in the rectosigmoid that occurred during retroflexion. In addition to intravenous antibiotics, what is the next step in management? A. B. C. D. E.

Insertion of a presacral drain Colostomy diversion Sigmoid resection Endoscopic clip closure of the defect Placement of a covered stent

E ANSWER:

As many as 26% of patients with initial Clostridium difficile infections (CDI) will not respond to conventional antibiotic regimens consisting of metronidazole and oral vancomycin. Effective treatment strategies for recurrent CDI are challenging. The Infectious Diseases Society of America and the American College of Gastroenterolog published guidelines on treatment of recurrent cases of CDI. In general, repeated, pulsed, and extended courses of oral vancomycin and fidaxomicin are recommended but this regimen becomes progressively less effective with each recurrence.

D

The incidence of iatrogenic colonic perforation during screening colonoscopy is low (<1%) but is often associated with significant morbidity and possible mortality. Risk factors for perforation include anticoagulation, contamination, malignancy, and use of electrocautery and polypectomy. Therapeutic colonoscopy carries a higher risk of perforation than screening colonoscopy. Immediate identification and repair of perforation is important, because a delayed diagnosis can lead to peritonitis, sepsis, and death. Perforations can be managed operatively or nonoperatively, with recent data supporting a minimally invasive approach.

Recently, fecal transplantation, consisting of enteral or colonic infusion of feces from healthy donors, emerged as an alternative treatment strategy for patients with chronic or recurrent CDI. Multiple studies in aggregate documented more than 300 patients successfully treated for CDI with fecal transplantation.

Nonoperative management of a patient is appropriate when the patient is asymptomatic with free air alone. These patients can be managed with oral antibiotics, serial abdominal examinations, and bowel rest. Insertion of a presacral drain is not indicated. Placement of a covered stent is also not necessary.

A recent New England Journal of Medicine study prospectively randomized patients with CDI refractory to vancomycin or metronidazole to receive 1 of 3 therapies: oral vancomycin for 14 days; oral vancomycin plus bowel lavage; or oral vancomycin with bowel lavage followed by infusion of a solution of healthy donor feces via nasoduodenal tube. Eighty-one percent of patients treated for recurrent CDI with the infusion of donor feces had resolution of the infection. Fecal transplant was a much more effective treatment strategy than oral vancomycin or bowel lavage.

Advances in endoscopic technology make endoscopic clip closure an ideal approach to the management of a small (<10 mm) colonic perforation that occurs during a screening colonoscopy. Intraluminal endoclipping allows repair of small perforations recognized at the time of colonoscopy. Endoscopic repairs should be performed with minimal air insufflation to avoid distention of the lumen and incomplete closure of the defect.

Probiotics are used to prevent and treat CDI, presumably to repopulate the colonic microflora. Lactobacillus, Bifidobacterium, and Saccharomyces are commonly used. No randomized controlled trials support the use of probiotics as a treatment of CDI.

Operative management (open or laparoscopic surgery) is indicated in patients with diffuse peritonitis and contamination or in patients who fail nonoperative or endoscopic management. Several surgical options exist. The best option depends on the patient's underlying pathology, the size of the perforation, and the surgeon's skill level. When done early, primary closure of the perforation with or without resection can be performed given the preceding bowel preparation. Colonic diversion is used only when patient is found to have significant peritoneal contamination.

Subtotal or total colectomy is the recommended surgical approach to fulminant CDI. Loop ileostomy with intraoperative colonic lavage is also described as an option for fulminant CDI. Neither surgical approach is used for recurrent CDI. Although the antimotility agent loperamide may reduce the duration of diarrhea, it may cause fecal stasis resulting in dangerous progression of CDI. Consequently, loperamide is not routinely recommended.

30. While performing a screening colonoscopy on a 66-yearold healthy male patient, you note a 5-mm, full-thickness tear

82

Alimentary Tract Part III

2. A 21-year old man with cystic fibrosis presents with crampy abdominal pain with distention. He has no surgical history. CT scan shows dilated loops of small bowel and heavy stool burden in his colon. After attempts at oral laxative treatment without results, what is an appropriate next step?

ITEMS 1-30 For each question, select the best possible response. 1. A 51-year-old patient presents with abdominal distention, nausea, and bilious vomiting 2 years after a cholecystectomy. He has not passed flatus for the last day. The patient is afebrile with normal vital signs. Exam demonstrates mild distention, tympany, and mild tenderness. A complete blood count is within normal limits. An abdominal CT scan is obtained, and the radiologist reports a small bowl dilation and a transition point with no other abnormalities. What is the appropriate next step? A. B. C. D. E.

A. B. C. D. E.

ANSWER:

Exploratory laparotomy Gastrografin small bowel follow through Diagnostic laparoscopy Gastrografin enema CT enterography

ANSWER:

Exploratory laparotomy Abdominal ultrasound Diagnostic laparoscopy Gastrografin enema CT enterography

D

Cystic fibrosis is an autosomal recessive genetic disorder that results in the thickening of many normally thin secretions throughout the body. Pulmonary toilet of these thick secretions is a lifelong challenge and results in frequent infections. The disease also causes abnormal pancreatic and intestinal function, which at times results in thickened small bowel succus and fecal material. Because this material can be difficult to pass through the gastrointestinal tract, patients may exhibit signs and symptoms of constipation or even large or small bowel obstructions. Supplemental digestive enzymes can prevent this from occurring.

B

The dictum that the sun neither rises nor sets on a small bowel obstruction is sometimes not appropriate because some patients do not present with worrisome physiology such as fever, leukocytosis, hypotension, tachycardia, or peritoneal signs on physical exam. Many small bowel obstructions resolve without surgical intervention. Recent data from a retrospective review and prospective trial suggest that absent signs of obstipation for more than 12 hours, CT scan findings of mesenteric edema, and fecalization of the small bowel, a Gastrografin challenge can be used to predict the need for operative management. This test uses 100 mL of watersoluble contrast, diluted with 50 mL of water, and infused via a gastric tube. An abdominal radiograph is taken 8-hours later. Visualized contrast in the colon suggests a nonobstructed small bowel and strongly predicts the ability to avoid surgery.

When intestinal obstruction results from thickened succus or fecal material (sometimes called meconium), imaging will show heavy stool burden distal to the dilated or obstructiveappearing intestinal loops. Laparotomy or laparoscopy with or without resection is the last solution. Gastrografin, a water soluble contrast, can pull fluid into the intestinal lumen relieving the obstruction caused by the inspissated meconium. Repeated applications are sometimes required. In this case, the diagnosis was made by CT scan. Further imaging, such as ultrasound or CT enterography, is not indicated.

Peritonitis or other signs of strangulated bowel should prompt immediate surgery. Likewise, the presence of fever, leukocytosis, hypotension, or tachycardia suggests the bowel is at risk for ischemia and should also prompt abdominal exploration. In this patient, such features were not reported, and an initial nonoperative evaluation with Gastrografin is appropriate. A contrast enema is not helpful because contrast is limited to the large intestine and would not interrogate the small bowel. CT enterography, although possible, is timeconsuming and not used in the acute setting. Diagnostic laparoscopy would not be indicated because the initial approach is nonoperative. Once the decision is made to intervene operatively, the next decision is whether to proceed in an open or laparoscopic fashion. This decision will depend on the patient presentation and the surgeon's expertise in laparoscopic surgery.

Items 3-6 Each lettered response may be selected once, more than once, or not at all. A. Follow-up surveillance colonoscopy now B. Follow-up surveillance colonoscopy in less than 3 years C. Follow-up surveillance colonoscopy in 3 years D. Follow-up surveillance colonoscopy in 5 years E. Follow-up surveillance colonoscopy in 10 years 3. 2-cm tubular adenoma, completely removed on screening colonoscopy 4. Multiple (12) adenomas found on screening colonoscopy 5. 1-cm tubular adenoma, incompletely removed on screening colonoscopy 6. No abnormalities seen on screening colonoscopy

84

ANSWERS:

including polyp size, the number of polyps, presence of gallstones or cholecystitis, and the patient's age and ethnicity (figure 7.1). In general, gallbladder polyps smaller than 10 mm are benign. However, a growing solitary polyp with a sessile morphology, coexisting with cholelithiasis, should raise concern for potential malignancy. Patients older than 60 and of Indian ethnicity are at higher risk. The probability that this patient has a covert gallbladder malignancy is very low, and she can be safely followed with ultrasound in 6 months, monitoring for polyp growth. Further imaging (MRI or endoscopic ultrasound) is not necessary, given the low malignancy suspicion. Patients with smaller polyps can be followed less frequently (figure 7.2).

C, B, A, E

The timing of follow-up surveillance or screening colonoscopy depends on the finding of the original study. Several groups, including the US Multi-Society Task Force (MSTF) on Colorectal Cancer and the European Society of Gastrointestinal Endoscopy (ESGE), have developed and updated recommendations. Guidelines are based on the risk of developing subsequent colorectal cancer and assume the initial colonoscopy thoroughly evaluates the mucosa throughout the colon and rectum, visualizes the ileocecal valve, and completely removes all visualized polyps. In general, any incomplete polyp removal requires a repeat colonoscopy or another attempt at removal Recent updates also account for evidence that some high-risk patients may develop cancer within 3 to 5 years of colonoscopy and polypectomy and hope to improve guideline adherence. Table 3-6.1 summarizes both sets of guidelines. Finding at Screening Colonoscopy Recommended Interval (years) MSTF ESGE Low No polyps 10 10 Risk <10 mm hyperplastic rectal or sigmoid polyps 10 10 3-10 tubular adenomas 5-10 3 >10 adenomas 3 3 and genetic counseling High Tubular adenoma >10 mm 3 3 Risk Villous adenoma 3 3 Adenoma with high-grade3 3 dysplasia

Table 3-6.1. US Multi-Society Task Force (MSTF) on Colorectal Cancer and the European Society of Gastrointestinal Endoscopy (ESGE) guidelines on the timing of follow-up surveillance or screening colonoscopy.

7. A 50-year-old woman who underwent a right upper quadrant ultrasound for vague postprandial symptoms has a 0.5-cm echogenic area in the gallbladder fundus that does not move with adjustment of the ultrasound probe or change of her physical position. Her ultrasound exam is otherwise normal, without cholelithiasis, sludge, or local discomfort during the exam. She is referred for possible cholecystectomy. What would you advise? A. B. C. D. E.

Repeat ultrasound in 6 months MRI Laparoscopic cholecystectomy Open cholecystectomy with resection of the hepatic bed Endoscopic ultrasound

ANSWER:

Figure 7.1. Recommended guidelines for management of gallbladder polyps

A

Gallbladder polyps are common, and most are benign cholesterol polyps. Regardless, patients are often referred for cholecystectomy out of concern for potential malignancy. The recommendation as to which patients can be observed and which require intervention depends on several factors,

85

advanced histologic features in less than 3% of such lesions, as well as less frequent exams. Repeat colonoscopy before 10 years after a negative, uncompromised colonoscopy appears to offer little benefit CT colonography is increasingly popular as a screening tool. Sensitivity for detection of adenomas of at least 6 mm is 80%, although radiologists do not routinely report lesions detected by CT coIonography that are smaller than 6 mm. The current recommendation is that polyps 6 to 9 mm, although likely to be benign, be removed by colonoscopy or followed with a second CT colonography in 3 years. For lesions larger than 10 mm, sensitivity approaches 88%. Sessile polyps can be difficult to identify by colonoscopy or CT colonography due to their flat morphology, primarily in the cecum. However, with careful preparation and attention to evidence of contrast adherence to the mucosa, identification of these worrisome lesions can match screening colonoscopy. Patient compliance may be greater for CT colonographic screening, but colonoscopy remains the only available one-step approach to colorectal cancer screening.

**Probability of malignancy (p) Risk Factors Single polyp Sessile polyp Patient age ≥50 Sessile, single polyp Patient age ≥50 & single polyp Patient age ≥50 & sessile polyp

Probability (p) 4.3% 13.9% 20.7% 24.8% 34.8% 65.6%

Patient age ≥50 & sessile, single polyp

79.6%

Stool DNA tests are becoming widely available. These tests detect tumor-associated genetic and epigenetic markers in cells shed in the stool. A recent cross-sectional study showed a sensitivity of 92% for a multitarget stool DNA for detecting colorectal cancer in a screening population, but it is less sensitive (42%) for detecting advanced adenomas. Fecal occult testing (FIT) has a 74% sensitivity for colorectal cancer and 24% for advanced adenomas. However, specificity of the FIT for nonadvanced or negative findings is superior to stool DNA testing (96.4% vs 89.8%). Given its false-positive rate and current cost, stool DNA testing has not supplanted FIT testing in the screening of low-risk patients.

Figure 7.2. Pragmatic clinical decisionmaking algorithm for gallbladder polyps.

8. Which of the following statements about screening tests for colorectal cancer is true? 9. One year after a Toupet fundoplication, a 56-year-old man undergoes surveillance upper endoscopy for his known Barrett esophagus. A transition from squamous to Barrett epithelium is noted to begin 32 cm from the teeth wjthout evidence of esophagitis. Four-quadrant biopsies immediately below the transition show low-grade dysplasia. On his gastroenterologist's advice, he has continued omeprazole 40 mg twice a day since the operation Upper gastrointestinal studies confirm an intact fundoplication with no active reflux. What would you advise for his endoscopic findings?

A. CT colonography is superior to colonoscopy for detecting sessile-serrated polyps. B. 6- to 9-mm polyps identified on CT colonography need follow-up in 3 years. C. Only polyps larger than 10 mm need to be removed at colonoscopy. D. Repeat colonoscopy before 10 years of a negative, uncompromised examination is recommended. E. Multitarget stool DNA testing has replaced fecal occult blood testing for screening average risk patients.

ANSWER:

A. B. C. D. E.

B

Colon cancer screening recommendations are evolving to improve patient compliance. Colonoscopy remains the current standard screening modality. Because more than 99% of diminutive polyps are benign, use of less invasive, more economical approaches to the management of patients at low risk for malignancy with polyps smaller than 1 cm are being assessed. Proposed strategies include removing and discarding diminutive polyps given the low incidence of

Conversion to Nissen fundoplication Radiofrequency ablation Repeat biopsies in 8 weeks Repeat endoscopy with biopsy in 6 months No further surveillance

ANSWER:

B

Barrett esophagus secondary to chronic reflux is currently the only identified precondition for esophageal adenocarcinoma. Antireflux procedures are touted as a means of limiting the

86

rate of endothelial dysplastic progression, and some studies showed efficacy in this regard. Management of residual lowgrade dysplasia, however, remains controversial. Reviews of patients undergoing surveillance have led to several best practice recommendations. The reported rate of progression from low-grade to highgrade dysplasia is highly variable (0.4-13.4% per year) and is dependent on the pathologist's interpretation of biopsy specimens. The presence of active erosive esophagitis will potentially confound diagnosis and biopsy in this setting should be avoided. The proximal half of the Barrett esophagus segment is most vulnerable to dysplasia. Persistent low-grade dysplasia on consecutive endoscopies, nodularity, and multifocality are risk factors for Barrett esophagus progression and for failure to regress after an antireflux procedure. Short-segment Barrett esophagus, smoking cessation, and proton pump inhibitor use are associated with regression. Eradication of Barrett esophagus should be encouraged. Endoscopic resection of Barrett esophagus offers a larger pathologic specimen for accurate grading of any dysplasia. Radiofrequency ablation significantly slows progression to high-grade dysplasia. This patient has persistence of a relatively long segment of Barrett esophagus 1 year after a successful antireflux procedure and has continued an intensive medical antireflux regimen. Repeat biopsies in 8 weeks would be appropriate if esophagitis was present. The most effective management choice to affect potential dysplasia progression is radiofrequency ablation, with a goal of complete endoscopic and histologic eradication of intestinal metaplasia. The current regimen involves treatment every 3 months for a maximum of 2 circumferential or 3 focal sessions. Should the patient refuse ablation, repeat endoscopy with biopsies in 4 quadrants every 1 to 2 cm at 6 and 12 months is recommended.

Figure 10.1.

10. A 28-y ear-old man undergoes an abdominal magnetic resonance cholangiopancreatography (figures 10.1 and 10.2) as part of an evaluation for abdominal pain and diarrhea. He has an elevated amylase and mild hyperbilirubinemia. Which of the following statements regarding his condition is true?

Figure 10.2

A. Cholangitis is a rare complication. B. Endoscopic ultrasound is required to confirm the diagnosis. C. Complete extrahepatic duct excision with cholecystectomy is recommend D. Placement of a biliary stent is preferred treatment. E. Malignancy is present in more than 50% of patients.

ANSWER:

C

This patient has a type I choledochal cyst, a fusiform dilation of the distal common bile duct (figure 10.3). Unlike children, adults with a choledochal cyst present with biliary or pancreatic symptoms associated with abdominal pain.

87

Concern for malignancy in choledochal cysts is the primary reason for surgical referral. The incidence in adults is approximately 12% overall, peaking at approximately 38% in the sixth or seventh decades. Although cancer can occur at any age, the incidence is less than 6% before age 30. Cholangiocarcinoma is most commonly associated with types I and IV cysts. If carcinoma is suspected, endoscopic ultrasound may be useful as a staging modality before operation. Management requires complete extrahepatic bile duct cyst excision down to the level of its communication with the pancreatic duct, cholecystectomy, and biliary-enteric reconstruction.

Figure 10.3. Classification of choledochal cysts (CC). Type I 11. What feature distinguishes a pancreatic intraductal papillary mucinous neoplasm from a mucinous cystic neoplasm?

cysts are fusiform dilatations of the common bile duct (CBD). Type II cysts are true diverticula of the CBD and type III CC (choledochoceles) are intraduodenal dilations of the common channel. Type IVA CC consist of multiple intrahepatic and extrahepatic biliary dilatations, while type IVB CC have extrahepatic biliary dilatation with a normal intrahepatic biliary tree. Type V CC, or Caroli's disease, consist of cystic dilation of the intrahepatic biliary tree. RHD right hepatic duct, LHD left hepatic duct, CHD common hepatic duct, DUO duodenum.

A. B. C. D. E.

Biliary stasis in the cyst contributes to symptoms often interpreted as cholecystitis, cholangitis, or symptomatic cholelithiasis. It is not uncommon for such patients to have previously undergone biliary stenting or cholecystectomy without resolution of their symptoms. Choledochal cysts are typically identified with multimodality imaging, including ultrasound, CT scan, and MRI with magnetic resonance cholangiopancreatography (MRCP). A common duct diameter greater than 10 mm on ultrasound should prompt further investigation. MRCP is highly sensitive (70-90%) and specific (90-100%) for both diagnosis and classification (figure 10.4).

Cyst fluid carcinoembryonic antigen level Cytology of mucus Cyst fluid amylase level Communication with the pancreatic duct Size greater than 3 cm

ANSWER:

D

Cystic lesions of the pancreas are commonly identified on abdominal imaging, and questions regarding their management often fall to the surgeon. Distinguishing the benign from malignant lesion (or potentially malignant lesion) and recommending appropriate management requires an understanding of the natural history of these lesions and an individual patient's oncologic risk. Although it is widely acknowledged that cyst size greater than 3 cm is a significant risk factor for malignancy in mucinous cystic neoplasms (MCNs), these solitary tumors typically occur in the body and tail of the pancreas and are not in communication with the main pancreatic duct. Intraductal papillary mucinous neoplasms (IPMNs), by definition, do communicate with the pancreatic duct Surgical resection is recommended for all main duct IPMN, Worrisome features for a branch duct IPMN include lesion size greater than 3 cm, thickened enhancing cyst walls, a nonenhancing solid component, an associated dilated main pancreatic duct, or an abrupt change in main pancreatic duct caliber with distal pancreatic atrophy and lymphadenopathy. Analysis of cyst contents obtained via endoscopic ultrasound is being studied as a means of distinguishing malignant from more benign lesions. Although elevated serum CA19-9 and carcinoembryonic antigen (CEA) are highly specific biomarkers associated with invasive pancreatic malignancy, cyst CEA levels alone cannot reliably discern benign from malignant and are variably present in both MCNs and IPMNs. Similarly, cyst amylase offers limited distinctions. Cyst fluid cytology can accurately diagnose a malignant pancreatic cyst lesion, but sensitivity is hampered by low cellular volume in mucous. The cytologic features of IPMNs and MCNs may be

Figure 10.4. MRI for type I choledochal cyst. 88

similar, each showing mucinous neoplastic epithelial cells. Although immunohistochemical analysis of mucous can distinguish the different histopathologic origin of an IPMN (e.g., differentiating gastric from intestinal or pancreatobiliary IPMNs), pancreatic mucin gene expression patterns, specific genetic mutations, and microRNA analyses remain under investigation as predictors of invasive cancer.

ANSWER:

C

Percutaneous endoscopic gastrostomy (PEG) tubes are placed for durable feeding access across a wide spectrum of diseases. The published complication rate is between 0.5 and 20%, with minor complications predominating. Typical complications associated with PEG tubes include those related to insertion and long-term maintenance. Insertionrelated complications include bleeding, hollow viscous or hepatic injury, and tumor seeding of the insertion site (seen in head and neck cases). Complications from long-term maintenance include abscess formation (including necrotizing soft tissue infections), distal tube migration, skin breakdown, and buried bumper syndrome (BBS; figure 12.3).

12. A 75-year-old man with dysphagia undergoes uncomplicated placement of a percutaneous endoscopic gastrostomy tube for durable feeding access. Approximately 3 weeks after placement, nurses at his facility notice pain with administration of feeds. He presents to the emergency department, and CT scan are obtained (figures 12.1 and 12.2). Other than pain with feeds, the patient is asymptomatic. What is the next step in the management of his tube? A. Urgent laparotomy with oversewing of the gastrotomy B. Wide local debridement of his anterior abdominal wall around his tube C. Tube removal D. Cutting the feeding tube flush to the skin and leaving it in situ E. Loosening of the retention flange

Figure 12.3. BBS occurs when the PEG fixation flange slowly migrates out of the intragastric space and into either the wall of the stomach or the anterior abdominal wall Typically, the stomach remains adherent to the abdominal wall and the presentation is an inability to administer feeds, with leakage around the PEG tube site. This complication is often seen after several months but can be seen as early as 3 weeks after placement. The etiology is likely due to too much tension on the flange causing slow erosion anteriorly of the bumper. The first step in management is to determine the etiology of the tube malfunction. Adherence of the stomach to the anterior abdominal wall should be confirmed. Patients with peritonitis, free air, or other findings consistent with an intraabdominal leak should be explored. For a true BBS, management can either be endoscopic or fluoroscopic. If the flange is partially visible on endoscopy, an attempt can be made to push it into the gastric lumen and then a new tube can be placed. If the bumper has completely migrated out of the stomach lumen, a wire can often be advanced through the old tube through the remaining small gastrocutaneous fistula. Alternatively, as in the case pictured, with axial evidence of the tube completely retracted into the abdominal wall, the tube should be removed, and the stomach can be accessed fluoroscopically to replace the tube or another percutaneous feeding tube can be placed. If there is significant tissue breakdown, the patient may require a period of wound

Figure 12.1.

Figure 12.2. 89

healing before replacing the tube. In the interim, the patient can be maintained on nasoenteric feeds, if necessary.

will go on to require surgical resection. The overall ischemic complications are low, ranging from 0 to 5% in the literature. This low rate is thought to be likely multifactorial and includes the use < very small microcatheters to restrict the embolization only to the distal arcades in the bleeding territory.

Cutting the tube flush to the skin will not address the problem and will likely lead to an abdominal wall abscess. This technique, described as "cut and leave," is reserved for only the direst of circumstances and patients without signs of local or systemic inflammation. Most patients, even those who are unable to tolerate any operative intervention, can have their tube removed at the bedside.

14. A 32-year-old woman recently completed a course of metronidazole for treatment of Clostridium difficile infection. She returns to the emergency department with complaints of abdominal pain and distension, watery diarrhea, and fevers. She is hemodynamically normal, and her white blood cell count is 11,000/mm3 (3600-11,200/mm3). Which of the following statements is true regarding her recurrent C. difficile infections?

Wide local debridement is necessary only if there is significant tissue destruction, abscess, or concern for a necrotizing process.

13. Which of the following statements is true regarding lower gastrointestinal bleeding?

A. Intravenous metronidazole is more effective than oral metronidazole. B. Oral metronidazole is more effective than oral vancomycin. C. Fidaxomicin is more effective than oral vancomycin. D. Duodenal infusion of donor feces is less effective than oral vancomycin, E. Administration of oral probiotics is more effective than oral vancomycin.

A. Superselective angioembolization carries an unacceptably high risk of bowel necrosis. B. Early colonoscopy for lower gastrointestinal bleeding is associated with increased colon perforation. C. Colonoscopy for lower gastrointestinal bleeding provides localization without options for successful hemostatic intervention. D. A rapid colon purge before early colonoscopy is safe. E. Patients with active bleeding will require surgical intervention.

ANSWER:

ANSWER:

C

Infectious colitis related Clostridium difficile infections have increased in incidence over the past 10 years. The number of asymptomatic carriers has increased, and the estimated incidence of infection in the United States is nearly 300,000 cases annually. Healthcare-related infections represent most infections, with up to half of these beginning in residents of long-term healthcare facilities. Vancomycin is used to treat initial episodes. Fidaxomicin, another macrocyclic antimicrobial similar to vancomycin, is also approved for the treatment of C. difficile; however, for initial infections, it is not superior to vancomycin. Metronidazole has been relegated to use when vancomycin or fidaxomicin are not available or for a mild C. difficile infection episode. Oral metronidazole is somewhat more effective than the same antimicrobial administered intravenously.

D

After resuscitation and stabilization, most lower gastrointestinal bleeds can be managed endoscopically. Colonoscopy is not only feasible but also safe and often therapeutic for the management of acute lower gastrointestinal bleeds. Several studies confirmed this finding. Despite advances in vascular and interventional radiology, endoscopy should be the first step in the diagnostic algorithm for lower gastrointestinal bleeds. Most lower gastrointestinal bleeds subside soon after presentation, so colonoscopy is often performed after a rapid colon purge A rapid bowel preparation is safe and allows for a more thorough examination of the colon, facilitating any therapeutic measures. Early colonoscopy decreases hospital length of stay. Therapeutic endoscopy includes epinephrine injection, thermal probe coagulation, and hemoclips placement. In one large study, no patient undergoing colonoscopic intervention for diverticular bleeds demonstrated a rebleed within the 3-year study period. This number was compared with 50% seen in historic controls where no endoscopic therapeutic maneuver was performed.

Recurrent infections occur in up to one-third of treated patients. The rate of recurrence increases with each recurrent episode (initial rate 15-30%, second recurrence 33-65%), emphasizing the importance of eradication and containment with the initial infectious episode. Vancomycin administered orally is more effective than metronidazole (either orally or intravenously) for the treatment of both initial and recurrent infections. Recent reviews of the available literature noted fidaxomicin to be superior to vancomycin for the treatment of recurrent infections.

For patients failing endoscopic management, both endovascular and surgical options exist. Superselective angioembolization (SSAE) is an option available to avoid surgical resection. Although the technical success rate of SSAE is documented at greater than 90%, one in 5 patients

Fecal transplant (either via nasoduodenal or endoscopic administration) has emerged as a nonantimicrobial method

90

for the treatment of infectious colitis related to C. difficile. The objective of fecal transplant is to restore gut microbiota to a healthy state. Low-strength evidence exists for the use of fecal transplant in the setting of recurrent C. difficile infections. Most of the studies to date have been small, although 3 notable randomized controlled trials have been conducted. The success rate for fecal transplantation in the setting of recurrent infections is between 60 and 100%. It appears to be superior to vancomycin in all of the randomized studies.

bleeding. Late complications include fecal incontinence and proctitis. Tenesmus is more common in stapled hemorrhoidopexy than in other methods of hemorrhoidectomy. Tenesmus is most likely from the presence of a low rectal suture. Septic complications are more common after stapled hemorrhoidectomy compared with all other techniques. The incidence of incontinence of feces and flatus is not lower with stapled hemorrhoidopexy compared with open hemorrhoidectomy.

Probiotics containing lactobacillus or other multiorganism mixes reduce the rate of C. difficile-related infections. Robust data demonstrating these agents as a method of reducing subsequent infections after a treatment course are lacking. Probiotic agents are not indicated for the treatment of an acute C. difficile infection (recurrent or otherwise.)

16. A 56-year-old woman presents with slowly progressive dysphagia for solids and liquids, with regurgitation of bland, undigested food, chest pain, weight loss, and chronic cough. Chest x-ray reveals absence of a gastric bubble, with the presence of a fluid-filled, dilated esophagus. Barium swallow reveals a dilated, aperistaltic esophagus with an air-fluid level and "bird's beak" appearance Esophageal manometry reveals a hypertensive lower esophageal sphincter and esophageal aperistalsis. Which treatment provides the best long-term relief of dysphagia, regurgitation, and weight loss?

15. Which of the following statements is true regarding stapled hemorrhoidopexy for a patient with grade 3 internal hemorrhoids? A. A complete bowel cleansing is necessary. B. The incidence of incontinence to feces and flatus is lower than with open hemorrhoidectomy. C. Tenesmus is common postoperatively. D. The rate of anal stenosis is higher compared with open hemorrhoidectomy. E. Postoperative pain is less compared with banding.

ANSWER:

A. B. C. D. E.

C

Thoracoscopic Heller myotomy Transhiatal esophagectomy Laparoscopic Heller myotomy Pneumatic dilation of the lower esophageal sphincter Botulinum toxin injection of the lower esophageal sphincter

ANSWER:

Hemorrhoidectomy can be performed as a closed, open, or semiopen procedure. For all 3, full mechanical bowel preparation is not necessary. Perioperative antibiotic administration provides no benefit. The open resection is associated with more complications, including stenosis. In a 3-arm randomized controlled trial (open, closed, or stapled), the worst outcomes were associated with the open technique. The closed and stapled approaches were associated with less stenosis and less pain.

C

Achalasia is an esophageal motility disorder with no known etiology. Patients present with symptoms of dysphagia, regurgitation of undigested food, and chest pain. Review of pathology slides reveals degeneration of the myenteric plexus. This results in a lack of peristalsis of the esophageal body as well as absence or incomplete relaxation of the lower esophageal sphincter. Esophagectomy is not indicated for motility disorders. Endoscopic injection of botulinum toxin into the lower esophageal sphincter is done with the intent of blocking the release of acetylcholine from the cholinergic neurons to lower both the basal and residual lower esophageal sphincter pressures. Although patients experience relief from dysphagia, this relief is short lived and requires repeated injections for continued relief. Lower esophageal sphincter pressure is not substantially reduced. Repeated injections create an inflammatory reaction in the distal esophagus, with resultant submucosal fibrosis, which can make subsequent myotomy more difficult.

In the stapled hemorrhoidopexy, a stapling device is used. This device resects and fixes the internal hemorrhoid tissues to the adjacent rectal wall. The staple line is above the dentate line, which produces less pain than a surgical hemorrhoidectomy. Rectal prolapse and tenesmus are higher with stapled hemorrhoidopexy. This approach is associated with less pain, less operating room time, and more rapid return to normal activity. Patients who had a stapled hemorrhoidopexy have an earlier return to work. Specific complications are associated with stapled hemorrhoidopexy. These include failure of the staple gun, urosepsis, and pelvic sepsis with rectal perforation. The most common complication of stapled hemorrhoidopexy is early

91

Pneumatic dilation decreases esophageal outflow resistance by tearing the fibers of the lower esophageal sphincter, with attendant risk of perforation. Its efficacy is longer lasting than botulinum toxin injection but not as efficacious as surgical myotomy, with only 38% of patients experiencing long-term symptom relief.

disease. The theory is that lymphoid cells accumulate in the stomach in response to H. pylori infection. The B cells proliferate monoclonally, resulting in a low-grade lymphoma. This proliferation is thought to require both the antigenic stimulation by H. pylori as well as production of IL-2 by normal T cells. Without treatment, low-grade lymphoma can progress to high-grade lymphoma.

Laparoscopic Heller myotomy is the most effective treatment for achalasia, with long-term symptom relief of dysphagia, regurgitation, and chest pain approaching 80% at 10 years.

Prognosis is better than that of equivalent non-Hodgkin lymphoma. Low-grade MALT lymphomas are typically localized and slow to spread to lymph nodes and other sites. As might be expected, survival for low-grade lymphoma is better than for high grade lymphoma.

Thoracoscopic Heller myotomy, although efficacious, has largely been abandoned due to a combination of cumbersome intraoperative management (double lumen endotracheal tube for single lung ventilation), postoperative pain related to tube thoracostomy, and an incidence of postoperative gastroesophageal reflux of up to 60%.

Initial treatment for low-grade MALT lymphoma should include the eradication of H. pylori, which results in successful and durable regression of the MALT lymphoma in most cases. Predictors of response to treatment include depth of penetration of the gastric wall (deep to the submucosa is less responsive), absence of AP-12-MALT1 translocation in the lymphoma cells, gastric site (proximal is more worrisome), microsatellite instability, and advanced age.

Per-oral endoscopic myotomy (POEM) is a new technique in which entry into the submucosal space allows dissection of the submucosa distally along the muscular layer using spray coagulation. This creates a submucosal tunnel that extends beyond the gastroesophageal junction. Myotomy of the circular esophageal and gastric muscle bundles can then be performed under direct vision. The mucosal incision is closed with hemostatic clips. POEM is safe, less invasive than conventional surgery, and appears to offer durable symptom relief approaching that of Heller myotomy, with very promising 3-year data.

If the patient does not have H. pylori infection or does not respond to treatment for these bacteria, treatment is then radiation therapy, with chemotherapy as salvage for radiation failure. Surgical excision for gastric MALT lymphoma is not considered appropriate therapy. Imatinib, a tyrosine kinase inhibitor, is used for treatment of gastrointestinal stromal tumors but is not described for treatment of gastric MALT lymphoma.

17. A 34-year-old man presents with epigastric pain that has persisted despite dietary modifications and proton pump inhibitor therapy. Upper endoscopy reveals a flat, slightly irregular, 3-cm mass in the distal stomach. Biopsy reveals low-grade lymphoma of mucosa-associated lymphoid tissue. The AP-12-MALT1 mutation was absent. Endoscopic ultrasound reveals no penetration of the submucosa and no involvement of perigastric nodes. What is the best initial management? A. B. C. D. E.

18. A 36-year-old woman undergoes upper endoscopy for evaluation of gastroesophageal reflux. No esophagitis or hiatal hernia is visualized. In the distal stomach, 3 subcentimeter polypoid lesions are visualized and biopsied. Pathology reveals carcinoid tumor. Repeat upper endoscopy with random biopsies of the antrum and fundus reveals no evidence of occult malignancy, but atrophic gastritis is identified. Endoscopic ultrasound reveals these lesions to be superficial. CT scan is unrevealing. Serum gastrin is markedly elevated. Chromogranin A is mildly elevated. Parietal cell and intrinsic factor antibodies are positive, consistent with autoimmune atrophic gastritis. Serum Vitamin B12 is low. The best management for these lesions is

Distal gastrectomy Imatinib Treatment for Helicobacter pylori Chemoradiation Subtotal gastrectomy

ANSWER:

C

A. B. C. D. E.

Gastric lymphoma accounts for 2 to 7% of primary gastric malignancies. The endoscopic appearance is variable and can be flat, polypoid, and ulcerated. It is primarily seen in the middle to distal third of the stomach, with multifocal and diffuse disease typical. Most primary gastric lymphoma arise from B cells, closely resembling mucosa associated lymphoid tissue (MALT). The connection of MALT lymphoma and Helicobacter pylori infection is well established, with these bacteria present in the stomachs of most patients with this

observation. endoscopic removal. antrectomy. vagotomy and pyloroplasty. total gastrectomy.

ANSWER:

B

Gastroenteropancreatic neuroendocrine tumors (NET or carcinoids) occur at a rate of 35 per 100,000 population.

92

Type I and type II gastric NETs that are less than 1 cm and without risk factors (muscle invasion, increased proliferation, angioinvasion) can be managed with either endoscopic surveillance (every 1-2 years) or endoscopic removal. Endoscopic removal is favored over observation and surveillance if there are fewer than 6 nodules, as in this patient. There is no survival benefit for surgical resection compared with endoscopic removal in this setting.

Treatment is based on correct World Health Organization (WHO) classification, which includes well-differentiated NETs (this patient), well differentiated neuroendocrine carcinomas, and poorly differentiated neuroendocrine tumors of the stomach. A tumor-node-metastases (TNM) staging and a grading system is used to further categorize these tumors. TNM stage is determined by tumor size, depth of invasion, and presence or absence of lymph node or liver metastases. Grading depends on proliferative activity (Ki-67 index, mitotic rate). A separate clinicopathologic categorization of NETs is also used, independent of the WHO and TNM classification, and breaks these into 4 types (table 18.1).

Solitary, larger gastric NETs often have associated risk factors and are managed surgically, with antrectomy for tumors larger than 1 cm and total gastrectomy with oncologic lymph node resection for poorly differentiated neuroendocrine carcinoma of the stomach.

Table 18.1. Features of gastric neuroendocrine tumors.

Vagotomy and pyloroplasty would address neither the

Type 1 GNET

Type II GNET

Type III GNET

Type IV GNET

Proportion of GNETs

70-80%

5-6%

14-25%

Rare

Tumor features

Usually multiple, small (1-2 cm), polypoid

Single, large (>2 cm, mean 5.1 cm)

Single, large (largest 16 cm)

Risk of metastases

Usually multiple, small (1-2 cm), polypoid or intramucosal 2-5%

10-30%

50-100%

100%

Tumor-related death

<0.5%

<5%

Well-differentiated: 25-30%, poorly differentiated: 75-87%

100% (Mean survival of 6.5-14.9 months)

Proliferation (Ki67)

<2%

<2%

>2%

>30%

Immunohistochemistry

CgA, NSE, VMAT 2 positive CgA negative

CgA positive

CgA negative

Histology

Mitoses (absent or occasionally)

Mitoses <1 per 2 HPFs

Mitoses >1 per IPF

Synaptophysin, NSE, PGP9.5 positive; CgA negative Severe grade 3 histology

This patient's tumor is type I, as are 70 to 80% of gastric NETs. They occur mostly in women ages 40 to 60, and present with multifocal, subcentimeter polypoid mucosal protrusions in the body or fundus. These lesions are associated with fundic chronic atrophic gastritis (often autoimmune) and enterochromaffin-like cell hyperplasia. As in this patient, almost all type I and II gastric NETs are asymptomatic and found on upper endoscopy performed for vague upper abdominal symptoms. Multiple biopsies are required, because larger tumors (>1 cm) can be associated with a synchronous gastric adenocarcinoma. Biopsies should be stained for Chromogranin A and synaptophysin, with mitotic count and Ki-67 index assessment.

primary problem nor its etiology and have not been used for this disease.

Endoscopic ultrasound is preferred for assessing both tumor size and depth of penetration. Serum gastrin and Chromogranin A are obtained; a serum gastrin greater than 120 pg/mL (0-100 pg/mL) is 94% sensitive and 100% specific for this diagnosis. Hypergastrinemia is associated with an acidic stomach with chronic atrophic gastritis on biopsy. If present, parietal cell and intrinsic factor antibodies, with serum Vitamin B12 and a complete blood count should be obtained as well.

A. B. C. D. E.

19. A 72-year-old man presents to urgent care with 4 days of intermittent fevers and persistent right upper quadrant pain. He is hemodynamically normal. His bilirubin is 5 mg/dL (0.21.9 mg/dL). His alkaline phosphatase is 110IU/L (<95 IU/L), and his aspartate aminotransferase is 400 IU/L (6-34 IU/L). His amylase is 200 U/L (23-85 U/L). Ultrasound shows gallbladder wall thickening and a common bile duct of 1 cm. Which of the following findings predicts the presence of a common bile duct stone? Total bilirubin Gallbladder wall thickening Pancreatitis Alkaline phosphatase Aspartate aminotransferase

ANSWER:

93

A

The presence of bile duct stones is a common problem in patients presenting with cholecystitis. Although endoscopic retrograde cholangiopancreatography (ERCP) is highly effective in extraction of choledocholithiasis, it is associated potentially severe complications, including post-ERCP pancreatitis, bleeding from the sphincterotomy, infection, and perforation. Patients should be risk stratified using biochemical tests and ultrasound findings, based on the likelihood of having choledocholithiasis, to minimize adverse outcomes.

20. A 47-year-old man presents to the emergency department with right upper quadrant pain after a large meal. He has no associated fever. His white blood cell count and liver function tests are normal. An abdominal ultrasound demonstrates a 1.4-cm gallbladder polyp in the fundus of the gallbladder. There is no evidence of pericholecystic fluid or gallbladder wall thickening. The optimal management is A. B. C. D. E.

The American Society of Gastrointestinal Endoscopy guidelines assign a likelihood of choledocholithiasis based on the presence of very strong, strong, and moderate clinical predictors (table 19.1). A high likelihood (>50%) of choledocholithiasis is defined as the presence of 1 very strong predictor or 2 strong predictors; a low likelihood exists in the absence of any predictors. All other patients are at intermediate risk for choledocholithiasis.

repeat ultrasound in 3 months. cholecystectomy. cholecystectomy with a liver margin. MRI of the abdomen. endoscopic retrograde cholangiopancreatography.

ANSWER:

B

A gallbladder polyp is defined as any projection of mucosa into the lumen of the gallbladder. The estimated prevalence is approximately 5%. Polyps are classified as neoplastic (adenomas, carcinoma in situ) and nonneoplastic (approximately 95% of all polyps). Although most gallbladder polyps are found incidentally after cholecystectomy, when identified on imaging studies, the management algorithm depends on the presence of symptoms and on the risk of malignancy. Cholecystectomy is recommended for symptomatic patients.

Predictors of choledocholithiasis Very strong CBD stone on transabdominal US Clinical ascending cholangitis Bilirubin >4 mg/dL Strong Dilated CBD on US (>6 mm with gallbladder in situ) Bilirubin level 1.8-4 mg/dL

Risk factors for malignancy include patient age, presence of gallstones, sessile morphology, polyp size, and a diagnosis of primary sclerosing cholangitis. Current consensus is that gallbladder polyps larger than 1 cm are at higher risk for malignancy, particularly for patients older than 45 to 50 years of age.

Moderate Abnormal liver biochemical test other than bilirubin Age older than 55 years Clinical gallstone pancreatitis Assigning a likelihood of choledocholithiasis based on clinical predictors Presence of any very strong predictor High Presence of both strong predictors High No predictors present Low All other patients Intermediate CBD, Common bile duct; US, ultrasound Table 19.1. A proposed strategy to assign risk of choledocholithiasis in patients with symptomatic cholelithiasis based on clinical predictors.

Surveillance is recommended for patients with gallbladder polyps between 5 and 10 mm and includes serial imaging every 6 to 12 months. The optimal duration of follow-up for polyps that are unchanged in size is unknown. For asymptomatic patients with small (<5 mm) polyps, surveillance may not be required.

21. A 65-year-old woman presents with 3 days of progressive abdominal distention and obstipation. She has no surgical history and is otherwise healthy except for declining mental status over the past several years. Her admitting abdominal plain film is shown (figure 21.1). With regard to her likely diagnosis, which of the following statements is true?

Several studies have examined the accuracy of these guidelines in predicting choledocholithiasis, particularly with respect to the impact of evolving laboratory values over time as well as anatomical considerations, such as mild dilation of the common bile duct (normal diameter 3-6 mm) related to older age. However, serial liver function tests do not improve the accuracy of existing guidelines. The most robust findings are the presence of a bilirubin greater than 4 mg/dL (specificity, 83-94%) and the presence of choledocholithiasis on ultrasound (specificity, 94-97%). Other predictors are more variable in accuracy and may lead to continued overuse of ERCP.

A. This condition has a lower incidence in African Americans. B. Sigmoidopexy is appropriate initial management. C. Colonoscopic decompression is more successful with sigmoid involvement than with a cecal location. D. The recurrence rate after colonoscopic sigmoid decompression is less than 25%. E. Placement of a rectal tube is sufficient management.

94

A. Patients will have an increased angle between the superior mesenteric artery and the aorta. B. Pain is secondary to malabsorption. C. There is an equal sex distribution. D. Abdominal pain is improved by lying prone. E. Endovascular stenting is recommended as initial therapy.

Figure 21.1.

ANSWER:

C Figure 22.1

Colonic volvulus is a common cause of large bowel obstruction worldwide (10-20%) but occurs less commonly in the United States. It occurs more frequently among older and debilitated patients. Colonic volvulus appears to have a higher incidence among African American patients because of a long sigmoid mesentery with a narrow stalk. This finding suggests that anatomic factors may play a more significant role in pathogenesis than diet alone. Recent reports suggest that cecal and sigmoid volvulus occur equally. When identified, volvulus must be treated rapidly to prevent progression of bowel ischemia, gangrene, and perforation. Colonoscopic decompression has a success rate of 70 to 90% in sigmoid volvulus but often fails in the management of cecal volvulus.

ANSWER:

D

Superior mesenteric artery (SMA) syndrome was first described by Rokitansky in 1842 and is caused by entrapment of the third part of the duodenum between the SMA and the aorta. The most important anatomic consideration is a decreased angle between the SMA and the aorta. Normally, this angle is between 38° and 65°. When reduced below 22%, obstruction of the third portion of the duodenum can occur (figure 22.2).

When successful for sigmoid involvement, colonoscopic decompression is not definitive treatment, because it is associated with a short-term recurrence rate of 20 to 70%. Sigmoidopexy has a recurrence rate of up to 10%. For these reasons, decompression is recommended for initial management, with resection with anastomosis rather than sigmoidopexy for definitive treatment. A limited number of studies have examined open versus laparoscopic management for uncomplicated sigmoid volvulus. Preliminary reports indicate no difference between the 2 techniques with regard to hospital stay, anastomotic leak rate, the need for reoperation, or the occurrence of serious morbidity. Placement of a rectal tube alone is not sufficient management because of the inability to decompress the bowl proximal to the area of torsion.

22. A 35-y ear-old woman presents with a 2-year history of abdominal pain, weight loss, and postprandial emesis. On physical examination, she is cachectic. An abdominal CT scan demonstrates a dilatated duodenum (figure 22.1). Which of the following statements regarding this condition is true?

Figure 22.2. The most important anatomic consideration of SMA syndrome is decreased angle between SMA and aorta (<20).

95

The cause of the narrowing is thought to be the loss of intraabdominal fat that narrows the aorta superior mesenteric angle resulting in functional obstruction of the duodenum. The 2 most common antecedent events are weight loss and corrective surgery for scoliosis. Women are affected more commonly than men, which may be a reflection of an overall lower body fat composition.

bleeding in hemodynamically normal patients, it is not a very sensitive or specific modality for identifying AEF. CT angiography, conversely, has a reported sensitivity approaching 94%, especially when considering findings associated with AEF. Such criteria include ectopic gas (i.e., air between the aorta and duodenum), perigraft fluid, perigraft soft tissue attenuation (i.e., loss of a distinct interface between aorta and bowel), focal bowel wall thickening, and the presence of a pseudoaneurysm. Given such sensitivity, it is the diagnostic modality of choice in hemodynamically abnormal patients with prior aortic prosthetic graft placement and suspected AEF.

Although malabsorption may lead to weight loss, epigastric postprandial pain associated with the SMA syndrome is caused by functional obstruction of the third portion of the duodenum leading to high-volume emesis of partially digested food, early satiety, and nausea. Several diagnostic studies are useful, including abdominal CT scan with intravenous contrast, barium swallow, angiography, and ultrasound. A barium study will typically show an abrupt cutoff at the level of the third portion of the duodenum. Having the patient lie prone or in a right lateral decubitus position will increase the aorto superior mesenteric angle and allow the barium to pass.

Colonoscopy, abdominal ultrasonography, and tagged red blood cell nuclear scintigraphy have very poor sensitivity and specificity in identifying AEFs. Although complete graft excision with oversewing of the aortic stump was the standard of care for AEF, treatment is evolving toward staged therapy involving endovascular stent placement followed by open repair.

Surgical procedures address the obstructed bowel rather than the SMA itself. Management includes both laparoscopic and open duodenojejunostomy to bypass the affected segment.

24. A 49-year-old man presents with right lower quadrant pain and a CT scan of the abdomen/pelvis consistent with appendicitis. He undergoes laparoscopic appendectomy. Pathology shows a 2.5-cm mass at the base of the appendix that is a well-differentiated neuroendocrine tumor and negative margins (carcinoid). The next step in treatment is

23. A 53-year-old man had open aortobifemoral grafting for an abdominal aortic aneurysm 2 years ago. he presents to the emergency department with tachycardia and hypotension after developing melena at home. After initiating resuscitative measures, which of the following should be undertaken immediately to evaluate for an aortoenteric fistula? A. B. C. D. E.

A. B. C. D. E.

Abdominal ultrasonography CT angiography Upper endoscopy Tagged red blood, cell scan Colonoscopy

ANSWER:

observation. total abdominal colectomy. C right hemicolectomy. 5-fluorouracil-based chemotherapy. E octreotide.

ANSWER:

C

The appendix is the most common location for intestinal neuroendocrine tumors (NETs). Well-differentiated intestinal NETs are also called carcinoid tumors. Right hemicolectomy is recommended for appendiceal NETs with any of the following characteristics: size larger than 2 cm, evidence of invasion at the base of the appendix, incomplete resection, lymphovascular invasion, invasion into the mesoappendix, intermediate to high grade, or goblet cell carcinoid/adenocarcinoid histology.

B

Aortoenteric fistula (AEF) is a potentially life-threatening condition arising after prosthetic graft placement for treatment of aortic disease (i.e., aneurysmal repair, bypass grafting). Up to 70% of patients present initially with gastrointestinal bleeding, although hypovolemic shock, sepsis, and abdominal pain with general malaise can occur. Exsanguinating hemorrhage is often preceded by a minor "herald bleed," which is self-limited secondary to vasoconstriction and thrombus formation at the site of the AEF. Herald bleeds can occur from hours to months before massive hemorrhage. The duodenum is the most common enteric site for an AEF, with more than half of cases occurring at this site, followed by the jejunum and ileum

Total hemicolectomy is not required, because the goal of the right hemicolectomy is to resect the nearby lymph node basin at risk for this appendiceal primary. Five-fluorouracil-based chemotherapy is standard treatment for advanced colorectal adenocarcinoma but not for localized intestinal NETs. Octreotide is a synthetic analog of somatostatin. Most intestinal NETs express somatostatin receptors. Octreotide is used in the treatment of locoregional unresectable NETs and patients with distant metastatic disease.

Prompt diagnosis is key to survival. Although upper endoscopy is undertaken to rule out other etiologies of

96

25. A 65-year-old woman presents with a complete small bowel obstruction. She has had no history of abdominal operations. CT scan of the abdomen/pelvis shows a mass in the mid-ileum and extensive mesenteric and retroperitoneal lymphadenopathy. After resuscitation and placement of a nasogastric tube, she is taken for exploratory laparotomy. Intraoperative biopsy of mesenteric lymph nodes suggests small bowel lymphoma. The best next step is to

E. CT scan precludes the need for esophagogastroduodenoscopy. 27. Pictures taken during the initial esophagogastroduodenoscopy (EGD) in figure 27.1. What is the next step in this patient's care? A. B. C. D. E.

A. A stop surgery; start postoperative chemotherapy. B. place an operative gastric tube. C. perform a small bowel resection with resection of the associated mesentery. D. biopsy the retroperitoneal lymphadenopathy. E. stop surgery; test for Helicobacter pylori.

ANSWER:

Early repeat endoscopy and dilation Pharmacologic steroids Esophagectomy Esophageal stenting CT scan

C

Patients with lymphoma involving the small bowel most often have systemic disease. In adults, lymphoma accounts for 25% of malignant small bowel tumors. The ileum is the most commonly affected site, due to the abundance of lymphoid tissue. In this patient, the complete small bowel obstruction mandates surgical exploration, even with the findings of extensive mesenteric and retroperitoneal lymphadenopathy on her preoperative imaging. The goal of surgery is to relieve her obstruction, and, if possible, to resect the affected bowel en bloc with the mesenteric disease. Stopping surgery and attempting chemotherapy would not be safe in the setting of a complete small bowel obstruction. Placing an operative gastric tube would temporize the patient but would be unlikely to relieve the obstruction enough to get her to definitive medical treatment A biopsy of the retroperitoneal lymphadenopathy is unlikely to give additional useful information. This disease likely also represents systemic lymphoma, and there is increased risk with performing the unnecessary additional biopsy. Treatment of Helicobacter pylori in MALT lymphoma can result in remission of disease in the majority of patients. Testing for H. pylori would not change the management for this patient.

Figure 27.1.

ANSWERS:

Items 26-27 26. A 17-year-old boy is rushed to the emergency department by his parents after drinking from a bottle containing a bathroom cleaner. The label indicates it contains sodium hydroxide and sodium hypochlorite. An attempt at self-harm is suspected. He is coughing, hoarse, and complains of throat and chest pain. Which of the following statements is true?

C, E

Ingestion of strong alkali bases, such as sodium hydroxide (NaOH) can produce severe esophageal burns and necrosis (caustic injury). Although sodium hypochlorite (NaOCl) is a strong alkali, ingestion of hypochlorite-only solutions, such as household bleach, typically cause only minor damage to the esophagus such as first-degree burns or hyperemia. Bleach ingestion only rarely produces severe complications such as perforation or stricture, usually with large volume ingestions. Attempts to use neutralizing agents in caustic injury should be avoided because they may generate exothermic reactions and further injury. Most accidental caustic ingestions produce mild injuries from which patients recover without complications.

A. A neutralizing solution should be given orally in the emergency department B. Sodium hypochlorite solutions have a higher risk of caustic injury than sodium hydroxide solutions. C. More severe injuries are associated with suicide attempts. D. Esophageal perforations due to alkali solutions occur within minutes of ingestion.

Large ingestions are usually associated with suicide attempts and produce more severe injuries. These can cause death from

97

hemorrhage, gastrointestinal perforation with sepsis, pulmonary injury, and aortoenteric fistulae in the first 2 to 3 weeks after ingestion. Long-term complications include gastrointestinal fistulae and pharyngeal, esophageal, and gastric strictures that compromise nutritional status.

Percutaneous endoscopic gastrostomy (PEG) tube insertion is commonly performed in patients who have long-term issues with oral intake, such as neurologic conditions Complication rates of PEG vary from 0.4 to 22.5% of cases, with major complications occurring in one-third of total complications. Button gastrostomies are popular in pediatric patients and may be safely placed as initial gastrostomy or to replace an existing PEG. This is a low-profile device that may be preferable in mobile adult patients on intermittent feeds where a standard PEG may be at risk of being pulled out The "buried bumper" syndrome is reported in 1% of PEG tube insertions, so-called because the internal fixation device migrates into the tract of the stoma and outside of the stomach. This can result in obstruction of the PEG or leakage of tube feeds into the abdominal wall or peritoneal cavity with possible infection, including necrotizing infections, peritonitis, and sepsis. Buried bumper syndrome is thought to be caused by excess compression of tissue between the external and internal fixation device of the PEG tube. Skin sutures on the external flange of the PEG alone will not prevent buried bumper syndrome. Avoiding excess tension or laxity of the PEG flanges during and after insertion and demonstration of the "PEG swirl" sign (twisting mucosa with rotation of the PEG tube) during insertion gastroscopy are proposed avoidance measures. Perioperative antibiotics for PEG tube insertion are recommended because a metaanalysis demonstrated a statistically significant reduction (50-75%) in the incidence of peristomal infection with administration of prophylactic antibiotics.

Initial management of the patient may require a critical care environment with endotracheal intubation and ventilation available. Multiple-agent ingestions (alcohol, drugs, other poisons) are common. Initial assessment usually includes evaluation of the injury via esophagogastroduodenoscopy. Caustic injury of the esophagus begins within minutes and may persist for hours. Clinical evidence of esophageal perforation is usually delayed, possibly as a result of transmural ischemia and subsequent necrosis due to alkaliinduced arteriolar and venous thrombosis of the esophageal wall. Figure 27.1 shows the initial esophagogastroduodenoscopy of the patient. Endoscopic grading is as follows: Grade 0 - Normal Grade 1 - Mucosal edema and hyperemia Grade 2A - Superficial ulcers, bleeding, exudates Grade 2B - Deep focal or circumferential ulcers Grade 3A - Focal necrosis Grade 3B - Extensive necrosis Grade 4 – Perforation Emergency surgery, including esophagectomy is often advocated for severe injuries seen at esophagogastroduodenoscopy (grades 3B and 4). However, poor survival and poor long-term functional outcomes are associated with emergency surgery for caustic injuries.

Early dislodgement of a PEG tube is usually described as less than 14 days after initial insertion. Dislodgement carries a significant risk of morbidity and mortality because the stomach may separate from the abdominal wall and leak gastric contents and tube feeds into the peritoneal cavity. Peritonitis usually requires open laparotomy to perform gastrostomy closure, irrigation and drainage of the peritoneal cavity, and placement of an open gastrostomy. After 14 days, the PEG tube tract is usually mature, and dislodgement is considered late. At this point, the tube can be inserted back through the tract and position confirmed by contrast fluoroscopy or CT scan. Ultrasound is used to avoid PEG insertion complications, and it may reveal the cause of unsuccessful transillumination of the abdominal wall due to intervening structures such as the left lobe of the liver.

Steroids are not beneficial in the management of esophageal injury. Strictures are late sequalae usually occurring within 8 weeks and sometimes as early as 3 weeks after esophageal injury. Strictures occur in 71% of grade 2B and 100% of grade 3 injuries. Stricture management may include stenting, dilation, or delayed esophagectomy.

28. Which of the following statements is true regarding percutaneous endoscopic gastrostomy (PEG) tube insertion? A. Button gastrostomy devices should not be placed as an initial procedure. B. Ultrasound-guided needle placement may decrease complications from PEG placement. C. "Buried bumper" syndrome is avoided by securing the external skin flange t the skin by sutures. D. Perioperative antibiotics are not required. E. Dislodgement or inadvertent removal of the PEG tube in the first 4 weeks after placement requires laparoscopy for replacement

ANSWER:

29. A patient has undergone previous endoscopic resection of a 2-cm malignant rectal polyp. Additional rectal surgery is necessary for which of the following clinical or pathologic findings? A. A 2-mm negative resection margin B. A moderately differentiated tumor with a negative margin C. Submucosal invasion of less than 1 mm D. Poorly differentiated tumor cells E. Polyp size

B 98

ANSWER:

Hyperplastic polyps most commonly occur in the setting of inflammation secondary to H. pylori infection. Hyperplastic polyps will regress with H. pylori eradication. Endoscopic findings of hyperplastic polyps include sizes ranging from 0.5 to 1.5 cm, usually with a smooth, dome-shaped characteristic, although they can be stalked. They usually cluster in the antrum but can occur throughout the stomach.

D

An abundance of evidence suggests that colon cancers arise from precursor adenomatous polyps. The polyp-to-cancer sequence represents a stepwise progression from normal epithelium to benign polyps, to malignant polyps, and ultimately invasive adenocarcinoma. Polyps can appear benign endoscopically but have occult malignant invasion identified only on microscopic pathologic evaluation after resection.

Fundic gland polyps tend to be smaller than hyperplastic polyps (0.1-0.8 cm) and are sessile. They occur exclusively in the gastric fundus and tend to have a normal underlying gastric mucosa. Their presence is associated with proton pump inhibitor use, hypergastrinemia, and familial adenomatous polyposis (FAP).

Malignant polyps will have cancer cells invading through the muscularis mucosa into the underlying submucosa. Histologic examination can identify patients who require surgical resection due to incomplete resection or the need to remove draining lymph nodes at high risk of metastatic disease.

Adenomatous polyps may be sessile or polypoid in shape but are usually less than 2 cm and solitary. They can occur spontaneously or as part of familial syndromes such as FAP. Alcohol intake is not associated with the development of any polyp disorders.

Complete endoscopic removal is necessary to allow accurate histologic classification. Characteristics of high-risk malignant polyps include poor differentiation, lymphovascular invasion, submucosal invasion of more than 1 mm, a margin of less than 1 mm, and resection in pieces, making margin assessment difficult. Although polyp size is associated with the presence of occult malignancy, it does not confer any specific risk over and above the previously outlined criteria. Patients medically fit for surgery who have high-risk polyps should be referred for definitive oncologic segmental resection.

30. A 45-year-old woman is undergoing an upper endoscopy for gastroesophageal reflux disease. Endoscopy demonstrates multiple smooth, dome-shaped polyps in the antrum of the stomach. They range in size from 0.5 to 1 cm. These polyps are most likely to be associated with A. alcohol consumption. B. familial adenomatous polyposis. C. hypergastrinemia. D. Helicobacter pylori. E. chronic proton pump inhibitor use.

ANSWER:

D

Most gastric polyps are asymptomatic. They are discovered incidentally during endoscopy in 6% of cases. Symptomatic gastric polyps tend to present with bleeding (acute or chronic) or nonspecific symptoms such as weight loss, abdominal pain, and nausea. Hyperplastic and fundic gland polyps are the 2 types of polyps most frequently identified in the stomach. They tend to occur most frequently on a background of chronic gastritis, that is, as a response to chronic inflammation. This gastritis may be chemical in nature (bile reflux), infectious (Helicobacter pylori), or reactive.

99

Alimentary Tract Part IV

Depth of invasion is related to the risk of lymph node metastasis. Complete endoscopic resection of mucosal cancer is curative, because there is no risk of lymph node involvement Meanwhile, submucosal involvement is associated with lymph node metastasis in 6 to 12%, which would require surgical resection and lymph node dissection. Pathologic findings of endoscopic submucosal excisions typically associated with increased risk of lymph node metastasis include massive submucosal invasion, lymphatic or vascular invasion, or poorly differentiated histology. In addition, sessile polyps with positive resection margins are at high risk of lymph node metastasis.

ITEMS 1-31 For each question, select the best possible response. 1. What is an appropriate indication for stress ulcer prophylaxis in the intensive care unit? A. B. C. D. E.

Mechanical ventilation for 24 hours Nothing per oral status Platelet count of 75,000/mm3 (150,000-400,000/mm3) Preadmission use of oral prednisone (5 mg/day) Traumatic spinal cord injury

ANSWER:

Despite the presence of these criteria, the rate of lymph node metastasis in the resected specimen is still low and does not sufficiently identify the colon in need of formal resection and lymphadenectomy. Thus, more rigid criteria are used to assess the endoscopic resection specimen. A multiinstitutional study from Japan of more than 800 endoscopic specimens of submucosal invasive colorectal carcinoma found

E

The physiology of critical illness often entails the development of systemic hypoperfusion, catecholamine surges, diminished cardiac output, circulatory hypovolemia, vasoconstriction, and the release of inflammatory cytokines. Release of inflammatory cytokines often results in splanchnic hypoperfusion and the predisposition to gastric ulceration. Hence, stress ulcer prophylaxis to prevent clinically significant gastric ulceration and hemorrhage is advocated.

that for nonpedunculated polyps, the rate of lymph node metastasis was 0% when the submucosal depth of invasion was less than 1 mm. Even when adjusted for other pathologic features representing higher risk, a minimum invasion depth associated with lymph node metastasis was 1 mm. The presence of other pedunculated polyps does not affect the risk of lymph node metastasis after resection of a sessile polyp with invasive submucosal colon cancer. Venous invasion is not a risk factor for lymph node metastasis on multivariate analyses.

Various risk factors are known to contribute to the development of stress ulcer gastritis. Mechanical ventilation for at least 48 hours, not 24 hours, predisposes patients to the disease process, as does coagulopathy with an international normalized ratio greater than 1.5 or platelet count less than 50,000/mm3 (150,000-400,000/mm3). Stress ulcer prophylaxis is appropriate for patients with traumatic brain injury, traumatic spinal cord injury, and severe bum injury (>35% body surface area). A nothing-by-mouth status does not confer any additional risk for stress ulcer gastritis. The use of high-dose steroids, equivalent to 250 mg of hydrocortisone, but not low-dose steroids is considered a risk factor for stress gastritis.

3. A percutaneous cholecystostomy tube A. B. C. D. E.

2. After complete endoscopic resection of a malignant, sessile colon polyp, what pathologic finding should prompt partial colectomy? A. B. C. D. E.

ANSWER:

1-mm negative radial margin Concurrent pedunculated polyp Moderately well-differentiated histologic grade Venous invasion Submucosal depth of invasion less than 1 mm

ANSWER:

is not indicated for acalculous cholecystitis. can be definitive treatment for acute cholecystitis. should be removed after 1 week. precludes subsequent laparoscopic cholecystectomy. is associated with a high periprocedural mortality.

B

Management of acute cholecystitis in patients with significant comorbid illnesses is challenging. Urgent cholecystectomy with a high risk of mortality is not an acceptable option in many cases. The gallbladder can be drained percutaneously in these cases with a procedural mortality of less than 3%. The timing of tube removal is somewhat controversial, but most practitioners agree that a minimum drainage period of 2 weeks is necessary for patients with transhepatic tube placement. Two weeks allows a tract to develop and minimizes the potential for bile leakage into the peritoneal cavity. If a transhepatic approach is not used, then a longer duration of tube drainage may be required. In a case series of 185 patients, more than 90% were successfully managed in the acute setting with a cholecystostomy tube alone.

A

The role of endoscopy in the management of colorectal cancer has evolved. Therapeutic endoscopy is considered definitive treatment in selective malignant polyps. Complete endoscopic resection of sessile colon polyps is now possible with advanced endoscopic techniques. Pathologic findings determine the adequacy of endoscopic resection and the need for formal colonic resection.

101

Gallstones may not be present in patients with acute cholecystitis, and 2 series documented a lack of cholelithiasis in 25 to 30% of patients successfully managed with percutaneous drainage. In most series, later cholecystectomy is performed for cholelithiasis, with the majority of patients receiving laparoscopic procedures. For patients without cholelithiasis, initial drainage with subsequent tube removal may be a definitive procedure. Historically, recurrent symptoms occur in 7.1 to 46.7% of patients. In a more recent 10-year retrospective study of 183 patients treated with cholecystostomy tubes, however, only 11% had recurrent symptoms after tube removal, with a median duration of follow-up of 38 months.

Figure 4.2. Splenic vain clot. 4. A 40-year-old man with a history of alcohol abuse and previous episodes of pancreatitis presents with a 3-day history of vague abdominal pain. His heart rate is 72 beats per minute, and his blood pressure is 140/76 mm Hg. His abdomen is soft on exam without rebound or guarding. His hemoglobin is 14 g/dL (13.5-17.5 g/dL), and his amylase and lipase are mildly elevated. CT scan is obtained (figure 4.1). What is the next appropriate step in his management? A. B. C. D. E.

With the increased use of CT scanning, however, splenic vein thrombosis is being diagnosed more frequently in 7 to 20% of patients with pancreatitis or pancreatic pseudocysts. In a 10-year study of 53 patients with pancreatitis or pancreatic pseudocysts, only 2 patients received splenectomy for gastrointestinal bleeding. No patients died from variceal bleeding, with a mean follow-up of more than 58 months In this patient with symptoms of pancreatitis but no evidence of gastric varices on CT scan or clinical evidence of gastrointestinal bleeding, splenectomy is not indicated. Upper endoscopy is not indicated at this time because his pain is easily explained by his chronic pancreatitis. Splenic vein thrombosis can spontaneously resolve with resolution of the inflammation from pancreatitis. No data support the use of endovascular thrombectomy in this setting. Vasopressin is a useful treatment for esophageal or gastric varices when actively bleeding but is not indicated at this time. The best option for treatment of this patient is observation.

Splenectomy Endoscopy Endovascular thrombectomy Vasopressin infusion Observation

The use of anticoagulants to treat splenic vein thrombosis remains controversial. Guidelines for anticoagulation from the American College of Chest Physicians suggest that symptoms should help guide anticoagulant therapy. Anticoagulation is recommended for symptomatic patients but not for asymptomatic patients with incidentally detected events.

Figure 4.1.

ANSWER:

5. A 62-year-old man experiences progressive difficulty swallowing, vomiting, and coughing over the past 12 months. He has lost 9 kg, and his wife notes his breath smells foul. What is the most appropriate next step in his evaluation? A. Ultrasound B. Contrast esophagram C. Esophagogastroduodenoscopy D. CT scan E. Esophageal manometry

E

The CT scan demonstrates splenic vein thrombosis (figure 4.2). This condition is recognized as a cause of gastrointestinal hemorrhage due to collateral venous drainage in patients with pancreatic tumors as well as pancreatitis. Historically, splenectomy was the treatment for this condition. In the past, 45 to 72% of patients presented with gastric variceal bleeding, and splenic vein thrombosis was confirmed on the CT scan at the time of the herald bleed. With this selection bias of already bleeding patients, little can be inferred about recommendations for asymptomatic patients.

ANSWER:

B

Zenker diverticulum (ZD) is a relatively uncommon problem of the pharyngoesophagus. It occurs predominately in elderly

102

men. The diverticulum contains only mucosa and submucosa and is therefore classified as a pseudodiverticulum. The primary pathophysiology is believed to be dysfunction of the cricopharyngeus muscle, resulting in increased intraluminal pressure and causing a pulsion diverticulum through the weakened area known as Killian triangle. The most common symptom is dysphagia, but other classic symptoms can include coughing, weight loss, and halitosis. When ZD is suspected, the best diagnostic study is a dynamic esophagram. This study will confirm the diagnosis, define the anatomy, and demonstrate retained contrast in clinically significant disease. An esophagogastroduodenoscopy is not necessary unless there is a suspicion of an underlying malignancy. It is not the initial diagnostic test of choice and, if done, care must be taken not to inadvertently enter the diverticulum and cause a perforation. Esophageal manometry is also not required in the diagnosis and management of ZD. Ultrasound can be used for patients who cannot tolerate an esophagram, but it is not the diagnostic study of choice. A CT scan is helpful for malignant esophageal disease but is not necessary for patients suspected of having a simple ZD. Historically, the management of ZD was through a left cervical excision. The most important principle in this technique is division of the cricopharyngeus muscle onto the cervical esophagus. This is the critical step of all surgical procedures. A larger diverticulum is then usually resected with a stapling device. A smaller diverticulum can be treated with a diverticulopexy by suturing the diverticulum to the prevertebral fascia.

Figure 6.1,

ANSWER:

A transoral approach using either rigid or flexible endoscopy is now often used. In this procedure, the common wall between the esophagus and diverticulum is divided, creating a common channel. As with the cervical approach, division of the cricopharyngeus muscle is critical for success. One advantage of this technique is the decreased risk of a fistula due to a leak from an open diverticulectomy. Although the transoral approach is gaining favor, few trials compare the techniques. Both techniques appear to have similar short-term outcomes; however, long-term outcomes comparing these procedures are lacking.

Anal fissures are a common cause of anal pain associated with minor anorectal bleeding. Fissures are most often associated with the passage of a hard bowel movement and are located in the posterior midline in the majority of cases. Fissures can be classified as acute if symptoms are less than 6 weeks or chronic for those causing symptoms greater than 6 weeks. Chronic anal fissures are often associated with a distal skin tag or a proximal hypertrophied anal papilla. The circular fibers of the internal sphincter are often visible at the base of the fissure, as seen in this photograph (figure 6.2). Acute fissures will generally heal with supportive care, inducting fiber supplementation, stool softeners, and sitz baths. After 6 weeks, however, healing rates tend to decrease and may require further therapy.

6. A 48-year-old otherwise healthy man has severe anal pain and blood on the toilet paper with each bowel movement. His symptoms have persisted for 6 months despite a high-fiber diet and a trial of nitroglycerin ointment. On physical exam, you see the following (figure 6.1). The next best step in management is A. B. C. D. E.

D

Because most anal fissures are associated with a hypertonic sphincter, the primary objective of all therapeutic options is to relax the anal sphincter. This can be done medically by using either topical nitric oxide donors, such as nitroglycerin ointment, or topical calcium channel blockers, such as diltiazem or nifedipine. Healing rates with these compounds are only a little bit better than placebo and recurrence is common. Nevertheless, there is little downside, so a trial of a topical agent remains the first-line therapy. Headaches, more

botulinum toxin injection. topical calcium channel blockers. anocutaneous advancement flap. lateral internal sphincterotomy. posterior sphincterotomy.

103

common with nitroglycerin ointment, limit its utility in some patients.

sphincter, often associated with either previous anorectal surgery or obstetrical trauma. In these situations, a lateral internal sphincterotomy should be avoided so as not to induce or exacerbate incontinence. Instead, a fissurectomy with an anocutaneous advancement flap is the procedure of choice. Posterior sphincterotomy effectively treats fissure but is associated with poor wound healing and creation of a "keyhole" deformity, so it was abandoned as a surgical option. In this patient scenario, the history and physical exam are consistent with chronic anal fissures. He has attempted conservative measures, including a trial of medical therapy, which were not effective. Because he has no risk factors for incontinence, a lateral internal sphincterotomy is the most appropriate next step. 7. A 63-year-old woman is 5 days postoperatively from a right colectomy for colon cancer. She has multiple loose stools, abdominal pain, and a leukocytosis. The most sensitive test for Clostridium difficile infection is A. B. C. D. E.

toxigenic stool culture. flexible sigmoidoscopy. polymerase chain reaction (PCR). enzyme immunoassay (ELISA). CT scan.

ANSWER:

A

clostridial difficile is a spore-forming anaerobic bacterium and is a common cause of antibiotic-associated diarrhea and colitis. C. difficile infection (CDI) is the most common cause of pseudomembranous colitis. Prior exposure to antibiotics is thought to alter a patient's natural microbiome, allowing for CDI. It is important to distinguish between patients who are colonized with C. difficile and those who have CDI, because only CDI requires treatment. The symptoms of CDI are mediated by toxin production (toxin A and toxin B). Therefore, for CDI to be diagnosed, either toxin A or B must be present. Toxigenic stool culture is the most reliable way to detect CDI. The process initially cultures stool for the presence of C. difficile but the further tests to detect either the toxin-producing gene or the toxin itself. Therefore, this process can distinguish between toxigenic and nontoxigenic strains of C. difficile Although toxigenic stool culture is the most sensitive test for CDI, it takes several days to complete and is therefore rarely used clinically.

Figure 6.2. For patients with continued symptoms, surgery is the best option with the highest rates of healing. The most successful surgery is a lateral internal sphincterotomy. Cutting the internal sphincter permanently relaxes the anal sphincter. The major concern is inducing incontinence as a result of the operation. Reported rates of incontinence are variable, but severe life-altering incontinence is uncommon. Limiting the sphincterotomy to the length of the fissure, instead of cutting the entire sphincter, can limit the effects on continence and effectively control the pain of the fissure. Because of the risk of incontinence, several investigators evaluated the use of botulinum toxin as an alternative to surgery. Botulinum temporarily paralyzes the internal sphincter, effectively causing a "chemical sphincterotomy," and allowing for healing of the fissure but only affecting continence transiently.

Enzyme immunoassays (ELISA) for toxin A and B were used as the primary diagnostic method for nearly 2 decades. However, recent studies comparing ELISA to toxigenic stool culture showed ELISA to be too insensitive. Therefore, ELISA is no longer considered adequate.

Multiple studies have shown that lateral internal sphincterotomy is more effective than botulinum toxin in healing chronic anal fissures. Botulinum, therefore, should be reserved for those who either refuse lateral internal sphincterotomy or who are at particular risk for incontinence with a lateral internal sphincterotomy.

Polymerase chain reaction (PCR) is now used to detect toxinproducing genes. This test is rapid, sensitive, and has a very high negative predictive value. It is now commonly used in the hospital setting. However, PCR testing cannot distinguish between patients colonized with C. difficile and those who

Although most fissures are associated with hypertonic sphincters, some patients develop a fissure with a hypotonic

104

truly have CDI and is susceptible to false-positive results. Therefore, current recommendations now stress that only patients with more than 3 loose stools over a 24-hour period be formally tested using PCR.

Although there are a number of screening tools, only 2 (fecal occult blood testing or FOBT and flexible sigmoidoscopy) have been studied in large randomized prospective studies. The Minnesota trial examined 46,000 patients using FOBT either annually or biennially and compared this strategy with nonscreened patients. The mortality of colorectal cancer decreased by 20% in the annually screened population without affecting the incidence of colorectal cancer. Four similar large studies from Sweden, France, Denmark, and the United Kingdom demonstrated similar results, making this the most thoroughly studied method of colorectal cancer screening. FOBT is nonspecific and detects the presence of heme (the protein in hemoglobin) from human and nonhuman sources, which is present in some food sources. Dietary restrictions are required before testing to limit the number of false-positive results.

Flexible sigmoidoscopy can be used to detect the pseudomembranes sometimes seen in CDI. However, because CDI can be present without pseudomembranes, flexible sigmoidoscopy lacks appropriate sensitivity and specificity. CT scan can be useful to diagnose CDI. Some studies have shown a positive predicative value as high as 88%. However, toxigenic stool culture remains the standard to which all other tests are compared. Once CDI is successfully treated, patients can carry toxigenic C. difficile for months. Therefore, current guidelines do not recommend repeat testing unless there are recurrent symptoms. For patients with actual recurrent CDI, oral vancomycin, fidaxomicin, and fecal transplant are all possible treatments.

FOBT also will detect blood from both upper and lower gastrointestinal sources. Therefore, most positive results will be falsely positive. A positive test should prompt a colonoscopy, which will discover a colorectal cancer in 2 to 10% of patients and a polyp in 20 to 30% of patients. To increase sensitivity, FOBT should be done on 3 successive stools and done annually if the test is normal.

Items 8-10 Each lettered response may be selected once, more than once, or not at all.

Flexible sigmoidoscopy is the other well-studied screening modality. Four large prospective and randomized studies examined flexible sigmoidoscopy worldwide, Pooled analysis suggested flexible sigmoidoscopy decreased the incidence of colorectal cancer by 18% with a 28% decrease in mortality. This finding was particularly true for left-sided cancers compared with right-sided disease. In the Norway study, the addition of a fecal immunochemical test (FIT) with flexible sigmoidoscopy improved these numbers, presumably by detecting right-sided lesions that would be missed by flexible sigmoidoscopy alone.

A. Fecal occult blood testing (FOBT) B. Fecal immunochemical testing (FIT) C. Multi-target stool DNA testing D. Colonoscopy E. CT colonography 8. Preferred screening test for patients at high risk for colon cancer 9. Well studied by randomized, prospective clinical trials 10. Detects human hemoglobin as primary target

ANSWERS:

The remaining screening modalities have not yet been tested as thoroughly, nor are they likely to be. The assumption from these data is that screening is effective and that additional studies are not necessary to evaluate each new test. Instead, the effectiveness of new tests is compared with existing technology and often compared directly with colonoscopy. It is also critically important to understand that therapeutic colonoscopy, in all of these studies, was used once there was a positive screening test

D, A, B

The care of colorectal cancer has improved significantly over the last several decades. Although the reason for this improvement is multifactorial, one factor is a better understanding of the importance of colorectal cancer screening. Colorectal cancer screening is now well established for all patients considered to be at average risk for colorectal cancer. In general, all patients, once they reach the age of 50, are considered to be at average risk and should be screened for colorectal cancer. Unfortunately, although the recommendations are quite clear, nearly 30% of adults eligible for colorectal on cancer screening have not been screened. The national campaign "80% by 2018" promoted the benefits of screening with the hope of achieving 80% compliance by the end of 2018 and beyond. Part of this campaign was for physicians to have a better understanding of screening options and the evidence for these options.

FIT uses immunochemistry techniques to specifically detect human heme. It also does not detect partially digested heme so will not detect blood from the upper gastrointestinal tract. For these reasons, FIT is more sensitive and specific than FOBT. There are no dietary restrictions and only one sample is required. FIT should be done annually if used as the primary screening tool. Multitargeted stool DNA testing uses quantitative molecular assays to detect abnormal DNA in the stool suggestive of

105

colorectal cancer. As part of this test, a FIT is also done but would not be considered the primary target of the test. Using

B. C. D. E.

colonoscopy as the standard of care, when multitargeted stool DNA testing was compared with FIT resting alone, it was more sensitive at detecting colorectal cancer (92% vs 74%) and more sensitive at detecting advanced precancerous lesions (42% vs 23%). However, it was less specific, so it had a higher rate of false-positives. Although recently approved for colorectal cancer screening, the recommended interval for multitargeted stool DNA testing remains unclear. Based on modeling data, this test is recommended every 3 years with the caveat that it has been poorly studied. Colonoscopy remains the standard of care for colorectal cancer screening, although the evidence for its effectiveness is mostly indirect. Several studies are under way, but it will take time for the data to fully mature. However, based on previous data and several case-controlled studies, colonoscopy has emerged as the preferred screening modality. Like flexible sigmoidoscopy, colonoscopy has the added benefit of removing precancerous lesions, which may actually prevent colorectal cancer instead of just detecting it early. Furthermore, colonoscopy is necessary as a diagnostic test for all of these other screening modalities. If colonoscopy is done for screening and is totally normal, another colonoscopy does not need to be done for 10 years. Because colonoscopy appears to be the most effective tool, it is the recommended test for patients considered to be at high risk for colorectal cancer, including patients with inflammatory bowel disease and those with a significant family history of colorectal cancer.

diffuse large B-cell lymphoma. mucosa associated lymphoid tissue (MALT). T-cell lymphoma. Burkitt lymphoma.

Figure11.12.1 12. After a complete evaluation, including a PET-CT scan, the disease appears to be limited to the colon. The most appropriate next step in treatments is A. B. C. D. E.

antibiotic treatment of Helicobacter pylori. chemotherapy. radiotherapy. right colectomy with lymphadenectomy. right colectomy with retroperitoneal lymphadenectomy.

ANSWERS:

B, D

Lymphoma can involve any part of the gastrointestinal tract. It affects the stomach most commonly, followed by the small intestine, and then the ileocecal region. Most lymphomas of the gastrointestinal tract are manifestations of diffuse nodal disease. Primary gastrointestinal lymphoma is relatively rare and lacks diffuse lymphadenopathy. Primary colonic lymphoma is an uncommon malignancy of the colon, accounting for less than 1% of colonic malignancies. The most common subtype for primary colonic lymphomas is diffuse large B-cell lymphoma. Other subtypes, including Tcell lymphoma, mucosa-associated lymphoid tissue (MALT), and Burkitt lymphoma are less common. Hodgkin lymphoma is uncommon in the gastrointestinal tract.

CT colonography uses x-ray technology to image the entire colon looking for colorectal cancers and colorectal polyps. Compared with colonoscopy, CT colonography is relatively good at detecting colorectal cancer and precancerous growths larger than 10 mm. Unfortunately, it still requires bowel preparation, which causes the greatest amount of patient dissatisfaction. The exact interval that this test should be done is also not well studied, but the recommended interval after a normal screening CT colonography is 5 years. Extraintestinal findings that may require further evaluation can be found in approximately 15% of patients. Although some findings are clearly beneficial to the patient, many results in unnecessary additional evaluations. Therefore, the value of discovering extraintestinal findings is not well understood.

The treatment for primary colonic lymphoma is somewhat controversial. Even though, by definition, the disease is localized to the colon, many researchers consider this to be a systemic disease that is best treated with chemotherapy. However, especially with large tumors involving the entire bowel wall, there is a reasonable risk of perforation with systemic chemotherapy. Moreover, recent data suggest that surgery followed by systemic chemotherapy may improve survival compared with chemotherapy alone This may be true for large tumors of the colon, most commonly affecting the cecum. Surgery should involve segmental resection of the colon and its associated lymph nodes. Because most physicians consider this to be a systemic disease,

Items 11-12 A 72-year-old man has right-sided abdominal pain and weight loss. A CT scan is obtained (figure 11-12.1). A colonoscopy confirms a large cecal mass. A biopsy is consistent with a lymphoma. 11. The most likely pathology is A. Hodgkin lymphoma.

106

chemotherapy should follow surgical resection for the best chance of long-term survival.

a central ulceration, which can result in either occult blood loss or a frank gastrointestinal bleed. If the diagnosis is i. question, a fine needle aspiration or core needle biopsy can be done at the time of the endoscopy. GISTs demonstrate benign-appearing spindle cells that stain positively for KIT.

In this patient with a large tumor mostly isolated to the cecum, surgical resection is warranted before chemotherapy (figure 11-12.2). The lymphadenectomy should be isolated to the segmental resection but would not include the retroperitoneum. MALT lymphomas, especially of the stomach, may respond to Helicobacter pylori treatment but would not be effective for this problem. Radiotherapy is not efffective for diffuse large B-cell lymphoma and would not be recommended for this patient.

Once a diagnosis is confirmed, resection is the treatment of choice. During surgery, the tumor should be removed with negative margins only, although a 1-cm margin seems reasonable when possible. Although RO resection is the goal, some data suggest that recurrence-free survival may not be different for low-risk tumors even with microscopically positive margins (Rl). This may be particularly important for a rectal GIST, for example, when obtaining larger margins may result in increased morbidity. Furthermore, these tumors do not spread via lymphatics, so a lymph node dissection is not necessary. Simple excision with negative margins is therefore the procedure of choice. Care should be taken not to rupture the tumor. When done laparoscopically, tumors should be removed with an endoscopic bag to prevent tumor rupture. Once excised, GISTs are risk stratified to assess for the potential of metastatic spread. The most significant risk factors include size larger than 5 cm, mitoses greater than 5 per 50 high powered fields, and location outside the stomach. Using these risk factors, nomograms can predict the risk of recurrence.

imatinib, a tyrosine kinase inhibitor, is used to treat high-risk GISTs. Sufficient evidence supports the use of at least 36 months of imatinib for high-risk tumors to improve both recurrence-free survival and overall survival There is little controversy in using this therapy for tumors larger than 5 cm, for a high mitotic rate, or for an estimated recurrence of 50%. More recently, studies suggest improved recurrence-free Survival for tumors 3 cm or larger. For tumors that are unresectable or only resectable with significant morbidity (e.g., an abdominal perineal resection), neoadjuvant imatinb should be considered to reduce the size of the tumor before surgical intervention. In this scenario, mitotic rate is the only choice that predicts a poor outcome. The other parameters are not predictive of poor outcome. Lactate dehydrogenase, often used to determine tumor lysis in lymphoma, is not used in the evaluation of GISTs.

Figure 11.12.2. Large tumor mostly isolated to the cecum

13. A 62-year-old man has an esophagogastro-duodenoscopy for reflux symptoms.A 3-cm submucosal mass is noted along the greater curve of the stomach. A core needle biopsy demonstrates spindle cells that stain positively for KIT. For this disease, what factor affects prognosis? A. B. C. D. E.

Lactate dehydrogenase value Mitotic rate Lymph node metastasis Central ulceration seen on endoscopy 1-cm margins

ANSWER:

B 14. A 52-year-old woman is diagnosed with a 3-cm welldifferentiated rectal adenocarcinoma 7 cm from the anal verge. Rectal MRI demonstrates a T2 tumor without a threatened radial margin or enlarged lymph nodes. A complete metastatic evaluation does not show any evidence of metastatic disease. She is otherwise healthy with a BMI of 28. The next best step in management is A. chemotherapy. B. chemoradiation. C. transanal endoscopic microsurgery. D. low anterior resection. E. abdominal perineal resection.

Gastrointestinal stromal tumors (GISTs) can occur throughout the gastrointestinal tract. The most common location is in the stomach, followed by the small intestine. GISTs grow in the bowel walL Nevertheless, up to 25% may erode through the mucosa and cause a gastrointestinal bleed. Otherwise, symptoms are generally nonspecific, such as pain and abdominal bloating. CT scan is the imaging modality of choice for most GISTs. They are often identified incidentally on scans obtained for nonspecific symptoms. Tumors in the bowel wall in any part of the gastrointestinal tract suggest the possibility of a GIST. On endoscopic evaluation, most GISTs appear as a submucosal mass, although a minority may show

107

ANSWER:

Therefore, neoadjuvant therapy is unlikely to improve her oncologic control and would not be necessary. Transanal endoscopic microsurgery, while the preferred technique for a local excision, is not appropriate given high local recurrence rates for T2 lesions, even with adjuvant chemoradiation. Given the location in the midrectum, an abdominal perineal resection is not necessary.

D

In rectal cancer, it is helpful to divide the rectum into thirds: the upper rectum, the mid rectum, and the lower rectum. Most clinicians measure the height of the tumor from the anal verge, which includes the anal canaL The top of the rectum is more difficult to define, but many clinicians use 15 cm as the division between the top of the rectum and the sigmoid colon. Using this number, the lower, mid, and upper rectum would be from 0 to 5 cm, 5 to 10 cm, and 10 to 15 cm, as measured from the anal verge. These values roughly correspond to the first, second, and third valves of Houston, which are other important anatomic landmarks. Defining this distance from the anal verge is critical in treatment strategies and can be done with a combination of physical exam, endoscopy, and pelvic MRI.

15. A 60-year-old man presents with increasing constipation, narrow stools, mucus discharge, and anal pain with bowel movements. He reports that a hemorrhoid procedure was performed 5 years ago. Physical examination demonstrates anal stenosis and with mucosal eversion. His hemorrhoid procedure was most likely A. injection sclerotherapy. B. 3-quadrant hemorrhoidectomy. C. rubber band ligation. D. transanal hemorrhoidal dearterialization. E. a Whitehead procedure.

Once the diagnosis is confirmed by biopsy and the location determined, local staging is critical. The goal of staging is to determine a clinical stage based on the depth of tumor invasion, the presence of local metastases, and the evaluation for distant disease. For years, endorectal ultrasound was the diagnostic procedure of choice for local staging. However, high-resolution pelvic MRI has evolved as the preferred imaging strategy. Using pelvic MRI, tumor depth can be estimated, lymph nodes can be evaluated, the relationship of the tumor to the anterior peritoneal reflection can be determined, and the mesorectal margin can be assessed for tumor invasion. CT scans of the chest and abdomen are then used to complete the metastatic evaluation. Once these studies are completed, a clinical stage can be assigned using the TNM staging system, which will affect therapeutic decisions.

ANSWER:

E

The most common cause of anal stenosis is a complication after hemorrhoidectomy The Whitehead procedure is a circumferential incision in the anal canal to facilitate removal of all hemorrhoid tissue of the distal rectum. The proximal rectal mucosa is then reapproximated by suturing it to the cut inferior edge of the anus. Although, a 3-quadrant hemorrhoidectomy, which involves longitudinal excision of the hemorrhoidal complexes, can be associated with anal stenosis, mucosal eversion (ectropion) with stenosis is associated with the Whitehead procedure.

Neoadjuvant chemoradiation improves local control for patients with locally advanced rectal cancer. This includes patients with either clinical stage II (T3, NO, MO) or stage III (Tx, N+, MO) rectal cancer. This is particularly true for patients with low- and midrectal cancers. The impact of neoadjuvant therapy in upper rectal cancers has been questioned by many clinicians and should be evaluated on a case-by-case basis using a multidisciplinary conference.

The Whitehead procedure can be tempting as a management approach for incarcerated and prolapsed circumferential hemorrhoids, because it serves to excise all thrombosed and inflamed tissue. However, because of risk of a Whitehead "deformity," which is difficult to repair, this procedure should be avoided. A 3-quadrant hemorrhoidectomy is not associated with mucosal eversion. Procedures that do not excise anoderm, such as injection sclerotherapy, rubber band ligation, and transanal hemorrhoidal dearterialization, have a negligible risk of anal stenosis and mucosal eversion.

For patients with stage I (T1/T2, NO, MO) rectal cancer, neoadjuvant therapy is not necessary. Very early rectal cancers (T1N0M0) without aggressive pathologic features may be appropriate for a local excision. For many/years, local excision followed by chemoradiation therapy was also considered adequate for T2N0M0 tumors. More recent data suggest that recurrence rates for these tumors is unacceptably high following this protocol. Therefore, the current recommendation for a T2N0M0 is a radical resection with a complete mesorectal excision.

Items 16-17 A 42-year-old woman undergoes an exploratory laparotomy after multiple hospitalizations for partial small bowel obstruction and weight loss. Her history includes treatment for stage IIIB cervical cancer. The pertinent operative finding in the distal ileum are demonstrated in figure 16-17.1.

In this patient, who is otherwise healthy and a good operative risk, preoperative imaging confirmed a T2N0M0 tumor. At 7 cm, a low anterior resection with a complete mesorectal excision should be possible. If surgery is done in the proper mesorectal planes, the MRI predicts a wide radial margin.

108

In a patient with the appropriate clinical history, radiation injury is the most likely explanation for the fibrosis of the bowel wall. Ischemic enteritis, which can be related to infection, embolus, or thrombosis, is unlikely to cause an obstruction and would show acute inflammatory changes on gross evaluation. A carcinoid tumor would show a focal lesion or cause an obstruction via a desmoplastic reaction in the mesentery. Carcinomatosis is an unlikely mechanism of spread for cervical cancer and would likely show focal tumor implants grossly. Adhesive disease would not result in the diffuse, homogeneous narrowing of the intestine as seen in the image. This patient's cancer is likely to be well controlled after chemoradiation. Therefore, the goal of surgical management is to safely relieve the obstruction and to decrease the risk of recurrent obstruction in the future. A bowel resection is the best approach to definitively remove the source of obstruction. Adhesiolysis would not remove the obstructing lesion. A diverting ileostomy may be appropriate for a hostile abdomen, but in this case, the intestinal loops are easy to eviscerate and manipulate.

Figure 16-17.1. 16. A. B. C. D. E.

The most likely diagnosis is ischemic enteritis. carcinoid tumor. carcinomatosis. radiation fibrosis. adhesive disease.

17. The most appropriate management for these operative findings is A. Adhesiolysis. B. small bowel resection. C. diverting ileostomy D. small bowel bypass E. strictureplasty.

ANSWERS:

Small bowel bypass is not as likely to prevent future recurrence. Small bowel bypass may be a safe option if the diseased small intestine is densely adherent to the pelvis, mandating a difficult lysis of adhesions to perform a resection, but imaging shows that diseased bowel can be delivered through the midline incision. Strictureplasty may be considered for a small focal lesion, but in this case, a long segment is affected.

D, B

Locally advanced-stage cervical cancer is managed with chemoradiation. Radiation to the pelvis can be associated with toxicity to the bowel. Radiation enteritis occurs acutely and results in abdominal pain and diarrhea. The chronic effects of radiation on the bowel are caused by fibrosis. Figure 16-17.2 depicts a segment of diffusely pale small intestine, with a narrow caliber adjacent to a dilated (proximal) loop of intestine that has a serosal surface of normal color.

18. A 44-year-old man with a history of paraplegia has a diverting colostomy for a large sacral decubitus. He now presents to the emergency department with abdominal bloating and decreased stool output. He is hemodynamically normal. Abdominal examination shows mild distension, with minimal tenderness, and external physical findings shown in figure 18.1. His white blood cell count is 11,400/mm3 (360011,200/mm3), other laboratory results are normal, and an abdominal plain film shows nonspecific bowel gas pattern without abnormalities. Which of the following is the appropriate next step in management? A. B. C. D. E.

Figure 16-17.2. A segment of diffusely pale small intestine, with narrow caliber adjacent to a dilated (proximal) loop of intestine

109

Re-siting of the stoma Manual reduction Subtotal colectomy with end ileostomy Submucosal sleeve resection Local revision

19. A 55-year-old woman who underwent uncomplicated hip replacement returns to the emergency department with no bowel function for 4 days, nausea, and bloating. A kidney, ureter, and bladder study show dilated colon from the cecum to the rectum Her vital signs are normal. Her abdomen is not tender. The appropriate first step in management should be A. B. C. D. E.

milk of magnesia. Gastrografin enema. methylnaltrexone. intravenous neostigmine. cecostomy.

ANSWER:

This patient demonstrates a lack of bowel function without signs of sepsis or peritonitis and has an imaging study suggestive of acute colonic pseudo-obstruction (also known as Ogilvie syndrome). This condition, a disorder of bowel motility, is thought to be related to increased sympathetic activity or reduced parasympathetic tone It is frequently seen in the postoperative state. The first critical step is excluding a distal obstruction, for example, at the distal rectum. A Gastrografin enema can rule out distal obstruction, may have therapeutic benefit, and is the appropriate first step in management. CT scan with rectal contrast or colonoscopy performed with cautious insufflation could also be used as the first step in management.

Figure 18.1

ANSWER:

B

E

Figure 18.1 shows a large colostomy prolapse. Stoma prolapse, a full-thickness intussusception of the bowel through its abdominal wall orifice, is more common in transverse and loop colostomies and occurs in up to 10% of

Milk of magnesia treats constipation, would not be effective, and could worsen colonic dilation. Although perioperative opioids exacerbate acute colonic pseudo- obstruction, methylnaltrexone is appropriate to use in opioid-induced constipation but not in this scenario. Intravenous neostigmine is an effective treatment modality for acute colonic pseudoobstruction; however, it is important to first rule out a distal obstruction. Cecostomy is not first-line management of this disorder.

patients over the long term. The mucosa of this prolapse appears edematous, and the tip of the prolapse is violaceous and dusky, suggesting ischemia. The patient is hemodynamically normal and does not have a bowel obstruction by imaging, so his symptoms of bloating and decreased stool output may be attributed to constipation. His white blood cell count is slightly elevated, possibly reflecting early bowel ischemia. If the prolapsed intestine were smooth and nonedematous in appearance, then despite the size, it could be reduced with plans for outpatient followup. However, given the concern for early ischemia, urgent operative management is appropriate. Re-siting the stoma requires a laparotomy, is more appropriate to manage a difficult parastoma hernia, and is more aggressive than required to address a colostomy prolapse.

20. The initial management of a localized 4-cm squamous cell carcinoma of the anal canal in a patient with HIV is A. B. C. D. E.

local excision. chemoradiation. chemotherapy. abdominoperineal resection. immunotherapy.

ANSWER:

A subtotal colectomy with ileostomy requires more bowel resection than necessary and exposes the patient to chronic volume loss from an ileostomy. A submucosal sleeve resection is not an adequate procedure for the prolapse. A local revision is the most suitable procedure for this prolapse and avoids a laparotomy. Any ischemic colon removed with a full-thickness resection.

E

The initial management of nonmetastatic anal canal cancer larger than 2 cm is chemoradiation. Many of these cancers will regress with chemoradiation alone, and no surgical management will be required. These principles are the same in patients with HIV. Local excision is inadequate treatment of a 4-cm tumor and will not be able to clear the tumor

110

without total disruption of the anal sphincter. Chemotherapy without radiation is insufficient treatment. Initial management with surgical resection not only commits the patient to a permanent colostomy, but also may not achieve clear margins, putting the patient at risk of local recurrence Immunotherapy using anti-PD-1 therapy may be a promising avenue of treatment for this disease, because many tumors express PD-L1 by immunohistochemistry. However, this treatment is investigational only and so far is limited to patients who have metastatic disease.

Figure 22.1 21. A 68-year-old man had a curative resection of a T2N1M0 adenocarcinoma of the rectum 1 year ago. He is asymptomatic, with a normal physical examination, and carcinoembryonic antigen of less than 0.5 mg/dL. In addition to a CT scan, surveillance at this time should include A. B. C. D. E.

ANSWER:

The intraoperative image depicts an intact, abnormal appendix with a swollen and well-circumscribed tip that appears to harbor a focal lesion (figure 22.2).

pelvic MRI. colonoscopy. rectal ultrasound. flexible sigmoidoscopy. PET-CT scan.

ANSWER:

B

This patient had a stage HI rectal cancer. Patients can develop a local or systemic recurrence. After curative treatment of a stage III rectal cancer, the goals are to detect recurrence early and to detect metachronous colorectal neoplasia. At 1 year, guidelines recommend a colonoscopy, in addition to an office visit, carcinoembryonic antigen laboratory test, and CT scan of the chest, abdomen, and pelvis. Although colonoscopy has the potential to detect a luminal recurrence in the area of the anastomosis, its primary purpose is to evaluate the colon and rectum for new adenomas and cancers. Pelvic MRI and rectal ultrasound are not strictly recommended for routine surveillance, because CT scan of the abdomen and pelvis is adequate. Routine flexible sigmoidoscopy every 6 months in rectal cancer patients is not consistently recommended by all specialty societies; however, this decision is obviated by the need for a colonoscopy at 1 year. A PET-CT scan is not part of routine surveillance for colorectal cancer.

Figure 22.2. These intraoperative findings are classic for a low-grade mucinous neoplasm of the appendix, also known as a mucocele. In an unruptured appendix, the neoplastic process is confined to the epithelium, and the appendix is filled with mucus. A small focus of invasive mucinous adenocarcinoma can be detected only by pathologic examination of the entire appendix. The most appropriate procedure at this point is an appendectomy with clear margins. However, great care must be taken to avoid perforation and spillage of neoplastic cells into the peritoneum, due to a concern that this may lead to pseudomyxoma peritonei.

22. A 68-year-old woman has an incidental finding of a 2-cm dilated appendix noted on a CT scan. There are no other abnormalities. The appendiceal lesion is shown in figure 22.1. The intraoperative findings are a clear appendiceal base and no abnormalities on evaluation of the rest of the peritoneal cavity. The most appropriate management of this patient is A. B. C. D. E.

A

Prophylactic oophorectomy is not necessary if there is no spread to the ovary, Completion right colectomy should be considered if the margins of resection are inadequate or if an invasive adenocarcinoma is found on final pathology, but in the image shown, the base of the appendix is not involved. Mesenteric lymphadenectomy is not necessary if there is no invasive carcinoma. Omentectomy is unnecessary because the patient does not have a malignancy. Although hyperthermic intraperitoneal chemotherapy can be

appendectomy. appendectomy with bilateral oophorectomy. right hemicolectomy. appendectomy with omentectomy. cytoreductive surgery with hyperthermic intraperitoneal chemotherapy.

111

considered for pseudo myxomatous disease, it is not of benefit as a prophylactic maneuver.

Peritoneal dialysis is a desirable option for many end stage renal disease patients because of patient autonomy, low cost, and improved quality of life compared with hemodialysis. A minilaparotomy is traditionally used to place the catheter semiblindly into the pelvis. Patients with prior abdominal surgery are candidates for peritoneal dialysis unless severe peritoneal adhesions preclude catheter flow and fluid exchange. Catheter tips may migrate out of the pelvis and become isolated from the peritoneal cavity because of omental wrapping and adhesions. Several techniques are used to prevent catheter dysfunction. Laparoscopic-assisted placement allows for more precise placement in the pelvis and for lysis of adhesions as indicated. It also reduces the incidence of mechanical dysfunction compared with open technique.

23. A 72-year-old woman is diagnosed with cecal cancer. When performing a curative segmental resection, the optimal mesenteric resection should include A. B. C. D. E.

proximal ligation of the ileocolic artery. sentinel lymph node biopsy. 10 cm proximal and distal bowel margins. removal of all palpable splanchnic lymph nodes. central lymph node (D3) dissection.

ANSWER:

A rectus sheath tunnel directs the catheter toward the pelvis, and the addition of omentopexy decreases the likelihood of omental entrapment. A meta-analysis comparing outcomes of laparoscopic-assisted peritoneal dialysis catheter and open placement showed a trend toward reduced peritoneal dialysis catheter removal but no difference in rates of peritonitis and exit-site infection. After infection, mechanical dysfunction is the second most common cause of peritoneal dialysis failure that necessitates conversion to hemodialysis.

A

Adequate mesenteric lymph node removal is the key to the oncologic resection of colon cancer. Removing and pathologically examining at least 12 mesenteric lymph nodes is recommended to render the most accurate cancer stage. Excised lymph nodes must be in the drainage basin of the cancer. For a cecal cancer, this drainage follows the path of the ileocolic artery, and a proximal ligation of the artery ensures adequate lymphadenectomy. Sentinel lymph node biopsy does not help guide the lymph node dissection. Although 5 cm proximal and distal bowel margins are recommended to clear the luminal component of the cancer, these margins do not affect the quality of the mesenteric lymph node dissection. A central lymph node (D3) dissection entails excising lymph nodes along the superior mesenteric artery and superior mesenteric vein. Neither a central lymph node dissection nor removal of all palpable splanchnic nodes improves the patient's outcome.

25. A key quality indicator for screening colonoscopy is A. an average withdrawal time of at least 10 minutes. B. an adenoma detection rate of 10%. C. an incidence of perforation of less than 1:1000 procedures. D. an incidence of postpolypectomy bleeding of 5%. E. a rate of cecal intubation of at least 98% with photo documentation.

ANSWER:

24. A 45-year-old woman with progressive end-stage renal disease is a potential candidate for peritoneal dialysis. Which of the following statements regarding catheter insertion is true?

The effectiveness of colonoscopy as a screening technique for colorectal neoplasia depends on the meticulousness of the procedure. To reduce the variability and to standardize the quality of screening, the American College of Gastroenterology (ACG) and the American Society for Gastrointestinal Endoscopy (ASGE) adopted key quality indicators. These include intubation of the cecum in at least 90% of all cases and in at least 95% of cases when screening a healthy adult patient, Photography of the cecum is mandatory to document successful cecal visualization. Careful examination of the colon lumen takes time, and an average withdrawal time of at least 6 minutes is the performance target. The proportion of screened patients with one or more adenomas found is defined as the adenoma detection rate. Given that adenomas are present in more than one-third of colonoscopic screenings, the ACG/ASGE set an adenoma detection rate target of at least 25%. Safety performance measures include perforations in fewer than

A. Prior abdominal surgery is a contraindication for peritoneal dialysis B. Open insertion has lower catheter mechanical dysfunction rates. C. Mechanical catheter dysfunction is the most common cause of conversion to hemodialysis. D. Laparoscopic insertion reduces the rate of exit-site infection compared with open insertion. E. Rectus sheath tunnel and omentopexy reduces the incidence of catheter mechanical dysfunction.

ANSWER:

C

E 112

1:1000 procedures and an incidence of postpolypectomy bleeding less than 1%.

27. A 42-year-old man with a history of familial adenomatous polyposis presents with a firm, 7.4-cm mass in the abdominal wall. A core needle biopsy is suspicious for a desmoid tumor. CT angiography shows no infiltration of intraperitoneal structures. What is the next step in treatment?

26. A 78-year-old woman with a history of atrial fibrillation and compensated congestive heart failure presents with a 2week history of diffuse abdominal pain, distention, and obstipation. Imaging studies confirm an obstructing lesion in the descending colon, and biopsy indicates an invasive adenocarcinoma. Compared with emergency resection, deployment of a self-expandable metal stent as a bridge to surgery can be expected to result in A. B. C. D. E.

A. B. C. D. E.

fewer complications. a higher mortality rate. more radical resections. fewer primary anastomoses. fewer temporary stomas.

ANSWER:

Enucleation Wide local excision C Imatinib Neoadjuvant chemotherapy Radiation

ANSWER:

B

Desmoid tumors are rare, accounting for approximately 3% of soft tissue neoplasms. Although most occur sporadically, desmoid tumors can be associated with familial adenomatous polyposis (FAP) syndrome due to mutations in the adenomatous polyposis coli (APC) gene. Up to 15% of patients with FAP syndrome develop desmoid tumors in the abdominal wall, abdomen, or retroperitoneum. Sporadically occurring desmoid tumors have a 2:1 female predilection, whereas FAP-associated tumors occur equally in men and women. Young adults are most commonly affected. Other risk factors include pregnancy, use of hormones, and local trauma or surgery.

E

Neoplastic colon obstruction is a life-threatening condition traditionally treated with emergency resection. Because most patients are older and present with bowel distension and serious comorbidities, temporary diversion with a stoma is often used as a bridging procedure to allow patient optimization and formal bowel preparation. Self-expanding metallic stents are now championed in patients with malignant colon obstruction as a means of bridging to elective resection without the need for a temporary stoma. Colonic stents are also used for palliative final treatment in patients with unresectable disease. The stents range in size from 20 to 30 mm in diameter and are placed endoscopically under fluoroscopic guidance. Once positioned, the stents are allowed to expand without direct dilation.

Desmoid tumors are also known as aggressive fibromatosis, which is perhaps a better description of their natural history. These tumors are not considered a true malignancy, because they do hot metastasize; however, they have a propensity for aggressive local growth and invasion of adjacent structures. Surgical resection with wide local excision to negative margins was traditionally considered the first-line treatment, but treatment of sporadic desmoids has recently evolved. In observational studies, some tumors showed long-term stability, and regression occurred in 5 to 10% of cases. Because of the morbidity of surgery, watchful waiting is the preferred approach for intra-abdominal desmoid tumors, especially in asymptomatic patients with tumors located in critical sites, such as the mesentery.

In a randomized controlled study published in 2017, researchers reported the results of a multicenter randomized trial comparing emergency resection versus bridging with a colonic stent for obstructing lesions in the descending or sigmoid colon. Technical success was achieved in 49 of the 56 stent cases (87.5%), with 8 (14%) perforations at the tumor site. The median time between stent placement and colon resection was 5 days. Stomas were performed in 39% of the emergency resection patients versus 22% of the stented group (p = .031). There were no differences in morbidity, extent of colon resection, or overall survival at 3 years between the 2 groups.

Surgical resection is first-line treatment for FAP-related desmoid tumors of the abdominal wall. Recurrence rates range from 20 to 45%, even in cases in which apparently clear margins were obtained Because of the infiltrative nature of these tumors, simple enucleation is not recommended Radiation therapy is effective in patients with unresectable tumors. Radiation therapy is also used as adjuvant therapy after re-resection for recurrent disease. Tyrosine kinase inhibitors, such as imatinib, are used in patients with advanced disease, but these agents cure associated with a low response rate.

In a case-matched, retrospective analysis, researchers reported a technical success rate of 93% for stenting as a bridge or palliative procedure in a series of patients with malignant proximal colon obstruction versus patients from an earlier era treated with emergency resection. There were no differences in morbidity, mortality, number of radical resections, or number of primary anastomoses between the patients undergoing stenting compared to those undergoing initial resection. However, stented patients were less likely to have a temporary stoma created.

28. A 46-y ear-old patient presents with a small bowel obstruction. He has a history of flushing episodes with antecedent diarrhea. During the operation, the patient is found to have numerous liver lesions along with an ileal mass with

113

enlarged lymph nodes. During the biopsy of one of the lesions on the surface of segment II, the patient becomes hypotensive and is unresponsive to fluids or norepinephrine. Frozen section of the lesion suggests a neuroendocrine tumor. What is the most appropriate management of the hypotension?

to decompress the colon. The patient also has a locally advanced distal rectal cancer, and the approach to the large bowel obstruction must be balanced with the need to provide oncologically appropriate care. Because the patient has CT scan findings consistent with stage III rectal cancer, upfront proctectomy should be avoided, if possible. The patient would be better served with neoadjuvant chemotherapy and radiation followed by definitive resection 8 to 12 weeks later. Therefore, proximal diversion is better than immediate low anterior resection when feasible. Upfront total proctocolectomy would be oncologically inappropriate for similar reasons. Instead, a loop sigmoid colostomy would effectively decompress the large bowel obstruction and allow for subsequent neoadjuvant therapy and surgery with curative intent. Using the sigmoid colon instead of the transverse colon will make the future coloanal anastomosis less technically demanding because it spares the transverse/descending colon conduit.

A. Transfuse packed red cells and fresh frozen plasma. B. Allow for permissive hypotension and continue the operation. C. Stop the operation and transfer the patient to the intensive care unit for resuscitation. D. Administer high-dose steroids. E. Administer high-dose octreotide.

ANSWER:

E

Gastroenteropancreatic neuroendocrine tumors, previously called carcinoid tumors, are indolent tumors of the midgut that can metastasize to the liver. These tumors are vasoactive and can release numerous peptides, including serotonin, histamine, bradykinin, and other peptides. Patients with liver metastases can develop carcinoid syndrome when the peptides bypass the hepatic circulation; symptoms include diarrhea, flushing, tachycardia, and bronchoconstriction.

Cecostomy tubes are effective in decompressing the cecum, but are generally very morbid, clog easily, and do not provide durable proximal diversion for patients like this who will not undergo proctectomy for several months. Colonic stent placement is appropriate for select obstructing left-sided colon cancers as a bridge to surgery or as palliation. However, stents are less effective for distal rectal cancers due to the high rates of migration, proctalgia, and incontinence, and technically, there is less room for distal stent deployment.

After manipulation of a hepatic metastasis, a carcinoid crisis can develop. This medical emergency involves extreme hypotension or hypertension. Under anesthesia, it manifests with unresponsive hypotension. In such situations, an octreotide infusion is titrated to reverse the crisis. Steroids are generally reserved for patients with adrenal insufficiency.

30. A 54-year-old woman diagnosed with ulcerative colitis 2 years ago is on infliximab therapy. She feels well overall and denies abdominal pain, diarrhea, or hematochezia. She undergoes surveillance colonoscopy, which reveals active colitis throughout the colon and rectum. There is a large mass in the cecum; biopsies reveal adenocarcinoma. Workup is negative for metastatic disease. What is the most appropriate surgical management? A. Cecectomy B. Right hemicolectomy C. Subtotal colectomy with ileosigmoid anastomosis D. Total proctocolectomy with ileal pouch-anal anastomosis E. Total proctocolectomy with continent ileostomy

Supportive measures, such as the use of crystalloid, are appropriate for patients with hypotension, but the use of blood or fresh frozen plasma without any clinical bleeding or anemia is not appropriate in this circumstance. Permissive hypotension is often allowed in hypovolemic shock but has not been studied in patients with a carcinoid crisis.

29. A 67-year-old man presents with progressive abdominal bloating and obstipation. Abdominal exam reveals severe distention with mild tenderness and no guarding. Digital rectal exam reveals a low-lying mass 5 cm from the anal verge that is circumferential. CT scan shows a complete large bowel obstruction with cecal distention to 11 cm, as well as a low rectal mass with surrounding mesorectal lymphadenopathy. What is the most appropriate management? A. Gecostomy tube B. Colonic stent placement C. Loop sigmoid colostomy D. Low anterior resection E. Total proctocolectomy

ANSWER:

ANSWER:

D

Patients with longstanding ulcerative colitis are at an almost 20% risk of developing colorectal cancer. The etiology is thought to be inflammation causing increased cell proliferation and turnover, which leads to dysplasia. When a cancer is found in the absence of metastatic disease, the standard of care is a total proctocolectomy. Total proctocolectomy with ileal pouch-anal anastomosis (IPAA) or restorative proctocolectomy, involves complete removal of all colorectal mucosa and then creation of a neorectum in the form of a J-shaped or S-shaped ileal pouch. Complication rates and long-term function are similar for ulcerative colitis patients with cancer and those undergoing

C

This patient has a complete large bowel obstruction with impending cecal perforation and requires urgent intervention

114

restorative proctocolectomy for other indications. Continent ileostomies are typically reserved for patients with pouch failure or those who are otherwise not candidates for a pouch. They are rarely used as first-line therapy.

excision followed by adjuvant radiotherapy. In this series, MD Anderson achieved an impressive local control rate of 82%, and 96% of patients remained colostomy free. Overall survival remained dismal because distant recurrence was very common. Nevertheless, this study suggested the possible role of adjuvant radiotherapy to improve local control. Although this study is promising, supporting data are limited. Therefore, this strategy has not been universally adopted. Furthermore, these data are from using radiation as adjuvant therapy. Radiation as the primary treatment would not be appropriate. There are very few promising data on chemotherapy for melanoma and certainly not as primary treatment. Ipilimumab is a monoclonal antibody that modulates the immune system and has shown some promise in the setting of metastatic melanoma, but it would not be used before resection. Although early reports were promising, long-term results, even with ipilimumab, remain dismal for most patients with metastatic disease.

Subtotal colectomy with ileosigmoid anastomosis or total colectomy with ileorectal anastomosis are less technically demanding, require less expertise, and avoid the morbidity of a pelvic dissection. However, they must be reserved for the rare and select group of patients who have rectal sparing, and thus would not be appropriate for this patient who has active proctitis. Additionally, up to 53% patients who undergo ileoproctostomy ultimately require subsequent proctectomy for poor function, ongoing proctitis, or subsequent rectal dysplasia and cancer. A cecectomy is not oncologically appropriate because it does not involve high ligation of the ileocolic artery and does not allow an adequate lymph node harvest, A right hemicolectomy would effectively treat the known cancer. However, in the setting of inflammatory bowel disease, right hemicolectomy would leave behind distal colon that is at a very high risk of metachronous malignancy. 31. A 62-year-old woman has increased anal pain associated with bleeding. A 1-cm mass is palpated in the anal canal and confirmed on colonoscopy. A biopsy is consistent with an anal melanoma. A whole-body PET-CT scan shows no evidence of disease other than that in the anal canal. What is the most appropriate primary treatment? A. Radiation B. Chemotherapy C. Ipilimumab D. Local excision E. Abdominal perineal resection

ANSWER:

D

Anal melanoma is an uncommon malignancy of the gastrointestinal tract with overall very poor prognosis. Survival is less than 25% and almost always associated with small tumors. Abdominal perineal resection (APR) was historically the treatment of choice for resectable disease, assuming this radical approach would provide the best option for long-term survival. Even with APR, both local and distant recurrences were common. This led to most experts recommending local excision when technically feasible. Local excision clearly has lower associated morbidity and, when compared with APR, a similar overall survival. Because the disease is not very common, most studies are small and retrospective in nature. Nevertheless, the conclusion of these studies shows no clear benefit to the more aggressive APR. Ideally, a local excision should include a 1-cm margin but achieving that can be challenging in the anal canal. Nevertheless, some data suggest this margin may affect both local control and perhaps overall survival. These data are likely to be influenced by significant selection bias. In 2011, MD Anderson reported its 20-year experience with local

115

Breast Part I

A. Total mastectomy with sentinel lymph node biopsy ± axillary dissection, chemotherapy, chest wall irradiation B. Chest wall radiation, chemotherapy, modified radical mastectomy C. Chemotherapy, modified radical mastectomy, chest wall irradiation D. Chemotherapy, chest wall irradiation, total mastectomy with sentinel lymph node biopsy E. Chemotherapy, total mastectomy with sentinel lymph node biopsy ± axillary dissection, chest wall irradiation

ITEMS 1-22 For each question, select the best possible response. 1. A 50-year-old woman has a routine screening mammogram. It shows microcalcifications in an area spanning 3 cm in the right breast. A core needle biopsy shows ductal carcinoma in situ. The patient opts for breastconserving therapy and undergoes partial mastectomy. Which of the following is considered an adequate margin? A. B. C. D. E.

No ink on tumor 1 mm 2 mm 5 mm 1 cm

ANSWER:

C

Figure 2.1.

Breast-conserving therapy is associated with high long-term disease-free survival in women with ductal carcinoma in situ (DCIS). Breast-conserving therapy is defined as removal of the primary tumor and a margin of surrounding normal tissue followed by whole breast irradiation. As with invasive carcinoma of the breast, the margin. width for breastconserving therapy in DCIS is controversial. This is in part because although 3 randomized trials required microscopically clear margins for DCIS, the fourth did not. These 4 studies did not answer the question of whether more widely clear margins reduced rates of ipsilateral breast tumor recurrence (IBTR) for DCIS. Although several nonrandomized studies concluded that larger margins were associated with decreased risk of recurrence, until recently there was no consensus.

ANSWER:

C

Inflammatory breast cancer is a rare but aggressive form of breast cancer. Current treatment for inflammatory breast cancer includes a trimodal approach— chemotherapy, surgery, and radiation therapy. Current guidelines recommend anthracycline-based neoadjuvant chemotherapy, modified radical mastectomy, and postmastectomy radiation therapy. Five- and 10-year survival rates are highest among patients who received trimodal treatment compared with any other treatment group (55.4% and 37.3%, respectively). Surgery is not the first step in the treatment of inflammatory breast cancer, because the dermal involvement precludes adequate clearance of disease. Therefore, systemic therapy is the first line of treatment. If sufficient response is achieved that the disease becomes "operable" (i.e., all inflammatory changes resolved, or residual inflammatory changes can be removed with reasonable resection margins), then surgery is the second line of treatment followed by comprehensive postmastectomy.

In 2016, consensus panel recommendations were published for margin width in DCIS. The panel included representatives from the Society of Surgical Oncology, the American Society for Radiation Oncology, and the American Society of Clinical Oncology. The primary question was "What margin width minimizes the risk of IBTR in patients with DCIS receiving breast-conserving therapy?" The panel concluded that a 2mm margin was adequate in DCIS treated with whole-breast irradiation. Clinical judgment should be used in determining the need for further surgery in patients with negative margins narrower than 2 mm.

radiation therapy, including all regional nodal basins. Rarely, the response to systemic therapy may be inadequate to proceed with surgery; palliative radiation may be considered in this scenario. A diagnosis of inflammatory breast cancer is one of the few remaining contraindications to sentinel lymph node biopsy, and axillary node dissection remains the standard of care for axillary staging and local control. No data support the long-term efficacy of breast-conserving surgery in patients with inflammatory breast cancer.Total mastectomy is removal of the breast alone and is inadequate for the treatment of inflammatory breast cancer.

2. A 47-year-old woman presents with an enlarged, erythematous, right breast with peau d'orange skin changes of 4 weeks duration (figure 2.1). The skin changes have progressed on antibiotic therapy. Core needle biopsy reveals high-grade ER", PR", HER2" cancer. MRI reveals diffuse non-mass enhancement throughout the left breast. CT scan and bone scan are negative for metastases. What is the recommended course and sequencing of treatment for this patient?

3. A 30-year-old woman has been treated for 3 periareolar breast abscesses in the same location within the last 4 months.

117

She now complains of noncyclic mastalgia and intermittent nonpurulent nipple discharge. She admits to smoking onehalf pack of cigarettes per day, but she is otherwise healthy and there is no family history of breast cancer. Findings at examination are noted in the photograph (figure 3.1). There are no palpable masses in the breast, and the axillary examination is normal. What is the most appropriate next step? A. B. C. D. E.

Oral antibiotics Incision and drainage Fistulotomy Mammography Fistulectomy with excision of underlying duct ectasia

Figure 3.2.

Figure 3.1.

ANSWER:

E

This patient has recurrent periductal mastitis with a periductal fistula. The fact that this patient has undergone 3 failed attempts at incision and drainage should alert the clinician that repeating this intervention is unlikely to be successful and that this condition may represent something more complicated than a simple abscess. The clinical findings of periductal mastitis are characterized by noncyclic mastalgia, nipple discharge and retraction, a subareolar breast mass, and a mammary fistula, as seen in this case. This is generally due to ectasia of a lactiferous duct, and smoking appears to be a factor in the etiology. If this patient does not quit smoking, recurrence is almost assured regardless of treatment.

Figure.3.3. 4. A 48-year-old woman presents with bilateral axillary enlargement that worsens with her menstrual cycle. It causes her no discomfort, and there are no overlying skin changes or abnormalities on examination. A representative photograph is shown (figure 4.1). What is the most likely diagnosis? A. Lymphedema B. Hidradenitis C. Ectopic breast tissue D. Granulomatous mastitis E. Cat scratch disease

lesions may become infected with bacteria and initially present as a periductal breast abscess. In this case, there is no evidence to suggest an active infection, thus oral antibiotics are not indicated. Although fistulotomy is proposed as a possible treatment for this condition, this does not effectively remove the ectatic ductal system and therefore is associated with a high rate of recurrence. Mammography is not indicated given her age and the fact that there is no family history of breast cancer. The most appropriate next step is to excise the fistulous tract en bloc with the ectatic ductal system, which offers the best chance of cure and the lowest recurrence rate (figures 3.2 and 3.3).

Figure 4.1. 118

ANSWER:

firm, discrete, nontender nodule is palpated. Breast ultrasound shows a corresponding 2- * 1.5-cm wellcircumscribed, ovoid, sharply marginated, anechoic lesion with posterior acoustic enhancement consistent with a simple cyst. What is the best next step in her management?

C

Ectopic breast tissue (i.e., accessory breast tissue) is breast tissue that persists beyond embryological development anywhere along the mammary ridge outside of the normal breast location. The mammary ridge in humans extends from the inner upper arm to the inner thigh and is often referred to as the "milk line." The axilla is the most common location for accessory breast tissue as demonstrated in this patient with bilateral involvement (figure 4.2). The reported incidence of ectopic breast tissue is 0.4 to 6%, and most patients are asymptomatic. However, symptoms of pain may develop during lactation or menstruation, and patients may be concerned with the cosmetic appearance of the accessory tissue. The breast tissue in an ectopic location is histologically normal yet has the potential for development of both benign and malignant breast disease. Little evidence exists to suggest that the risk of malignancy is higher in accessory tissue. However, surveillance may be a challenge, and the clinician must be cognizant of including accessory breast tissue in both clinical examination and routine surveillance imaging protocols.

A. B. C. D. E.

Mammogram Aspiration Reassurance Surgical excision Repeat ultrasound imaging in 6 months

ANSWER:

C

Cysts are the most common benign breast lesions and can be found in as many as one-third of women ages 35 to 50. Most cysts are nonpalpable (microcysts) and are found on routine mammography, ultrasonography, or MRI. In approximately 20 to 25% of cases, patients may present with palpable or "gross" cystic change, which prompts further evaluation with imaging. In patients younger than 30, a palpable mass should be evaluated initially with ultrasonography, followed by mammography if there is clinical suspicion for malignancy. For patients 30 years or older, mammography is usually performed first with attention to the palpable area; however, mammography cannot accurately distinguish cysts from solid masses, Ultrasonography and fine needle aspiration cytology are both highly accurate and are the diagnostic methods of choice if there is any concern that the findings represent a solid mass. Cysts can be classified as simple or complex. Simple cysts are epithelium lined, fluid filled, round or oval, and not associated with an increased risk of malignancy. They likely occur due to obstructed ducts and can either be isolated in the breast or diffuse. On ultrasonography, simple cysts appear avascular, anechoic, oval, or round, with posterior enhancement and an imperceptible wall. With gentle compression of the lesion by ultrasound probe, a cyst can flatten, whereas solid masses do not. Simple cysts are usually asymptomatic. If the cyst is round and tense, the provider can aspirate the cyst for symptomatic relief.

Figure 4.2. Routine surgical excision is not warranted in the asymptomatic patient, and liposuction may be a reasonable alternative when cosmesis is the primary concern. Cat scratch disease and granulomatous mastitis are both inflammatory conditions that would not be expected to change with the menstrual cycle and would likely cause significant pain. Likewise, the severity of lymphedema is unlikely to change with the menstrual cycle and is generally not as localized or bilateral as in this patient. Hidradenitis suppurativa is another inflammatory condition that may present in both axillae at the same time but generally presents with symptoms of pain or drainage and is likewise unlikely to change in severity with menstruation.

By contrast, ultrasonographic features of complex cysts include internal echoes or thin septations, thickened or irregular walls, and lack of posterior enhancement. The risk of malignancy is approximately 0.3% for complex cysts. Therefore, they can be managed with routine follow-up imaging studies.

6. A 30-year-old premenopausal woman is referred with a new growth involving her right nipple. She nipple discharge, other skin changes, or masses. She family history of breast malignancy. A skin punch reveals an adenoma. These lesions

5. A 27-year-old healthy woman presents with a newly palpable,right breast nodule. On physical exam, a smooth,

A. require no further treatment.

119

to you denies has no biopsy

B. C. D. E.

are malignant entities. are a type of intraductal papilloma. are typically accompanied by nipple discharge. occur only in women.

ANSWER:

Evidence is not sufficient to recommend for or against annual breast MRI screening in women with a personal history of lobular carcinoma in situ or ductal carcinoma in situ. There have also been no phase III randomized trials on the utility Abreast MRI versus mammography in women with dense breast tissue.

C

A nipple adenoma is a rare, benign breast tumor. It can present with symptoms of nipple erosion, crusting, or nodule formation. Nipple adenomas are more commonly found to be unilateral but can present bilaterally. They occur in women ages 30 to 40 years but are also found in men. A nipple adenoma is a type of intraductal papilloma and is a benign tumor of the ductal epithelium localized around the collecting ducts. On histology, the adenoma is characterized by proliferating ductal structures that invade the surrounding stroma.

8. A 55-year-old premenopausal woman presents with a firm, large, nontender right breast mass with no nipple discharge. She has no family history of breast cancer, and a history of a previous benign breast biopsy. The only medication she takes is insulin. On physical exam, there is a palpable 3-cm firm, mobile, nontender mass in the right breast with no axillary lymphadenopathy. There are no overlying skin changes. Bilateral mammograms show extremely dense tissue, and an ultrasound shows a hypoechoic mass. What is the most appropriate next step in her management?

On clinical examination, a nipple adenoma is located under the nipple tissue, which can lead to difficulties in visualization on imaging studies. The adenoma has clear margins, and the surface of the nipple can show erosion, crusting, or ulceration. Thus, the diagnosis of a nipple adenoma is typically made by surgical excision. Surgical excision is also generally warranted to mitigate the local symptoms caused by the lesion itself.

A. B. C. D. E.

ANSWER:

7. Screening MRI is recommended by the American Cancer Society for patient

B

The diagnostic evaluation of a palpable breast lesion includes physical examination, imaging, and tissue diagnosis. The physical examination includes the clinical breast . exam, which also consists of palpation of regional lymph nodes. Mammography or ultrasonography can be used for imaging, and pathology can be obtained via fine needle aspiration or core needle biopsy. This work-up precludes the need for the more invasive incisional or excisional biopsy. If pathology indicates a benign lesion and this is thought to be concordant with physical examination or imaging findings, surgery can be avoided.

A. who are BRCA mutation carriers. B. with dense breast tissue. C. who have a lifetime risk of breast cancer of 10 to 15%, based on family history. D. with a personal history of lobular carcinoma in situ. E. with a personal history of ductal carcinoma in situ.

ANSWER:

MRI Core needle biopsy Fine needle aspiration Excisional biopsy Incisional biopsy

A

Mammography is the standard of care for breast cancer screening in women of average risk It is the only breast cancer screening modality that reduces breast cancer mortality. Women at increased risk for breast cancer, generally defined as lifetime risk greater than 20%, benefit from additional imaging with breast MRI.

Fine needle aspiration is the traditional method used to obtain cytology; it is both rapid and safe. However, core needle biopsy is increasingly used due to the advantage of providing preoperative knowledge of the histology and prognostic factors of the breast lesion. Studies directly comparing fine needle aspiration with core needle biopsy demonstrate that core needle biopsy is superior in terms of yielding more conclusive diagnoses for both malignant and benign breast lesions. Core needle biopsy can differentiate between ductal carcinoma in situ and invasive ductal carcinoma and can better distinguish between invasive lobular carcinoma and invasive ductal carcinoma. These distinctions are fundamental in the preoperative assessment and treatment planning for the patient with breast cancer.

In 2007, the American Cancer Society recommended breast MRI screening as an adjunct to mammography in women at high risk for breast cancer. This includes those with a BRCA mutation, a first-degree relative who is a BRCA carrier, a prior history of mantle irradiation, and a lifetime risk of breast cancer of at least 20 to 25% based on a careful family history. Although survival or mortality benefit is not available, MRI has consistently demonstrated higher sensitivity for breast cancer detection in these patient cohorts compared with mammography alone, and current guidelines now include breast MRI in addition to mammography as part of annual screening for these categories of high-risk women.

MRI is not part of the standard diagnostic evaluation for women presenting with a palpable breast mass, and MRI cannot replace tissue confirmation of a benign versus malignant-appearing lesion. In this case, the diagnostic

120

evaluation of the palpable breast mass with mammography was inconclusive, and the ultrasound confirmed a hypoechoic mass. The most definitive means to determine whether the lesion is benign or malignant is to obtain a tissue specimen via a core needle biopsy.

enlarged; the skin is warm, thickened, and erythematous. A breast ultrasound does not demonstrate an underlying fluid collection but shows a poorly marginated 3-cm hypoechoic area. Mammogram demonstrates only increased density of the breast with skin thickening. What is the best next step in her management? A. B. C. D. E.

9. A 40-year-old woman has a 2-cm area of pleomorphic calcifications in the upper outer quadrant of the left breast. Stereotactic biopsy demonstrates ductal carcinoma in situ (DCIS), intermediate to high grade with comedo necrosis, and immunohistochemistry shows the DCIS to be estrogen receptor positive. She is interested in breast conservation. What is the optimal margin width to reduce her risk of local recurrence? A. B. C. D. E.

ANSWER:

A

The diagnosis of inflammatory breast cancer continues to be a challenge for many providers. Classically described as presenting with the rapid onset of diffuse breast erythema and edema with or without an underlying breast mass, it is often confused with infectious conditions, and antibiotics are pursued for varying periods of time without notable improvement, contributing to delays in diagnosis. Despite significant advances in our understanding of breast cancer molecular subtypes, patients with inflammatory breast cancer have inferior survival outcomes compared with other patients with locally advanced disease.

No ink on tumor 1 mm 2 mm 5 mm 10 mm

ANSWER:

Ultrasound-guided core biopsy Breast MRI Skin biopsy of the breast Course of intravenous antibiotics Ultrasound-guided needle aspiration

C

Breast-conserving surgery or lumpectomy is defined as the excision of the breast malignancy with a margin of surrounding normal tissue. Breast-conserving surgery is typically followed by whole-breast irradiation to reduce the risk of in-breast recurrence. When breast-conserving therapy is pursued, the goal is to obtain a negative surgical margin while preserving cosmesis of the breast. The definition of a clear margin or the optimal width of the margin to minimize the risk of local recurrence is an area of controversy.

All patients suspected of having inflammatory breast cancer should undergo imaging, including a diagnostic mammogram and ultrasound of the breast and regional lymph nodes. Breast MRI is generally not indicated to confirm the diagnosis; however, if diagnostic mammogram and ultrasound fail to detect an underlying breast mass/parenchymal lesion, an MRI may be helpful. When imaging demonstrates an underlying breast mass/parenchymal lesion or abnormal regional lymph nodes, an image-guided core biopsy of both the breast and any abnormal-appearing lymph nodes is recommended for diagnosis, histologic tumor classification, immunohistochemical (ER, PR, HER2) subtyping, and staging.

In 2016, the Society of Surgical Oncology, the American Society for Radiation Oncology, and the American Society of Clinical Oncology published a consensus guideline on margins for breast-conserving surgery followed by wholebreast irradiation in patients with ductal carcinoma in situ (DCIS). This consensus was based on the results of a metaanalysis that included studies of margin width and ipsilateral breast tumor recurrence for patients with a DCIS diagnosis undergoing breast-conserving therapy.

Some practitioners have suggested that skin biopsies should be used to confirm the diagnosis, because the inflammatory skin changes of the breast represent dermal lymphovascular invasion that can be histologically visualized via skin biopsies. However, even with adequate sampling, dermal lymphovascular invasion may not be visualized in up to 75% of patients with confirmed inflammatory breast cancer. Thus, the diagnosis of inflammatory breast cancer remains a clinical diagnosis characterized by the rapid onset of diffuse erythema and edema of the Breast. Diagnosis requires an appropriate level of clinical suspicion when encountered and should be followed by appropriate diagnostic imaging. The absence of dermal lymphovascular invasion does not rule out a diagnosis of inflammatory breast cancer.

The authors concluded that a positive margin, defined as ink on tumor, was associated with increased rates of in-breast recurrence; whereas a 2-mm margin width minimized the risk of in-breast tumor recurrence compared with lesser margin widths. Importantly, margins larger than 2 mm did not further decrease the risk of in-breast tumor recurrence. Therefore, the consensus panel concluded that a margin with of 2 mm was the appropriate margin width for patients with DCIS undergoing breast conservation

10. A 58-year-old woman presents with right breast pain and erythema with no improvement after 2 weeks of oral antibiotics. On physical exam, the breast is firm, tender, and

11. You perform a lumpectomy and sentinel lymph node biopsy on a 37-year-old patient with ER/PR-negative HER2negative invasive breast cancer. The pathology report shows

121

a 1.9-cm invasive ductal cancer with the closest margin measuring 1 mm. One of 2 sentinel lymph nodes show a 3mm metastasis without extracapsular extension. What would you advise the patient regarding her need for further surgery?

C. Perform SLNB before and after neoadjuvant chemotherapy. D. Do not perform SLNB in this setting. E. Only perform SLNB if preoperative imaging demonstrates a complete radiographic response.

A. B. C. D.

No further surgery is indicated. Re-excision of the lumpectomy cavity is necessary. She should undergo completion axillary node dissection. Re-excision of the lumpectomy cavity with completion axillary node dissection is necessary. E. She should undergo modified radical mastectomy.

ANSWER:

ANSWER:

A

Until recently, breast cancer patients with biopsy proven node-positive disease (cNl) underwent axiliary node dissection and were not candidates for sentinel lymph node biopsy (SLNB).

A

The increased use of neoadjuvant chemotherapy in patients presenting with node-positive disease and the high rates of nodal pathologic complete response (pCR), particularly in patients with HER2+ and triple negative breast cancer, led to investigations of the feasibility of SLNB in this setting with the goal of avoiding axillary lymph-node dissection in patients who achieve a nodal pCR. Several studies demonstrated the feasibility of SLNB in patients who present with cNl disease and convert to cNO disease after neoadjuvant chemotherapy. In these trials, clinical node status after neoadjuvant chemotherapy was defined by physical examination, and negative axillary imaging was not required.

For patients with early-stage invasive breast cancer undergoing breast conservation therapy (lumpectomy plus radiation therapy), the accepted definition of a negative margin is "no ink on tumor." This definition is supported by the joint consensus guideline for patients with stage I and II invasive breast cancer undergoing breast conserving surgery and whole-breast irradiation from the Society of Surgical Oncology and the American Society for Radiation Oncology. In the meta-analysis that informed this consensus, margins wider than no ink on tumor were not associated with a lower risk of in-breast recurrence. Thus, re-excision to obtain a wider negative margin is not indicated. Young age and triple negative breast cancer subtype do not change this recommendation, because no data support that bigger surgery, wider margins, or mastectomy (the widest possible margin) are associated with lower rates of recurrence, even among high-risk groups.

When performing SLNB in this setting, the recommendations are to minimize the false-negative rate of the procedure using dual tracer, identify and remove at least 3 sentinel lymph nodes, and remove any node that was biopsied and marked with a clip before treatment. The use of clips to mark nodes that undergo biopsy remains variable and is not required if the SLNB procedure is otherwise successful and at leasts sentinel lymph nodes are removed.

Similarly, management of the axilla should follow the algorithm of the ACOSOG ZOO 11 trial, a randomized clinical trial comparing sentinel lymph node dissection (SLND) alone to SLND plus axillary lymph node dissection (ALND) in patients with cTl-2 NO breast cancer found to have 1 or 2 positive sentinel lymph nodes. Even though additional nodal disease was present in 27% of patients in the ALND arm of ZOO 11, there was no difference in rates of local recurrence, disease-free survival, or overall survival based on the performance of an ALND. The 5-year overall survival rates were 92.5% and 91.8% in the SLND-alone and ALND groups, respectively. Thus, there is no indication for further axillary surgery in the patient with a 3-mm metastasis in 1 of 2 sentinel nodes.

Performing SLNB before neoadjuvant chemotherapy is not indicated because the patient is already known to have positive nodes. Any further nodal surgery before neoadjuvant therapy denies the patient the opportunity to benefit from downstaging of the axillary disease. Similarly, there is no role for SLNB before and after neoadjuvant chemotherapy.

13. A 16-year-old girl presents to the general surgery clinic with a 1.5-cm palpable breast mass. The most likely diagnosis is A. B. C. D. E.

12. A 55-year-old woman is about to begin neoadjuvant chemotherapy for a stage II, biopsy-proven node positive (cNl) invasive ductal carcinoma (ER+, PR+, Her2+). When considering the role of sentinel lymph node biopsy (SLNB) to evaluate the axilla in this scenario, which of the following is the most appropriate strategy?

tubular adenoma. phyllodes tumor. fibroadenoma. pseudoangiomatous stromal hyperplasia. simple cyst.

ANSWER:

A. Perform SLNB after neoadjuvant chemotherapy. B. Perform SLNB before neoadjuvant chemotherapy.

C

Most breast masses in adolescents are benign. Fibroadenomas are the most common breast masses in adolescent girls and

122

are a benign overgrowth of stromal connective tissue from the breast lobule. They typically are rubbery, mobile, and painless masses on physical exam. Ultrasound will demonstrate a well-circumscribed hypoechoic mass. Management ranges from excision to watchful waiting with serial ultrasound exams.

pain, tenderness, erythema, and swelling. Patients can present with fever, chills, and body aches if the mastitis is severe. Treatment of lactational mastitis is effective milk emptying with encouragement to continue breastfeeding and other supportive measures, such as rest and hydration. Patients may take anti-inflammatory medications and use warm compresses for pain control. The use of antibiotics varies, as some studies demonstrated adequate resolution with supportive measures alone. However, most experts recommend antibiotics to cover common bacterial pathogens.

Tubular adenomas are benign breast tumors that some surgeons consider to be a variant of fibroadenoma. Phyllodes tumors are rare tumors that also arise from stromal connective tissue from the breast lobule and are clinically and radiographically indistinguishable from fibroadenomas. Malignant and benign forms of phyllodes tumors exist and cannot be distinguished by fine needle aspiration; therefore, the patient should undergo biopsy. Large phyllodes (>5 cm) or tumors that rapidly enlarge should be excised.

This patient does not have evidence of a breast abscess; however, many experts use ultrasound of the breast to rule out abscesses. Less than 3% of patients with lactational mastitis present with an abscess. If an abscess is suspected and identified, aspiration is the treatment of choice along with the therapy described here. Incision and drainage is not recommended in the initial treatment of breast abscesses.

Pseudoangiomatous stromal hyperplasia (PASH) is a benign proliferation of breast stromal tissue with interconnected channels lined with spindle cells. The spaces in these tumors are lined by myofibroblasts and contain mucopolysaccharide. Pseudoangiomatous refers to the similarity of the interconnected slit-like spaces because they resemble vessels. PASH can also present as a rubbery mass and is sonographically similar to fibroadenomas. Biopsy of the lesion is indicated to rule out sarcoma. Simple cysts are benign and may be followed clinically.

Although rare, inflammatory breast cancer should always be a consideration. Thus, appropriate follow-up is necessary to ensure that the patient's symptoms have resolved with medical therapy.

15. A 63-year-old woman undergoes routine screening mammography with the results shown (figures 15.1 and 15.2). She has no associated mass. Image-directed core needle biopsy reveals an invasive cancer. The patient opts for breast-conserving therapy and undergoes partial mastectomy. Which of the following is considered a clear margin for the invasive cancer? A. No ink on tumor B. 1 mm C. 2 mm D. 5 mm E. l cm

14. A 24-year-old woman presents to the emergency department with complaints of left breast pain, swelling, and redness. She has a 1-month old child and is currently breastfeeding. On examination, her left breast is swollen with cellulitis present at the 3 o'clock position near the areola. There is no fluctuance or purulent material draining or expressed from the nipple. What is the best next step in management? A. Antibiotics and warm compresses B. Incision and drainage C. Discontinuation of breastfeeding D. Breast biopsy with specimen to include skin E. Aspiration

ANSWER:

A

Mastitis is a common benign breast disease encountered in the lactating woman, occurring in 2 to 33% of breastfeeding women. Given its common association with lactation, it is also commonly referred to as lactational mastitis or puerperal mastitis. The primary cause is milk stasis due to ineffective baby positioning, limited feeding, or restricted feeding. The mastitis may be sterile or may grow bacteria. The most common bacteria associated with infective mastitis are Staphylococcus aureus and Staphylococcus albus. Delayed or inappropriate therapy leads to unnecessary cessation of breastfeeding, tissue damage, abscess, and greater costs and morbidity. Patients typically present with unilateral breast

Figure 15.1.

ANSWER: 123

Figure 15.2.

A

The patient's mammogram reveals a suspicious lesion (figures 15.3 and 15.4). The biopsy confirms breast cancer. She desires breast-conserving therapy.

Figure 15.3.

The treatment of invasive breast cancer continues to evolve from radical surgical procedures based on physical examination to multimodality therapy based on the biology of the tumor. Biomarkers, including estrogen and progesterone receptor and HER2, play important roles in determining treatment. For a patient with a node negative, hormone receptor positive, HER2 negative breast cancer, European guidelines recommend multianalyte genetic expression profiles to predict outcome and assist in making decisions about adjunct therapy. They also recommend all available gene expression profiles, except the Breast Cancer Index, for patients with hormone receptor positive cancers and 1 to 3 positive lymph nodes. The 21-gene RT-PCR (Oncotype DX®) gene expression profile is an option for patients with ER positive, node negative tumors to help estimate the likelihood of recurrence and the benefit of chemotherapy. The 21-gene RT-PCR can be used in select patients with 1 to 3 involved ipsilateral nodes to guide the addition of chemotherapy to standard endocrine therapy. The next step in this patient would be a gene expression profile to guide further treatment.

Figure 15.4.

Breast-conserving therapy is complete removal of the primary tumor including normal tissue followed by whole breast irradiation. Multiple randomized trials with mature follow-up have proved breast-conserving therapy equivalent to mastectomy for the treatment of stage I and II breast cancer. Although breast-conserving therapy was incorporated widely into practice in the late 1990s, rates of re-excision for margins remained high. In 2014, a consensus panel including representatives from the Society of Surgical Oncology and the American Society for Radiation Oncology published guidelines for margin width for invasive cancer. The goal was to answer the question "What margin width minimized the risk of ipsilateral breast tumor recurrence (IBTR)?" The meta-analysis that formed the basis of the guidelines included 33 eligible trials. The panel concluded that "the use of no ink on tumor as the standard for an adequate margin in invasive cancer in the era of multidisciplinar therapy is associated with low rates of IBTR and has the potential to decrease reexcision rates, improve cosmetic outcomes, and decrease healthcare costs."

17. A 65-y ear-old otherwise healthy woman with no family history of breast or ovarian cancer presents with enlarging axillary adenopathy. The rest of her physical examination is unremarkable. Core biopsy of the nodes reveals invasive ductal cancer, intermediate grade, hormone receptor positive, her2 neu negative. MRI and bilateral mammograms are read as normal. CT scan of the chest shows only the axillary adenopathy, and CT scan of the abdomen and pelvis and a bone scan are negative. The patient should be told that A. B. C. D. E.

ANSWER:

16. A 67-year-old woman has a 2.2-cm, grade 2, ER positive, PR negative, HER 2 negative invasive ductal breast cancer. She has a partial mastectomy and sentinel lymph node biopsy. Pathology shows the 2.2-cm tumor with clear margins and 2 negative sentinel nodes. The next step in her management is A. B. C. D. E.

A

Despite the sensitivity of current breast imaging, approximately 1% of all breast cancers still present as axillary adenopathy without a known breast primary, so-called "occult breast cancer." Specifically, MRI detects a primary in 80% of women with breast cancer metastatic to regional nodes and a negative mammogram, leaving 20% of such women with the primary still undetected by imaging. Mastectomy still fails to reveal the primary in 30 to 60%, of such cases and survival rates with mastectomy or radiation therapy are similar. These findings are the reason mastectomy or radiation are equally recommended. A national study in Korea of 142 occult breast cancer patients showed equivalent survival rates after axillary dissection with or without mastectomy. Regarding systemic therapy for occult breast

endocrine therapy. genetic expression profile of the tumor. chemotherapy. chemotherapy and endocrine therapy. no systemic therapy.

ANSWER:

a primary tumor will rarely be found. hormone therapy is contraindicated. treatment should include mastectomy. tumor gene testing will reveal a dominant mutation. survival is worse than in cases with known primary.

B 124

cancer, there is not enough evidence to recommend one treatment over another. In the Korean study, 43% of patients received endocrine therapy for hormone positive tumors; the average age of patients tended to be older (65 in one series), so that chemotherapy was not always offered.

In addition to gynecomastia, men who use anabolic androgenic steroids are typically well virilized, but have small testes, a low sperm count, and low levels of sex hormone binding globulin. Additional characteristics may include an elevated hematocrit and acne.

The data regarding survival in patients with occult breast cancer are mixed. Most series of occult breast cancers showed outcomes that were better than those in which the primary is known, with similar amounts of nodal involvement, whereas a few showed comparable outcomes and only one study showed that occult breast cancer is associated with a worse prognosis. Occult breast cancer is a subset of the broader group of "cancers of unknown primaries" (CUPs). Although genetic testing is an interesting avenue for future study in CUPs, no data as yet show clinical utility of genetic testing of the patient or the tumor, a link between occult breast cancer and known breast cancer gene mutations, or dominant driver mutations in the tumor. This is likely due to the rarity of the condition, the older mean age of patients, and the complex gene signature of such tumors.

19. A 42-year-old GOPO woman presents for evaluation of spontaneous nipple discharge from her right breast. She has no personal or family history of breast cancer and has never taken exogenous hormones. On physical examination, both breasts are soft without dominant masses. There is no nipple discharge on the left. On the right, serous fluid emanates from a single duct. Mammogram is normal. What is the next step in her management? A. B. C. D. E.

18. A 30-year-old male bodybuilder presents for evaluation of bilateral breast enlargement. He takes anabolic androgenic steroids. What characteristic is this patient likely to have in addition to breast enlargement? A. B. C. D. E.

ANSWER:

B

Nipple discharge is very common and occurs in up to 5 to 10% of women. Nipple | discharge is a common reason that women seek medical care. Physiologic, or benign nipple discharge is bilateral, arises from multiple ducts, is provoked, and can be either longstanding or intermittent in nature. The character of the expressed fluid is milky or dark green. Physiologic nipple discharge is usually due to pregnancy, breastfeeding, medications, or benign hyperplasia and typically does not require workup other than a detailed history and thorough breast examination.

Enlarged testicles High sperm count Low estradiol levels Acne Anemia

ANSWER:

Ductoscopy Ultrasound MRI Galactography Subareolar duct excision

D

Gynecomastia is benign enlargement of male breast tissue and is a common finding. Normal male breast tissue has receptors for both estrogens and androgens. The growth of breast tissue is stimulated by estrogens and inhibited by androgens. Most cases of gynecomastia result from androgen deficiency or estrogen excess; more specifically, an absolute or relative deficiency of androgens, deficient androgen action, an increase in levels of estrogen, or an increase in estrogen action. The histologic characteristics of gynecomastia include ductal epithelial hyperplasia and an increase in stromal and periductal connective tissue.

Pathologic nipple discharge is unilateral, arises from a single duct, and is typically spontaneous and persistent. The character of the expressed fluid is serous (clear, yellow, orange) or bloody. Pathologic nipple discharge is associated with an | increased risk of underlying disease. The majority of pathologic nipple discharge is caused by papilloma or benign duct ectasia. Malignancy, including ductal carcinoma in situ and invasive ductal carcinoma, is much less frequently encountered. For patients with pathologic nipple discharge, after history and physical examination, the standard of care for the next step in workup is mammography and breast ultrasound. An ultrasound is performed even when mammography is normal, looking for abnormalities not seen on mammography.

Physiologic gynecomastia occurs during infancy, puberty, and in the elderly. Medications are the most common cause of nonphysiologic gynecomastia in adults. Anabolic androgenic steroids are performance-enhancing drugs often used by athletes, bodybuilders, and even nonathletes to augment muscle development. Aromatizable androgens such as testosterone or androstenedione undergo peripheral aromatization to estradiol or estrone, in turn resulting in an androgen-estrogen imbalance.

The central question underpinning the management of pathologic nipple discharge is whether a duct excision should be performed in patients with a normal physical examination (other than the nipple discharge) and a negative mammogram and ultrasound. Historically, the standard approach for management of these patients was to proceed with duct

125

excision to exclude malignancy. Although the operation is relatively straightforward, there is associated risk, including interference with breastfeeding, loss of nipple sensation, and nipple areolar necrosis. The more recent literature recognizes that for many patients, proceeding directly to duct excision may be too invasive an approach, and additional diagnostics may be indicated to further characterize the process. Therefore, to avoid unnecessary duct excision, a selective approach for surgical management may be considered.

Cyclical mastalgia typically starts with the luteal phase and ends with the onset of menstruation. This scenario implies a hormonal etiology. Many hormonal, herbal, and dietary remedies can be tried. Hormonal therapies include tamoxifen, bromocriptine, and danazol. Of these, tamoxifen has the fewest side effects and is considered a first-line therapy. Many different herbal and vitamin supplements are used to treat mastalgia. The most common is evening primrose oil. Evening primrose oil was evaluated in several studies, including 2 randomized controlled trials. Neither showed a benefit to evening primrose oil.

Selection of patients most likely to benefit from surgery is determined by additional diagnostic testing. Previously, when mammography and ultrasound were diagnostically unrevealing in patients with pathologic nipple discharge, the next step was typically galactography. However, breast MRI is gradually replacing conventional galactography, as the recent literature indicates a higher diagnostic performance of breast MRI compared with galactography. If the breast MRI is normal, short-term monitoring with the patient's agreement is reasonable.

Additionally, a low-fat diet, not a high-fat diet, is recommended for cyclic mastalgia. Some studies showed an improvement in lipid profiles, but due to lack of randomization and the limit on dietary fat to 15%, which many people find difficult to follow, low fat diets are not recommended.

An alternative to galactography and breast MRI is mammary ductoscopy, which has been used to improve the diagnosis of intraductal lesions in patients with pathologic nipple discharge since the late 1980s. However, mammary ductoscopy has significant limitations. This modality is highly performer dependent, with a substantial learning curve to acquire the necessary skill set.

21. A 46-year-old woman presents with a solitary 1.5-cm breast mass. A needle biopsy is consistent with the diagnosis of sclerosing adenosis. Which of the following statements about this finding is true? A. It is most common in women younger than 45 years old. B. This finding increases the risk of developing breast cancer. C. Is not associated with hormone replacement therapy. D. It is associated with an increased BMI. E. E This lesion can be followed with ultrasound.

20. A 34-year-old woman presents with significant bilateral breast pain cyclical with her menstrual periods. She denies any palpable masses on self-breast exam and denies nipple discharge. On physical exam, no masses are palpated. Digital mammography is negative. The next step in managing this patient is A. B. C. D. E.

ANSWER:

Sclerosing adenosis is a proliferative lesion commonly found in benign breast biopsies (figure 21.1)

tamoxifen. evening primrose oil. increased fat diet. whole breast ultrasound. breast MRI.

ANSWER:

B

A

Breast pain is a common issue managed by general surgeons. Breast pain can be unilateral or bilateral and can be secondary to chest wall pain or to true breast pain. The initial assessment should be a thorough history and physical exam. If chest wall pain is identified, treatment should be based on the etiology. Unilateral breast pain is more concerning, and imaging should be considered. Bilateral breast pain that is cyclical with menses is known as cyclical mastalgia. In this patient, history, physical J exam, and mammography are negative. Further imaging will not change the diagnosis of cyclical mastalgia, thus breast ultrasound or MRI would not be indicated.

Figure 21.1.A

126

drainage of the abscess is a traditional and effective treatment, this approach is also associated with the need for serial dressing changes, potential for poor cosmetic result, parenchymal loss, formation of milk fistula, and prolonged time to resolution. With the exception of very large abscesses, where a percutaneous drain may be needed, most breast abscesses can be treated with ultrasound-guided needle aspiration combined with antibiotics. Factors favoring successful treatment using needle aspiration include low pus volume (<5 mL) and early treatment (<6 days). If required, repeated aspiration can be performed until the abscess has resolved. Potential complications of aspiration include hematoma or seroma formation as well as failure to clear the abscess. Should aspiration fail, incision and drainage of the abscess can be performed. Keeping the breast empty decreases pain due to engorgement and avoids obstruction of the ducts, which may worsen the infection. Discontinuation of breast feeding is not required, as the infant is not adversely affected by nursing from an infected breast Closure of the skin would result in recurrence of the abscess. A breast abscess is a benign, infectious process; therefore, wide local excision is not necessary.

Figure 21.1.B x20 photomicrographs of sclerosing adenosis. Sclerosing adenosis will show proliferation of ductal architecture. Involutional changes are also present. Fibrosis around duct proliferation (arrow, 21.1 A). Ductal proliferation with microcysts (arrow, 21.1B).

The histology consists of enlarged and distorted lobules, containing duplicated and crowded acini, with prominent myoepithelium and stromal fibrosis. In a series of 13,434 women in the Mayo Benign Breast Disease Cohort, sclerosing adenosis occurred on biopsies in 27.8% of patients. It occurs most commonly in women aged 45 to 55 years (34% of the patients with sclerosing adenosis) followed by women older than 55 years (28.3% of patients). Sclerosing adenosis was more common in women who used hormonereplacement therapy (32.8%) compared with patients who have never used hormone replacement therapy (25%). The presence of sclerosing adenosis was not associated with BMI. Women with sclerosing adenosis have double the risk of developing breast cancer compared with the general population. Sclerosing adenosis is a histologic finding often found in association with atypical lobular hyperplasia, but it does not have any ultrasound features that would support following it on serial ultrasonography.

22. A 19-year-old woman who is breastfeeding presents with cellulitis of the right breast associated with fever and leukocytosis. An ultrasound reveals a 3- x 4-cm hypoechoic lesion 4 cm deep within the breast. In addition to antibiotics, what is the best treatment option? A. B. C. D. E.

Percutaneous drain placement Ultrasound-guided needle aspiration Discontinuation of breastfeeding Wide local excision Incision, drainage, and skin closure

ANSWER:

B

A breast abscess typically presents with fever, erythema, tenderness, and leukocytosis. Staphylococcus aureus is the most common causative organism. Although incision and

127

Breast Part II

intraductal papilloma. Which of the following statements is true of intraductal papilloma?

ITEMS 1-23 For each question, select the best possible response.

A. Mammography can reliably distinguish between benign and malignant lesions. B. It is the most common diagnosis in women with pathologic nipple discharge. C. Fine needle aspiration is preferred to establish the initial diagnosisD. Surgical excision is rarely indicated. E. Bilateral disease is common.

1. A 12-year-old boy is seen with gynecomastia. His breast size has been enlarging for 2 months. His parents are concerned about cancer. The best step in management at this time would be • • • • •

tamoxifen. breast ultrasound. mammogram. danazol. observation.

ANSWER:

ANSWER:

B

Papillary breast lesions are epithelial proliferations representing a histologic spectrum of disease that includes benign, atypical, and malignant lesions. Papillary breast lesions have a wide variety of clinical presentations, with intraductal papilloma being the most common diagnosis in women with pathologic nipple discharge. Imaging studies alone are not sufficient to make a definitive diagnosis. Neither mammography nor sonography can distinguish between benign and malignant papillary lesions. A percutaneous stereotactic biopsy or ultrasound-guided coreneedle biopsy is used to make an initial diagnosis; these have largely replaced fine needle aspiration.

E

Gynecomastia is very common in men. It is typically seen at 3 time points: infancy, puberty, and advanced age. During infancy, gynecomastia is caused by circulating maternal estrogen. Pubertal gynecomastia is seen in up to 70% of boys, and male gynecomastia is histologically present in approximately 50% of men who have autopsies performed. The pathogenesis of gynecomastia is primarily due to an imbalance of androgenic and estrogenic influence on breast tissue. Estrogens stimulate production of breast tissue, whereas androgens inhibit production of breast tissue. Therefore, gynecomastia occurs when there is a deficiency of androgens or an increased level of estrogens.

Core biopsy carries the potential risk of sampling error or specimen fragmentation, with an upstage rate to atypia of potentially more than 20% using this technique. Atypia can also be found in the immediately surrounding tissue. In the presence of histologic atypia, the risk of malignancy can be high. In the presence of atypia, surgical excision is required for these lesions. For this reason, surgical excision for papillary breast lesions is a reasonable treatment option.

During puberty in boys, the serum androgen-to-estrogen ratio is altered due to an estrogen surge, allowing gynecomastia to develop. As puberty progresses, this ratio corrects, and gynecomastia resolves as androgen secretion dominates. In a boy of this age, the process is self-limited, and there is no indication for breast imaging with either ultrasound or mammogram. Male breast cancer typically does not present at this age and presents as a hard, fixed mass located peripheral to the nipple.

3. A 35-y ear-old woman with a known BRCA-1 mutation and a normal breast exam does not wish to consider prophylactic mastectomy. The optimal screening regimen is A. B. C. D. E.

The management of this patient is observation. In some patients, gynecomastia can be the result of alcohol, marijuana, or other drugs and is reversible with discontinuation of these agents. Hormonal manipulation with tamoxifen (an antiestrogen) in a population of grown men resulted in resolution in 78% of patients, but it was associated with a higher recurrence rate. Danazol (an antiandrogen) used in this same age group had only 40% resolution. Given the age of the patient and the self-limited nature of gynecomastia associated with puberty, neither medication is appropriate.

annual mammogram. biannual mammogram. annual mammogram and breast MRI. annual mammogram and whole breast ultrasound. biannual mammogram and breast MRI.

ANSWER:

C

Although annual mammography is a successful screening tool, up to 15% of breast cancers are missed using mammogram alone, often in women with dense breast tissue. Mammographic sensitivity is diminished in younger women with dense breast tissue. Younger women at high risk with dense breast tissue should undergo alternate methods of screening.

2. A 50-year-old woman is seen by her primary care physician for spontaneous nipple discharge of the right breast associated with a small palpable mass adjacent to the nipple. She reports no fevers. Workup confirms a diagnosis of

129

MRI detects small foci of cancer in dense breasts that would not be visible on mammography. MRI has a higher sensitivity than mammography for detecting breast cancer, and the sensitivity is not altered by breast density. However, due to its high cost and limited availability, MRI screening is used judiciously.

A recent review and meta-analysis showed that CPM does not offer a survival benefit for patients with unilateral breast cancer and no heritable mutations. Among this group of patients, the risk of metachronous contralateral breast cancer is already low and likely does not warrant the morbidity of CPM.

In 2007, the American Cancer Society developed guidelines for annual MRI screening. Patients who are BRCA mutation carriers, untested first-degree relatives of mutation carriers, and women with a lifetime risk of breast cancer development of 20% or greater determined by family history should have annual MRI screening in addition to annual mammogram.

According to the Breast Cancer Prevention Trial, tamoxifen reduced breast cancer incidence among BRCA2 patients by 62% but did not reduce breast cancer incidence in patients with BRC Al mutations. Among male patients with the BRCA1 mutation, the lifetime risk of breast cancer is 1.2%. There is no documented evidence that prophylactic mastectomy reduces this risk.

Recommendations for high-risk screening with MRI are supported by clinical research. The American Cancer Society supports the use of annual MRI in high-risk women as an adjunct to annual mammography. Screening mammography more frequently than annually does not enhance the early diagnostic capability of the screening regimen.

Sentinel lymph node biopsy (SLNB) is not recommended universally for patients undergoing CPM. There is some evidence that patients who have an extremely high risk for invasive cancer (e.g., patients with a history of inflammatory carcinoma) may benefit from SLNB at the time of CPM. Some authors suggest that patients should be stratified for their risk by evaluation of the breast with MRI before CPM.

Annual ultrasound is not an effective supplemental screening tool for breast cancer because of the high incidence of falsepositive results.

Multiple studies indicate that nipple-sparing mastectomy is an acceptable choice for patients with BRCA mutations undergoing prophylactic mastectomy, with no diminishment in the oncologic effectiveness of the procedure.

4. Which of the following statements about prophylactic mastectomy is true? A. In women with unilateral breast cancer and no heritable mutations, contralateral prophylactic mastectomy offers no survival advantage. B. In women with BRCA-1 mutation, tamoxifen offers similar survival advantage to bilateral prophylactic mastectomy. C. Bilateral prophylactic mastectomy is recommended for men with known BRCA-1 mutation. D. Sentinel lymph node biopsy is standard practice in patients undergoing prophylactic mastectomy. E. Nipple-sparing mastectomy is considered inappropriate for women with BRCA-1 mutation undergoing prophylactic mastectomy.

ANSWER:

5. A 62-year-old healthy, postmenopausal woman has new calcifications seen in her left breast on screening mammogram. A stereotactic biopsy is performed. Pathology reveals calcifications involved with lobular carcinoma in situ (LCIS). Her breast and axilla exam are unremarkable. What is an appropriate management option? A. Lumpectomy and radiation therapy B. Lumpectomy, sentinel node biopsy, and radiation therapy C. Left mastectomy D. Lumpectomy followed by raloxifene E. E Raloxifene

A

ANSWER:

Among women with a family history of breast cancer or a genetic mutation in BRCA1 or BRCA2, incidence of metachronous contralateral breast cancer can be as high as 47%. Contralateral prophylactic mastectomy (CPM) has historically been prescribed for these higher risk patients to decrease mortality. However, women diagnosed with unilateral breast cancer are increasingly requesting and receiving CPM at a rate that has more than doubled over the past 15 years. Despite this movement toward more CPM, without a BRCA mutation, less than 10% of women would be expected to develop metachronous contralateral breast cancer.

D

Lobular carcinoma in situ (LCIS) and atypical lobular hyperplasia (ALH) are part of a breast disease spectrum referred to as lobular hyperplasia. A diagnosis of LCIS is associated with an 8-to-l0-fold increased risk of breast cancer. The risk of breast cancer is equal in both breasts and is not increased in the breast found to have LCIS. LCIS is not a direct precursor of invasive lobular carcinoma. Therefore, patients with a diagnosis of LCIS have an equal risk of developing ductal and lobular carcinomas. At a minimum, breast cancer screening in patients known to have LCIS should include annual mammogram. In some high-risk patients, breast MRI can be added. Typically, MRI is

130

performed annually and scheduled at 6-month intervals with the mammogram.

of ADH cases are upgraded to DCIS after excisional biopsy. Therefore, excision of ADH is critical to obtaining the correct final diagnosis.

This patient is best treated by a lumpectomy followed by raloxifene. Lumpectomy is recommended to evaluate all adjacent breast tissue not sampled with core biopsy. Omitting lumpectomy is recommended only when LCIS is found incidentally on core biopsy (i.e., when other pathology explains the imaging finding and LCIS is incidental). Because this patient's calcifications were associated with the LCIS, this patient does not meet criteria to omit lumpectomy. Pharmacologic interventions, such as tamoxifen and raloxifene decrease the incidence of subsequent invasive, hormone positive breast cancers in patients with known LCIS. Typically, tamoxifen is used in premenopausal patients and raloxifene in postmenopausal patients.

Fibroadenomas are benign breast tumors that often present clinically as a mobile and palpable mass. Biopsy-proven fibroadenomas do not require excision unless they have pathologic atypia, are rapidly growing, or are symptomatic. Sclerosing adenosis is a benign lesion histologically characterized by proliferation of the lobular acini. A diagnosis of sclerosing adenosis is not associated with an increased risk of cancer or pathologic upgrade. Therefore, excision or increased screening is not warranted. Fat necrosis is a benign lesion that often appears after trauma or breast surgery. Fat necrosis does not infer an increased risk of breast cancer and does not need to be surgically excised.

Because LCIS is a risk marker and not true breast carcinoma, these patients are not recommended to undergo radiation after surgery. In addition, LCIS does not metastasize to axillary nodes; therefore, sentinel lymph node biopsy is not warranted. Because LCIS increases the risk of bilateral breast cancer, unilateral mastectomy will not sufficiently decrease risk of subsequent breast cancer. Some patients will opt for bilateral mastectomy as a risk-reducing measure.

Pseudoangiomatous stromal hyperplasia (PASH) is a benign proliferative lesion in the breast. PASH does not increase breast cancer risk and does not need to be surgically excised.

7. A 30-year-old healthy woman presents with a rapidly growing right breast mass. Ultrasound-guided core needle biopsy is consistent with fibroadenoma. The patient is taken to the operating room for excision of the mass. The surgical specimen is pictured (figure 7.1). The pathology report reveals a malignant phyllodes tumor that abuts all surgical margins. The best next step in the management of this patient is

6. In which of the following scenarios is surgical excision recommended if it is found on core needle biopsy? A. B. C. D. E.

Fibroadenoma Sclerosing adenosis Fat necrosis Pseudoangiomatous stromal hyperplasia Atypical ductal hyperplasia

ANSWER:

A. re-excision of the lumpectomy bed to achieve 1-cm margins. B. re-excision of the lumpectomy bed to achieve 1-cm margins with sentinel lymph node biopsy. C. right breast radiation therapy. D. doxorubicin and cisplatin-based chemotherapy. E. doxorubicin and cisplatin-based chemotherapy followed by radiation therapy.

E

Patients with abnormal findings on breast imaging are often recommended to undergo core needle biopsy for diagnostic purposes. Core needle biopsy allows for many asymptomatic patients to avoid surgical excision when the findings are benign. After core needle biopsy, further management depends on pathology. Patients with benign core biopsy should have further evaluation if findings are symptomatic or the finding portends an increased risk for breast cancer. Management of patients with benign pathology after core biopsy require a multidisciplinary approach. Of the lesions listed, only atypical ductal hyperplasia (ADH) is routinely recommended for excisional biopsy. ADH is localized intraductal proliferation with many of the same features as low grade ductal carcinoma in situ (DCIS). On imaging, ADH most often presents as visible microcalcifications. The differentiation between ADH and DCIS is sometimes difficult to make from a core biopsy sample. Differentiating between ADH and DCIS involves both quantitative and qualitative histologic assessments. Approximately 10 to 20%

Figure 7.1.

131

ANSWER:

performed and reveals invasive ductal carcinoma that is ER positive, PR positive, and HER2 negative. Bone scan and CT scan of the abdomen and pelvis are negative for metastatic disease. In addition to neoadjuvant chemotherapy, the patient should be treated with

A

Phyllodes tumors of the breast are a rare form of fibroepithelial breast tumors that represent approximately 1% of all breast malignancies. Phyllodes tumors typically present as enlarging mobile breast masses. The work-up should include mammogram and ultrasound. Phyllodes tumors have a typical benign appearance on imaging and are often misdiagnosed as fibroadenomas. Core biopsy before

A. tamoxifen and trastuzumab. B. tamoxifen and radiation therapy. C. central lumpectomy with sentinel lymph node biopsy and radiation therapy. D. mastectomy with sentinel lymph node biopsy and radiation therapy. E. modified radical mastectomy and radiation therapy.

surgical excision is not requisite and often does not provide the definitive diagnosis, because phyllodes tumors have a high degree of tumor heterogeneity. The histologic diagnosis is most accurate when the entire mass is evaluated after surgical excision. The treatment of phyllodes tumors should start with surgical excision (figure 7.1). If the diagnosis is not known at the time of initial surgery, the mass can initially be shelled out of the breast with no margin of benign breast tissue. If final pathologic evaluation does reveal phyllodes tumor, a 1-cm margin of benign tissue is generally recommended. This can be accomplished with re-excision of seroma cavity if the original mass was shelled out. When diagnosis is known preoperatively, either breast-conserving surgery or mastectomy are reasonable options for initial management. Breast conserving surgery is achieved if the tumor-to-breast size ratio is adequate to allow for a 1-cm margin. Margins of less than 1 cm result in higher local recurrence rates. Fiveyear survival is similar in patients initially treated with mastectomy or breast-conserving surgery.

ANSWER:

E

This patient has inflammatory breast cancer, an aggressive form of breast cancer that portends a worse survival compared with locally advanced, noninflammatory breast cancer. Inflammatory breast cancer is a clinical diagnosis. The disease often presents with diffuse erythema with no discrete palpable breast mass. Prompt diagnosis is paramount; patients should be staged for metastatic disease, and treatment should be initiated expeditiously. Approximately 30% of patients will have distant metastatic disease at the time of diagnosis. The treatment of inflammatory breast cancer consists of trimodality therapy including targeted neoadjuvant chemotherapy, surgery, and radiation therapy. Overall and median disease-free survival are both improved when patients are treated with trimodality therapy with a curative intent. This requires a coordinated multidisciplinary approach. Chemotherapy is typically anthracycline-based. Patients with HER2+ disease are also started on HER2-targeted therapies when chemotherapy is initiated. Neoadjuvant chemotherapy is followed by modified radical mastectomy. Due to the widespread nature and skin involvement in inflammatory breast cancer, breast-conserving surgery is not indicated. In addition, axillary node dissection is the standard of care. Currently, sentinel node biopsy is not an adequate treatment option in this patient population.

Due to the very low incidence of axillary nodal metastases, axillary staging with sentinel lymph node biopsy and axillary node dissection are not indicated in patients with phyllodes tumors. Phyllodes tumors behave similarly to other sarcomas. The most common areas to find early metastases are the lungs and bone. Histologic grade of the tumor is the most important factor associated with local ant distant disease recurrence. Neoadjuvant therapy does not change surgical management or affect survival and recurrence rates. Indications for adjuvant therapy are not clearly defined, and neither chemotherapy nor radiation is routinely recommended. When adjuvant chemotherapy is used, the regimen most often consists of doxorubicin and dacarbazine.

After modified radical mastectomy, chest wall and axillary radiation follows. After the completion of radiation therapy, patients with hormone receptor positive disease will also be treated with tamoxifen or an aromatase inhibitor.

Radiation therapy may be indicated in tumors larger than 2 cm after breast-conserving surgery or in very large tumors (>10 cm) after mastectomy. Radiation may improve local disease control but does not improve survival.

Treating with chemotherapy, tamoxifen, and trastuzumab is not adequate. Treatment with radiation but not surgery is also not warranted.

8. A 72-year-old woman reports 3 weeks of increasing erythema of her right nipple areola complex and surrounding skin. Physical exam is significant for an indurated central right breast with peau d'orange skin changes and palpable right axillary adenopathy. She is unable to tolerate a mammogram. Punch biopsy of the right breast skin is

9. A woman with a palpable 1-cm breast mass and no palpable axillary lymph nodes undergoes lumpectomy and sentinel lymph node biopsy (SLNB). Her SLNB reveals 1 metastatic node without extranodal extension. She agrees to

132

have chemotherapy and whole breast radiation. Comparing axillary lymph node dissection with no axillary lymph node dissection, which of the following statements is true? A. B. C. D. E.

ANSWER:

Occult breast cancer presenting within an axillary lymph node is an uncommon presentation of breast cancer. Preoperative breast MRI is the standard of care for patients with breast cancer axillary metastases and no documented breast primary on mammogram. Whole breast ultrasound is proposed, but no studies show this method is superior to breast MRI. If an MRI correlate or lesion is identified, histologic confirmation with clip placement should be performed.

Overall survival is equivalent. Disease-free survival is decreased. Overall survival is increased. Disease-free survival is increased. Overall survival is decreased.

ANSWER:

A

Breast conservation (axillary lymphadenectomy alone) with whole breast radiation therapy is an option for patients diagnosed with occult breast cancer who have a negative preoperative breast MRI. Patients with occult breast cancer are more likely to receive radiation therapy and axillary lymphadenectomy if they receive care at an academic center. These patients tend to have significantly better overall survival compared with patients treated with modified radical mastectomy.

After nearly a century of surgical science and tradition suggesting the need for axillary lymph node dissection (ALND) for the treatment of breast cancer, sentinel lymph node dissection (SLND) emerged as a more targeted approach for diagnosis of regional metastatic disease. SLND limits the long-term complications of axillary lymphadenectomy such as lymphedema, numbness, and range of motion limitations, among others. Although SLND was known to be equivalent to ALND for the diagnosis of regional metastatic disease and despite short-term follow-up suggesting the noninferiority of SLND for survival, the question remained as to whether the same would be true with a longer follow-up.

A PET scan may be reasonable if multiple axillary lymph nodes are suspected to be involved with metastatic breast cancer. Neoadjuvant chemotherapy for triple negative breast cancer that is biopsy proven lymph node positive is reasonable, but a breast MRI should be performed before initiating chemotherapy.

A 10-year follow-up of the American College of Surgeons Oncology Group Z0011 trial demonstrated continued noninferiority of SLND compared with ALND for overall and disease-free survival among women with T1 or T2 invasive primary breast cancer, no palpable axillary lymphadenopathy, and 1 or 2 sentinel lymph nodes containing metastatic disease. These data suggest that overall survival is equivalent No difference, increase or decrease, was observed in overall or disease-free survival.

11. You are seeing a 66-year-old woman who has a clinical diagnosis of stage IA (T1cN0M0) infiltrating ductal carcinoma. She had an ultrasound-guided partial mastectomy with sentinel lymph node dissection. Adjuvant radiation therapy is planned. Which of the following final reports regarding the margins would support re-excision? A. Invasive ductal carcinoma on ink at the deep margin that included pectoralis fascia B. Invasive lobular carcinoma within 1 mm of the margin but not extending to the inked margin C. Ductal carcinoma in situ (DCIS) less than 1 mm from the inked margin D. Classic lobular carcinoma in situ (LCIS) at the inked margin E. Pleomorphic LCIS 2.5 mm from inked margin

10. A 64-year-old woman presents with axillary swelling 3 months after upper arm surgery. The patient attributes the swelling to the anesthesia, which was a regional block in the axilla. She became concerned when the swelling recently increased. Her preoperative chest x-ray was normal. Routine screening mammography 3 months ago was normal. An axillary ultrasound identifies a 2.3-cm hypoechoic oval mass. A whole breast ultrasound is negative bilaterally. A biopsy of the mass reveals a lymph node replaced with poorly differentiated adenocarcinoma consistent with a breast primary. Biomarker analysis is as follows:

ANSWER:

C

Pathology reports should list margins in quantitative measurements rather than categories of close, negative, and positive. The width of ink on tumor should also be quantified rather than using categorical terms such as broad, focal, and diffuse. Breast conservation specimens should be oriented at the time of surgery to allow for proper margin evaluation. This is important in determining the necessity for re-excision. A margin at the fibroglandular border (skin, pectoralis fascia)

Estrogen receptor: 0% staining Progesterone receptor: 0% staining Her2Neu FISH: not amplified. What is the next step in her management? A. B. C. D. E.

D

PET scan Modified radical mastectomy Axillary dissection Breast MRI Chemotherapy

133

does not need re-excision where a margin associated with residual remaining breast tissue should be re-excised.

ducts, leading to infection of secretions with aerobic and anaerobic organisms that are sensitive to first-line antibiotics. Ultrasound is the recommended imaging modality for diagnosis, and collections can be drained using needle aspiration under ultrasound guidance. Recurrent and chronic abscesses are most commonly associated with smoking, and cessation is an important intervention to prevent recurrence.

Negative margin is no ink on tumor for invasive breast cancer and is endorsed by the Society of Surgical Oncology and the American Society for Radiation Oncology. LCIS is now reported as classic LCIS or pleomorphic LCIS. Classic LCIS is considered a high-risk lesion treated as benign that does not require surgical removal with a negative margin. The margin status of classic LCIS is not required to be reported. The significance of pleomorphic LCIS at the margin is uncertain. Some guidelines advocate for a 2 mm margin for pleomorphic LCIS, similar to ductal carcinoma in situ (DCIS).

13. Which of the following statements regarding sentinel lymph node (SLNB) and axillary lymph node dissection (ALND) for breast cancer is true? A. ALND is indicated when a sentinel lymph node is negative by hematoxylin and eosin staining. B. ALND is indicated when ultrasound-guided fine needle aspiration of an axillary node identifies carcinoma cells. C. ALND is not performed with a mastectomy in the setting of a positive sentinel lymph node. D. SLNB is ideal for staging women with inflammatory breast cancer. E. ALND is suitable for women with ductal carcinoma in situ when breast-conserving surgery is planned.

Consensus guidelines are endorsed by the Society of Surgical Oncology, the American Society for Radiation Oncology, and the American Society of Clinical Oncology regarding margins for breast conserving surgery in a patient with DCIS having whole breast radiation. A 0.2-cm margin is standard and is associated with lower rates of ipsilateral breast tumor recurrence. A positive margin is defined as DCIS on inked margin and is associated with an increased risk of ipsilateral breast tumor recurrence. It is not necessary to regularly perform surgery to obtain greater than 0.2-cm margin for DCIS treated with whole breast radiation. In patients who are treated for DCIS with excision alone, the optimal margin is not known but is suggested to be at least 0.2 cm. Breast conservation specimens should be oriented at the time of surgery to allow for proper margin evaluation. Re-excision of close (<2 mm) or positive margins should be selectively used.

ANSWER:

In patients with localized breast cancer, the staging and management of the axillary lymph node basin has evolved. If clinical examination identifies suspicious lymph nodes, these are easily examined by ultrasound and confirmed as malignant by fine needle aspiration biopsy. Axillary lymph node dissection (ALND) is recommended in these patients. ALND is not recommended for women with in situ cancer in which breast conservation is planned. In this case, sentinel lymph node biopsy (SLNB) is the preferred evaluation for occult lymph node metastases. SLNB is an intraoperative method used when preclinical staging fails to identify metastatic lymph nodes in the axillary nodal region.

12. Which of the following statements regarding periareolar abscesses is true? A. This is a disease of postmenopausal women. B. Pathology starts with papillary cancer obstructing the duct. C. Abscesses commonly grow multidrug resistant organisms. D. MRI is recommended for diagnosis. E. Cigarette smoking cessation is important for preventing recurrence.

ANSWER:

B

Lymphoscintigraphy maps the lymphatic drainage from the breast to the axillary nodes and allows the surgeon to identify and remove nodes for pathologic examination. Based on randomized studies, patients with a negative SLNB by pathologic evaluation should not receive ALND. Additionally, women with sentinel lymph node metastases who will undergo mastectomy should be offered ALND. Although not supported by randomized data, patients with inflammatory breast cancer are not candidates for SLNB because of the potential for dermal lymphatic spread of malignant cells.

E

A nonlactational abscess is an infection in the breast occurring in women who are not pregnant or breastfeeding. Nonlactational breast abscesses can be peripheral (away from the nipple) or periareolar (near or deep to the nipple areolar complex). Periareolar abscesses occur in women within a broad age range (18-50 years) and are not exclusive to postmenopausal women. Regardless of age, periareolar abscesses occur more often in smokers versus nonsmokers. The cause is believed to be related to "squamous metaplasia" or a flattening out of the cells within the terminal ducts beneath the nipple. This is often manifested clinically as a central cleft in the nipple. The cellular debris obstructs the

14. What is the most likely diagnosis in a 30-year-old menstruating woman with pain in the right breast, induration of the nipple, and bloody nipple discharge? A. Bacterial mastitis B. Ductal carcinoma in situ C. Intraductal papilloma

134

D. Nipple adenoma E. Fibrocystic disease

ANSWER:

uncommon, breast cancer can anc does occur in this age group, and observation of a palpable mass would not be sufficient treatment. Further diagnostic imaging is indicated. Mammography in a young patient with dense breasts may be difficult to interpret and consequently, nondiagnostic. A wellcircumscribed mass may be solid or cystic, and ultrasonography will easily distinguish between them. Echogenic images with a posterior enhancement is diagnostic of a simple cyst (figure 15.1). Immediate aspiration is not recommended because this is a benign process, but repeat physical examination is recommended after the next menstrual cycle. Excisional biopsy before ultrasound examination is clearly not indicated. If masses persist after multiple menstrual cycles, repeat evaluation is warranted.

C

For most patients, nipple discharge is associated with benign breast pathology, with malignancy reported in 9 to 21% of patients in large case series. For patients without a palpable mass, the incidence is much less (3-6%). Univariate analysis showed that in a patient with nipple discharge, malignancy is most often associated with older age and the presence of a mass. The association of malignancy with the color of nipple discharge remains controversial and has not been definitively established as a marker for malignancy. Consequently, ductal carcinoma in this young patient is not likely. With pain and induration, bacterial infection is more common in a nursing mother, but a bloody nipple discharge is not typical of an infectious process. Fibrocystic disease presents with breast pain associated with the menstrual cycle and clear, milky, green, brown, or black nipple discharge. By contrast, intraductal papillomas most often present with bloody or pink nipple discharge and account for 35 to 48% of patients presenting with nipple discharge.

Figure 15.1. Simple cyst with and without compression Both aspiration and excisional biopsy have a role in the management of cystic masses of the breast in specific circumstances. Neither is the best next step in the management of the patient presented here. Complicated cysts will demonstrate similar characteristics with the additional finding of echogenic homogenous contents on ultrasound (Breast Imaging Reporting and Data System [BI-RADS®] 3, figure 15.2).

A full breast examination is indicated in this patient. Cytologic examination of the discharge may not be diagnostic because sensitivity is reported to range from 45 to 82% for identification of malignancy in cases of nipple discharge (of any type), with a false positive rate of 0.9 to 2.6%. Mammography may not be helpful in a young patient with dense breast tissue, and further imaging with ultrasonography or MRI is recommended. Any associated masses should be biopsied with a core needle or excised for diagnosis. If a mass is not discovered, surgical excison of the duct is the standard of care. Gentle dilation of the identifed duct with lacrimal probes will allow insertion of a flexible intravenous catheter for methylene blue injection. The identified duct may then be excised under local anesthesia for definitive histologic diagnosis.

15. A 38-year-old woman presents with a painless, wellcircumscribed, mobile 2-cm mass in the lower outer quadrant of the left breast discovered on routine breast self-exam. What is the next best step in her management? A. B. C. D. E.

Observation Mammography Ultrasound Aspiration of fluid for cytology E Excisional biopsy

Figure 15.2. A. At 1 year screening US, 4 mm oval mass(arrow) was category 3 BI-RADS lesion. B. 8 mm in 2 year C. 12 mm in 3 years.

ANSWER:

C

Fine needle aspiration and cytologic examination may help in the diagnosis of these patients. Microscopic examination can determine the fluid to be benign, noncontributory, atypical, or

Abreast cyst in a premenopausal woman is the most common palpable breast abnormality in this age group. Although more

135

malignant. Benign fluid in a complicated cyst should prompt a repeat ultrasound examination in 6 months. All other cytologic reports should be evaluated further with percutaneous biopsy. For patients with a BRCA mutation or those with a high-risk family history, excisional biopsy is recommended rather than aspiration because medullary carcinoma can be difficult to distinguish by ultrasonography.

B. C. D. E.

tamoxifen therapy. radiation of the breast. mastectomy. aromatase inhibitor therapy.

ANSWER:

E

Elderly patients with breast cancer pose challenges to longterm management. For physically fit patients, standard recommendations for breast cancer treatment are indicated. Patients and care givers should be involved in preoperative planning to determine the level of fitness, and geriatric physician referrals should be made as indicated. Side effects of all contemplated treatments should be clearly discussed with the patient. Palliative strategies with few side effects may be considered for patients not expected to survive for 2 to 3 years due to other comorbidities. Based on actuarial tables, an otherwise healthy 87-year-old woman can be expected to live for another 6 years.

Cysts with a solid component on ultrasound (BI-RADS 4; figure 15.3) are called complex cysts and have an incidence of malignancy of 3 to 36%. Percutaneous biopsy is recommended for these patients as the best method for diagnosis.

Mastectomy after lumpectomy did not increase overall survival in elderly patients (>80 years old) in the European Organization for Research and Treatment of Cancer trial. Radiation therapy in these patients is also controversial, with multiple trials demonstrating a decreased risk of ipsilateral recurrence and breast cancer mortality but no improvement in overall survival. The 9343 trial from the Cancer and Leukemia Group B enrolled women over the age of 70 with ER positive, Tl, node-negative breast cancers, randomizing them to either hormone therapy or hormone therapy with radiation to the breast. Although locoregional recurrence was higher in patients not receiving radiation therapy, there was no difference in survival between the 2 groups.

Figure 15.3.A

Chemotherapy may be required if the tumor is ER, PR, and HER 2 negatives (triple negative breast cancer). Hormonal therapy is key to the management of ER positive breast cancer, with tamoxifen being the accepted standard of care. Tamoxifen provides an overall 5-year risk reduction in death of 31 %, regardless of age or concomitant chemotherapy. Complicating its use are menopausal-like symptoms and an absolute increased risk of death of 0.2% at 10 years due to thromboembolic events and uterine cancer. An aromatase inhibitor should be considered in this patient because she is postmenopausal. The ATAC (Arimidex, Tamoxifen Alone or in Combination) trial compared anastrozole to tamoxifen and found an absolute improvement in disease-free survival of 3% compared with tamoxifen. Subgroup analysis of the Breast International Group 1-98 Collaborative study reported significantly improved disease-free survival and overall survival with treatment with aromatase inhibitors compared with tamoxifen. Side effects are different for patients treated with aromatase inhibitors, with an increased risk of osteoporotic fractures (3% at 5 years), myalgia, and arthralgia in addition to an adverse effect on lipid profiles.

Figure 15.3.B Irregularly shaped cyst (A) with a solid component (B)

16. An 87-year-old woman presents with a 10-mm mass in the lower outer quadrant of the right breast. She has no palpable lymph nodes. She has no family history of cancer. Mammography confirms the mass, with localized microcalcifications. Needle biopsy confirms ductal carcinoma. Lumpectomy reveals a 12-mm ER-positive HER2-negative tumor with clear margins. Her sentinel lymph node biopsy is negative. The best next step in her management is A. chemotherapy.

136

17. A 48-year-old woman has a core needle biopsy of a 1-cm lesion noted on a mammogram. The final pathology shows atypical ductal hyperplasia. What is the most appropriate next step in management? A. B. C. D. E.

18. In a woman with macromastia, which of the following is a contraindication to breast reduction surgery? A. B. C. D. E.

Repeat core needle biopsy Excisional biopsy Breast MRI Raloxifene Tamoxifen

A history of back and neck pain BMI of 35 Age younger than 18 years Absence of a screening mammogram Active smoking

ANSWER: ANSWER:

E

Physicians who refer patients with macromastia for plastic surgery should understand the indications and risk factors for breast reduction surgery. Although the traditional recommendation was that obese patients should lose weight before breast reduction surgery, this recommendation is substantiated only for patients with a BMI greater than 40. Data also indicate that patients with macromastia who undergo breast reduction surgery, including adolescents, have enhanced body image perception and participation in social activities. Therefore, age younger than 18 should not contraindicate breast reduction surgery. The adage that surgery does not improve pain is refuted by evidence that patients report decreased pain after a reduction mammoplasty.

B

Atypical hyperplasia is a high-risk lesion found in approximately 10% of breast biopsies. It can be classified as either atypical ductal hyperplasia (ADH) or atypical lobular hyperplasia (ALH). Both lesions are considered to be highrisk lesions with a relative risk of nearly 4 for a future breast cancer. This correlates to a risk of breast cancer that approaches 30% over 25 years. With the increasing use of core needle biopsy, physicians may encounter a pathology consistent with ADH. With this finding, a complete excisional biopsy is mandatory because approximately 30% of these women will have a final diagnosis c breast cancer. Unfortunately, investigators have not been able to identify any "low risk" ADH, so all lesions with ADH should be fully excised to ensure there is no evidence of a more advanced lesion.

An analysis of more than 13,000 patients in the National Surgical Quality Improvement Program® data repository identified smoking to be an independent risk factor for wound complications, including superficial and deep infections and wound disruptions. Recommendations for mammography should follow American Cancer Society, American College of Obstetricians and Gynecologists, and US Preventive Services Task Force guidelines, but mammography is not a requirement before breast reduction surgery.

There is good evidence that chemoprevention will lower the risk of breast cancer for these high-risk women. Various agents are used, including antiestrogens (tamoxifen), selective estrogen-receptor modulators (raloxifene), and aromatase inhibitors. Despite evidence of significant risk reduction, many women with ADH either do not choose or are not offered chemoprevention. Because this decision is complicated, a thorough discussion of the risks and benefits is crucial for women at risk. In addition to risk reduction, some experts believe breast MRI should be used for screening these high-risk women. However, current guidelines state only that breast MRI should be "considered." Nevertheless, these women need to clearly understand the significant risk for breast cancer over time once ADH is identified.

19. A 35-year-old woman with well-controlled type II diabetes and a 20 pack-year smoking history presents with a painful mass in her right breast. Examination reveals a warm, firm, and tender nonfluctuant mass in the right outer quadrant of the right breast. Ultrasound evaluation reveals a 2.5-cm abscess. The next step in management should be A. B. C. D. E.

In this patient, a core needle biopsy of a suspicious lesion noted on a mammogram is consistent with ADH A repeat biopsy may not give a definitive diagnosis. An excisional biopsy should be done to rule out the possibility of a missed breast cancer. If on the final pathology report there is no cancer requiring treatment, then chemoprevention with either raloxifene or tamoxifen would be a risk-reduction strategy. Given the risk of nearly 30% of a future breast cancer, annual breast MRI could be used for future surveillance.

open incision and drainage. ultrasound-guided aspiration. intravenous antibiotics. oral antibiotics. percutaneous drain placement.

ANSWER:

B

Breast infections in nonlactating women (nonpuerperal abscess) are associated with increased age, diabetes mellitus

137

type II, and a smoking history. The mainstay of treatment of a nonpuerperal abscess was surgical incision and drainage with a high rate of cure. However, this treatment often requires a procedure in the operating room and is associated with long healing times when the wounds are left open. Percutaneous drainage has emerged as a safe and reliable alternative to incision and drainage. No high-quality trials demonstrate superiority of image-guided percutaneous techniques over surgical drainage; however, numerous small trials prove its efficacy. Abscesses greater than 5 cm in size, chronic or long durations of infection, and multiloculated collections are risk factors for failure of percutaneous techniques. The initial therapy for most abscesses should be ultrasound-guided needle aspiration. Some patients may require repeat aspiration. For larger collections, typically greater than 3 cm, an indwelling catheter may be left in place to aid in drainage. Most patients (80-90%) treated with percutaneous techniques and concurrent antibiotic therapy do not require surgical incision and drainage. Neither oral nor intravenous antibiotic therapy alone is sufficient treatment for an abscess of this size.

all increase the risk of developing breast cancer. Family history can also increase risk of breast cancer development, but a breast cancer history outside of first-degree family members does not affect this risk. Families with the BRCA-1 and BRCA-2 gene mutation have increased risk for developing breast cancer along with ovarian cancer, melanoma, and prostate cancer for BRCA-1 and male breast cancer, pancreatic cancer, prostate cancer for BRCA-2 Because of her great uncle's breast cancer, the mutation of BRCA-2 is suspected but this factor alone does not increase her risk of breast cancer development because her great uncle is not a first-degree relative. She would need to be tested for this mutation. Receptor status has prognostic value for those who develop breast cancer. The presence of estrogen and progesterone receptors in a tumor indicate that the malignancy is better differentiated. As such, this represents a favorable prognostic sign. In this patient, her race is associated with a higher chance of having an ER negative status. 21. A 55-year-old woman presents with a red, swollen right breast. She reports progressive edema, heaviness, and erythema of her entire breast over the past 3 months. She has been on antibiotics for 10 days with no improvement. On exam, her contralateral breast is normal. The right breast is mildly tender and erythematous, with pitting of the skin around areola. There is no discrete mass, but there is generalized fullness in the breast. The next step in management should be A. mammography. B. core needle biopsy in the center of the breast. C. dermal biopsy. D. breast MRI. E. PET-CT scan.

20. A 55-year-old black postmenopausal woman presents with tenderness in her left breast and a possible mass. Her BMI is 42 She denies a history of breast disease. Her menarche was at age 13 and she has a G3P2 obstetrics history, with her first child at age 26. Her maternal aunt was diagnosed with breast cancer at age 54, and she had a paternal great uncle with breast cancer. On physical exam, she has large pendulous breasts, but you cannot palpate a mass. Which of the following statements is true regarding her risk for breast cancer? A. Obesity increases her risk for advanced-stage disease. B. Weight loss would reduce her future cancer risk. C. Her weight decreases the risk of having a large tumor. D. Her race increases her risk for ER-positive tumors. E. E Her race increases her risk for developing breast cancer.

ANSWER:

ANSWER:

A

Inflammatory breast cancer is a rare presentation of a locally advanced breast cancer (<3% of breast cancer diagnoses). Inflammatory breast cancer is characterized by diffuse tumor infiltration of the breast tissue and dermal lymphatics. This produces the characteristic clinical presentation of inflammatory breast cancer-a swollen, edematous breast with peau d'orange skin changes and diffuse erythema. This presentation can be misdiagnosed as an infection, but it will not respond to systemic antibiotics.

A

Many risk factors for breast cancer affect either the risk for developing breast cancer or the prognosis. Black race does not increase the risk of developing breast cancer, but it does increase the risk of having breast cancer diagnosed at a later stage due to inequities of care access. This may be because screening mammography is not readily accessible for all. Obesity increases the risk of developing breast cancer over the lifetime, and it increases the risk of having advanced disease at the time of diagnosis, including nodal disease, larger tumors, and distant disease. Mortality is also directly affected negatively by obesity. This risk increases as the BMI increases. There is no proof that weight loss specifically reduces breast cancer risk.

The diagnosis of inflammatory breast cancer is based largely on the clinical presentation. Patients commonly report rapid onset of symptoms, often less than 6 months in duration, with the described physical exam features. The workup of patients suspected to have inflammatory breast cancer starts with breast imaging. Bilateral mammography and breast ultrasound of the affected breast and ipsilateral axilla are performed to identify any discrete mass or suspicious lymph nodes. Full-thickness skin biopsy or open surgical biopsy are not needed to confirm the diagnosis. Standard image-guided biopsy should be performed on any identified mass.

Prolonged exposure to endogenous estrogen may increase the lifetime risk of developing breast cancer. Therefore, in addition to obesity, increasing estrogen exposure, early menarche, late menopause, and a nulliparous obstetric history

138

Breast MRI is not the initial imaging study of choice but can be used to identify a target to biopsy if no mass is found on mammography or ultrasound. Because 20 to 35% of patients with inflammatory breast cancer will have distant metastases at presentation, a full staging workup should be performed after the diagnosis is made. CT scans of the brain, chest, abdomen, and pelvis are obtained to evaluate for distant metastases.

23. A 68-year-old woman with a normal breast exam had pleomorphic calcifications spanning 3 cm on screening mammography. Core needle biopsy was consistent with ductal carcinoma in situ, and she underwent a lumpectomy. The margins were 25 mm. What factor is associated with an increased risk of local recurrence? A. Microcalcifications B. Final margins 2.5 mm C. Size of the primary lesion D. Presence of comedo necrosis E. Age older than 40 years

22. A 42-year-old woman underwent her first screening digital mammogram. She has a Gail Model 5-year breast cancer risk of 1.7%. Her mammogram report notes extremely dense breasts and a Breast Imaging Reporting and Data System (BI-RADS®) category 1 What is the next step in her care? A. Repeat digital mammogram in 1 year B. Whole breast screening ultrasound C. Digital breast tomosynthesis D. Breast MRI E. Contrast-enhanced spectral mammography

ANSWER:

ANSWER:

D

Ductal carcinoma in situ (DCIS) is increasing in incidence with the advent of screening mammography and now represents up to 50% of screen-detected cases of breast cancer. DCIS is a heterogeneous disease with 25 to 50% of patients progressing to invasive disease. The primary goal of initial treatment is to prevent recurrence of invasive disease, while also minimizing treatment-related morbidity and optimizing cosmesis. Therefore, identification of patients who are at increased risk of local recurrence is critically important.

C

A woman with an average risk of breast cancer should begin digital mammography at age 40. Further recommendations are based on the results of that study. Breast cancer screening continues to evolve as surgeons gain a better understanding of risk. This patient has a Gail Model risk of 1.7%. This places her into the high-risk category. For patients with a high risk of breast cancer, initial screening is done with digital mammography. If this test is equivocal or if factors exist that limit the accuracy of digital mammography, alternative testing is needed. Whole breast screening ultrasound is currently used as a secondary screening tool. Although it shows promise, no randomized clinical trials exist evaluating screening ultrasound. Breast MRI is a useful adjunct to screening mammography in patients who are at high risk for cancer, including those ages 10 to 30 with a prior history of chest wall radiation, a known genetic predisposition for breast cancer, or a strong family history.

In the National Surgical Adjuvant Breast and Bowel Project (NSABP) B-24 trial, women who underwent breastconserving surgery with radiotherapy for DCIS were assigned to tamoxifen or placebo. In both groups, local recurrence rates were higher for margin-positive than for margin-negative patients. Recommendations for the optimal margin threshold is 2 mm; the risk of ipsilateral recurrence is lower at this margin threshold than for margins of 1 mm or no ink on tumor. Rates of ipsilateral recurrence were also higher for tumors with a component of comedo necrosis (regardless of adjuvant therapies) than for tumors without. The presence of necrosis, however, is a weaker predictor of recurrence than cellular architecture and nuclear grade. Estimates of risk for recurrence should incorporate these factors as well. Other factors associated with increased risk of local recurrence include initial detection by palpation and tumor size (>10 mm). The presence of microcalcifications is not associated with increased risk of recurrence.

Digital breast tomosynthesis (DBT), or 3-dimensional mammography, is an advanced form of breast imaging. In DBT, multiple 2-dimensional images of the breast are obtained. These images are then combined to form a 3dimensional image of the breast. DBT is currently recommended when digital mammography shows dense breasts and the patient is at high risk for breast cancer. DBT identifies more neoplastic lesions in this population than screening mammography. Similar to whole breast ultrasound, contrast-enhanced spectral mammography is a newer technique in which intravenous contrast is injected and digital mammography is then performed. The technology was approved in 2011. Unfortunately, no randomized trials support its use at this time.

Molecular testing to predict the risk of local recurrence for DCIS continues to evolve. These include use of a 12-gene subset of the 21-gene recurrence score used to predict recurrence after breast-conserving surgery for invasive disease, as well as studies using individualized expression of ER, PR, Ki67 antigen, pl6, p53, ERBB2, and COX-2 These tests may lead to better identification of patients with DCIS at low, intermediate, or high risk of developing subsequent invasive cancer and individualizing adjuvant therapies accordingly.

139

Endocrine

2. The most common type of functional pancreatic neuroendocrine tumor in a patient with multiple endocrine neoplasia type 1 3. Symptoms of confusion, tremors, weakness ameliorated with administration of glucose

ITEMS 1-25 For each question, select the best possible response.

4. Migratory necrolytic erythema and deep venous thrombosis

1. A 62-year-old active woman presents 10 years after radioiodine treatment for Graves disease. Her medications include a thiazide for hypertension and 0.1 pg of levothyroxine. She now presents with weakness, weight gain, delayed ankle reflexes, and scaly skin. Her serum thyroidstimulating hormone level is high. Which intervention should be considered? A. B. C. D. E.

5. Achlorhydria, hypokalemia, and dehydration

ANSWERS:

Neuroendocrine tumors (NETs) are a diverse group of neoplasms that may arise from a host of different organs within the thoracic and abdominal cavities. Pancreatic neuroendocrine tumors (pNETs—previously termed "islet cell tumors") are relatively uncommon, yet they are found in 0.8 to 10% of patients in autopsy studies, suggesting that most patients are asymptomatic. Of those who do develop symptoms, some 90% are caused by compressive effects of the tumor (nonfunctional), whereas only 10% are characterized as functional due to the effects of hormone hypersecretion.

Stopping thiazide Increasing levothyroxine dose Checking a free T4 level Decreasing levothyroxine dose Monitoring serum thyroglobulin

ANSWER:

B

Iodine-131 (131I) is generally regarded as the treatment of choice for patients with Graves disease who do not improve after a course of antithyroid medications. A consequence of 131 I treatment is development of hypothyroidism. The incidence of hypothyroidism increases with higher doses of radioactive iodine. Although lower doses of 131I are currently used, this strategy is not entirely effective, with up to 15% of patients developing hypothyroidism. Levothyroxine is used after radioactive iodine treatment to prevent hypothyroidism, but hypothyroidism can still occur while taking this drug. Hypothyroidism typically presents with weakness, weight gain, delayed ankle reflexes, and scaly skin. Hypothyroidism is usually confirmed by checking a thyroid-stimulating hormone level. A low level while taking levothyroxine, especially with signs and symptoms of hypothyroidism, is adequate for a diagnosis. Further laboratory testing is usually unnecessary. Treatment is to increase levothyroxine.

Fifty percent of patients with multiple endocrine neoplasia (MEN) 1 will develop Zollinger-Ellison syndrome (refractory peptic ulcer disease and secretory diarrhea) from the hypersecretion of gastrin as the result of a gastrinoma the most common type of pNET in a patient with this syndrome. Insulinoma is the most common functional pNET outside of the multiple endocrine neoplasia syndromes. The hypersecretion of insulin from a functional insulinoma manifests clinically as the Whipple triad—symptoms of hypoglycemia (confusion, tremors, weakness, palpitations, sweating)., with documented hypoglycemia during the episode, and resolution of symptoms with glucose administration. Glucagonomas and VIPomas each account for less than 10% of functional pNETs. The classic finding in a patient with hypersecretion of glucagon is migratory necrolytic erythema, a dermatitis characterized by erythematous lesions that become necrotic and result in a pigmented scar. Other symptoms attributed to a glucagonoma are glucose intolerance, diarrhea, and deep venous thrombosis. The hypersecretion of vasoactive intestinal peptide from a VIPoma causes achlorhydria, hypokalemia, and dehydration from large-volume watery diarrhea.

Some drugs may cause or mimic hypothyroidism, but thiazide is not one of these. Drugs that contain iodine can be problematic. Amiodarone is another drug that can cause hypothyroidism. Serum thyroglobulin is used to monitor thyroid cancer patients after treatment for persistent or recurrent disease. Items 2-5 Each lettered response may be selected once, more than once, or not at all. A. B. C. D. E.

E, A, D, B

The least common pNET is the somatostatinoma. Patients with functional tumors rarely exhibit all the symptoms associated with this tumor, which are diabetes mellitus, gallbladder disease, steatorrhea, anemia, and weight loss.

Insulinoma VIPoma (vasoactive intestinal polypeptide tumor) Somatostatinoma Glucagonoma Gastrinoma

141

6. A 31-year-old asymptomatic woman was referred for management of thyroid nodules. A CT scan after an automobile crash incidentally demonstrated a nodular thyroid. Subsequent ultrasound showed a 1-cm cyst on the left and a 7-mm indeterminate lesion on the right. The most appropriate next step in management is

Gynecomastia has an incidence of 1.8% in the male population and is often associated with obesity. This condition is caused by changes in the ratio of free androgen to estrogen. These changes include enhanced peripheral aromatization of androgen to estrogen (obesity), increases in available estrogen, alterations in sex hormone-bindingglobulin, or decreases in free androgen. The cause is pharmacologic in 10 to 25% of cases. Hypogonadism predisposes individuals to gynecomastia, accounting for a higher prevalence in the elderly male population.

A. ultrasound-guided aspiration of both thyroid nodules. B. ultrasound-guided aspiration of the right-sided thyroid nodule. C. total thyroidectomy. D. right thyroid lobectomy. E. follow-up ultrasound in 6 months.

ANSWER:

Gynecomastia is a potentially treatment-limiting side effect in men being treated with hormones for prostate cancer, many of whom may already have pre-existing gynecomastia. Treatment for prostate cancer is multimodal and may include prostatectomy. Treatment for advanced-stage prostate cancer may include bilateral orchiectomy but more often includes drugs that decrease testosterone production. These therapies may be less distressing or morbid alternatives to bilateral orchiectomy. All androgen deprivation therapies are associated with some degree of gynecomastia, but the incidence seems greater with antiandrogen hormonal therapy. Gynecomastia is often a reason for premature withdrawal from therapy. Prompt recognition, evaluation, and management of the condition are essential. Mitigation of gynecomastia may include withdrawal of a causative drug, prophylactic irradiation of the breasts, or use of drugs that inhibit aromatase. Aromatase or estrogen synthetase is a cytochrome P-450 enzyme that has a key role in the production of estrogens in men, particularly in subcutaneous fat. Overexpression of aromatase is accompanied by gynecomastia. Medical interventions are associated with good outcomes during the active proliferative phase of gynecomastia. Only short-term, drug-induced gynecomastia responds well to removal of the contributing agent. Long-standing gynecomastia is associated with irreversible changes, including fibrosis, and is relatively unresponsive to anything but surgery. Surgical modalities have evolved to less-invasive techniques and include bilateral subcutaneous mastectomies or ultrasound-assisted liposuction.

E

Incidental thyroid lesions (incidentalomas) are often discovered on CT scan or MRI performed for other reasons. When found, thyroid ultrasound is recommended, and the size and appearance on thyroid ultrasound are used to guide next steps. Generally, size criteria are stratified as smaller than 1 cm, 1 to 1.5 cm, and larger than 1.5 cm. Nodules less than 1 cm in size can be followed with repeat ultrasound in 6 months. Thyroid nodules 1 to 1.5 cm with intermediate- or high-risk features should undergo fine needle aspiration. Those with low-risk features can be followed. Nodules at least 1.5 cm should undergo fine needle aspiration regardless of suspicious nature on ultrasound. Suspicious ultrasound features include solid versus cystic nodules, hypoechogenicity, microcalcifications, intranodular vascularity, irregular or infiltrative margins, and shape taller than wide. Partially cystic nodules without any of these features have a risk of malignancy of less than 3%, and purely cystic nodules have a risk of malignancy of less than 1 %. Inpatients suspected to be at increased risk—such as those with a family history of thyroid cancer, history of radiation exposure, isolated uptake on PET scan, suspicious cervical lymphadenopathy, or hoarseness—fine needle aspiration biopsy can be considered for smaller nodules. There is no indication to proceed to surgery without a preoperative diagnosis by fine needle aspiration.

8. A 45-year-old woman has a calcium level of 11 g/dL (8.510.2 g/dL) on routine screening laboratory examination for a life insurance policy. Her parathyroid hormone level is at the upper end of normal, and her serum calcium levels are persistently 1 g/dL above normal. Which of the following tests should be ordered as part of her initial biochemical evaluation? A. 25-hydroxyvitamin D B. Beta human chorionic gonadotropin C. Cortisol D. Liver function E. Thyroid stimulating hormone

7. A 74-year old man presents with symptomatic gynecomastia associated with treatment for prostate cancer. The condition has been present for a little more than 12 months. What treatment will have the best chance of success in treating his gynecomastia? A. B. C. D. E.

Bilateral orchiectomy Prophylactic radiotherapy Nonsteroidal antiandrogen therapy Estrogen therapy Subcutaneous mastectomy

ANSWER:

E 142

ANSWER:

D. Serum aldosterone E. MRI

A

Figure 9.1. Algorithm for the evaluation of incidentalomas. ACTH * adrenocorticotropic hormone; AVS = adrenal vein sampling.

Asymptomatic primary hyperparathyroidism (pHPTH) is a commonly encountered condition across the world. The American Association of Endocrine Surgeons and an international group of experts developed guidelines for evaluating a patient with suspected pHPTH. Part of the history should determine whether the pHPTH is a component of an inherited syndrome. Biochemical evaluation should include total calciun (appropriately corrected for serum albumin), parathyroid hormone levels, creatinine, and 25hydroxyvitamin D levels. Patients should also undergo 24hour urine testing for creatinine and calcium. A recent addition to the guidelines was the recommendation that patients with asymptomatic pHPTH undergo abdominal imaging to detect nephrocaldnosis or nephrolithiasis. Measurement of beta human chorionic gonadotropin, cortisol, liver function tests, and thyroid stimulating hormone are not currently recommended as part of the diagnostic evaluation of pHPTH

ANSWER:

A

9. A 55-year-old man underwent a CT scan of the abdomen and pelvis after a motor vehicle crash. No intra-abdominal injuries were noted; however, a 2-cm homogenous right adrenal mass was found. He is normotensive. Which of the following is the next step in evaluating the clinical significance of this adrenal lesion?

Laboratory examination should include 24-hour urine collection for fractionated catecholamines and metanephrines or plasma-free metanephrines to rule out pheochromocytoma.

Adrenal incidentalomas are adrenal tumors identified during radiographic testing performed for unrelated causes. They are among the most common solid organ tumors. Adrenal incidentalomas are found in 4% of CT scans, and incidence increases with age. Although most adrenal incidentalomas are benign, the differential is broad and must consider aggressive tumors such as adrenal cortical cancer. Adrenal incidentalomas may be hormonally functional. Failure to recognize and treat patients with functional adrenal incidentalomas leads to an increased risk of long-term morbidity and mortality. Although only 3.1% of adrenal incidentalomas are pheochromocytomas, nearly half of all pheochromocytomas are initially discovered as adrenal incidentalomas. Regardless of the size and radiographic appearance, all adrenal tumors should be screened to exclude biochemical hyperfunction.

All patients should also be screened with a thorough history and physical (figure 9.1). Further biochemical workup may include a number of tests. An overnight 1-mg dexamethasone suppression test should be used to evaluate for Cushing syndrome.

A. Plasma-free metanephrines B. No further evaluation C. CT-guided biopsy

143

If the patient is hypertensive, an aldosterone:plasma renin activity ratio should be obtained. MRI may demonstrate findings consistent with pheochromocytoma; however, routine imaging with MRI is neither necessary nor recommended in the initial evaluation of adrenal incidentalomas. CT-guided biopsy is rarely necessary and should be reserved for patients with a previous history of cancer to rule out metastatic disease if it will change overall management.

papillary thyroid cancer tumors in the absence of prior head and neck radiation, familial thyroid cancer, or clinically detectable cervical nodal metastasis. However, total thyroidectomy may be chosen if adjuvant radioactive iodine treatment is planned, if high-risk features are present, or for patient preference. Prophylactic lymph node dissection is not indicated for noninvasive, clinically node negative papillary thyroid cancer tumors smaller than 4.0 cm.

11. A 50-year-old woman presents after her annual physical examination. She had a serum calcium of 11.5 mg/dL (8.510.2 mg/dL). Her parathyroid hormone level was 70 pg/mL (10-65 pg/mL). Her urinary excretion of calcium was low (<100 mg/day). She states she has no complaints. Ultrasound of the neck is normal. What is the most appropriate next step?

10. A 35-year-old healthy woman presents for evaluation of a small lump that she recently noticed in her anterior neck. Her medical, surgical, and family history are unremarkable. Physical examination is normal, with the exception of a single small, firm nodule in the mid-neck to the right of the trachea. Ultrasound of the neck demonstrates a 3.0-cm solid nodule in the right lobe of the thyroid and no evidence of abnormal lymph nodes. Ultrasound-guided fine needle aspiration of the nodule demonstrates papillary thyroid carcinoma. What is the most appropriate management option?

A. B. C. D. E.

A. B. C. D.

Right thyroid lobectomy Total thyroidectomy Total thyroidectomy with central node dissection D. Total thyroidectomy, central node dissection, with right level II through V node dissection E. E. Total thyroidectomy, central node dissection, with bilateral level II through V node dissection

ANSWER:

Sestamibi scan Chest CT scan Serum Vitamin D level Bilateral neck exploration Bone density test

ANSWER:

C

Hyperparathyroidism is subdivided into 3 types: primary, secondary, and tert Primary hyperparathyroidism (pHPTH) is caused by the overproduction of parathyroid hormone from parathyroid hormone secreting cells known as the chief cells. Secondary hyperparathyroidism is most commonly caused by the increased production of parathyroid hormone in response to an external stimulus, such as kidney disease. The exact mechanism is not completely understood. Tertiary hyperparathyroidism is a sequela to the treatment of secondary hyperparathyroidism, for example, the persistence of hyperparathyroidism after kidney transplant.

A

Papillary thyroid cancer comprises 90% of all thyroid cancers. It is indolent in nature, with an excellent prognosis if appropriate therapy is undertaken. Surgical resection is the mainstay of treatment. Until 2016, the American Thyroid Association guidelines recommended total or near-total thyroidectomy for papillary thyroid cancer tumors larger than 1 cm. Thyroid lobectomy was recommended only for tumors less than 1 cm in size. Studies published before the 2016 guidelines revision called into question whether tumor size should be an absolute indicator for total thyroidectomy, which carries with it a greater risk of hypoparathyroidism and recurrent laryngeal nerve injury than does unilateral thyroid lobectomy. More recent data failed to demonstrate a survival advantage of total thyroidectomy for papillary thyroid cancer tumors 1.0 to 4.0 cm in size. This finding held true for stratified analyses of 1.0 to 2.0 cm tumors and 2.1 to 4.0 cm tumors Therefore, tumor size is only one factor to be considered in determining the extent of resection for papillary thyroid cancer tumors.

Approximately 80% of the primary hyperparathyroid cases are cause by one gland. Roughly 10 to 15% are caused by 4gland overproduction. Approximately 4% are caused by 2 or 3 glands hypersecreting the hormone. The diagnosis of pHPTH is based entirely on laboratory analyses. For the patient who presents with elevated calcium and parathyroid hormone levels, 2 other diagnoses must be excluded before giving the patient a diagnosis of primary hyperparathyroidism. First, a Vitamin D level should be checked. Low Vitamin D levels will cause a compensatory increase in parathyroid hormone. If correction of the Vitamin D level takes place and the parathyroid hormone level is still elevated, the diagnosis of pHPTH should be considered. The second diagnosis to consider is familial hypocalciuric hypercalcemia. As its name suggests, it is caused by low urinary excretion of calcium along with elevated parathyroid hormone and calcium levels.

In keeping with more recent data, the 2016 American Thyroid Association guidelines suggested that thyroid lobectomy is adequate treatment for up to 4-cm unifocal, intrathyroid

This patient has elevated parathyroid hormone levels and calcium levels. Of the choices listed, the next best test would

144

be a Vitamin D level. A sestamibi scan would be used to identify a hyperactive parathyroid gland, which would not be done before a Vitamin D level is checked. Neither a CT scan of the chest nor a bone density scan are necessary based on her presentation. Bilateral neck exploration is appropriate for some patients with pHPTH but not in this patient without a clear diagnosis yet.

Acute and symptomatic hypercalcemia require prompt therapy to restore the glomerular filtration rate and normalize the serum calcium levels. These patients are usually extremely volume depleted. Additionally, these patients often present with poor oral intake and may have experienced nausea and vomiting that worsened the volume depletion. Thus, initial therapy should begin with intravenous volume expansion with 0.9% saline at 200 to 500 mL per hour. A loop diuretic may be added to saline therapy to increase urine calcium excretion by a forced saline diuresis. While commonly used previously, loop diuretics have fallen out of favor as a treatment approach for severe hypercalcemia. These drugs may worsen electrolyte derangements and volume depletion when administered at high doses. Loop diuretics should be used cautiously and only if the patient is manifesting signs and symptoms of fluid overload.

12. An 80-year-old man with a history of small cell lung cancer with brain and bone metastases is in the emergency department. His wife states that this past week he has experienced nausea, vomiting, abdominal pain, excessive thirst, and weakness. Today he became very confused. He is extremely cachectic, stuporous, and moaning in bed while holding his abdomen. His heart rate is 110 beats per minute, and his blood pressure is 95/60 mm Hg. Severe hypercalcemia is confirmed. After initiation of intravenous saline and bisphosphonates, what is the next step in treatment for this patient? A. B. C. D. E.

The pathophysiology of hypercalcemia is related to bony resorption from activated osteoclasts. Intravenous bisphosphonates are the initial treatment of choice because they inhibit osteoclastic bone resorption, and they should be given as soon as possible. However, bisphosphonates do not reach maxi effectiveness until days 2 through 4, highlighting the importance of proper hydration as the first step in treatment. Another important component in the treatment of these patients is initiating a rapid reduction in serum calcium levels. Thus, calcitonin should be added to the immediate treatment regimen. Calcitonin is particularly effective due to the rapidity of its effect Renal calcium absorption is blocked and beneficial effects are seen as soon as 6 hours after administration. Glucocorticoids are beneficial only in lymphomas secreting calcitriol. Calcium levels are usually slow to decrease with corticosteroid treatment alone. Cinacalcet corrects hypercalcemia by decreasing the production of PTH and is useful for lowering calcium levels in patients with parathyroid cancer and ectopic production of PTH. In patients with malignant hypercalcemia, PTH levels are typically suppressed, rendering cinacalcet ineffective.

Calcitonin Cinacalcet Glucocorticoids Mithramycin Furosemide

ANSWER:

A

As many as 30% of patients with metastatic cancer may suffer from hypercalcemia. The symptoms of hypercalcemia mimic the symptoms of the underlying malignancy and its treatment. Patients experience nausea, vomiting, constipation, abdominal pain, anorexia, weight loss, bone pain, polyuria, fatigue, and weakness. In patients with severe degrees of hypercalcemia, neurologic symptoms, such as confusion and even coma, may be present. Because these symptoms are nonspecific and often develop gradually, they can delay diagnosis and increase morbidity and mortality. The most important laboratory test to guide initial treatment is the serum calcium level. In this population, malnutrition may be present, and correcting the total serum calcium for a low albumin level or measuring an ionized serum calcium level to assess the true degree of hypercalcemia is important. Patients with low albumin levels and normal serum calcium levels actually may be hypercalcemia

Mithramycin, a cytotoxic antibiotic that inhibits bone resorption by binding to osteoclasts and inhibiting RNA synthesis, was used to treat hypercalcemia. However, it is less effective and convenient compared with bisphosphonates and is no longer used to treat acute malignant hypercalcemia. In renal compromised patients, saline diuresis may lead to hazardous volume overload. The most appropriate treatment in this subset of hypercalcemic patients may be hemodialysis.

Further laboratory evaluation may be performed to identify the particular etiology of the patient's hypercalcemia but is less useful in terms of immediate therapeutic intervention. A more extensive laboratory evaluation should include intact parathyroid hormone (PTH), which will confirm whether the patient has a PTH-independent process. Intact PTH is typically suppressed in hypercalcemia of malignancy but would be elevated in other conditions such as primary hyperparathyroidism or parathyroid cancer.

13. You have performed a fine needle aspiration of a 2.2-cm thyroid nodule in die right lobe on a 35-year-old euthyroid patient. The cytopathologist reports the finding as Bethesda Category III, follicular lesion of undetermined significance. Neck ultrasound shows no lymphopathy. What is the best next step in management?

145

A. B. C. D. E.

Repeat fine needle aspiration Surveillance ultrasound Total thyroidectomy Right lobectomy Subtotal thyroidectomy

ANSWER:

In the patient mentioned here, a Bethesda Category III diagnosis (follicular lesion of undetermined significance) has been made. Due to the low chance of malignancy (5-15%), a repeat fine needle aspiration is indicated. Surveillance ultrasound does not provide repeat pathology that is needed with a follicular lesion of undetermined significance. Surgical options—including total thyroidectomy, right lobectomy, and subtotal thyroidectomy—would be considered once a more definitive diagnosis was made.

A

Thyroid nodules are common and can be seen in one of 7 patients. Less than 5% of these nodules are cancerous. The best first test to assess a thyroid nodule is an ultrasonography evaluation. The following ultrasonographic descriptors are concerning for cancer: large nodules, hypoechoic, microcalcifications, irregular borders, and dimensions that are taller than wide. Cancer is usually not suspected for lesions that are cystic, spongiform, and with sharp borders. Fine needle aspiration is recommended for nodules between 1 and 2 cm based on the suspicion of malignancy. Before fine needle aspiration, assurance should be made that the patient is euthyroid. The Bethesda System for Thyroid Cytopathology was developed to aide in the management of the fine needle aspiration result (table 13.1).

14. A 55-year-old man was referred to you for evaluation of a 5.2-cm, incidentally discovered mass in his right adrenal gland. He has a history of hypertension and diabetes. What is the next step in his management? A. B. C. D. E.

Measurement of plasma-free metanephrines Right adrenalectomy Follow-up CT scan in 6 months CT-guided adrenal biopsy Measurement of urine vanillylmandelic acid

ANSWER:

1. Nondiagnostic or Unsatisfactory • Cyst fluid only • Virtually acellular specimen • Other (obscuring blood, clotting artifact, etc.) II. Benign • Consistent with a benign follicular nodule (includes adenomatoid nodule, colloid nodule, etc.) • Consistent with lymphocytic (Hashimoto) thyroiditis in the proper clinical context • Consistent with granulomatous (subacute) thyroiditis • Other III. Atypia of Undetermined Significance or Follicular Lesion of Undetermined Significance

A

Adrenal incidentalomas can be found in approximately 1% of patients undergoing axial imaging for unrelated reasons. These incidentalomas can be divided into 3 groups: functional benign, nonfunctional benign, and malignant. Approximately 60% of adrenal incidentalomas are nonfunctional cortical adenomas, but all should be evaluated for functional status and malignant potential. Adrenocortical carcinomas are typically large (6-8 cm) and have characteristic features on imaging such as irregular borders and heterogenous attenuation. As the size of the tumor increases, so does the likelihood of carcinoma, even without typical imaging features. Clinical guidelines suggest that any lesion larger than 4 cm undergo resection because of increased cancer risk.

IV. Follicular Neoplasm or Suspicious for a Follicular Neoplasm • Specify if Hurthle cell (oncocytic) type V. Suspicious for Malignancy • Suspicious for papillary carcinoma • Suspicious for medullary carcinoma • Suspicious for metastatic carcinoma • Suspicious for lymphoma • Other VI. Malignant • Papillary thyroid carcinoma • Poorly differentiated carcinoma • Medullary thyroid carcinoma • Undifferentiated (anaplastic) carcinoma • Squamous cell carcinoma • Carcinoma with mixed features (specify) • Metastatic carcinoma • Non-Hodgkin lymphoma • Other Table 13.1. The Bethesda System for Reporting Thyroid Cy topathology; recommended diagnostic categories.

Pheochromocytomas need to remain in the differential diagnosis of incidentalomas, even in the face of other indications for resection, due to the risk of hypertensive crisis in the operating room. The initial screening test for a pheochromocytoma is plasma-free metanephrines. If these are elevated, 24-hour urine collection for catecholamines and metanephrines should be performed. Urine vanillylmandelic acid is no longer used for screening because it has a low specificity. All asymptomatic patients with adrenal incidentaloma should also undergo screening for cortisol excess using a dexamethasone suppression test. Adrenal biopsy has an extremely limited role in the evaluation of incidentalomas. Fine needle aspiration carries risks and cannot differentiate benign from malignant tumors. Given the size of this lesion, observation with 6-month follow up CT scan is not appropriate.

146

15. A 64-year-old woman presents with nephrolithiasis and primary hyperparathyroidism. Her family history is unremarkable A Tc-99 sestamibi scan demonstrates a focus of activity near the left lower pole of the thyroid gland. The next step in her management should be

reliable in experienced hands, and office based. It has the added benefit of ruling out any additional thyroid pathology in the setting of planned neck exploration.

A. B. C. D.

left neck exploration. confirmatory 24-hour urinary calcium level. neck ultrasound. observation with repeat calcium and parathormone levels in 6 months. E. contrast-enhanced dynamic CT scan of the neck and chest (4-dimensional CT).

16. In classical multiple endocrine neoplasia type 2A, most patients develop

ANSWER:

ANSWER:

A. B. C. D. E.

C

The diagnostic workup for primary hyperparathyroidism (pHPTH) is relatively simple biochemically. The serum calcium level will be elevated with a concomitant inappropriately elevated parathormone level (PTH). Confirmatory testing can include elevated levels of urinary calcium excretion, effectively ruling out familial hypocalciuric hypercalcemia. A normal Vitamin D level will exclude low levels as a cause of the elevated PTH level. Other associated biochemical findings can include depressed serum phosphate with a serum chloride-to-phosphate ratio of greater than 33. Once the biochemical diagnosis of pHPTH is made, further testing is unnecessary and the decision for surgery is made in concert with a surgeon familiar with endocrine surgery. Criteria for surgical intervention are well established in pHPT. In general, the symptomatic patient or the patient with evidence of organ dysfunction from hypercalcemia should have a parathyroidectomy.

medullary thyroid cancer. pheochromocytoma. primary hyperparathyroidism. insulinoma. pituitary adenomas.

A

Multiple endocrine neoplasias (MEN) are autosomal dominant inherited familial disorders characterizing a predisposition to tumors of endocrine organs (figure 16.1). The tumors include both benign and malignant tumors and are frequently multicentric. The tumors may arise synchronously or metachronously. Classical MEN2A includes medullary thyroid cancer, pheochromocytoma, and primary hyperparathyroidism. Medullary thyroid cancer and pheochromocytoma are also seen inMEN2B. Nearly all patients with classical MEN2A develop medullary thyroid cancer; fewer patients develop pheochromocytomas or primary hyperparathyroidism. However, pheochromocytomas should be excluded whenever a diagnosis of medullary thyroid cancer is made because pheochromocytomas should be treated first when any intervention is planned. Pancreatic neuroendocrine tumors, pituitary adenomas, and parathyroid hyperplasia are characteristic of MEN 1.

The decision for further imaging is based solely on the operative technique planned. If traditional 4-gland neck exploration is planned, no further imaging is necessary. In the hands of an experienced surgeon, additional imaging only increases costs without significant benefit. If a patient is a candidate for focused parathyroidectomy, or "minimally invasive" parathyroidectomy, imaging techniques to localize a solitary adenoma are appropriate. The sensitivity of Tc-99 sestamibi scan ranges from 65 to 85% and varies across institutions widely. More importantly, the relatively low specificity of any single imaging test to diagnosis a solitary adenoma, as opposed to multiple adenomas or hyperplastic disease, is not sufficient to guide focused surgical techniques. Most experienced surgeons use a 2-imaging test algorithm, and if concordant and confirmatory of solitary adenoma, focused exploration may be appropriate. It is inappropriate to offer focused parathyroidectomy based on a single positive image or discordant imaging results. The most commonly used imaging tests to guide exploration options, from least expensive to most expensive, are neck ultrasonography, Tc99 sestamibi scanning, and contrast-enhanced CT scan of the neck. Ultrasonographic imaging of the neck is inexpensive,

147

Metabolic expressions

Similarities

Differences

• • • •

• •

High penetrance for hormonal tumour Parathyroid adenomas in each Many good treatments available for most symptoms Includes life-threatening hormone/metabolic expression (La gastrin In MEN1 or catecholamines in MEN2) that now is well managed

Distinct variants only In MEN2 Unique organ grouping for each MEN

Malignant expressions

Similarities

Differences

• •

• •

Unique organ grouping for each MEN Good prevention or cure for an associated cancer by early surgery In MEN2 only

Figure 16.1. Endocrine tumors expressed in multiple endocrine neoplasia types 1 and 2.

148

Usually Indolent Cancer Is potentially lethal in 30% of all carriers if not cured

17. A 49-year-old woman is found to have hypercalcemia on routine bloodwork. Detailed questioning reveals no symptoms related to hypercalcemia. She has normal bone density. Laboratory evaluation reveals the following: Component Value Calcium 10.9 mg/dL Phosphorus 2.0 mg/dL Chloride 100 mEq/L Intact parathyroid hormone 177 Pg/mL 24-hour urine calcium 300 mg Creatinine 0.6 mg/dL 25(OH) Vitamin D 33 ng/mL Parathyroidectomy is indicated based on A. B. C. D. E.

Most patients younger than 50 years develop disease progression if observed, and this is a clear indication for parathyroidectomy in patients with primary hyperparathyroidism.

Normal 8.5-10.2 mg/dL 2.5-4.5 mg/dL 95-105 mEq/L 10-65 pg/mL 100-300 mg/day 0.40-1.30 mg/dL 20-50 ng/mL

Serum calcium

(>upper limit of normal) 1.0 mg/dL (0.25 mmol/L)

Skeletal

A. BMD by dual-energy x-ray absorptiometry: Tscore < -2.5 at lumbar spine, total hip, femoral neck, or distal 1/3 radiusa B. Vertebral fracture by x-ray, CT, MRI, or VFA A. Creatinine clearance <60 mL/minute B. 24-hour urine for calcium >400 mg/day (>10 mmol/day) and increased stone risk by biochemical stone risk analysisb C. Presence of nephrolithiasis or nephrocalcinosis by x-ray, ultrasound, or CT <50

Renal

the chloride/phosphorus ratio greater than 40. the degree of parathyroid hormone elevation. the degree of hypercalciuria. her age. her creatinine level.

Age, years a

ANSWER:

Consistent with the position established by the ISCD, the use of Z-zones instead of T-scores is recommended in evaluating bone mineral density (BMD) in premenopausal women and men younger than 50 years.

D

b

Most clinicians will first obtain a 24-hour urine for calcium excretion. If marked hypercalciuria is present (>400 mg/day [>10 mmol/day]), further evidence of calcium-containing stone risk should be sought by a urinary biochemical stone risk profile, available through most commercial laboratories. In the presence of abnormal findings indicating increased calcium- containing stone risk and marked hypercalciuria, a guideline for surgery is met. Table 17.1 Current Guidelines for surgery in Asymptomatic PHPT

This patient has a diagnosis of primary hyperparathyroidism. Symptomatic patients are often helped with parathyroidectomy. However, in the absence of symptoms, objective criteria for operation are needed. An international workshop and recent surgical guidelines elucidated these criteria. In the current scenario, the patient has clearly documented hyperparathyroidism based on elevated parathyroid hormone in the presence of hypercalcemia. She is asymptomatic, and objective criteria are useful to help determine whether she would benefit from surgical intervention. Table 17.1 outlines the clearly established indications for parathyroidectomy in asymptomatic patients. In this patient, the calcium is not more than 1 mg/dL above normal. Calcium should be corrected for albumin level. Phosphorus, chloride, and the phosphorus:chloride ratio do not have a role in determining which patients benefit from parathyroidectomy. Hypercalciuria (>400 mg/24 hours) is an indication for operation, and the current patient's level is below that. Kidney stones (current or history of) and nephrocalcinosis are also indications for surgical treatment. Serum creatinine level is not a determinant of which patients will benefit from parathyroidectomy.

18. A 45-year-old man undergoes abdominal CT scan after a motor vehicle collision. CT scan reveals only an incidental 3.5-cm right adrenal lesion (<10 Hounsfield units). The patient is otherwise healthy and has no other injuries related to the collision. His electrolytes are normal. Plasma-free metanephrines are normal Which of the following steps is also appropriate to evaluate his adrenal lesion? A. B. C. D. E.

The presence of renal dysfunction (glomerular filtration rate <60 mL/minute) is an indication for operation. The presence of an elevated intact parathyroid hormone level in the presence of hypercalcemia establishes the diagnosis of primary hyperparathyroidism. However, the intact parathyroid hormone level does not determine the basis for operation. In patients with hyperparathyroidism, Vitamin D levels should be assessed and replenished if low.

MRI. dexamethasone suppression test. metaiodobenzylguanidine (MIBG) scan. plasma aldosterone concentration and renin activity. observation.

ANSWER:

B

Adrenal incidentaloma is encountered in 3 to 8% of patients undergoing imaging for other reasons. Evaluation is directed at determining malignant potential and the lesion's functional activity (figure 18.1).

149

Metaiodobenzylguanidine (MIBG) scanning was formerly used for localization of pheochromocytoma but has been replaced by MRI; PET scan is used in selected cases.

1For

patients with history of extra-adrenal malignancy, see special section 5.6.4. 2Only in patients with concomitant hypertension and/or hypokalemia 3Only in patients with clinical or Imaging features suggestive of adrenocortical carcinoma.

19. A healthy 50-year-old woman underwent single parathyroidectomy for primary hyperparathyroidism 1 year ago. Preoperative imaging at that time did not localize an adenoma. She presents now with recurrent elevated calcium and elevated parathyroid hormone levels. Which of the following is the recommended next step?

Figure 18.1 Radiographic characteristics assist in determining malignant potential. Size and Hounsfield units (HU) on CT scan are significant predictors of malignant potential. Size less than 4 cm with 10 HU or fewer are unlikely to be malignant; therefore, further imaging such as MRI is not needed.

A. B. C. D. E.

Functional activity should be evaluated through careful history and physical examination. All patients with adrenal incidentaloma should undergo screening for cortisol excess using a dexamethasone suppression test. All patients should be screened for pheochromocytoma using plasma-free metanephrines or urinary fractionated metanephrines. Screening for aldosteronoma using plasma aldosterone concentration and renin activity can be reserved for patients with hypertension and hypokalemia.

Bilateral neck exploration Repeat neck ultrasound in 6 months 99m Technetium sestamibi scintigraphy Bone density scan Radioactive iodine uptake scan

ANSWER:

C

Surgery fails to cure primary hyperparathyroidism (pHPTH) in 10% or fewer patients. By definition, persistent pHPTH is failure of cure before 6 months postoperatively. Recurrent pHPTH occurs in a patient with initial normalization of

150

calcium with recurrence after 6 months. All previous imaging, operative reports, and pathology reports should be reviewed. If the patient meets the criteria for surgery, noninvasive imaging, such as 99mTechnetium sestamibi scintigraphy, should be repeated. If a parathyroid adenoma is visualized, the patient should be I taken to surgery. If the imaging does not localize an adenoma, invasive imaging should be considered (e.g., ultrasound aspiration of potential adenomas with PTH measurement, parathyroid angiography with selective venous sampling). If no localization is found on invasive imaging, nonoperative management should be considered. Reoperative surgery is associated with increased risk of recurrent laryngeal nerve injury and permanent hypoparathyroidism.

needle aspiration biopsy only if it would affect treatment. The absence of a corresponding focal lesion on ultrasound or CT scan is highly predictive of benign pathology. Ninety-five percent of malignancies incidentally identified on PET scan are primary thyroid tumors, with papillary carcinoma as the most common malignant pathology. The remainder are metastases from other cancers. 21. A 42-year-old man undergoes lithotripsy for caldum/oxylate renal stones. Associated laboratory values include serum calcium of 10.6 mg/dL (8.5-10.2 mg/ dL), phosphorus of 3 mg/dL (2.5-4.5 mg/dL), chloride 102 mEq/L (95-105 mEq/L), creatinine of 1.2 mg/dL (0.4-1.30 mg/dL), and intact parathyroid hormone of 60 pg/mL (10-65 pg/mL). Preoperative imaging is nonlocalizing. Which of the following statements regarding his care is true?

Although a bone density scan may document osteopenia or osteoporosis in this patient, the patient already has an indication to proceed with surgery (age 50 or younger and documented recurrent pHPTH). A radioactive iodine uptake scan is useful in the management of well-differentiated thyroid cancer patients, but it is not used for imaging in pHPTH.

A. Primary hyperparathyroidism is unlikely to be a contributor to his stone disease. B. Observation is the best management. C. Vitamin D replacement will precipitate a hypercalcemic crisis. D. Dietary calcium restriction is recommended. E. Parathyroidectomy is indicated.

20. Which of the following statements regarding thyroid uptake identified on an 18F-fluorodeoxyglucose positron emission tomography (18F-FDG PET) performed as part of the workup for a distal esophageal adenocarcinoma is true?

ANSWER:

Primary hyperparathyroidism (pHPTH) presents as a spectrum of signs and symptoms. A strong family history, nephrocalcinosis/nephrolithiasis, loss of bone density with potential bone fragility, vertebral compression fracture or osteoporosis, and asymptomatic hypercalcemia should prompt investigation. Patients with neurocognitive/neuropsychiatric symptoms potentially attributable to hypercalcemia should be offered parathyroidectomy. Individuals diagnosed at age 50 years or younger should be counseled for parathyroidectomy, regardless of symptom status. Although gastrointestinal complaints (constipation) and hypertension are not considered primary indicators, patients may perceive improvement after a successful operation.

A. Diffuse thyroid uptake rules out thyroid malignancy. B. Focal thyroid uptake suggests stage IV esophageal carcinoma. C. Maximum standard uptake values (SUVmax) greater than 3 indicate a malignant lesion. D. The absence of a corresponding focal lesion on neck CT scan suggests a benign pathology. E. Incidence of malignancy is greater than 75% in thyroid lesions incidentally identified on FDG-PET scan.

ANSWER:

E

D

The diagnosis is often subtle. Although easily recognizable when hypercalcemia is coupled with a significantly elevated intact parathyroid hormone (PTHi) level, insufficient hormonal suppression in the face of hypercalcemia is also evidence of hyperparathyroidism. A chloride/phosphate ratio of more than 33 and evidence of significant hypercalciuria bolster the diagnosis. Imaging aimed at localizing an adenomatous gland(s) is significantly less accurate for multigland disease and should not be relied on to confirm or exclude the diagnosis. As a result, a nonlocalizing imaging study should not influence surgical referral. Rather, imaging serves as an adjunct or a "road map" once the decision to proceed with surgery has been made.

Incidentally identified thyroid uptake on staging PET scans obtained during the workup of various cancers is increasingly common. Although no guidelines on best management exist to inform diagnosis and management, retrospective analysis of biopsied focal hypermetabolic thyroid lesions has documented malignancy in approximately 21% of such lesions. Current data suggest that as the metabolic activity of thyroid lesions increases, so does the malignancy rate, although risk appears to be dependent on whether uptake is diffuse (malignancy prevalence up to 13%) or focal (prevalence between 10-60%). Diagnostic ultrasound should be performed for all lesions with intense uptake (thyroid lesion/thyroid background ratio >2.0), and suspicious nodules should be subjected to fine

The recommended daily calcium intake for the average adult is 1000 to 1200 mg. Patients with pHPTH and concomitant Vitamin D deficiency can and should safely undergo

151

replacement therapy (to include oral calcium) before neck exploration. Careful preoperative repletion of Vitamin D to at least low-normal levels will not precipitate a hypercalcemic crisis and is essential to avoid severe bone hunger in the perioperative period after successful parathyroidectomy. 23. A 48-year-old woman presents 8 years after laparoscopic Roux-en-Y gastric bypass for a BMI of 43 associated with, type II diabetes mellitus and obstructive sleep apnea. She has maintained a loss of 75% of her excess weight. Her diabetes and sleep apnea have resolved. She adheres to a bariatric diet and takes an adult multivitamin, calcium + Vitamin D 3, and iron supplements twice a day. She has never had biochemical evidence of malnutrition. She visits you in clinic today after sustaining an ankle fracture while exercising. She also notes fatigue, muscle weakness, and joint pain. Her physical exam is normal. What laboratory abnormality most likely explains her symptoms?

22. Which of the following statements is true regarding the evaluation of a 25-year-old patient with a 1.5-cm thyroid nodule and a fine needle aspiration demonstrating medullary cells A. Serum or urinary metanephrine levels are necessary for cancer staging. B. Testing for mutations in RET proto-oncogene is necessary. C. The minimal planned surgery should include a total thyroidectomy with bilateral neck dissection. D. Radioactive iodine imaging should be performed after medullary thyroid cancer resection. E. Detectable postoperative calcitonin levels after resection should prompt re-exploration of the neck.

ANSWER:

A. B. C. D. E.

Hypoalbuminemia Low Vitamin B12 Elevated parathyroid hormone Low iron Hypocupremia

B

Identification of medullary cells on fine needle aspiration of a thyroid nodule is considered diagnostic of medullary thyroid cancer. Medullary thyroid cancer originates from calcitonin-secreting neuroendocrine parafollicular or C cells of the thyroid. Approximately 20% of medullary thyroid cancer is associated with germline mutations of the RET gene associated with the inherited endocrine neoplastic syndromes MEN2A, MEN2B, and FMTC (familial medullary thyroid cancer).

ANSWER:

C

Malnutrition is common after bariatric surgery, even in purely restrictive surgeries like adjustable gastric banding and the sleeve gastrectomy. Malabsorptive procedures like the Rouxen-Y gastric bypass and duodenal switch have malnutrition rates up to 40%. Low levels of Vitamin B12, folate, iron, Vitamin D, calcium, and protein are the most likely deficient nutrients because they either require an intact foregut for their absorption (Vitamin B12) or they are absorbed in the stomach, duodenum, or proximal jejunum. Other deficiencies are seen in copper, the fat-soluble vitamins (A, D, E, and K), thiamine, and zinc. All of these deficiencies can occur, even if patients are taking the recommended supplementation.

The remaining 80% of medullary thyroid cancer is sporadic, although mutations in RET can be identified within tumor DNA in 40 to 50% of sporadic cases. Therefore, initial evaluation of a young patient with apparently localized medullary thyroid cancer should include a neck ultrasound examination, measurement of serum calcium, calcitonin and carcinoembryonic antigen levels, urine screening studies for pheochromocytoma, and genetic counseling for consideration of RET proto-oncogene testing.

Calcium and Vitamin D deficiencies can lead to bone resorption resulting in low bone mineral density. The estimated risk of fracture is increased 23 times after bariatric surgery. Other symptoms of hypocalcemia are fatigue, muscle weakness, and joint pain. Long-term Vitamin D deficiency leads to secondary hyperparathyroidism. Parathyroid hormone controls serum levels of calcium by causing mobilization from bone to maintain levels of serum calcium. This also results in an elevated alkaline phosphatase.

If this patient has a solitary 1.5-cm tumor and no clinical evidence of metastases to the lateral regional lymph nodes, the minimal recommended surgical procedure should include a total thyroidectomy with a central neck dissection. Serum calcitonin levels should be measured after surgical resection; persistent elevation of calcitonin levels is most commonly due to persistent disease in the neck or distant metastatic disease, and imaging is indicated. Medullary thyroid cancer cells do not concentrate iodine, so radioactive iodine ablation is not a useful strategy. External beam radiation is used to achieve local control in some cases.

Hypoproteinemia is common after bariatric surgery, and it too can lead to weakness and fatigue but does not typically result in fractures. Hypoproteinemia causes hyperphagia, edema, and muscle wasting. Vitamin B12 and folate deficiencies can also cause weakness and fatigue from megaloblastic anemia; they also cause paresthesias and peripheral neuropathies.

152

Homocysteine levels tend to elevate when Vitamin B12 and folate levels are low.

25. Which of the following statements best describes asymptomatic hyperparathyroidism?

Iron is absorbed in the duodenum and proximal jejunum, and its deficiency can result in weakness and fatigue as well as impaired thermoregulation, immune dysfunction, gastrointestinal disturbances, memory impairment, and pica. It does not lead to fractures.

A. Approximately 60% of patients referred for surgical evaluation are asymptomatic. B. In-depth neuropsychiatric testing often detects symptoms attributable to hyperparathyroidism. C. The distal radius should be included for bone mineral density evaluation. D. Parathyroidectomy reduces cardiovascular risks. E. Age is not a criterion for parathyroidectomy.

Copper is absorbed in the stomach, and its deficiency is underappreciated in postbariatric patients. Its deficiency can cause symptoms similar to Vitamin B12 deficiency like anemia, ataxia, paresthesias, and optic neuropathies. But it can also cause leukopenia. Homocysteine levels are likely to be normal, and copper deficiency does not result in fractures.

ANSWER:

Primary hyperparathyroidism (pHPT) is a common endocrine disorder that leads to hypercalcemia, nephrolithiasis, neuropsychiatric symptoms, and increased bone resorption. With the advent of routine laboratory screening, most patients (>80%) are diagnosed with pHPT without overt signs of disease and are considered "asymptomatic." In particular, the presence of neuropsychiatric symptoms is often nonspecific and difficult to attribute to pHPT. In-depth neuropsychiatric testing is therefore not indicated as part of screening.

24. A 50-year-old woman being evaluated for asymptomatic hyperparathyroidism has a skeletal bone mineral density Tscore less than -2.5 at the hip. Compared with no surgery, what is the long-term bone health benefit for parathyroidectomy in this patient? A. Her hip T-score will decrease less over time. B. She will be able to maintain her current hip T-score. C. Her hip T-score will increase over 5 to 8 years of followup. D. Her future fracture risk at the lumbar spine will not change. E. Her hip T-score will increase only if she is premenopausal.

ANSWER:

C

Bone resorption, quantified by bone scan, is more severe in areas with high cortical bone content, such as the distal third of the radius. By contrast, the hip, the femoral neck, and the lumbar spine have a higher proportion of cancellous bone and therefore experience less bone resorption. The 2013 National Institutes of Health consensus conference recommended parathyroidectomy with severe osteopenia or with radiologic evidence of vertebral fracture. Other indications for surgery in asymptomatic patients include age younger than 50, hypercalcemia, and laboratory or radiographic evidence of renal disease. Although cardiovascular disease may be associated with pHPT, parathyroidectomy does not improve cardiovascular risks.

A

Parathyroidectomy is the only curative treatment in primary hyperparathyroidism and yields prompt and sustained improvement in bone mineral density. Parathyroidectomy increases hip bone mineral density scores in both premenopausal and postmenopausal women by 2 years follow-up, but the effect is transient. By 5 to 8 years follow-up, hip bone mineral density is lower than baseline regardless of surgical therapy or observation. Women who undergo parathyroidectomy can anticipate that by 8 years after surgery, their hip bone mineral density will be lower than baseline, although with much less reduction than in the no-surgery group. Parathyroidectomy is associated with a 64% absolute risk reduction of hip fracture and a 24% risk reduction of all fractures at 10 years.

153

Head and neck

reveals a round mass of uniform low echogenicity, no acoustic enhancement, and no internal septations. Which structures most likely associated with this congenital lesion? A. Eustachian tube B. Tonsil C. Thymus D. Superior parathyroid gland E. Inferior parathyroid gland

ITEMS 1-15 For each question, select the best possible response. 1. A 20-year-old woman has a thyroid nodule on a routine physical examination. She has no concerning historical or examination findings. Ultrasound confirms a 2-cm nodule in the left hemithyroid without enlarged lymph nodes in the neck. Fine needle aspiration confirms the diagnosis of papillary thyroid cancer. During thyroidectomy, no clinically enlarged nodes are found in the central compartment. Which of the following statements is true regarding central neck dissection in this patient? A. It is contraindicated. B. Node micrometastases are found in 10% of patients. C. It will improve this patient's 5-year disease-free survival. D. It is associated with a 2-fold increase in unilateral recurrent laryngeal nerve injury compared with thyroidectomy alone. E. The superior border of the central compartment is the hyoid bone.

ANSWER:

ANSWER:

B

Branchial cleft anomalies (BCAs) represent the second most common congenital neck mass after thyroglossal duct cyst. They can present as cysts, sinuses, or fistulas. They arise from aberrant development or incomplete obliteration of the external clefts and internal pouches that develop from the 6 mesodermal arches that form on the superior-lateral aspect of a fetus during the fourth to seventh weeks of gestation. Each branchial arch, except the fifth, is associated with the development of specific structures of the head and neck. The first arch forms into the middle ear, eustachian tube, and external auditory canal. The second arch develops into the lining of the palatine tonsils. The third arch becomes the inferior parathyroid glands and the thymus. The fourth arch evolves into the superior parathyroid glands and C-cells of the thyroid. BCAs arising from the second arch are the most common, accounting for 70 to 90%. These anomalies can present anywhere along a tract arising from the tonsillar fossa, running by the glossopharyngeal and hypoglossal nerves, coursing between the internal and external carotids, and ending at the anterior border of the sternocleidomastoid. They are classified into 4 types, based on whether they lie superficial to, adjacent to, within, or posterior to the carotid sheath, respectively. Second arch BCAs most commonly manifest in the third to fifth decade as cysts, presenting as a nontender, soft mass lying deep to the sternocleidomastoid. Ultrasound imaging reveals a round mass of uniform low echogenicity, no acoustic enhancement, and no internal septations. CT scans and MRI also may be used to delineate the BCA and its course. In adults, fine needle aspiration should be performed to rule out malignancy as the origin of the mass. As with all BCAs, complete surgical excision of the cyst or tract is the therapy of choice.

E

The preoperative clinical staging for this patient with papillary thyroid carcinoma (PTC) is cTlb (tumor size >1 cm < 2 cm limited to the thyroid gland), NO, MO. Although the prognosis for this patient is favorable, the potential for locoregional recurrence serves as the rationale for some surgeons to advocate for routine prophylactic central lymph node dissection (PCLND). The incidence of lymph node micrometastases ranges from 25 to 50%, and subsequent follow-up using thyroglobulin can lead to unnecessary radioiodine therapy if PCLND is not performed. Detractors of routine PCLND in patients with clinically node-negative disease cite that routine PCLND does not alter overall 5-year survival in patients younger than 45 years. Although PCLND for patients with cNO disease remains controversial, it is certainly not contraindicated. Unfortunately, the American Thyroid Association (ATA) has determined that a randomized clinical trial to examine the potential benefits of PCLND for patients with cNO disease is not feasible because the morbidity of PCLND is quite low and the large sample size required to power a study to demonstrate significance would be prohibitive. There is no evidence to support a higher risk of recurrent laryngeal nerve injury with routine PCLND. To standardize the definition of the central neck compartment, the ATA defined the boundaries of level VI and VII nodes to be the hyoid bone superiorly, the carotid arteries laterally, the superficial layer of the deep cervical fascia anteriorly, the deep layer of the deep cervical fascia posteriorly, and the innominate artery on the right and its corresponding axial plane on the left inferiorly.

3. One week after a wide excision and sentinel node biopsy for a 1.3-mm nonulcerated melanoma of the right upper back, a 37-year-old, otherwise healthy man presents to the emergency department complaining of shoulder pain and inability to raise his right arm. On exam, there are healing scars on his right back and neck, his right shoulder shrug is weak, and he can abduct his right arm to only 90°. Sensation in the face, neck, and arm is intact. Which of the following statements is true? A. His sentinel node was most likely in the submandibular triangle (level I). B. Urgent exploration of the neck wound is needed.

2. A 25-year-old man presents with a nontender, soft mass deep to the right anterior sternocleidomastoid muscle. Fine needle aspiration is negative for malignant cells. Ultrasound

155

C. The patient has sustained injury to the greater auricular nerve. D. Electromyography can be diagnostic. E. Brain CT scan should be done.

ANSWER:

Electromyography can be diagnostic, revealing a spectrum of nerve injury ranging from near complete recordable nerve action potential at one extreme to complete denervation on the other. The patient presented can be evaluated by electromyography, and, if the nerve seems innervated, conservative management with physical therapy can be attempted. Electromyography can be monitored to look for signs of regeneration, which can take up to 18 months.

D

The patient presents with classic history and exam for a shoulder syndrome secondary to spinal accessory nerve (SAN; 11th cranial nerve) injury. SAN injury impairs motor function to the trapezius muscle, leading to weakness of shoulder abduction and drooping of the shoulder. Other symptoms and signs include shoulder pain and stiffness and winging of the scapula.

The neck should be explored within 3 to 6 months if the patient shows no improvement on clinical exam and electromyography. In a series of 156 patients who failed conservative management and eventually came to operative repair, most nerve injuries were in the upper portion of the triangle, 29 patients needed neurolysis only, and 123 required nerve repair or grafting; 88% of patients improved with operative intervention. This is a peripheral nerve injury; therefore, ruling out a brain metastasis is unnecessary. The patient has stage T2aN0, IB melanoma. Staging with any imaging modality is not indicated for this low stage. Further, the preferred imaging modality for the brain in melanoma patients is MRI, not CT scan.

The spinal root of the SAN exits the skull through the jugular foramen, pierces the sternocleidomastoid muscle, innervates it, and the runs lateral to it, where it is close to the greater auricular nerve (which is a sensory nerve and can serve as a landmark for surgeons to find the SAN). The SAN then runs diagonally through the posterior triangle (level V) with a variable course and ends by innervating the trapezius at that muscles' edge a few fingerbreadths above the clavicle. The extracranial course of the SAN is superficial as it lies between the investing and prevertebral layers of fascia. It is small and interdigitates with nodal tissue, leading to its tendency to be injured (figure 3.1).

4. A 30-year-old woman presents with a slowly enlarging, painless mass in the cheek just in front of the ear (figures 4.1 and 4.2). Ultrasound shows a mass in the parotid gland with no associated cervical adenopathy. Fine needle aspiration suggests pleomorphic adenoma. She is referred for surgical resection. The patient should be told that A. a confirmatory core needle biopsy of the mass is needed. B. a neck dissection in addition to parotidectomy is necessary. C. this mass is probably related to smoking. D. the pathology specimen has a low chance of showing malignancy. E. E operative intervention will result in a permanent facial nerve deficit.

Figure 3.1 Accordingly, such injuries are not uncommon during lymph node biopsy in the posterior triangle (level V) of the neck. Approximately 3 to 10% of all lymph node biopsies in that area result in some SAN injury, which can lead to functional deficit in 60 to 90% of cases if not treated. In a review of malpractice cases for a SAN injury, 95% involved lymph node biopsy in the posterior triangle. In that review, only 46% of cases were decided for the defendant, and the average award was in excess of $500,000. SAN injury cases are considered high-risk litigation.

Figure 4.1. 156

5. A 5-year-old girl has a 1.5-cm neck mass to the left of midline just above the laryngeal cartilage noted during her annual visit with a pediatrician (figure 5.1). It moves with swallowing and is nontender. She denies any difficulty with breathing or swallowing. An ultrasound examination reveals the mass is solid, and the thyroid is difficult to visualize. What is the most appropriate next step in management? A. B. C. D. E.

Thyroid scan Excision including the hyoid bone Fine needle aspiration of the mass Serum calcitonin level Direct laryngoscopy

Figure 4.2. ANSWER:

D

Most salivary tumors are in the parotid (the largest salivary gland) and most of these are benign. Conversely, minor salivary gland (the smallest salivary glands) tumors are uncommon, and most are malignant. As a rule of thumb, the likelihood of a salivary tumor is directly proportional to the size of the gland of origin, yet the chance that the tumor is malignant is inversely proportional to the size of the gland. Pleomorphic adenomas, the most common parotid tumors, are usually benign, are accurately diagnosed on fine needle aspiration, and have a low chance (3-10%) of harboring occult malignancy on final pathology. Malignant transformation of a pleomorphic adenoma is more likely if the pleomorphic adenoma recurs. The case presented here is classic: benign parotid tumors are usually slowly growing and painless. Fine needle aspiration is highly accurate (figure 4.3). In one study, confirmatory core needle biopsy was needed only 21% of the time, even when malignancy was suspected. Superficial parotidectomy without node dissection has a high chance of cure for pleomorphic adenomas and should be accomplished with a low risk of facial nerve injury in experienced hands. Warthin tumors (monomorphic adenomas), the second most common parotid tumors, are more often seen in older patients, men, and smokers, and they are often bilateral.

Figure 5.1 Midline neck mass.

ANSWER:

E

Neck masses in children must be evaluated. The most common midline mass in a child is a thyroglossal duct cyst, which is a remnant from the migration of the thyroid. Other midline masses include dermoids and ectopic thyroid tissue. Ultrasound is commonly used to determine whether the mass is cystic or solid. In the case of a solid midline mass, ectopic thyroid should be considered. Thyroid scan can confirm the diagnosis (figure 5.2). Ectopic thyroid tissue is most common at the base of the tongue, called lingual thyroid tissue, although it may occur anywhere along the migration route of the thyroid anlage.

Figure 4.3

157

outcomes are similar. There is no evidence that intensive care unit length of stay is favored by either technique. Delayed tracheal stenosis occurs rarely with both techniques and does not occur more frequently with either. Accidental decannulation occurs more frequently with percutaneous tracheostomy. Procedural cost is typically thought to favor the percutaneous technique, especially when comparing bedside percutaneous technique to open tracheostomy in the operating room. However, the cost difference is primarily for the operating room time. When comparing both techniques at the bedside, the open technique costs less. This finding is even more dramatic when bronchoscopy is used to support the percutaneous technique. Finally, although infrequent, most studies support fewer infectious complications with the percutaneous approach because there is less soft tissue dissection with this procedure.

Figure 5.2. A technetium-99 thyroid scan uptake in the region of the mass but no uptake in the area of the thyroid gland.

These patients are often hypothyroid; measuring a thyroid stimulating hormone will confirm a hypothyroid state. Malignancy is rare in children and obtaining a fine needle aspiration (FNA) to exclude malignancy is not necessary with a positive thyroid scan. Although FNA can be done, the risk of malignancy is low and such an invasive procedure would not be the next step. Direct laryngoscopy looking for a malignancy would also not be necessary in this child. Calcitonin would be appropriate if a family history of endocrinopathy was established, including medullary thyroid cancer, which is also not present in this case.

7. which of the following statements is true and can be used to counsel patients with thyroid disease concerning the use of intraoperative nerve monitoring (IONM)? A. Operative time is reduced. B. Operative cost is reduced. C. Injurv to the recurrent laryngeal nerve during the initial operation is decreased. D. Recurrent laryngeal nerve injury is decreased in thyroid reoperative surgery. E. Vagus nerve stimulation is a substitute for actual identification of the recurrent laryngeal nerve at surgery.

Ectopic thyroid tissue can be confused with a thyroglossal duct cyst. A Sistrunk procedure is not necessary and would remove the only thyroid tissue the patient may actually have.

ANSWER:

Management is variable given the rarity of the condition. Treating a hypothyroid state with thyroid replacement is necessary and such replacement is associated with a reduction in mass size. Surgical excision is also possible followed by thyroid hormone replacement. When the diagnosis is delayed or diagnostic findings suggest malignancy, excluding thyroid cancer with an FNA is necessary. Treatment would be guided by the FNA findings.

Careful dissection to identify the recurrent laryngeal nerve before removal of the thyroid gland remains the standard of care for nerve protection. The recurrent laryngeal nerve is most vulnerable to injury when neck dissections occur near the inferior thyroid artery, which also supplies all 4 parathyroid glands. The superior glands lie behind the recurrent laryngeal nerve, and the inferior glands lie in front of the nerve. Intraoperative nerve monitoring (IONM) for thyroid surgery as a means of improving this standard is controversial. The use of IONM adds both operative time and cost by an average of 5 to 7% to routine thyroid surgery with no observed increase in value. IONM appears to have a role in repeat operations on the thyroid gland. The prevalence of recurrent laryngeal nerve injury is higher in repeat thyroid surgery, approaching 12.5% for temporary and 3.8% for permanent events In a retrospective cohort study of patients undergoing thyroid reoperation, IONM resulted in an observed reduction in both temporary and permanent paresis of the recurrent laryngeal nerve compared with patients who had nerve visualization only.

6. Which of the following is lower for percutaneous tracheostomy compared with open tracheostomy? A. B. C. D. E.

Infection rate Delayed tracheal stenosis Accidental decannulation Procedure costs Intensive care unit length of stay

ANSWER:

B

C

Tracheostomy is frequently used to manage patients with pulmonary failure. Although the merit of open tracheostomy versus percutaneous tracheostomy is still debated, most

158

8. A 29-year-old man complains of right cheek swelling, pain, and redness. One week ago, he was stabbed in the right cheek just above the angle of the mandible and underwent simple closure of the wound in the emergency department. On examination today, his right cheek is swollen, erythematous, and tender to palpation. The sutures are pulling through the wound. What is the most likely underlying cause of this patient’s findings? A. B. C. D. E.

Pseudoaneurysms of the facial artery are another complication of missed injury and, similar to sialoceles, present as swelling without erythema or significant inflammation. A superficial wound infection is possible after any laceration closure; however, infections after closure of facial lacerations are not common, occurring in 3% of patients or less. A laceration on the cheek presenting with local inflammatory changes after closure should prompt concern for missed parotid gland or ductal injury. The description of this patient's wound location and severe inflammatory reaction are not consistent with pharyngeal or periodontal abscesses that rarely present with external facial findings.

Parotid duct injury Pseudoaneurysm of the facial artery Superficial wound infection Pharyngeal abscess Periodontal abscess

ANSWER:

9. A 32-year-old healthy woman with no significant family history has a palpable 25cm left thyroid nodule found on a routine annual exam. She has no symptoms; thyroid function is normal; and the nodule is smooth, firm, and mobile. The neck exam is otherwise unremarkable. Fine needle aspirate is performed and is consistent with a Bethesda category III follicular lesion of uncertain significance. The next step in management is

E

Patients presenting with penetrating facial trauma must have their wounds examined closely to rule out deeper anatomic structures. Lacerations to the cheek may cause significant injury to the facial nerve, vascular structures, the parotid gland, or the parotid duct (figure 8.1). The parotid gland lies superficial to the posterior portion of the masseter muscle. The parotid duct is also called the Stenson duct. It measures approximately 7 cm long and ascends from the anterior portion of the gland. It passes anteriorly, superficial to the masseter muscle near the transverse facial artery and buccal branch of the facial nerve. Hence, evidence of injury to either the artery or nerve should raise concern for parotid duct injury. Failure to recognize and treat parotid duct injuries at the time of initial presentation may lead to development of sialocele, cutaneous fistula, or a salivary duct cyst. The most common presentation of missed parotid duct injury is wound breakdown and salivary drainage in the early days after injury. Delayed recognition of parotid duct injury is common and frequently leads to complications. Inspect all cheek lacerations for evidence of injury to the gland, duct, nerve, or artery. If a ductal injury is identified, microsurgical repair is the treatment of choice and should be undertaken at the time of initial presentation. Delayed repair makes primary repair extremely challenging and often necessitates drainage through the parotid duct orifice, an oral fistula, or saliva production suppression.

A. B. C. D.

dedicated neck ultrasound in 6 months. total thyroidectomy. left thyroidectomy. repeat fine needle aspirate for gene expression classifier testing. E. E PET-CT scan.

ANSWER:

B

The management of thyroid nodules is a rapidly changing field in the era of molecular diagnostic techniques guiding surgical decisions. A commonly used and sensitive test for the diagnosis of thyroid cancer is the fine needle aspirate (FNA). This test is cost-effective, safe, and relatively sensitive and specific. Experienced cytopathologists can accurately diagnose threatening lesions based on FNA. The Bethesda classification system, recently revised and updated, categorizes adequate FNA samples into 6 diagnostic classes. Category I and II lesions can be observed, and category V and VI lesions should be surgically removed in the absence of a reason not to intervene. Management of category III and IV lesions are controversial in terms of background malignancy rates. It is in these categories that the sensitivity and specificity of FNA makes the test less reliable for definitive management of thyroid nodules Until recently, category III and IV nodules diagnosed on FNA were submitted for surgical pathology after resection harbored malignancy in only 15 to 40% of cases; thus, surgery was unnecessary in most patients. In addition, the frozen section diagnosis of such lesions is tricky and unreliable, oftentimes resulting in secondary cancer operations after initial hemithyroidectomy for diagnosis.

Figure 8.1. 159

Management options for a Bethesda category III or IV lesion include observation, surgery, or repeat FNA. Further imaging has no role in management, unless to guide repeat FNA PETCT scan has no role in the management of thyroid nodules. Repeat FNA may not yield any new information, but with the addition of gene expression profiling, a high probability of malignancy or benignancy may be obtained and is a very reasonable option for management. FNA can confidently guide the role of interval imaging and observation. Repeat FNA with cytopathology alone does not always add information for decision making, but in combination with gene expression profiling, the test is associated with a high negative predictive value for cancer. Newer molecular techniques to interrogate FNA specimens are associated with a higher positive predictive value for cancer, guiding a decision for surgery.

and develop granulation tissue at the end of the stent. Consequently, tracheal stents should be used only in carefully selected patients with PITS who are unfit for surgery or who have a limited life expectancy. For patients with PITS who are not in distress, the optimal management is single-stage tracheal resection and reconstruction. The length of trachea that can be resected safely varies with age and body type, but the amount of resection considered feasible for appropriate reconstruction can involve up to half the length of the trachea. Neck flexion alone is sufficient for most patients having primary repair of PITS to facilitate healing without tension. If additional length is needed, a suprahyoid laryngeal release can provide an additional 1.0 to 1.5 cm. Primary single-stage reconstruction results in good or satisfactory results in 93% of patients. Laser ablation can be effective in destroying intraluminal tumors and restoring tracheal patency. However, laser therapy is not effective at treating PITS due to the circumferential, transmural nature of the lesion. Although tracheal transplantation has been attempted in a handful of patients with variable success, it is still considered an experimental treatment.

10. A 50-vear-old previously healthy man undergoes a course of prolonged intubation after a motor vehicle crash. He recovers and is discharged. He returns 6 weeks after the incident neurologically and hemodynamically normal. He is complaining of shortness of breath and wheezing on exertion. On physical exam, he has inspiratory and expiratory stridor. CT scan of the neck/chest and fiberoptic bronchoscopy reveal a 4-mm diameter, 2-cm long segment of mid-tracheal stenosis with thickened granulation tissue. What is the recommended treatment? A. B. C. D. E.

11. The anatomic boundaries of level IV jugular nodes are

Serial balloon dilation Laser ablation of granulation tissue Tracheal resection Self-expanding tracheal stent Tracheal transplant

ANSWER:

A

postintubation tracheal stenosis (PITS) remains the most common long-term airway complication after prolonged intubation. PITS is caused by cicatricial healing and fibrosis after an endotracheal tube balloon creates transmural ischemia or necrosis of the trachea. Despite improvements in design of endotracheal and tracheostomy tubes, specifically high-volume, low-pressure cuffs, PITS develop in 1 to 2% of patients after prolonged intubation. Symptoms related to PITS will usually develop within 3 to 6 weeks. Exertional dyspnea begins to occur when the diameter of the trachea is reduced below 10 mm, and wheezing or stridor develops once the airway has a diameter less than 5 mm.

Superior

Inferior

Medial

Lateral

A. Hyoid

Cricoid

B. Cricoid

Clavicle

C. Hyoid

Cricoid

Lateral border sternohyoid m Lateral border sternohyoid m Carotid sheath

D. Cricoid

Clavicle

Carotid sheath

E. Hypoglossal n

Spinal accessory n

Stylohyoid m

Lateral border SCM m Posterior border SCM m Spinal accessory n Spinal accessory n Lateral border SCM m

ANSWER:

C

Lymph node drainage is based on anatomy. Knowledge of lymph node stations of neck is critical because general surgeons often perform lymph node biopsy and must accurately convey anatomic location. If compartmentoriented lymph node dissection is performed, it is important to accurately describe and label the specimen. Figure 11.1 outlines the anatomic boundaries of the different lymph node basins.

Bronchoscopy with dilation is the initial recommended treatment for acute symptoms of PITS for a patient in respiratory distress. Bronchoscopy can improve symptoms temporarily, but it is not definitive therapy. The transmural nature of the scarring and stenosis explains the limited success of purely endoluminal therapy such as serial dilation and tracheal stents. In addition, tracheal stents can migrate

160

12. A 49-year-old man with a 20-pack-year smoking history presents with a nontender 2-cm mass in his right neck at the anterior border of the sternocleidomastoid muscle and inferior to the hyoid bone. The patient has no significant medical or family history. The remainder of his physical exam is unremarkable. The most appropriate next step is A. B. C. D. E.

oral antibiotics. excisional biopsy of the mass. laryngoscopy. esophagoscopy. ultrasound-guided fine needle aspiration of the mass.

ANSWER:

A

Cancer is the most common cause of a new neck mass in patients older than 40. A neck mass in an adult should be considered malignant until proven otherwise. Prompt diagnosis is crucial, as delayed diagnosis and treatment worsens prognosis. A trial of oral antibiotics and no additional diagnostic workup is not appropriate in this patient who reports no infectious symptoms. After detailed history and focused head and neck examination, ultrasound-guided fine needle aspiration (FNA) is the most appropriate next step. Biopsy of the solid portion of the mass is important for correct diagnosis, and ultrasound will help facilitate the procedure. FNA is diagnostic in more than 90% of cases. FNA is also amenable to real-time cytologic evaluation. The risk of seeding a tumor in the biopsy tract is negligible.

Figure 11.1. Level IA nodes are located in the submental region. They drain the floor of the mouth, anterior tongue, anterior mandibular alveolar ridge, and lower lip. Level IB nodes are located in the submandibular region. They drain the oral cavity, anterior nasal cavity, midface, and submandibular gland.

Excisional biopsy is rarely indicated and is reasonable only after repeat FNA, consideration of core needle biopsy, and thorough investigation of the primary disease. If FNA demonstrates carcinoma but a primary site is not identified on physical exam, laryngoscopy and then esophagoscopy and bronchoscopy should be performed. CT scan of the neck with intravenous contrast will identify the primary source of malignancy in a patient with a neck mass believed to be associated with a head and neck malignancy.

Level II nodes are the upper jugular nodes. They are divided into IIA (anterior to spinal accessory nerve) and IIB (posterior to spinal accessory nerve). These nodes provide drainage to oral and nasal cavities, nasopharynx, oropharynx, hypopharynx, larynx, and parotid gland. Level III nodes are the mid jugular nodes. They drain the oral cavity, nasopharynx, oropharynx, hypopharynx, and larynx. Level IV nodes are the lower jugular nodes. This nodal basin drains the hypopharynx, thyroid, cervical esophagus, and larynx.

13. A 70-year-old man presents with foul-smelling breath and repeated episodes of aspiration pneumonia. On examination, there are wheezes auscultated at the bases of both lungs and no neck masses. He reports of a sticking feeling in the throat with dysphagia. A CT scan of the chest shows signs of pneumonia. After pneumonia treatment, what is the best diagnostic test?

Level V nodes are located in the posterior triangle. This group is divided into VA (superior to level of cricoid) and VB (inferior to level of cricoid). This group drains the nasopharynx, oropharynx, and skin and soft tissue of the posterior scalp and neck.

A. B. C. D. E.

Level VI nodes are the pretracheal and paratracheal nodes. They provide drainage to the thyroid gland, glottis and subglottic larynx, apex of the piriform sinus, and cervical esophagus.

161

Esophagogastroduodenoscopy Esophageal manometry Barium esophagram Capsule endoscopy Neck ultrasound

ANSWER:

Esophagogastroduodenoscopy is not the recommended initial diagnostic test, because the small opening of a diverticulum in a proximal location may be easily missed. However, diverticula are found incidentally in patients being evaluated for dysphagia. Esophageal manometry is useful in the diagnosis of esophageal motility disorders but does not aid in the diagnosis of Zenker diverticulum. Capsule endoscopy is used in diagnosing disorders of the small intestine but does not aid in the diagnosis of esophageal diverticulum. Ultrasound of the neck can confuse a diverticulum with a thyroid nodule. If air is seen in the cystic structure, then a diverticulum must be considered instead of a thyroid nodule.

E

Patients with a pharyngoesophageal (Zenker) diverticulum are initially asymptomatic until the diverticulum grows. Symptoms may then include a sticking feeling in the throat with dysphagia, retrosternal pain, excessive salivation, halitosis, and respiratory infections. Barium esophagram, with lateral views, will show a barium-filled diverticulum at the level of the cricothyroid cartilage, posterior to the esophagus (figures 13.1 and 13.2).

14. A 20-year-old man presents with hoarseness and subcutaneous emphysema after sustaining a blow to his neck during a tackle football game. He is hemodynamically normal and has no other injuries. Plain film shows a large pneumomediastinum. During flexible bronchoscopy, a 3-cm tear is noted in the lower cervical trachea, which extends to approximately one-quarter of the circumference of the trachea. The tissue appears viable without much displacement. What is the most appropriate next step in his management? A. B. C. D. E.

Placement of an endotracheal tube Primary repair of the injury through a median sternotomy Primary repair through a cervical incision Stent placement Voice rest, antibiotics, and a proton pump inhibitor

ANSWER:

Figure 13.1.

D

Not all traumatic tracheobronchial injuries require repair. Low-energy, nonpenetrating injuries, in particular, can be managed nonoperatively despite seemingly impressive damage. The diagnosis is suspected by hoarseness, subcutaneous emphysema, and a mechanism that involved the airway system such as a traumatic injury or recent bronchoscopy or even intubation, especially in someone with a stenotic trachea. The diagnosis is confirmed by either rigid or flexible bronchoscopy. Features that favor nonoperative management are injuries that are 4 cm or smaller, involve less than one-third of the circumference of the trachea, have wound edges that are well opposed, and those that do not show significant tracheal tissue loss. Additionally, the patient should be hemodynamically normal, without an associated esophageal injury, and without signs of sepsis. This patient is hemodynamically normal with no other injuries and favorable anatomy. Elective intubation secondary to the tracheobronchial injury is not necessary. If intubation is necessary, care should be taken during intubation so as not to make a partial disruption into a complete one. A nonoperative approach can be used even in patients who require mechanical intubation and would consist of humidified air, voice rest, antibiotics, a

Figure 13.2. 162

proton pump inhibitor, continued observation, and repeat bronchoscopy. A low tidal volume is recommended. There is no role for stenting in these patients. If the patient has difficulty with intubation or mechanical ventilation, then operative management should occur.

pleomorphic adenoma, also known as benign mixed tumor. Unlike some benign tumors, pleomorphic adenomas tend to recur and undergo malignant transformation (carcinoma ex pleomorphic adenoma) if inadequately treated.

For patients who require operative repair, most cervical injuries can be managed through a transverse collar incision 2 cm above the sternal notch. The chest should be prepared in case a median sternotomy is required. If the injury is noted to be 2 to 3 cm from the carina, then a right posterolateral thoracotomy through the fourth or fifth intercostal space should be used unless the injury is distal to the carina on the left. In those cases, a left posterolateral thoracotomy should be used through the fifth intercostal space. However, these distal injuries are unlikely in low-energy blunt trauma situations (figure 14.1).

The mainstay of treatment for pleomorphic adenomas Is surgical resection. Tumor enucleation (or lumpectomy) that leaves the tumor capsule in situ is inadequate, with reported recurrence rates as high as 45%. Dissection of the facial nerve with superficial parotidectomy is the traditional management when the tumor is located in the superficial lobe. Total parotidectomy Is used when the tumor is located in the deep lobe. In the current scenario, because the tumor is located in the superficial lobe, superficial parotidectomy is the indicated treatment. Neither modified neck dissection nor adjuvant radiation therapy is necessary in the treatment of pleomorphic adenomas. Although tumor recurrence is greatly reduced with superficial parotidectomy, associated complications such as facial weakness, Frey syndrome, and esthetic hollowing are debilitating. Thus, some head and neck surgeons have championed lesser resections that dissect fewer branches of the facial nerve and leave more parotid tissue. Partial parotidectomy, as opposed to enucleation, appears to be as effective as superficial parotidectomy when performed by expert surgeons in appropriately selected patients.

Figure 14.1

15. A 47-year-old male smoker is found to have a firm 3-cm mass in the right parotic gland. CT scan indicates that the tumor is located in the superficial lobe. Fine needle aspiration biopsy returns pleomorphic adenoma. Treatment should consist of A. B. C. D. E.

enucleation of the tumor. superficial parotidectomy. total parotidectomy. total parotidectomy and right modified neck dissection. total parotidectomy and postoperative radiation treatment.

ANSWER:

B

The parotid gland is the most common site for salivary neoplasms (approximately 85% of cases). Most parotid tumors are benign, and the most common type is the

163

Ethics

ultimately provide the patient with a satisfactory result at the end of the discussion.

ITEMS 1-20 For each question, select the best possible response.

Many researchers have studied the effectiveness of end-oflife discussions. Communication skills training is beneficial for healthcare professionals, but not in all areas. Healthcare professionals who participate in a course that is learner focused and involves facilitators who are experienced in endof-life discussions can improve empathy and information gathering. Additionally, healthcare professionals who participate in such courses have increased self-efficacy.

1. A woman with severe dementia who lives in a nursing home is admitted to the hospital with a diagnosis of left lower extremity cellulitis. She is able to smile and interact with family. She is started on antibiotics. She has a living will, does not want to be resuscitated, and has a healthcare power of attorney (HCPOA). The HCPOA is her son who is present at the bedside and wants to discuss whether the treatment plan for the cellulitis is worth implementing. You respond that

These studies do not show an improvement in patientcentered outcomes. No correlation exists between taking a course on end-of-life communication and improved patient anxiety, mental health, or satisfaction. Although these aspects of end-of-life discussions are important markers of effective communication, they are not affected.

A. B. C. D.

treatment is futile. the outcome is undesirable. the treatment plan may achieve its goal. the plan of care should be determined by a consensus all family members. E. a hospital ethics committee should review the case.

ANSWER:

3. A single 71-year-old man presents with a new diagnosis of a 5-cm squamous cell carcinoma involving his chest wall. After the examination, the patient's only child pulls you aside and shares with you that her father has been diagnosed with early stages of dementia, and she is concerned about his decision-making capacity. Which of the following statements is true?

C

According to medical ethics scholar, Griffin Trotter, medical futility occurs when 1. 2. 3.

There is a goal, There is an action to achieve this goal There is certainty that action will not achieve the goal.

A. Medical decisions should be made by his daughter. B. Medical paternalism takes priority over patient selfdetermination. C. Consultation with geriatric psychiatry is required to obtain proper informed consent D. Capacity requires this patient to understand his situation, proposed treatment, and risks. E. Competency requires a multidisciplinary assessment.

Interventions can be medically futile if they do not produce any benefit to the patient. Medical futility should not be applied to the overall care of the patient but to specific interventions at a particular time. Medical treatment in this woman with dementia is not futile. The outcome of treating the cellulitis with antibiotics is desirable. The plan of care does not need to be determined by all family members, and an ethics committee does not need to be involved. Therefore, responding to the son that the treatment plan may achieve its goal is the correct answer.

ANSWER:

2. With regard to discussions about end-of-life care, communication skills training can help healthcare providers A. B. C. D. E.

The requirement to obtain informed consent has been the law of the land in the United State since 1914 based on Schloendorff v Society of New York Hospital. The requirements vary from state to state and are ambiguous at best. Decision-making capacity Is a requirement for truly informed consent. In general, most physicians are capable of assessing a patient's decision-making capacity for the proposed Intervention, but consultation with a geriatric psychiatrist or neurologist may be needed for a formal evaluation of the patient's cognition.

show increased empathy. reduce patient anxiety. improve patient mental health. improve patient satisfaction. increase ability to use closed, directed questions.

ANSWER:

D

A The Patient Self-Determination Act of 1990 established as US law that a patient's right of self-determination was the highest standard of medical ethics—taking precedence over medical paternalism and beneficence. Competency is a legal term, and the determination of incompetence is a judicial decision. On the contrary, a physician determines capacity. Capacity usually requires the patient to understand his or her situation, proposed treatment, and risks. The US Supreme

Most surgeons will be involved with end-of-life discussions. Although this is a critical skill, most surgeons do not receive any formal training in these types of discussions. As providers, we should attempt to be empathetic, reduce patient anxiety, improve the patient's understanding of the process, use open-ended questions to facilitate the discussion, and

165

Court has repeatedly ruled that people have the right to control decisions regarding their medical treatment. Thus, if capable, the patient has the right to refuse or even discontinue treatment. The patient's right to refuse treatment holds for guardians as Will, With some notable exceptions. States and the federal government have enacted statutes to protect incapacitated adults, as they are the most vulnerable of its citizens.

E. transfuse blood products.

ANSWER:

The appropriate intervention for this child in hemorrhagic shock is to transfuse blood against the wishes of the parents. In Prince v Massachusetts, the court specifically stated "Parents may be free to become martyrs themselves. But it does not follow that they are free, in identical circumstances, to make martyrs of their children." Several additional court rulings have stated that parents cannot refuse live-saving treatment for a child. The summary points of these court rulings focus on 3 main themes. First, children's and state's interests outweigh parental rights to refuse treatment Second, parental rights do not afford life-and-death dominion over their children. Third, parents do not have an absolute right to refuse medical treatment for their children based on religious beliefs.

4. A 58-year-old man who is functioning well with activities of daily living, but who has widely metastatic colon cancer, presents with an acute colonic obstruction. He has a do-notresuscitate (DNR) order, and he consents to a palliative ileostomy. The surgeon, the patient, and his healthcare power of attorney do not want to rescind his DNR wishes. The operating room team is concerned about what should be done if the patient experiences intraoperative complications, including cardiac arrest. What would be the next step in the management of this patient?

Although many Jehovah's Witness patients have a church member with them at all times during hospitalization, they are not legal authorities to make medical decisions for patients. Asking children what they would like to do is not appropriate, because children cannot consent to treatment. They are unable to weigh risks and benefits, compare alternatives, or understand the long-term consequences of the choices they make. There is not enough time in this case to consult the hospital ethics committee.

A. Transfer the pat lent to a different hospital. B. Do not perform the ileostomy until the DNR is rescinded. C. Seek a healthcare team who will honor the patient's request. D. D Perform the ileostomy and, if needed intraoperatively, proceed with full resuscitative efforts. E. E Perform the ileostomy with limited resuscitation attempts.

ANSWER:

A suggestion to facilitate this conundrum for Jehovah's Witness patients is to have a pre-emptive discussion with local church leaders regarding how this situation should be managed. The result would be an agreement that is acceptable by both the church and the medical leadership in the hospital.

C

his right of self-determination. Failing this, a multidisciplinary patient centered-care conference with the patient and his family should be arranged to discuss other options. These might include a "procedure-directed approach" determining what procedures he would allow in the event of cardiac arrest (e.g., discussion of options of pharmacologic therapy, defibrillation, CPR) or a "goaldirected approach" discussing what his goals would be if he were to arrest. An example of this decision would include that he wishes to wake up with the same quality of life as before the arrest and treatment can only be delivered with these constraints in mind.

6. An 84-year-old woman with mild congestive heart failure presents with nausea and vomiting. She has right lower quadrant pain and tenderness. CT scan, confirms a closedloop small bowel obstruction in the right lower quadrant with signs of intestinal ischemia. You quote a projected 20% mortality rate with operation. After an extensive discussion with the patient and her daughter, they are unable to make a decision regarding operation. The next step is A. B. C. D. E.

5. A5-year-old boy was involved in a motor vehicle collision. He is hemodynamically abnormal with grade IV liver and spleen lacerations. His parents, who are Jehovah's Witnesses, refuse blood product transfusion for the child with the knowledge that without a transfusion, the boy may lie. The next appropriate step is to A. B. C. D.

E

an ethics consult. a psychiatry consult. a palliative care consult. identifying a new surrogate. surgical exploration.

ANSWER:

ask a church representative if an exception can be made. ask the child what he would like to do. consult the hospital ethics team. honor the parents' wishes.

C

Medical decision-making around goals of care and end-of-life decisions can be very challenging, especially in urgent clinical situations in which there is time pressure for the

166

patient and family to make a decision. These situations also pose a challenge for the surgeon, who often does not have a pre-existing relationship with the patient.

neurologic death may also be present, he cannot be declared dead by neurologic criteria. A resuscitative thoracotomy is a therapeutic intervention for which indications and contraindications can be identified either by individual surgeons or through clinical practice guidelines. Performance of this procedure is not a prerequisite for pronouncing death, even in patients with penetrating torso injuries. There are currently no telemetry or ultrasound requirements for death pronouncement. Thus, the declaration of circulatory death remains a clinical determination as indicated by the absence of circulation and breathing with no possibility to resume spontaneously. This occurs after 2 to 5 minutes if cardiopulmonary resuscitation (CPR) has not been provided and approximately 7 minutes after CPR termination. In this patient, CPR is likely futile given his penetrating torso wound with presumed exsanguination and clinical evidence of a severe traumatic brain injury from a penetrating mechanism; therefore, a full 7-minute waiting period after cessation of CPR would not be necessary to pronounce death after the initial assessment.

An ethics consult would be indicated if there was some concern that the daughter was not acting in her mother's best interests or trying to inappropriately influence her mother's decision-making. Unless there were concerns about the patient's mental status or capacity to make decisions, a psychiatry consult would also not be indicated. There is no indication that this patient has lost her decision-making capacity and appointing a surrogate without her permission would violate her autonomy. Finally, in this case, the medical facts are not in dispute. This patient and her daughter need assistance in their decision-making. This patient has a clear indication for operation (ischemic bowel) and perioperative risk factors (age and congestive heart failure). Thus, the appropriate next step would be to obtain a palliative care consult to help her in medical decision-making. Taking the patient to the operating room for surgical exploration would constitute battery and is not appropriate.

8. A 55-year-old surgeon complains on Facebook about fits recent call night which he says was busy because of a highprofile shooting that resulted in a police officer being killed. He does not mention the police officer's name. The post

7. A 22-year-old man presents to the emergency department approximately 30 minutes after sustaining multiple gunshot wounds. Paramedics were on the scene approximately 15 minutes after the event and had a 5-minute scene time. He had no pulse on assessment, and cardiopulmonary resuscitation was continued during the 15-minute transport. Cardiopulmonary resuscitation is held at the direction of the trauma team leader to facilitate the patient's assessment. On examination, he is apneic, he has absent breath sounds, absent cardiac sounds, no carotid pulse, no pupillary response, and his extremities are flaccid. He has large penetrating wounds to the back of his head and to his right flank. He is connected to the telemetry monitor, which demonstrates a wide complex, bradycardic rhythm. Subxiphoid and parasternal ultrasound windows demonstrate no pericardial fluid; however, there is intermittent hypokinetic movement of the cardiac muscle. Which of the following statements regarding this patient is true?

A. is acceptable if his Facebook profile and posts are designated as "friends only." B. is acceptable if he uses a pseudonym for his profile. C. is acceptable within the confines of a closed Facebook group for surgeons. D. is acceptable on social media platforms designed for physicians only. E. could result in professional consequences for the surgeon.

ANSWER:

With the increasing popularity and use of social media, surgeons must be cognizant to maintain online professionalism. Unprofessional online behavior is disturbingly common among surgical residents and practicing surgeons, and such behaviors can be associated with professional and legal consequences, including job loss and revocation of medical licenses.

A. He can be pronounced dead based on neurologic criteria. B. He can be pronounced dead based on circulatory criteria. C. He cannot be pronounced dead until a resuscitative thoracotomy is performed. D. He cannot be pronounced dead until there is asystole on telemetry. E. He cannot be pronounced dead until there is no cardiac motion on ultrasound.

ANSWER:

E

It is the surgeon's obligation to use social media wisely and to always make patient privacy the main priority. In general, social media such as Facebook and other similar platforms are not compliant with the Health Insurance Portability and Accountability Act of 1996, even in the setting of a closed group. Surgeons should refrain from sharing any information that can compromise patient privacy and trust. The use of enhanced privacy settings (e.g., "friends only") and

B

This patient meets criteria for circulatory death based on his clinical assessment and can thus be pronounced dead at the conclusion of his evaluation. He has not been formally assessed for brain death through a neurologic exam and has not undergone confirmatory testing. Consequently, although

167

pseudonyms have not been historically effective in preventing the wide dissemination of private information.

10. A surgeon at a certified bariatric center of excellence performs a laparoscopic sleeve gastrectomy on a 42-year-old woman with a BMI of 42 and associated hypertension. On postoperative day 1, she is doing fine and is discharged to home, which is 90 miles away. On postoperative day 2, she calls her surgeon's office with a complaint of abdominal pain and gets the answering service. She does not receive a call back from the covering bariatric surgeon. She calls the office on postoperative day 3, and a triage nurse responds telling her that the covering surgeon will call her back, but no one does. On postoperative day 5, the patient presents to her local emergency department in septic shock. CT scan shows evidence of a leak. The local emergency department does not have a bariatric sturgeon, and the on-call surgeon is not comfortable taking care of this patient. The patient is transferred back to the operating surgeon's hospital after discussion with the covering bariatric surgeon. The patient makes it to the emergency department but then dies shortly thereafter. The patient's family decides to sue. In deposition, the bariatric surgeon claims no calls were made to the covering bariatric surgeon. Who is mostly at fault for this outcome?

Surgeons must remember that posting on social media can result in an unintentional overlap between their personal and professional lives, and comments may reach a larger audience than originally intended.

9. Concerning surgical care in the United States, which of the following is more likely to be increased in black patients compared with white patients? A. B. C. D. E.

Regional anesthesia Perioperative blood transfusions Minimally invasive surgery Enrollment in clinical trials Treatment at a high-volume hospital

ANSWER:

B

Disparate healthcare delivery in the United States represents an ethical crisis. Race and ethnicity, lower-income status, and insurance status are associated with limited access to surgical care and poorer surgical outcomes. Multiple studies document that patients of color are less likely than white patients to receive procedures for a myriad of vascular and oncologic diagnoses. In spite of a higher prevalence of aortic aneurysms, black men older than 65 years undergo repair less frequently compared with white men. Likewise, a national study of patients with pancreatic adenocarcinoma demonstrated that black patients were less likely to have a medical oncologist or radiation oncologist and less likely to receive surgical resection.

A. B. C. D. E.

The operative bariatric surgeon The answering service The covering bariatric surgeon The local emergency department The local general surgeon

ANSWER:

A

In this case, the bariatric surgeon is most at fault because he or she is supposed to provide reliable 24-hour coverage in the perioperative period for all bariatric patients according to the consensus statement from the American Society of Metabolic and Bariatric Surgery. This perioperative care includes providing reliable access to care after discharge should complications arise. The answering service is responsible only for delivering the messages and not for any triage or clinical care. A lack of communication between the patient and the clinician is one of the main reasons that telephone care fails patients. A failure in this communication results in more than 25% of patients having ongoing pain and discomfort, 3% sustaining clinical harm, and 8% requiring admission to the hospital. As such, an answering service should be more than willing to contact the on-call physician.

For several abdominal procedures, black patients undergo minimally invasive surgery less often compared with white patients and undergo regional anesthesia less often with longer operative times than either white or Hispanic patients. Despite receiving fewer procedures and having less access to surgical care, black patients have higher rates of in-hospital complications and disease recurrence. Black and Hispanic patients have significantly higher rates of postoperative complications, blood transfusions, perioperative mortality, and longer hospital stays compared with white patients. Inequities in morbidity and mortality among blacks and Hispanics persist even when controlling for insurance status. Since complication rates have been correlated with surgical volume, the fact that blacks are less likely to be treated at a high-volume hospital or by a highvolume surgeon may explain their higher complication rates. Finally, a large discrepancy exists between the percentage of minorities (30%) in the United States and the minority enrollment rate in clinical trials (1%)—especially cancer trials.

A hospital and its emergency department are not responsible for care of bariatric patients if it does not provide bariatric care. This includes the covering surgeon at that hospital. The covering bariatric surgeon is not at fault because it does not appear that he received any messages about this patient until he or she was called by the nonbariatric hospital. Again, the primary bariatric surgeon is most at fault for not providing reliable 24-hour service in the perioperative period.

168

11. A patient is having a surgical procedure at a teaching institution. The surgeon commonly schedules overlapping surgeries. The surgeon's responsibility for overlapping surgeries to the involved patients is to

attending surgeon's responsibility is to ensure all portions of an operation are performed safely, regardless of who is performing the procedure.

A. explain the process in general terms to the patient whose case is scheduled second. B. provide additional disclosure, because the practice is well understood by the public C. explicitly describe the components of the procedure that the surgeon will perform. D. inform that 30-day mortality is slightly higher in this situation. E. refer the patient to the hospital administrator who is responsible for setting the critical components of each surgery.

ANSWER:

12. Which of the following is an example of a futile intervention in the intensive care unit? A. Placement of a feeding tube in a malnourished patient without an established diagnosis and prognosis B. Induced hypothermia in a comatose patient after out-ofhospital cardiac arrest C. Venovenous extracorporeal membrane oxygenation in a patient with acute respiratory distress syndrome on maximal mechanical ventilator support D. Prolonged cardiopulmonary resuscitation to treat refractory cardiac arrest due to malignant hyperthermia E. Placement of a ventricular assist device in a patient with end-stage heart failure who is not a candidate for cardiac transplantation

C

Overlapping surgery is defined by the American College of Surgeons as 2 or more procedures performed by the same surgeon that overlap in their incision and closure but have critical components that do not occur at the same time. Overlapping surgery is a frequent practice in many academic training programs where the second procedure is being performed by a trainee who has done the procedure or similar procedures before. Concurrent surgery is defined similarly, but the critical components occur simultaneously. The American College of Surgeons clearly states that overlapping surgery is acceptable in many instances, but that concurrent surgery is never acceptable.

ANSWER:

A

The advancement of medical technology has made it possible to support patients for extended, and potentially indefinite, periods of time in the intensive care unit For surgeons working in the environment of the intensive care unit, it is therefore imperative to understand which therapies or treatments are not likely to benefit the patient in any meaningful sense and therefore represent futile interventions. Placement of a feeding tube in a malnourished patient who does not have an established diagnosis and prognosis is a futile intervention. Without an established diagnosis, no prognosis can be made regarding the patient's likelihood of recovery and how long the tube feedings should be administered.

Morbidity and mortality rates as well as length of hospitalization after surgery for overlapping surgery are not increased. Current data focus on elective cases. Risk adjustment was used in these studies and did not appear to affect the outcomes, although low-risk procedures and patients were included. Fewer data exist for concurrent surgery.

Induced hypothermia in a comatose patient after out-ofhospital cardiac arrest can improve neurologic outcomes and decrease mortality after cardiopulmonary resuscitation. Venovenous extracorporeal membrane oxygenation in a patient with acute respiratory distress syndrome on maximal mechanical ventilator support can reduce ventilator-induced lung injury and improve survival.

Key elements associated with overlapping surgery include the ability to return to the operating room at any time if a problem arises and to have a back-up surgeon available who could return when called. This is a Centers for Medicare and Medicaid Services billing mandate as well. The surgeon is responsible for defining the critical components of each case. How this is monitored is often not defined, but this information should be shared during the consent process.

Prolonged cardiopulmonary resuscitation has markedly improved survival in patients with refractory cardiac arrest without an underlying terminal diagnosis. Placement of a ventricular assist device in patients with end-stage heart failure can provide for several years of improved quality of life in patients who are not candidates for cardiac transplantation.

The idea of overlapping surgery must be folded into the consent process, because the public's understanding of overlapping surgery is poor. A recent investigation concluded that overlapping surgery should be explicitly disclosed to the patients during the consent process. This conclusion is consistent with the legal intent of informed consent, which suggests that the physician should disclose material information necessary for a reasonable patient to make an informed decision about the planned procedure. The

13. The use of patient decision aids to enhance the informed consent process

169

A. B. C. D. E.

increases healthcare costs. increases malpractice litigation. delays the implementation of therapy. replaces direct patient-physician communication. may direct patients to a particular choice of treatment.

ANSWER:

E. There is an ethical distinction between a medical treatment that is inside the body (i.e., implanted implanted cardioverter defibrillator or pacemaker) versus a treatment or intervention that is outside the body (i.e., mechanical ventilation).

E

ANSWER:

Patient decision aids are information tools provided to patients to enhance understanding of their condition, the alternative treatment options available, the risks and benefits of the treatment options, and expected outcomes. Patient decision aids improve the consent process by providing patients with sufficient information regarding their decisions to lead the discussion and focus on the aspects that matter most to them. Available evidence suggests that patient decision aids actually decrease healthcare costs and litigation. These tools are designed to supplement, not supplant, direct patient-to-physician communication. There is no evidence that indicates patient decision aids delay therapy.

D

A patient who has decision making capacity appropriate to the decision at hand has the right to decline any medical intervention or ask that an intervention be stopped, even when that decision is expected to lead to his or her death and regardless of whether or not the individual is terminally ill American Medical Association Principles of Medical Ethics It has been assumed for several years that the controversy regarding the difference between withholding and withdrawing treatment is resolved, but advances in biotechnology have resurrected the debate. Clinicians and patients continue to harbor values that cast doubt on whether it is ethically, morally, or legally appropriate to withdraw treatment that is certain to result in the death of a patient in a relatively short period of time. This is particularly true for interventions that are placed inside a patient, such as an internal cardioverter defibrillator or pacemaker. Cultural ethos and religious convictions may influence a patient's views and values regarding a distinction between withdrawing and withholding medical treatments, but medical ethics and the rule of law have not wavered in the position that there is no distinction between them.

The short coming of patient decision aids is the potential for bias, intentional or unintentional, by directing patients to a particular treatment strategy. National standards for the certification of patient decision aids attempt to ensure that patient information is complete, accurate, current, and presented in an unbiased and balanced manner. The Centers for Medicare Medicaid Services (CMS) as well as many healthcare payors are incorporating patient decision aids into pay-for-performance incentives. Provisions of Section 3506 of the Affordable Care Act established a division within the CMS tasked with certification of patient decision aids that meet rigorous standards. Currently this provision has not received congressional funding.

A patient is considered competent unless a court of law decides otherwise. A competent patient has the right to accept, refuse, or terminate a treatment that is already in place. Terminating a treatment that is already in place is usually prompted by the patient's decision that the burden of continuing therapy outweighs any potential benefits or the treatment no longer aligns with the patient's values or goals for care. This applies to all types of treatment and is not dependent on the location of the treatment, internal or external (e.g., internal implanted cardioverter defibrillator vs a mechanical ventilator); the duration of treatment (e.g., hemodialysis for years or inotropes for hours); or whether treatment is continuous or intermittent (e.g., continuous cardiac pacing vs intermittent discharge of an implanted cardioverter defibrillator). The only exception guiding these principles involves a treatment that has become a living part of the patient such as a transplanted organ.

14. A 76-year-old man is terminally ill with metastatic colon cancer and has an implanted cardioverter defibrillator. Over the last week, the patient has endured multiple daily discharges from the device, frequently awakening him from sleep and causing significant pain and discomfort The patient requests that his device be deactivated. Which of the following legal principles apply to this situation? A. There is a legal distinction between withholding and withdrawing life-sustaining treatment. B. The deactivation of implanted devices (defibrillators) involves a different legal standard compared with discontinuation of external support devices (mechanical ventilation). C. Legal precedent requires physicians to comply with a patient's request for the discontinuation of a medical intervention, even if the intervention is contrary to the physician's own personal values. D. The legal distinction between withdrawal of unwanted therapy from patient-assisted suicide or euthanasia involves the concepts of "intent" and "cause of death."

The withdrawal of a life-sustaining treatment known or expected to result in a patient's death, even imminent death, is not intentional killing of the patient. This is based on the principles of "intent" and "cause" of death. Deactivation of an implanted cardioverter defibrillator at a patient's request is the acknowledgement that the patient has decided that the burden of the intervention has exceeded its benefit under the current circumstances and not an intent by the physician to kill the patient. The deactivation of the device itself is not the cause

170

of the patient's death; the cause is the underlying cardiac condition.

published by the American Medical Association in 1847. These 2 principles can be in conflict as well. An example might include ordering a laboratory test that will have no effect on the treatment plan and that could result in laboratory error prompting additional nonbeneficial interventions.

Although the legal and ethical principles apply as described, physicians are not compelled to comply with a patient's request for treatment that the physician consider futile or to withdraw therapy when the request is contrary to the physician's own personal beliefs or values. In such a case, the treating physician is responsible for identifying another provider who is willing to abide by the patient's wishes and accept the responsibility for the patient's care or to seek the assistance of the hospital committee in identifying another provider.

Of the 4 principles, autonomy is considered the most important. It was not until the early 20th century that this concept was applied to patient care. judge Benjamin Cordozo in Schloendorff v. New York Hospital first opined that "every human being of adult years and sound mind has a right to determine what shall be done with his own body: and a surgeon who performs an operation without his patient's consent commits an assault for which he is liable in damages." This decision represented a major shift from the traditional practice based on the physician's expertise and the understanding that the physician therefore knows what is best for the patient, the concept of paternalism. This right of selfdeterminism is the basis for an individual's right accept or to refuse any proposed medical intervention, based on the patient's understanding of the potential benefits and risks and whether the intervention coincides with his or her values and beliefs. Physicians are bound both legally and ethically to adhere to the patient's wishes, even if the physician feels that the patient's decision is in error.

15. Of the following legal and ethical principles that govern a patient's or surrogate' right to choose or refuse medical intervention, the most paramount is A. B. C. D. E.

justice. autonomy. beneficence. nonmalfeasance. appropriateness of intervention.

ANSWER:

B 16. A 35-year-old man is hospitalized with a massive intracranial hemorrhage after blunt trauma. After 2 weeks in the intensive care unit, he remains unresponsive. Neurosurgery and neurology consultants both concur that his chances of meaningful neurologic recovery are nil. However, he breathes on an apnea trial. The family has decided that lifesustaining treatment should be withdrawn but would like organ donation after cardiac death. Five minutes after terminal extubation, the patient remains unresponsive with no spontaneous respirations noted. The cardiac monitor reveals persistent electrical activity. What is the next step necessary to proceed with organ donation?

Four legal and ethical principles form the foundation of a patient's right to choose, or refuse, medical treatment, autonomy, beneficence, nonmalfeasance, and justice. Although the appropriateness of an intervention falls within the context of good medical and surgical practice, it is not a foundational ethical principle. These principles, although appealing to the moral values of both patient and treating physician, frequently come in conflict with one another. Individual justice is defined as the application of treatments in a fair and equitable fashion for all. Distributive justice, however, often becomes problematic when the distribution of healthcare resources for an individual patient compromises the application of resources for the betterment of the community at large. An example would be the utilization of intensive care unit beds, generally in limited supply. Decisions regarding who the sickest patients are or determining which patients are the most likely to derive benefit from intensive care unit care and offering these beds to those will prevent others from using this resource.

A. Auscultate the precordium to confirm absence of heart sounds B. Confirm complete cessation of pulsations on the arterial catheter C. Obtain transcranial Doppler to confirm brain death D. Obtain 12-lead electrocardiogram to confirm pulseless electrical activity E. Return the patient to mechanical ventilatory support

Beneficence implies that interventions offered to patients will be in their best interest with the intent of improving their wellbeing. This also involves an accurate and honest appraisal of the risk of benefit versus harm of any particular intervention under consideration.

ANSWER:

B

The usual scenario for organ donation after cardiac death involves mechanically ventilated patients who have sustained severe and irreversible brain injuries, either traumatic or vascular in etiology. Those with high spinal cord injuries or end-stage musculoskeletal disease are also potential donors.

Nonmalfeasance implies the intent to do no harm, primum non nocere. Beneficence and nonmalfeasance are the 2 oldest of the principles, have roots in the Hippocratic code, and formed the sole basis of the first Code of Medical Ethics

171

The United Network for Organ Sharing (UNOS) is a private nonprofit group that operates the Organ Procurement and Transplantation Network under contract with the US government. UNOS developed the rules for organ donation after cardiac death.

A. B. C. D. E.

Mechanical asystole is defined as the absence of arterial pulsations. Arterial pulsations may be absent even when a coordinated electrical cardiac rhythm persists. Electromechanical asystole is defined as the absence of both electrical and or mechanical cardiac activity. There is broad consensus that mechanical asystole (and not electromechanical asystole) is sufficient to determine death, because the standard definition of death is the absence of circulation, not the absence of cardiac electrical activity. Pulseless electrical activity (mechanical asystole) that does not generate circulation is thus immaterial in determining death.

Focused assessment with sonography for trauma Diagnostic peritoneal lavage CT scan of the chest and abdomen with pelvic shielding CT scan of the chest, abdomen, and pelvis MRI of the chest and abdomen

ANSWER:

D

After initial evaluation for blunt force trauma, imaging to investigate for occult blood loss is indicated in a patient who has responded to resuscitation. In the pregnant trauma patient, the edict "what's good for the mother, is good for the fetus" applies. At 35 weeks' gestation, organogenesis is complete, and the fetus is viable. Therefore, concerns for radiation exposure in the secondary evaluation should be overruled by the need to rapidly diagnose and treat any ongoing hemorrhage. CT scanning of the chest, abdomen, and pelvis is a quick and sensitive way to search for occult injury and ongoing blood loss.

The organ donation process begins with the identification of a suitable candidate, followed by full informed consent of the legal next of kin to the withdrawal of care and the retrieval of organs. Life-sustaining measures are then withdrawn under controlled drcumstances in the intensive care unit or in the operating room. When the criterion for cardiac death is met by the donor, a doctor pronounces the patient dead. The time from the onset of mechanical asystole to the declaration of death is generally 5 minutes, but it can be as short as 2 minutes. Multiple studies suggest that circulation does not spontaneously return after it has stopped for 2 minutes. To avoid conflict of interest, neither the surgeon who will perform organ retrieval nor any transplant personnel participate in end-of-life care or the declaration of death.

The pregnancy should not prevent scanning of the pelvis for a source of blood loss, because diagnostic peritoneal lavage is relatively contraindicated in advanced gestations. MRI, although associated with less fetal radiation exposure, is reserved for secondary studies in hemodynamically normal blunt force trauma patients. The focused assessment with sonography for trauma exam would be difficult to interpret in a pregnant female at 35 weeks' gestation.

18. When obtaining informed consent from a patient before elective surgery, which of the following statements is true?

Asystole can be assessed by palpating arterial pulses and listening for a heartbeat; these are typically done in routine determinations of death. However, in donation after cardiac death, more accurate and reproducible measurements of pulselessness are used. Indwelling arterial catheters sensitively and specifically measure arterial pulsations and pressures, and, in this case, will confirm asystole. Percutaneous Doppler ultrasound of peripheral arteries and echocardiography to show absence of flow through the aortic valve can also be helpful.

A. A patient's competence to sign an informed consent may be determined by the surgeon. B. Family members are best suited as translators for nonEnglish speaking patients. C. Mild cognitive impairment precludes the patient's ability to make a competent decision. D. Failure to obtain consent constitutes a legal action for battery. E. A patient must be able to verbalize as a condition of signing a consent form.

Transcranial Doppler is not indicated, because the patient is not brain dead, but rather is an organ donor because of prognosis. Returning the patient to mechanical ventilatory support is not indicated; once mechanical asystole confirms death, organ retrieval should proceed.

ANSWER:

D

Obtaining informed consent before elective surgery is a critical part of surgical practice. When obtaining consent, a surgeon must determine whether the patient has capacity to understand what he or she is being told. Competence, conversely, denotes a legal status determined by judicial review. In most situations, it is reasonable for the surgeon to determine whether the patient has the capacity to understand and consent or whether a substituted consent needs to be sought. Although legal criteria for consent vary between states, they generally include the ability to (1) understand

17. A 25-year-old woman who is pregnant at 35 weeks' gestational age is the unbelted passenger in a high-speed motor vehicle crash. She is alert, breathing spontaneously, with an initial systolic blood pressure of 88 mm Hg and a heart rate of 100 beats per minute. Her systolic pressure improves with resuscitation. After placement of a fetal monitor, which of the following tests should be performed?

172

relevant information, (2) communicate a choice, (3) appreciate the medical consequences of the situation, and (4) reason about treatment choices. The act of communicating a treatment choice requires indication of the choice made and does not necessarily require a verbal response. Mild cognitive impairment or early dementia do not necessarily preclude a patient's ability to make a competent decision. When such conditions are present or suspected, however, psychiatric consultation are helpful and should be documented carefully. As to the consent itself, translators may be required when English is not the patient's primary language. A systematic review of the literature found that use of professional interpreters, rather than family members, has the positive benefit of decreasing communication errors and improving facility use, clinical outcomes, and satisfaction with care. A translator is important in obtaining consent when surgeons and patients speak different languages, and the use of translators should be documented.

from an action for medical negligence: (1) the treatment must be voluntary and the physician must not owe the patient a prior duty, (2) the treatment rendered must be in a true emergency, and (3) treatment must be rendered outside a hospital or other place with medical equipment. The concept of "duty" is important, because if the surgeon provides emergency voluntary care to his own patient, there is a preexisting duty before the emergency that would likely nullify a Good Samaritan claim. If a surgeon agrees to assist another surgeon before the occurrence of the emergency, a duty to the patient is created. Another implication of the duty requirement involves being "on call." In such a situation, the on-call element implies a duty to anyone who requires the oncall physician's services, potentially excluding a Good Samaritan claim.

The process of obtaining informed consent is important. As detailed in Morvillo v Shenandoah Memorial Hospital, as well as many other cases, the court found that "the law requires informed consent... [and that] ... the failure to obtain informed consent is tantamount to no consent."

A. should be provided on a case-by-case basis. B. is defined similarly for all patients. C. suggests an intervention that cannot meet the goals of the patient's family. D. implies treatment that cannot obtain an intended physiologic goal. E. is an obligation of the caring physician if those are the wishes of the patient.

20. Futile treatment in the intensive care unit

A tort occurs when one person commits an action against another and may be either intentional or unintentional. Battery is defined as an unauthorized touching or invasion of a patient's body and is considered an intentional tort. Claims against doctors based on battery typically are restricted to cases in which physicians did not obtain informed consent or exceeded the scope of a consent that was obtained.

ANSWER:

Futile treatment is defined as treatment provided to patients who cannot obtain the intended physiologic goal. In the intensive care unit setting, this typically implies the provision of life-sustaining interventions when survival is not expected beyond the acute setting or when severe irreversible neurologic injury is sustained. Futility should be an uncommon determination in the acute setting and is determined based on individual clinical factors. The Society of Critical Care Medicine Ethics Committee recommends that futile interventions not be provided.

19. In which of the following situations would a physician likely be able to claim immunity from an action for medical negligence under a state Good Samaritan Law? A. A physician agrees to function as an assistant in a case and the patient dies. B. While on call at the hospital, a physician performs an urgent tracheotomy on his own patient for angioedema, and the patient dies. C. A physician performs cardiopulmonary resuscitation on a patient of his who has suddenly collapsed in the hallway outside his office. D. A physician comes to the aid of an unconscious player while attending a little league game. E. A physician places a chest tube for a pneumothorax occurring after he placed a central line and the patient dies.

ANSWER:

D

Healthcare providers have an obligation to respect the autonomy of patients and the wishes of their surrogates. However, they have no ethical obligation to provide treatment that they deem to be futile, even if requested by the patient. The patient or surrogate always maintains the opportunity to transfer care to a different physician or different institution if a disagreement arises.

D

Good Samaritan laws, also known as volunteer protection acts, exist at both the federal and state levels, and vary somewhat between jurisdictions. In general, there are 3 requirements to qualify a licensed physician for immunity

173

Oncology

appropriate margins within 4 to 6 weeks of diagnosis. Narrow excision margins are appropriate based on Breslow tumor thickness. For in situ tumors, the recommended excision margin is 0.5 cm. For tumors 2 mm or less in thickness, the recommended margin is 1 cm. For tumors larger than 2 mm in thickness, the recommended margin is 2 cm.

ITEMS 1-15 For each question, select the best possible response. 1. A healthy 48-year-old man presents 5 years after resection of a right calf synovial sarcoma. Surveillance imaging of the chest, abdomen, and pelvis shows a new 2-cm solitary right lower lobe lung mass. Percutaneous needle biopsy confirms metastatic sarcoma. He has no recurrence in the right calf. Pulmonary metastasectomy in this case A. B. C. D. E.

Sentinel lymph node dissection allows for evaluation of the first draining lymph node in the regional lymphatic system and is a staging procedure used when palpation and diagnostic imaging (typically, ultrasound) do not suggest lymph node metastases. For patients with tumors less than 1 mm in thickness, sentinel lymph node dissection is not recommended because the positivity rate for these thin tumors is extremely low. Sentinel lymph node dissection is required for thickness between 1 and 2 mm.

is not recommended. should include right lobectomy. improves overall survival. should be followed by whole lung irradiation. should be preceded by radiofrequency ablation.

ANSWER:

In patients who have negative sentinel lymph nodes, no further lymph node surgery is indicated. For patients with micrometastases in the sentinel lymph nodes, no clear survival benefit for complete lymph node dissection currently exists, and this fact should be critically discussed with patients. Potential benefits of complete lymph node dissection should be discussed when larger tumor deposits (>1 mm) are Identified in the sentinel lymph node.

C

Extremity sarcomas can metastasize hematogenously to the lungs. Pulmonary metastasectomy improves overall survival in sarcoma patients with isolated pulmonary metastases. More recent studies demonstrate a 30 to 50% 5-year overall survival after pulmonary metastasectomy. Patients with a longer disease-free interval and complete resection have improved outcomes. Complete resection implies removal of all disease with negative margins but does not mandate an anatomic resection (i.e., segmentectomy or lobectomy). Wedge resection is the most commonly used approach. Whole-lung irradiation is not recommended for sarcomas, with the exception of Ewing sarcoma.

There is no therapeutic advantage for elective lymph node dissection in patients who have clinically negative lymph nodes. However, if lymph node metastases are identified by physical examination or diagnostic imaging, radical lymph node dissection is standard therapy. Items 3-7

Radiofrequency ablation has been used in place of surgery for pulmonary metastases, with one small retrospective study showing a 3-year survival rate of 65.2% and a median disease-free survival of 7 months. However, it is not routinely delivered before surgical resection.

Each lettered response may be selected once, more than once, or not at all. A. B. C. D. E.

2. A 50-year-old man undergoes excisional biopsy of a mole overlying the lateral aspect of his left scapula. Other than the mole, his physical examination is normal. Pathology demonstrates a 2.5-mm thick melanoma. Initial surgical management includes A. B. C. D. E.

Dabrafenib Trastuzumab Erlotinib Imatinib Cetuximab

3. Breast cancer 4. Colon cancer 5. Gastric cancer

0.5-cm margins with sentinel lymph node dissection. 1.0-cm margins with sentinel lymph node dissection. 2.0-cm margins with sentinel lymph node dissection. 2.5-cm margins with elective lymph node dissection. 5.0-cm margins with elective lymph node dissection.

6. Gastrointestinal stromal tumor 7. Melanoma

ANSWER: ANSWER:

C

B, E, B, D, A

Trastuzumab is a targeted agent used for Her-2 positive tumors. It is indicated for the treatment of breast cancer as adjuvant therapy or for metastatic disease that shows Her-2 overexpression. It has also been approved by the US Food and Drug Administration for Her-2 overexpressing metastatic esophagogastric or gastric adenocarcinoma.

Surgical excision is the mainstay of therapy for cutaneous melanoma. With respect to diagnosis, excisional biopsy is preferred, and incisional biopsy should be avoided if possible. Definitive surgical excision should be performed with

175

Cetuximab is approved for EGFR-expressing metastatic colorectal cancer. If the tumor is KRAS positive, there is no benefit for cetuximab. Cetuximab is also approved for squamous cell head and neck cancers (locally advanced and metastatic).

macroscopically negative margins, when possible. Debulking of primary or recurrent tumors fails to improve overall survival; thus, subtotal resection should be limited to relieving intestinal obstruction when deemed appropriate. Paraneoplastic hypoglycemia is rare but may be associated with leiomyosarcomas or fibrosarcomas.

Imatinib is an oral tyrosine kinase inhibitor approved for treatment of C-Kit (or CD-117) positive gastrointestinal stromal tumors, ALL, CML, some leukemias, and other rare tumors.

9. A 65-year-old man develops weight loss and abdominal pain. CT scan of the abdomen and pelvis shows a large right colon mass. Colonoscopy is otherwise normal and colonoscopic biopsies of the mass reveal large B-cell lymphoma. PET scan reveals no other sites of disease. What is the most appropriate treatment?

Erlotinib is an oral EGFR inhibitor approved for use in metastatic non-small cell lung cancer with EGFR mutations. EGFR mutations are present in approximately 15% of lung cancer patients who are current or former smokers and nearly 50% of never-smokers. It is also approved for locally advanced or unresectable pancreatic cancer.

A. B. C. D. E.

Dabrafenib is an oral kinase inhibitor indicated for treatment of unresectable or metastatic melanoma that harbors BRAF V600E mutations. It was approved in June 2017 for metastatic non-small-cell lung cancer that also harbors BRAF mutations, which is 1 to 3% of all lung cancers.

Observation Chemotherapy Chemoradiation Right hemicolectomy Right hemicolectomy followed by chemotherapy

ANSWER:

Colonic lymphomas are uncommon, representing less than 1% of primary colonic malignancies. Large B-cell lymphoma is the most common histologic subtype. Colonic lymphomas are slightly more common in men (1.5:1) and typically present in the fifth to seventh decades of life. Risk factors include chronic immunosuppression and inflammatory bowel disease. Common symptoms include abdominal pain and weight loss. Hematochezia is also present in 20% of patients, but perforation is uncommon. Thinner patients will often have a palpable mass on abdominal exam. These cancers are typically right sided, with more than 70% of colonic lymphomas being proximal to the hepatic flexure.

8. A 50-year-old woman present to the surgical clinic with a large retroperitoneal fatty tumor identified on CT scan and confirmed on MRI. Resection of a 10 x 10 x 6-cm fatty tumor is performed, and pathology confirms a well-differentiated liposarcoma. Which of the following statements regarding this patient’s case is true? A. Recurrence is most likely to be local. B. Metastatic lesions are common. C. Risk of death is higher than for dedifferentiated liposarcomas. D. Debulking of recurrent or primary tumors improves survival. E. Paraneoplastic hypoglycemia is a common associated finding.

ANSWER:

E

The diagnosis of colonic lymphoma is obtained via colonoscopic biopsy. Staging CT scan and PET scan are indicated to exclude extracolonic disease. In the absence of metastasis, the initial treatment for colonic lymphoma is surgical resection. Adjuvant chemotherapy with cyclophosphamide, doxorubicin, vincristine, and prednisone improves long-term survival over surgery alone. Unfortunately, recurrence rates are very high (33-75%), and patients tend to re-present with diffusely metastatic disease. The median overall survival for patients who receive surgery and chemotherapy is 36 to 53 months.

A

Retroperitoneal soft tissue sarcomas are uncommon neoplasms. Due to their location, they typically present as locally advanced tumors when discovered. Only 10 to 20% of dedifferentiated tumors present with distant metastases, despite the locally advanced nature of most of these cases. Liposarcomas are the most common histologies of all tumors. There is significant variability in outcomes depending on whether the tumor is high or low grade. A well-differentiated tumor has a significantly higher survival rate than a poorly differentiated or dedifferentiated tumor.

Observation is not appropriate for patients who are fit for surgery with disease localized to the colon. Chemotherapy alone is less effective than resection followed by chemotherapy. Chemotherapy alone is typically used only in the presence of metastatic disease. Chemoradiotherapy is used for primary lymphoma in other sites and the addition of radiation can improve local control. However, external-beam radiation to the right lower quadrant of the abdomen can be morbid and is not typically used for colonic lymphomas.

The management of retroperitoneal soft tissue sarcomas is primarily surgical with complete resection to

176

however, no survival benefit is noted. This patient is a candidate for neoadjuvant radiation, but the surgical resection margins should not be reduced to 1 cm. Because sarcomas have a hematogenous route of metastases, sentinel node biopsy is not appropriate in the surgical management of these patients.

10. A 35-year-old woman presents with a firm, nontender lump over her right bicep, noted to have slowly grown over past 2 years. MRI shows a 6-cm well-circumscribed mass in the bicep muscle, adjacent to the fascia, which is close to, but does not involve, the neurovascular bundle. Core needle biopsy is diagnostic for myxofibrosarcoma. What is the appropriate surgical approach?

11. The genetic defect in patients with a Lynch-syndrome associated colon cancer results in

A. Resect the mass with a 2-cm margin, including the fascia B. Resect the entire bicep muscle, including the fascia C. Resect the bicep muscle with associated neurovascular bundle D. Resect the mass with 1-cm margin and sentinel node biopsy E. Preoperative radiation followed by resection with 1-cm margins

A. B. C. D. E.

chromosome deletion. microsatellite instability. constitutively active K-ras. mitochondrial DNA mutation. disruption of binding in beta-catenin.

ANSWER: ANSWER:

A

B

Lynch syndrome is the most common cause of hereditary colorectal cancer and accounts for 2 to 4% of cases. It has an autosomal-dominant inheritance pattern and is caused by a germline mutation in one of the mismatch repair (MMR) genes; the most commonly affected genes are MLH1, MSH2, MSH6, and PMS2 Their tumors are characterized by microsatellite instability, which can be demonstrated with polymerase-chain-reaction testing of the tumor specimens. Microsatellites are short segments of repeated DNA and are prone to increased or decreased numbers during DNA replication, the repair of which depends on MMR protein complexes. The underlying etiology of Lynch syndrome is not associated with a chromosome deletion, constitutively active K-ras, mitochondrial DNA mutation, or disruption of beta-catenin function.

Soft tissue sarcomas are a rare group of malignancies that include almost 100 different subgroups. Extremity soft tissue sarcomas account for 40% of all soft tissue sarcomas, with the remainder in the retroperitoneum and abdomen. Due to concerns for high local and regional recurrence rates, amputation of the limb was often recommended, but limbsparing wide local excision in combination with multimodal therapy provides similar overall and disease-specific survival compared with amputation. Each of the numerous histologic subtypes of soft tissue sarcomas has varying biologic activity. Many soft tissue sarcomas have characteristic imaging findings and can be diagnosed on cross-sectional imaging. If the diagnosis is not definitive with imaging, core needle or incisional biopsy can provide the histologic diagnosis. Two specific subtypes are important when discussing extremity soft tissue sarcomas. Both myxofibrosarcoma and dermatofibrosarcoma protuberans have an infiltrative growth pattern with a higher rate of local recurrence. The recommended surgical margin for both of these subtypes is 2 cm instead of the standard 1 cm. The remainder of the surgical principles remain similar.

12. Regarding anorectal infections in neutropenic patients, A. the most common symptom is perianal drainage. B. ultrasound is the preferred imaging modality. C. antibiotic therapy should be reserved for progression after initial treatment, D. cultures most often show mixed infection E. early surgical drainage is essential.

Several surgical principles guide the operative management of extremity soft tissue sarcomas. Any core needle or incisional biopsy scars should be included in the planned excision to reduce the risk of local recurrence. Most soft tissue sarcomas tumors do not require the entire muscle body to be resected; a 1-cm margin is sufficient for noninfiltrating subtypes. If a neurovascular bundle is within the planned margin, the vessels and nerve should be skeletonized but left intact. If the mass encases the neurovascular bundle, the tumor can be bivalved in an attempt to preserve these structures.

ANSWER:

D

Anorectal infections in healthy patients are most often treated with early drainage, often without antibiotics. Outcomes are generally good. By contrast, the management of such infections in neutropenic patients (e.g., patients undergoing chemotherapy and bone marrow transplantation) is complex and potentially associated with mortality. Treating physicians must consider factors such as chemotherapy toxicity, degree of neutropenia, immunosuppression, thrombocytopenia, and corticosteroid use. Mortality rates have dropped in recent

Neoadjuvant radiation therapy reduces the local recurrence rate for patients with high-grade soft tissue sarcomas;

177

years due to the use of multidisciplinary teams that understand the disease and monitor the associated neutropenia.

that may have some merit is the avoidance of needle sticks; data on skin puncture as a lymphedema risk are particularly conflicting, likely because a minority of sticks lead to infections, and infections and cellulitis are definite risk factors.

Because of the neutropenia, pus is actually a later sign, often seen only as the neutropenia begins to resolve. Accordingly, in a study of 100 patients with neutropenia-associated anorectal infection, the most common symptom (90%) was pain, often occurring 2 to 3 days earlier than other more typical symptoms such as swelling (30%) and drainage (25%). Because of this early absence of the typical findings of an abscess, MRI is preferred over ultrasound or CT scan for imaging. MRI is better at showing edema and fistulous tracts. Early antibiotic therapy is the standard of care. Because most infections in these patients are mixed, initial antibiotic regimens should be broad spectrum and cover Gram-positive, Gramnegative, anaerobic, and fungal organisms. Surgical drainage is reserved for later in the patient's course when neutropenia resolves and abscesses or fluid collection develop. Surgical incisions at this time are usually smaller and localized. Earlier surgical exploration was often associated with larger incisions, creating wound management problems.

Reverse lymphatic mapping is a surgical technique in which the extremity is injected preoperatively with radioactive or colored dyes to allow the surgeon to intraoperatively identify and leave nodes that drain the extremity. Several retrospective and prospective studies showed a decrease in lymphedema rates with these procedures. 14. A 40-year-old woman presents with an enlarging, painful mass in the lateral aspect of her right thigh. Physical exam reveals a 5- x 5-cm firm well-circumscribed, immobile mass. Imaging reveals the mass to be contained in the lateral compartment. What is the best approach to obtain a pathologic diagnosis? A. B. C. D. E.

With this multidisciplinary and reverse sequence approach to anorectal sepsis in neutropenic patients, mortality rates have fallen dramatically, although deaths still occur, especially in patients in whom the neutropenia does not resolve.

ANSWER:

13. The risk of upper extremity lymphedema after axillary node removal for cancer is increased most by A. B. C. D. E.

E

Pathologic diagnosis of a suspected extremity sarcoma is best accomplished by core needle biopsy. Preoperative documentation of malignancy helps plan definitive surgical resection and is useful for patients who may be candidates for neoadjuvant radiation therapy. Fine needle aspiration does not produce enough tissue to satisfactorily diagnose and subtype most sarcomas. Incisional biopsy may be used when core needle biopsy fails to establish a diagnosis. It is important to plan any incision in line with the final planned incision (axial orientation on the extremity).

axillary reverse lymphatic mapping. weight gain. blood pressure measurements in the involved arm. air travel. exposure to extreme temperatures.

ANSWER:

Excision of the lateral compartment Excisional biopsy Surgical resection of the mass with 1-cm margins Fine needle aspiration Core needle biopsy

B

Excisional biopsy is not recommended, because the diagnosis can typically be made with a simpler core needle biopsy, and excisional biopsy increases the risk of bleeding, infection, and contamination of surgical planes. Excision of the entire lateral compartment is more extensive surgery than recommended for this mass. Sarcomas are commonly resected with 2-cm gross surgical margins, including investing fascia.

Lymphedema affects 1 in 6 patients who undergo therapy for solid tumors in the United States. Patients who still require removal of groin or axillary nodes for cancer are particularly at risk of lymphedema. Current data suggest better outcomes if lymphedema is recognized and treated early. Accordingly, it is important to know which factors increase or decrease lymphedema risk. Meta-analyses show that the extent of node removal, radiation, and high BMI are the factors most associated with increased lymphedema risk. Weight gain after operation increases the risk 3- to 5-fold. Weight loss is a highly effective method of treating lymphedema, and patients who undergo groin and axillary node removal should be counseled to maintain a healthy weight.

15. A 45-year-old woman undergoes breast conservation surgery and sentinel lymph node biopsy for a 1.2-cm invasive ductal carcinoma of her right breast. Lumpectomy margins are clear, and only 1 of 4 sentinel lymph nodes is positive for metastatic disease. The Elston/Nottingham score is 8. The tumor is strongly ER/PR positive and Her2/neu negative. What biomarker would provide information that would mitigate her need for adjuvant chemotherapy?

Lymphedema is not related to air travel, blood pressure measurements, exercise, or extremes of temperature. Therefore, cautioning patients to avoid these things, as is still commonly done, is being re-evaluated. The one precaution

178

A. B. C. D. E.

A p53 B. 21-gene expression profile (Oncotype DX®) Ki-67 D. BRCA1/BRCA2 PD-L1

ANSWER:

with a high recurrence score (>31) achieve a significant benefit from adjuvant chemotherapy. The benefit of chemotherapy in patients with intermediate recurrence score (18-30) remains controversial and may depend on the age or menopausal status of the patient. A recent large prospective clinical trial confirmed that recurrence scores of 10 or less in node-negative patients are associated with a 5-year diseasefree survival of more than 98% with endocrine therapy alone. The data are reported as shown in figure 15.1.

B

Historically, the recommendation for adjuvant chemotherapy in patients with breast cancer was made based on clinicopathologic features, including patient age, tumor size, nodal status, and grade. Over the last 20 years, the importance of the biologic features of the tumor, specifically the presence or absence of the estrogen receptor (ER), progesterone receptor (PR), and Her2/neu receptor (HER2), have become increasingly clear. The ER/PR/HER2 receptor status is now a major determinant of the need for adjuvant chemotherapy. Antiestrogen therapy, by blocking estrogen receptors on tumor cells and thus preventing signaling stimulating proliferation or by suppressing the production of native estrogen, is very effective in limiting tumor growth, improving survival, and decreasing recurrence in ER positive breast cancer. Similarly, anti-HER2 therapies (e.g., trastuzumab, pertuzumab) are routinely used for tumors that overexpress HER2. These therapies have led to dramatic improvements in survival for patients with HER2+ breast cancer. The interest in immune- and targeted-therapy has encouraged the search for additional biomarkers that might lead to the development of additional therapeutic options. Oncotype DX® is a tissue-based analysis of 21-genes developed and validated to predict 10-year risk of distant recurrence in women with node negative, ER+/ HER2-breast cancer. Reverse transcriptase-polymerase chain reaction (RTPCR) is used to quantitate RNA found in formalin-fixed paraffin blocks, obviating the need for fresh or fresh-frozen tissue and thus expanding its utility. From a pool of 250 candidate genes identified in the medical literature, 16 genes were selected based on their ability to predict clinical outcome among patients treated in the NSABP B14 and B20 trials. Five genes were associated with proliferation/antiapoptosis: 4 representing the hormonal signaling pathway; 2 associated with tumor invasion; 2 linked to HER2; and 1 each related to detoxification, macrophage function, and apoptosis. The quantitative analysis of RNA was then normalized to the expression of 5 reference genes, allowing the development of a formula that generates a recurrence score of 0 to 100.

In the NSABP B-20 study, relatively few patients had a Recurrence Score result > 50. The chemotherapy benefit for these patients is expected to be at least as great as it is for those with Recurrence Score results = 50.

Figure 15.1. Report showing data for the 21-gene recurrence score. Two important observations emerged from the available data: (1) the use of the 21-gene recurrence score avoids adjuvant chemotherapy in a group of patients unlikely to derive benefit, and (2) the assay identifies patients likely to gain benefit from adjuvant therapy who might otherwise not be offered it based on traditional clinicopathologic features. The 21-gene recurrence score has also been incorporated into the 8th edition of the American Joint Committee on Cancer staging system such that any patient with an ER+/HER2/node-negative breast cancer with a recurrence score of less than 11 is staged as prognostic stage IA breast cancer, regardless of tumor size. Other gene expression assays are available (e.g., Mammaprint®, genomic grade index, and EndoPredict®), but only the Oncotype DX® is validated as a tool to predict the benefit of adjuvant chemotherapy. A prospective study, POsitive Node, Endocrine Responsive Breast Cancer (PONDERx) trial, by the Southwestern Oncology Group is an ongoing evaluation of the 21-gene

The 21-gene recurrence score provides a point estimate for the 10-year risk of distant disease recurrence with a 95% confidence interval, assuming that the patient completes 5 years of adjuvant hormonal therapy. Based on the analysis of patients treated in NSABP B20, a trial of endocrine therapy alone versus endocrine therapy plus chemotherapy in earlystage breast cancer, patients with a low recurrence score (<18) do not benefit from adjuvant chemotherapy. Patients

179

analysis in HR+/HER2-/ node-positive (1-3) women with a recurrence score of 25 or less. Results are anticipated in 2022. p53 is a tumor suppressor protein encoded by the TP53 gene. Mutations are found in many tumors, including breast cancer, that render the p53 protein ineffective. Current data are lacking to determine whether p53 alone is sufficient to use in clinical decisionmaking regarding prognosis. No clinical evidence exists regarding this marker's ability to predict benefit from adjuvant therapy. As such, it is not included in any of the guidelines for the management of breast cancer. Ki-67 is a protein expressed on cell membranes during the Sphase (proliferative) of the cell cycle. The level of evidence for the use of this marker for prognosis is mostly level IV. Ki67 levels are highest in triple-receptor negative breast cancers. Lack of standardization for laboratory identification and the high false-negative and false-positive rates, approximately 25% each, render it unsuitable as an independent marker for prognosis. Like other markers, data are lacking for its utility in predicting response to chemotherapy. Its use is not included in the American Society of Clinical Oncology guidelines. BRCA1/BRCA2 genes have great utility in predicting a woman's life-time risk of developing breast or ovarian cancer and might serve to influence the decision regarding preventive interventions, the decision for prophylactic surgery, or the extent of therapeutic surgical intervention. Obviously, the determination of BRCA gene mutation status is important information for family members who may also be at risk and are considering genetic testing. BRCA gene mutations do not provide useful information for decisionmaking regarding adjuvant chemotherapy. PD-L1, programed death ligand 1, is a receptor that has generated significant interest recently, not only in the medical community but also among lay persons due to the commercialization of targeted therapy touted in the media. PD-L1 is present in many tissues. When present on tumor cells, it binds to the PD-1 receptor on T-cells, inhibiting Tcell destruction of the tumor tissue. It is most commonly expressed in triple negative breast cancers, but only 20% of the time. PD-L1 identification has not been used as a prognostic marker for breast cancer or as an indicator of the benefit of chemotherapy.

180

Related Documents

Sesap 17th Vol 1
February 2021 0
Sesap 17th Vol 2
February 2021 0
Creative-beading-vol-1
February 2021 1
Mad Ter Vol.1
January 2021 4

More Documents from "Cristina"

Sesap 17th Vol 1
February 2021 0
Sesap 17th Vol 2
February 2021 0
Tourstournage_02.pdf
February 2021 2
Le Nectar Cachete
January 2021 0
Bremmer-humansacrifice
January 2021 2